You are on page 1of 168

San Beda College of Law Alabang

Constitutional Law 2 Case Digests

INTRODUCTION TO CONSTITUTIONAL LAW 2


THE NATURE OF THE CONSTITUTION AND ITS RELATION WITH THE COURTS
FRANCISCO VS. HOUSE OF REPRESENTATIVES
[415 SCRA 44; G.R. No. 160261; 10 Nov 2003]
Facts:
Impeachment proceedings were filed against Supreme Court Chief Justice Hilario Davide. The justiciable
controversy poised in front of the Court was the constitutionality of the subsequent filing of a second
complaint to controvert the rules of impeachment provided for by law.
Issue:
Whether or Not the filing of the second impeachment complaint against Chief Justice Hilario G. Davide,
Jr. with the House of Representatives falls within the one year bar provided in the Constitution and
whether the resolution thereof is a political question has resulted in a political crisis.
Held:
In any event, it is with the absolute certainty that our Constitution is sufficient to address all the issues
which this controversy spawns that this Court unequivocally pronounces, at the first instance, that the
feared resort to extra-constitutional methods of resolving it is neither necessary nor legally permissible.
Both its resolution and protection of the public interest lie in adherence to, not departure from, the
Constitution.
In passing over the complex issues arising from the controversy, this Court is ever mindful of the essential
truth that the inviolate doctrine of separation of powers among the legislative, executive or judicial
branches of government by no means prescribes for absolute autonomy in the discharge by each of that
part of the governmental power assigned to it by the sovereign people.
At the same time, the corollary doctrine of checks and balances which has been carefully calibrated by
the Constitution to temper the official acts of each of these three branches must be given effect without
destroying their indispensable co-equality. There exists no constitutional basis for the contention that the
exercise of judicial review over impeachment proceedings would upset the system of checks and
balances. Verily, the Constitution is to be interpreted as a whole and "one section is not to be allowed to
defeat another." Both are integral components of the calibrated system of independence and
interdependence that insures that no branch of government act beyond the powers assigned to it by the
Constitution.
When suing as a citizen, the interest of the petitioner assailing the constitutionality of a statute must be
direct and personal. He must be able to show, not only that the law or any government act is invalid, but
also that he sustained or is in imminent danger of sustaining some direct injury as a result of its
enforcement, and not merely that he suffers thereby in some indefinite way. It must appear that the
person complaining has been or is about to be denied some right or privilege to which he is lawfully
entitled or that he is about to be subjected to some burdens or penalties by reason of the statute or act
complained of. In fine, when the proceeding involves the assertion of a public right, the mere fact that he
is a citizen satisfies the requirement of personal interest.
In the case of a taxpayer, he is allowed to sue where there is a claim that public funds are illegally
disbursed, or that public money is being deflected to any improper purpose, or that there is a wastage of
public funds through the enforcement of an invalid or unconstitutional law. Before he can invoke the
power of judicial review, however, he must specifically prove that he has sufficient interest in preventing
the illegal expenditure of money raised by taxation and that he would sustain a direct injury as a result of
the enforcement of the questioned statute or contract. It is not sufficient that he has merely a general
interest common to all members of the public.
At all events, courts are vested with discretion as to whether or not a taxpayer's suit should be
entertained. This Court opts to grant standing to most of the petitioners, given their allegation that any

Page
Section 1-C, SY 06-07

impending transmittal to the Senate of the Articles of Impeachment and the ensuing trial of the Chief
Justice will necessarily involve the expenditure of public funds.
As for a legislator, he is allowed to sue to question the validity of any official action which he claims
infringes his prerogatives as a legislator. Indeed, a member of the House of Representatives has standing
to maintain inviolate the prerogatives, powers and privileges vested by the Constitution in his office. 83
The framers of the Constitution also understood initiation in its ordinary meaning. Thus when a proposal
reached the floor proposing that "A vote of at least one-third of all the Members of the House shall be
necessary to initiate impeachment proceedings," this was met by a proposal to delete the line on the
ground that the vote of the House does not initiate impeachment proceeding but rather the filing of a
complaint does.
To the argument that only the House of Representatives as a body can initiate impeachment proceedings
because Section 3 (1) says "The House of Representatives shall have the exclusive power to initiate all
cases of impeachment," This is a misreading of said provision and is contrary to the principle of reddendo
singula singulis by equating "impeachment cases" with "impeachment proceeding."
Having concluded that the initiation takes place by the act of filing and referral or endorsement of the
impeachment complaint to the House Committee on Justice or, by the filing by at least one-third of the
members of the House of Representatives with the Secretary General of the House, the meaning of
Section 3 (5) of Article XI becomes clear. Once an impeachment complaint has been initiated, another
impeachment complaint may not be filed against the same official within a one year period.
The Court in the present petitions subjected to judicial scrutiny and resolved on the merits only the main
issue of whether the impeachment proceedings initiated against the Chief Justice transgressed the
constitutionally imposed one-year time bar rule. Beyond this, it did not go about assuming jurisdiction
where it had none, nor indiscriminately turn justiciable issues out of decidedly political questions. Because
it is not at all the business of this Court to assert judicial dominance over the other two great branches of
the government.
No one is above the law or the Constitution. This is a basic precept in any legal system which recognizes
equality of all men before the law as essential to the law's moral authority and that of its agents to secure
respect for and obedience to its commands. Perhaps, there is no other government branch or
instrumentality that is most zealous in protecting that principle of legal equality other than the Supreme
Court which has discerned its real meaning and ramifications through its application to numerous cases
especially of the high-profile kind in the annals of jurisprudence. The Chief Justice is not above the law
and neither is any other member of this Court. But just because he is the Chief Justice does not imply that
he gets to have less in law than anybody else. The law is solicitous of every individual's rights irrespective
of his station in life.
Thus, the Rules of Procedure in Impeachment Proceedings which were approved by the House of
Representatives on November 28, 2001 are unconstitutional. Consequently, the second impeachment
complaint against Chief Justice Hilario G. Davide, Jr is barred under paragraph 5, section 3 of Article XI of
the Constitution.
MANILA PRINCE HOTEL VS. GSIS
[267 SCRA 408; G.R. No. 122156; 3 Feb 1997]
Facts:
The controversy arose when respondent Government Service Insurance System (GSIS), pursuant to the
privatization program of the Philippine Government under Proclamation No. 50 dated 8 December 1986,
decided to sell through public bidding 30% to 51% of the issued and outstanding shares of respondent
Manila Hotel Corporation. In a close bidding held on 18 September 1995 only two (2) bidders participated:
petitioner Manila Prince Hotel Corporation, a Filipino corporation, which offered to buy 51% of the MHC or
15,300,000 shares at P41.58 per share, and Renong Berhad, a Malaysian firm, with ITT-Sheraton as its
hotel operator, which bid for the same number of shares at P44.00 per share, or P2.42 more than the bid
of petitioner.
Pending the declaration of Renong Berhad as the winning bidder/strategic partner and the execution of
the necessary contracts, matched the bid price of P44.00 per share tendered by Renong Berhad.
On 17 October 1995, perhaps apprehensive that respondent GSIS has disregarded the tender of the
matching bid and that the sale of 51% of the MHC may be hastened by respondent GSIS and
consummated with Renong Berhad, petitioner came to this Court on prohibition and mandamus.

Page
Section 1-C, SY 06-07

San Beda College of Law Alabang


Constitutional Law 2 Case Digests

In the main, petitioner invokes Sec. 10, second par., Art. XII, of the 1987 Constitution and submits that the
Manila Hotel has been identified with the Filipino nation and has practically become a historical
monument which reflects the vibrancy of Philippine heritage and culture. It is a proud legacy of an earlier
generation of Filipinos who believed in the nobility and sacredness of independence and its power and
capacity to release the full potential of the Filipino people. To all intents and purposes, it has become a
part of the national patrimony. 6 Petitioner also argues that since 51% of the shares of the MHC carries
with it the ownership of the business of the hotel which is owned by respondent GSIS, a governmentowned and controlled corporation, the hotel business of respondent GSIS being a part of the tourism
industry is unquestionably a part of the national economy.
Issue:
Whether or Not the sale of Manila Hotel to Renong Berhad is violative of the Constitutional provision of
Filipino First policy and is therefore null and void.
Held:
The Manila Hotel or, for that matter, 51% of the MHC, is not just any commodity to be sold to the highest
bidder solely for the sake of privatization. The Manila Hotel has played and continues to play a significant
role as an authentic repository of twentieth century Philippine history and culture. This is the plain and
simple meaning of the Filipino First Policy provision of the Philippine Constitution. And this Court, heeding
the clarion call of the Constitution and accepting the duty of being the elderly watchman of the nation, will
continue to respect and protect the sanctity of the Constitution. It was thus ordered that GSIS accepts the
matching bid of petitioner MANILA PRINCE HOTEL CORPORATION to purchase the subject 51% of the
shares of the Manila Hotel Corporation at P44.00 per share and thereafter to execute the necessary
clearances and to do such other acts and deeds as may be necessary for purpose.
PEOPLE VS. POMAR
[46 Phil 126; G.R. No. L-22008; 3 Nov 1924]
Facts:
Macaria Fajardo was an employee of La Flor de la Isabela, a Tobacco factory. She was granted a
vacation leave, by reason of her pregnancy, which commenced on the 16 th of July 1923. According to
Fajardo, during that time, she was not given the salary due her in violation of the provisions of Act No.
3071. Fajardo filed a criminal complaint based on Section 13 and 15 of said Act against the manager of
the tobacco Factory, Julio Pomar, herein defendant. The latter, on the other hand, claims that the facts in
the complaint did not constitute an offense and further alleges that the aforementioned provisions of Act
No. 3071 was unconstitutional. Section 13, Act No. 3071 provides that, Every person, firm or corporation
owning or managing a factory, shop or place of labor of any description shall be obliged to grant to any
woman employed by it as laborer who may be pregnant, thirty days vacation with pay before and another
thirty days after confinement: Provided, That the employer shall not discharge such laborer without just
cause, under the penalty of being required to pay to her wages equivalent to the total of two months
counting from the day of her discharge. Section 15 of the same Act provides for the penalty of any
violation of section 13. The latter was enacted by the legislature in the exercise of its supposed Police
Power with the purpose of safeguarding the health of pregnant women laborers in "factory, shop or place
of labor of any description," and of insuring to them, to a certain extent, reasonable support for one month
before and one month after their delivery. The trial court rendered a decision in favor of plaintiff,
sentencing the defendant to pay the fine of fifty pesos and in case of insolvency, to suffer subsidiary
imprisonment. Hence, the case was raised to the Court of Appeals which affirmed the former decision.
Issue:
Whether or not Section 13 of Act No. 3071 is unconstitutional.
Whether or not the promulgation of the questioned provision was a valid exercise of Police Power.
Held:
The Supreme Court declared Section 13 of Act No. 3071 to be unconstitutional for being violative or
restrictive of the right of the people to freely enter into contracts for their affairs. It has been decided
several times, that the right to contract about one's affairs is a part of the liberty of the individual,
protected by the "due process of law" clause of the constitution. The contracting parties may establish any
agreements, terms, and conditions they may deem advisable, provided they are not contrary to law,
morals or public policy

Page
Section 1-C, SY 06-07

The police power of the state is a very broad and expanding power. The police power may encompass
every law for the restraint and punishment of crimes, for the preservation of the public peace, health, and
morals. But that power cannot grow faster than the fundamental law of the state, nor transcend or violate
the express inhibition of the constitution. The Police Power is subject to and is controlled by the
paramount authority of the constitution of the state, and will not be permitted to violate rights secured or
guaranteed by the latter.
LAMBINO VS. COMELEC
[G.R. No. 174153; 25 Oct 2006]
Facts:
Petitioners (Lambino group) commenced gathering signatures for an initiative petition to change the 1987
constitution, they filed a petition with the COMELEC to hold a plebiscite that will ratify their initiative
petition under RA 6735. Lambino group alleged that the petition had the support of 6M individuals
fulfilling what was provided by art 17 of the constitution. Their petition changes the 1987 constitution by
modifying sections 1-7 of Art 6 and sections 1-4 of Art 7 and by adding Art 18. the proposed changes will
shift the present bicameral- presidential form of government to unicameral- parliamentary. COMELEC
denied the petition due to lack of enabling law governing initiative petitions and invoked the Santiago Vs.
Comelec ruling that RA 6735 is inadequate to implement the initiative petitions.
Issue:
Whether or Not the Lambino Groups initiative petition complies with Section 2, Article XVII of the
Constitution on amendments to the Constitution through a peoples initiative.
Whether or Not this Court should revisit its ruling in Santiago declaring RA 6735 incomplete, inadequate
or wanting in essential terms and conditions to implement the initiative clause on proposals to amend the
Constitution.
Whether or Not the COMELEC committed grave abuse of discretion in denying due course to the
Lambino Groups petition.
Held:
According to the SC the Lambino group failed to comply with the basic requirements for conducting a
peoples initiative. The Court held that the COMELEC did not grave abuse of discretion on dismissing the
Lambino petition.
1. The Initiative Petition Does Not Comply with Section 2, Article XVII of the Constitution on Direct
Proposal by the People
The petitioners failed to show the court that the initiative signer must be informed at the time of the
signing of the nature and effect, failure to do so is deceptive and misleading which renders the
initiative void.
2. The Initiative Violates Section 2, Article XVII of the Constitution Disallowing Revision through
Initiatives
The framers of the constitution intended a clear distinction between amendment and revision, it is
intended that the third mode of stated in sec 2 art 17 of the constitution may propose only
amendments to the constitution. Merging of the legislative and the executive is a radical change,
therefore a constitutes a revision.
3. A Revisit of Santiago v. COMELEC is Not Necessary
Even assuming that RA 6735 is valid, it will not change the result because the present petition
violated Sec 2 Art 17 to be a valid initiative, must first comply with the constitution before complying
with RA 6735
Petition is dismissed.
SANTIAGO VS. COMELEC
[270 SCRA 106; G.R. No.127325; 19 Mar 1997]
Facts:

Page
Section 1-C, SY 06-07

San Beda College of Law Alabang


Constitutional Law 2 Case Digests
Private respondent Atty. Jesus Delfin, president of Peoples Initiative for Reforms, Modernization and
Action (PIRMA), filed with COMELEC a petition to amend the constitution to lift the term limits of elective
officials, through Peoples Initiative. He based this petition on Article XVII, Sec. 2 of the 1987
Constitution, which provides for the right of the people to exercise the power to directly propose
amendments to the Constitution. Subsequently the COMELEC issued an order directing the publication of
the petition and of the notice of hearing and thereafter set the case for hearing. At the hearing, Senator
Roco, the IBP, Demokrasya-Ipagtanggol ang Konstitusyon, Public Interest Law Center, and Laban ng
Demokratikong Pilipino appeared as intervenors-oppositors. Senator Roco filed a motion to dismiss the
Delfin petition on the ground that one which is cognizable by the COMELEC. The petitioners herein
Senator Santiago, Alexander Padilla, and Isabel Ongpin filed this civil action for prohibition under Rule 65
of the Rules of Court against COMELEC and the Delfin petition rising the several arguments, such as the
following: (1) The constitutional provision on peoples initiative to amend the constitution can only be
implemented by law to be passed by Congress. No such law has been passed; (2) The peoples initiative
is limited to amendments to the Constitution, not to revision thereof. Lifting of the term limits constitutes a
revision, therefore it is outside the power of peoples initiative. The Supreme Court granted the Motions
for Intervention.
Issue:
Whether or not Sec. 2, Art. XVII of the 1987 Constitution is a self-executing provision.
Whether or not COMELEC Resolution No. 2300 regarding the conduct of initiative on amendments to the
Constitution is valid, considering the absence in the law of specific provisions on the conduct of such
initiative.
Whether the lifting of term limits of elective officials would constitute a revision or an amendment of the
Constitution.
Held:
Sec. 2, Art XVII of the Constitution is not self executory, thus, without implementing legislation the same
cannot operate. Although the Constitution has recognized or granted the right, the people cannot exercise
it if Congress does not provide for its implementation.
The portion of COMELEC Resolution No. 2300 which prescribes rules and regulations on the conduct of
initiative on amendments to the Constitution, is void. It has been an established rule that what has been
delegated, cannot be delegated (potestas delegata non delegari potest). The delegation of the power to
the COMELEC being invalid, the latter cannot validly promulgate rules and regulations to implement the
exercise of the right to peoples initiative.
The lifting of the term limits was held to be that of a revision, as it would affect other provisions of the
Constitution such as the synchronization of elections, the constitutional guarantee of equal access to
opportunities for public service, and prohibiting political dynasties. A revision cannot be done by initiative.
However, considering the Courts decision in the above Issue, the issue of whether or not the petition is a
revision or amendment has become academic.
GONZALES VS. COMELEC
[21 SCRA 774; G.R. No. L-28196; 9 Nov 1967]
Facts:
The case is an original action for prohibition, with preliminary injunction.
The main facts are not disputed. On March 16, 1967, the Senate and the House of Representatives
passed the following resolutions:
1. R. B. H. (Resolution of Both Houses) No. 1, proposing that Section 5, Article VI, of the Constitution of
the Philippines, be amended so as to increase the membership of the House of Representatives from a
maximum of 120, as provided in the present Constitution, to a maximum of 180, to be apportioned among
the several provinces as nearly as may be according to the number of their respective inhabitants,
although each province shall have, at least, one (1) member;
2. R. B. H. No. 2, calling a convention to propose amendments to said Constitution, the convention to be
composed of two (2) elective delegates from each representative district, to be "elected in the general
elections to be held on the second Tuesday of November, 1971;" and

Page
Section 1-C, SY 06-07

3. R. B. H. No. 3, proposing that Section 16, Article VI, of the same Constitution, be amended so as to
authorize Senators and members of the House of Representatives to become delegates to the
aforementioned constitutional convention, without forfeiting their respective seats in Congress.
Subsequently, Congress passed a bill, which, upon approval by the President, on June 17, 1967, became
Republic Act No. 4913, providing that the amendments to the Constitution proposed in the
aforementioned Resolutions No. 1 and 3 be submitted, for approval by the people, at the general
elections which shall be held on November 14, 1967.
Issue:
Whether or Not a Resolution of Congress, acting as a constituent assembly, violates the Constitution.
Held:
Inasmuch as there are less than eight (8) votes in favor of declaring Republic Act 4913 and R. B. H. Nos.
1 and 3 unconstitutional and invalid, the petitions in these two (2) cases must be, as they are hereby,
dismiss and the writs therein prayed for denied, without special pronouncement as to costs. It is so
ordered.
As a consequence, the title of a de facto officer cannot be assailed collaterally. It may not be contested
except directly, by quo warranto proceedings. Neither may the validity of his acts be questioned upon the
ground that he is merely a de facto officer. And the reasons are obvious: (1) it would be an indirect inquiry
into the title to the office; and (2) the acts of a de facto officer, if within the competence of his office, are
valid, insofar as the public is concerned.
"The judicial department is the only constitutional organ which can be called upon to determine the proper
allocation of powers between the several departments and among the integral or constituent units
thereof."
Article XV of the Constitution provides:
. . . The Congress in joint session assembled, by a vote of three-fourths of all the
Members of the Senate and of the House of Representatives voting separately, may
propose amendments to this Constitution or call a contention for that purpose. Such
amendments shall be valid as part of this Constitution when approved by a majority of the
votes cast at an election at which the amendments are submitted to the people for their
ratification.
From our viewpoint, the provisions of Article XV of the Constitution are satisfied so long as the electorate
knows that R. B. H. No. 3 permits Congressmen to retain their seats as legislators, even if they should run
for and assume the functions of delegates to the Convention.

SANIDAD VS. COMELEC


[78 SCRA 333; G.R. No. 90878; 29 Jan 1990]
Facts:
This is a petition for certiorari assailing the constitutionality of Section 19 of Comelec Resolution No. 2167
on the ground that it violates the constitutional guarantees of the freedom of expression and of the press.
On October 23, 1989, Republic Act No. 6766, entitled "AN ACT PROVIDING FOR AN ORGANIC ACT
FOR THE CORDILLERA AUTONOMOUS REGION" was enacted into law. Pursuant to said law, the City
of Baguio and the Cordilleras which consist of the provinces of Benguet, Mountain Province, Ifugao, Abra
and Kalinga-Apayao, all comprising the Cordillera Autonomous Region, shall take part in a plebiscite for
the ratification of said Organic Act originally scheduled last December 27, 1989 which was, however, reset
to January 30, 1990 by virtue of Comelec Resolution No. 2226 dated December 27, 1989. The
Commission on Elections, by virtue of the power vested by the 1987 Constitution, the Omnibus Election
Code (BP 881), said R.A. 6766 and other pertinent election laws, promulgated Resolution No. 2167, to
govern the conduct of the plebiscite on the said Organic Act for the Cordillera Autonomous Region. In a
petition dated November 20, 1989, herein petitioner Pablito V. Sanidad, who claims to be a newspaper

Page
Section 1-C, SY 06-07

San Beda College of Law Alabang


Constitutional Law 2 Case Digests
columnist of the "OVERVIEW" for the BAGUIO MIDLAND COURIER, a weekly newspaper circulated in
the City of Baguio and the Cordilleras, assailed the constitutionality of Section 19 of Comelec Resolution
No. 2167, which provides:
Section 19. Prohibition on columnists, commentators or announcers. During the
plebiscite campaign period, on the day before and on the plebiscite day, no mass media
columnist, commentator, announcer or personality shall use his column or radio or
television time to campaign for or against the plebiscite Issue.
It is alleged by petitioner that said provision is void and unconstitutional because it violates the
constitutional guarantees of the freedom of expression and of the press enshrined in the
Constitution. Unlike a regular news reporter or news correspondent who merely reports the news,
petitioner maintains that as a columnist, his column obviously and necessarily contains and
reflects his opinions, views and beliefs on any issue or subject about which he writes. Petitioner
likewise maintains that if media practitioners were to express their views, beliefs and opinions on
the issue submitted to a plebiscite, it would in fact help in the government drive and desire to
disseminate information, and hear, as well as ventilate, all sides of the issue.
Issue:
Whether or not Section 19 of Comelec Resolution No. 2167 is unconstitutional.
Held:
The Supreme Court ruled that Section 19 of Comelec Resolution No. 2167 is unconstitutional. It is clear
from Art. IX-C of the 1987 Constitution that what was granted to the Comelec was the power to supervise
and regulate the use and enjoyment of franchises, permits or other grants issued for the operation of
transportation or other public utilities, media of communication or information to the end that equal
opportunity, time and space, and the right to reply, including reasonable, equal rates therefor, for public
information campaigns and forums among candidates are ensured. The evil sought to be prevented by
this provision is the possibility that a franchise holder may favor or give any undue advantage to a
candidate in terms of advertising space or radio or television time. This is also the reason why a
"columnist, commentator, announcer or personality, who is a candidate for any elective office is required
to take a leave of absence from his work during the campaign period (2nd par. Section 11(b) R.A. 6646).
It cannot be gainsaid that a columnist or commentator who is also a candidate would be more exposed to
the voters to the prejudice of other candidates unless required to take a leave of absence.
However, neither Article IX-C of the Constitution nor Section 11 (b), 2nd par. of R.A. 6646 can be
construed to mean that the Comelec has also been granted the right to supervise and regulate the
exercise by media practitioners themselves of their right to expression during plebiscite periods. Media
practitioners exercising their freedom of expression during plebiscite periods are neither the franchise
holders nor the candidates. In fact, there are no candidates involved in a plebiscite. Therefore, Section 19
of Comelec Resolution No. 2167 has no statutory basis.
Plebiscite Issue are matters of public concern and importance. The people's right to be informed and to
be able to freely and intelligently make a decision would be better served by access to an unabridged
discussion of the Issue, including the forum. The people affected by the Issue presented in a plebiscite
should not be unduly burdened by restrictions on the forum where the right to expression may be
exercised. Comelec spaces and Comelec radio time may provide a forum for expression but they do not
guarantee full dissemination of information to the public concerned because they are limited to either
specific portions in newspapers or to specific radio or television times.
The instant petition is GRANTED. Section 19 of Comelec Resolution No. 2167 is declared null and void
and unconstitutional.

BONDOC VS. PINEDA


[201 SCRA 792; G.R. No. 97710; 26 Sep 1991]
Facts:
In the elections held on May 11, 1987, Marciano Pineda of the LDP and Emigdio Bondoc of the NP were
candidates for the position of Representative for the Fourth District of Pampanga. Pineda was proclaimed
winner. Bondoc filed a protest in the House of Representatives Electoral Tribunal (HRET), which is
composed of 9 members, 3 of whom are Justices of the SC and the remaining 6 are members of the
House of Representatives (5 members belong to the LDP and 1 member is from the NP). Thereafter, a

Page
Section 1-C, SY 06-07

decision had been reached in which Bondoc won over Pineda. Congressman Camasura of the LDP
voted with the SC Justices and Congressman Cerilles of the NP to proclaim Bondoc the winner of the
contest.
On the eve of the promulgation of the Bondoc decision, Congressman Camasura
received a letter informing him that he was already expelled from the LDP for allegedly helping to
organize the Partido Pilipino of Eduardo Cojuangco and for allegedly inviting LDP members in Davao Del
Sur to join said political party. On the day of the promulgation of the decision, the Chairman of HRET
received a letter informing the Tribunal that on the basis of the letter from the LDP, the House of
Representatives decided to withdraw the nomination and rescind the election of Congressman Camasura
to the HRET.
Issue:
Whether or not the House of Representatives, at the request of the dominant political party therein, may
change that partys representation in the HRET to thwart the promulgation of a decision freely reached by
the tribunal in an election contest pending therein
Held:
The purpose of the constitutional convention creating the Electoral Commission was to provide an
independent and impartial tribunal for the determination of contests to legislative office, devoid of partisan
consideration.
As judges, the members of the tribunal must be non-partisan. They must discharge their functions with
complete detachment, impartiality and independence even independence from the political party to which
they belong. Hence, disloyalty to party and breach of party discipline are not valid grounds for the
expulsion of a member of the tribunal. In expelling Congressman Camasura from the HRET for having
cast a conscience vote in favor of Bondoc, based strictly on the result of the examination and
appreciation of the ballots and the recount of the votes by the tribunal, the House of Representatives
committed a grave abuse of discretion, an injustice and a violation of the Constitution. Its resolution of
expulsion against Congressman Camasura is, therefore, null and void.
Another reason for the nullity of the expulsion resolution of the House of Representatives is that it violates
Congressman Camasuras right to security of tenure. Members of the HRET, as sole judge of
congressional election contests, are entitled to security of tenure just as members of the Judiciary enjoy
security of tenure under the Constitution. Therefore, membership in the HRET may not be terminated
except for a just cause, such as, the expiration of the members congressional term of office, his death,
permanent disability, resignation from the political party he represents in the tribunal, formal affiliation with
another political party or removal for other valid cause. A member may not be expelled by the House of
Representatives for party disloyalty, short of proof that he has formally affiliated with another

MIRASOL VS CA
[351 SCRA 44; G.R. No. 128448; 1 Feb 2001]
Facts:
The Mirasols are sugarland owners and planters. Philippine National Bank (PNB) financed the Mirasols'
sugar production venture FROM 1973-1975 under a crop loan financing scheme. The Mirasols signed
Credit Agreements, a Chattel Mortgage on Standing Crops, and a Real Estate Mortgage in favor of PNB.
The Chattel Mortgage empowered PNB to negotiate and sell the latter's sugar and to apply the proceeds
to the payment of their obligations to it.
President Marcos issued PD 579 in November, 1974 authorizing Philippine Exchange Co., Inc. (PHILEX)
to purchase sugar allocated for export and authorized PNB to finance PHILEX's purchases. The decree
directed that whatever profit PHILEX might realize was to be remitted to the government. Believing that
the proceeds were more than enough to pay their obligations, petitioners asked PNB for an accounting of
the proceeds which it ignored. Petitioners continued to avail of other loans from PNB and to make
unfunded withdrawals from their accounts with said bank. PNB asked petitioners to settle their due and
demandable accounts. As a result, petitioners, conveyed to PNB real properties by way of dacion en
pago still leaving an unpaid amount. PNB proceeded to extrajudicially foreclose the mortgaged properties.
PNB still had a deficiency claim.
Petitioners continued to ask PNB to account for the proceeds, insisting that said proceeds, if properly
liquidated, could offset their outstanding obligations. PNB remained adamant in its stance that under P.D.

Page
Section 1-C, SY 06-07

San Beda College of Law Alabang


Constitutional Law 2 Case Digests
No. 579, there was nothing to account since under said law, all earnings from the export sales of sugar
pertained to the National Government.
On August 9, 1979, the Mirasols filed a suit for accounting, specific performance, and damages against
PNB.
Issue:
Whether or not the Trial Court has jurisdiction to declare a statute unconstitutional without notice to the
Solicitor General where the parties have agreed to submit such issue for the resolution of the Trial Court.
Whether PD 579 and subsequent issuances thereof are unconstitutional.
Whether or not said PD is subject to judicial review.
Held:
It is settled that Regional Trial Courts have the authority and jurisdiction to consider the constitutionality of
a statute, presidential decree, or executive order. The Constitution vests the power of judicial review or
the power to declare a law, treaty, international or executive agreement, presidential decree, order,
instruction, ordinance, or regulation not only in this Court, but in all Regional Trial Courts.
The purpose of the mandatory notice in Rule 64, Section 3 is to enable the Solicitor General to decide
whether or not his intervention in the action assailing the validity of a law or treaty is necessary. To deny
the Solicitor General such notice would be tantamount to depriving him of his day in court. We must
stress that, contrary to petitioners' stand, the mandatory notice requirement is not limited to actions
involving declaratory relief and similar remedies. The rule itself provides that such notice is required in
"any action" and not just actions involving declaratory relief. Where there is no ambiguity in the words
used in the rule, there is no room for construction. 15 In all actions assailing the validity of a statute,
treaty, presidential decree, order, or proclamation, notice to the Solicitor General is mandatory.
Petitioners contend that P.D. No. 579 and its implementing issuances are void for violating the due
process clause and the prohibition against the taking of private property without just compensation.
Petitioners now ask this Court to exercise its power of judicial review.
Jurisprudence has laid down the following requisites for the exercise of this power: First, there must be
before the Court an actual case calling for the exercise of judicial review. Second, the question before the
Court must be ripe for adjudication. Third, the person challenging the validity of the act must have
standing to challenge. Fourth, the question of constitutionality must have been raised at the earliest
opportunity, and lastly, the issue of constitutionality must be the very lis mota of the case.
DUMLAO VS. COMELEC
[95 SCRA 392; G.R. No.L-52245; 22 Jan 1980]
Facts:
Petitioner Dumlao questions the constitutionality of Sec. 4 of Batas Pambansa Blg 52 as discriminatory
and contrary to equal protection and due process guarantees of the Constitution. Sec. 4 provides that any
retired elective provincial or municipal official who has received payments of retirement benefits and shall
have been 65 years of age at the commencement of the term of office to which he seeks to be elected,
shall not be qualified to run for the same elective local office from which he has retired. According to
Dumlao, the provision amounts to class legislation. Petitioners Igot and Salapantan Jr. also assail the
validity of Sec. 4 of Batas Pambansa Blg 52, which states that any person who has committed any act of
disloyalty to the State, including those amounting to subversion, insurrection, rebellion, or other similar
crimes, shall not be qualified for any of the offices covered by the act, or to participate in any partisan
activity therein: provided that a judgment of conviction of those crimes shall be conclusive evidence of
such fact and the filing of charges for the commission of such crimes before a civil court or military
tribunal after preliminary investigation shall be prima facie evidence of such fact.
Issue:
Whether or Not the aforementioned statutory provisions violate the Constitution and thus, should be
declared null and void
Whether or not the requisites of judicial review are complied with
Held:

Page
Section 1-C, SY 06-07

No constitutional question will be heard and decided by the Court unless there is compliance with the
requisites of a judicial inquiry, which are: 1) There must be an actual case or controversy; 2) The question
of constitutionality must be raised by the proper party; 3) The constitutional question must be raised at the
earliest possible opportunity; and 4) The decision of the constitutional question must be necessary to the
determination of the case itself.
As to (1), Dumlao has not been adversely affected by the application of the provision. His question is
posed merely in the abstract, and without the benefit of a detailed factual record. As to (2), neither Igot nor
Salapantan has been charged with acts of loyalty to the State, nor disqualified from being candidates for
local elective positions. They have no personal nor substantial interest at stake. Igot and Salapantan have
institute the case as a taxpayers suit, but the institution of a taxpayers suit per se is no assurance of
judicial review. As to (4), there is no cause of action in this particular case. Therefore, the necessity for
resolving the issue of constitutionality is absent.
In regards to the unconstitutionality of the provisions, Sec. 4 of BP Blg 52 remains constitutional and valid.
The constitutional guarantee of equal protection of the laws is subject to rational classification. One class
can be treated differently from another class. In this case, employees 65 years of age are classified
differently from younger employees. The purpose of the provision is to satisfy the need for new blood in
the workplace. In regards to the second paragraph of Sec. 4, it should be declared null and void for being
violative of the constitutional presumption of innocence guaranteed to an accused.
LACSON VS. PEREZ
[357 SCRA 756; G.R. No. 147780 ;10 May 2001]
Facts:
President Macapagal-Arroyo declared a State of Rebellion (Proclamation No. 38) on May 1, 2001 as well
as General Order No. 1 ordering the AFP and the PNP to suppress the rebellion in the NCR. Warrantless
arrests of several alleged leaders and promoters of the rebellion were thereafter effected. Petitioner
filed for prohibition, injunction, mandamus and habeas corpus with an application for the issuance of
temporary restraining order and/or writ of preliminary injunction. Petitioners assail the declaration of Proc.
No. 38 and the warrantless arrests allegedly effected by virtue thereof. Petitioners furthermore pray that
the appropriate court, wherein the information against them were filed, would desist arraignment and trial
until this instant petition is resolved. They also contend that they are allegedly faced with impending
warrantless arrests and unlawful restraint being that hold departure orders were issued against them.
Issue:
Whether or Not Proclamation No. 38 is valid, along with the warrantless arrests and hold departure orders
allegedly effected by the same.
Held:
President Macapagal-Arroyo ordered the lifting of Proc. No. 38 on May 6, 2006, accordingly the instant
petition has been rendered moot and academic. Respondents have declared that the Justice Department
and the police authorities intend to obtain regular warrants of arrests from the courts for all acts
committed prior to and until May 1, 2001. Under Section 5, Rule 113 of the Rules of Court, authorities
may only resort to warrantless arrests of persons suspected of rebellion in suppressing the rebellion if the
circumstances so warrant, thus the warrantless arrests are not based on Proc. No. 38. Petitioners prayer
for mandamus and prohibition is improper at this time because an individual warrantlessly arrested has
adequate remedies in law: Rule 112 of the Rules of Court, providing for preliminary investigation, Article
125 of the Revised Penal Code, providing for the period in which a warrantlessly arrested person must be
delivered to the proper judicial authorities, otherwise the officer responsible for such may be penalized for
the delay of the same. If the detention should have no legal ground, the arresting officer can be charged
with arbitrary detention, not prejudicial to claim of damages under Article 32 of the Civil Code. Petitioners
were neither assailing the validity of the subject hold departure orders, nor were they expressing any
intention to leave the country in the near future. To declare the hold departure orders null and void ab
initio must be made in the proper proceedings initiated for that purpose. Petitioners prayer for relief
regarding their alleged impending warrantless arrests is premature being that no complaints have been
filed against them for any crime, furthermore, the writ of habeas corpus is uncalled for since its purpose is
to relieve unlawful restraint which Petitioners are not subjected to.
Petition is dismissed. Respondents, consistent and congruent with their undertaking earlier adverted to,
together with their agents, representatives, and all persons acting in their behalf, are hereby enjoined

Page
Section 1-C, SY 06-07

10

San Beda College of Law Alabang


Constitutional Law 2 Case Digests
from arresting Petitioners without the required judicial warrants for all acts committed in relation to or in
connection with the May 1, 2001 siege of Malacaang.
SANLAKAS VS. EXECUTIVE SECRETARY
[421 SCRA 656; G.R. No. 159085; 3 Feb 2004]
Facts:
During the wee hours of July 27, 2003, some three-hundred junior officers and enlisted men of the AFP,
acting upon instigation, command and direction of known and unknown leaders have seized the Oakwood
Building in Makati. Publicly, they complained of the corruption in the AFP and declared their withdrawal of
support for the government, demanding the resignation of the President, Secretary of Defense and the
PNP Chief. These acts constitute a violation of Article 134 of the Revised Penal Code, and by virtue of
Proclamation No. 427 and General Order No. 4, the Philippines was declared under the State of
Rebellion. Negotiations took place and the officers went back to their barracks in the evening of the same
day. On August 1, 2003, both the Proclamation and General Orders were lifted, and Proclamation No.
435, declaring the Cessation of the State of Rebellion was issued.
In the interim, however, the following petitions were filed: (1) SANLAKAS AND PARTIDO NG
MANGGAGAWA VS. EXECUTIVE SECRETARY, petitioners contending that Sec. 18 Article VII of the
Constitution does not require the declaration of a state of rebellion to call out the AFP, and that there is no
factual basis for such proclamation. (2)SJS Officers/Members v. Hon. Executive Secretary, et al,
petitioners contending that the proclamation is a circumvention of the report requirement under the same
Section 18, Article VII, commanding the President to submit a report to Congress within 48 hours from the
proclamation of martial law. Finally, they contend that the presidential issuances cannot be construed as
an exercise of emergency powers as Congress has not delegated any such power to the President. (3)
Rep. Suplico et al. v. President Macapagal-Arroyo and Executive Secretary Romulo, petitioners
contending that there was usurpation of the power of Congress granted by Section 23 (2), Article VI of the
Constitution. (4) Pimentel v. Romulo, et al, petitioner fears that the declaration of a state of rebellion
"opens the door to the unconstitutional implementation of warrantless arrests" for the crime of rebellion.
Issue:
Whether or Not Proclamation No. 427 and General Order No. 4 are constitutional?
Whether or Not the petitioners have a legal standing or locus standi to bring suit?
Held:
The Court rendered that the both the Proclamation No. 427 and General Order No. 4 are constitutional.
Section 18, Article VII does not expressly prohibit declaring state or rebellion. The President in addition to
its Commander-in-Chief Powers is conferred by the Constitution executive powers. It is not disputed that
the President has full discretionary power to call out the armed forces and to determine the necessity for
the exercise of such power. While the Court may examine whether the power was exercised within
constitutional limits or in a manner constituting grave abuse of discretion, none of the petitioners here
have, by way of proof, supported their assertion that the President acted without factual basis. The issue
of the circumvention of the report is of no merit as there was no indication that military tribunals have
replaced civil courts or that military authorities have taken over the functions of Civil Courts. The issue of
usurpation of the legislative power of the Congress is of no moment since the President, in declaring a
state of rebellion and in calling out the armed forces, was merely exercising a wedding of her Chief
Executive and Commander-in-Chief powers. These are purely executive powers, vested on the President
by Sections 1 and 18, Article VII, as opposed to the delegated legislative powers contemplated by Section
23 (2), Article VI. The fear on warrantless arrest is unreasonable, since any person may be subject to this
whether there is rebellion or not as this is a crime punishable under the Revised Penal Code, and as long
as a valid warrantless arrest is present.
Legal standing or locus standi has been defined as a personal and substantial interest in the case such
that the party has sustained or will sustain direct injury as a result of the governmental act that is being
challenged. The gist of the question of standing is whether a party alleges "such personal stake in the
outcome of the controversy as to assure that concrete adverseness which sharpens the presentation of
Issue upon which the court depends for illumination of difficult constitutional questions. Based on the
foregoing, petitioners Sanlakas and PM, and SJS Officers/Members have no legal standing to sue. Only
petitioners Rep. Suplico et al. and Sen. Pimentel, as Members of Congress, have standing to challenge
the subject issuances. It sustained its decision in Philippine Constitution Association v. Enriquez, that the
extent the powers of Congress are impaired, so is the power of each member thereof, since his office
confers a right to participate in the exercise of the powers of that institution.

Page
Section 1-C, SY 06-07

11

JOYA VS. PCGG


[225 SCRA 568; G.R. No. 96541; 24 Aug 1993]
Facts:
On 9 August 1990, Mateo A.T. Caparas, then Chairman of PCGG, wrote then President Corazon C.
Aquino, requesting her for authority to sign the proposed Consignment Agreement between the Republic
of the Philippines through PCGG and Christie, Manson and Woods International, Inc concerning the
scheduled sale on 11 January 1991 of eighty-two) Old Masters Paintings and antique silverware seized
from Malacaang and the Metropolitan Museum of Manila alleged to be part of the ill-gotten wealth of the
late President Marcos, his relatives and cronies. On 14 August 1990, then President Aquino, through
former Executive Secretary Catalino Macaraig, Jr., authorized Chairman Caparas to sign the
Consignment Agreement allowing Christie's of New York to auction off the subject art pieces for and in
behalf of the Republic of the Philippines. On 15 August 1990, PCGG, through Chairman Caparas,
representing the Government of the Republic of the Philippines, signed the Consignment Agreement with
Christie's of New York. According to the agreement, PCGG shall consign to CHRISTIE'S for sale at public
auction the eighty-two Old Masters Paintings then found at the Metropolitan Museum of Manila as well as
the silverware contained in seventy-one cartons in the custody of the Central Bank of the Philippines, and
such other property as may subsequently be identified by PCGG and accepted by CHRISTIE'S to be
subject to the provisions of the agreement.
On 26 October 1990, the Commission on Audit through then Chairman Eufemio C. Domingo submitted to
President Aquino the audit findings and observations of COA on the Consignment Agreement of 15
August 1990 to the effect that: the authority of former PCGG Chairman Caparas to enter into the
Consignment Agreement was of doubtful legality; the contract was highly disadvantageous to the
government; PCGG had a poor track record in asset disposal by auction in the U.S.; and, the assets
subject of auction were historical relics and had cultural significance, hence, their disposal was prohibited
by law.
After the oral arguments of the parties on 9 January 1991, we issued immediately our resolution denying
the application for preliminary injunction to restrain the scheduled sale of the artworks on the ground that
petitioners had not presented a clear legal right to a restraining order and that proper parties had not been
impleaded.
On 11 January 1991, the sale at public auction proceeded as scheduled and the proceeds of
$13,302,604.86 were turned over to the Bureau of Treasury.
Issue:
Whether or not petitioners have legal standing.
Whether or not the Old Masters Paintings and antique silverware are embraced in the phrase "cultural
treasure of the nation".
Whether or not the paintings and silverware are properties of public dominion on which can be disposed
of
through
the
joint
concurrence
of
the
President
and
Congress.
Whether or not PCGG has complied with the due process clause and other statutory requirements for the
exportation and sale of the subject items.
Whether or not the petition has become moot and academic, and if so, whether the above Issue warrant
resolution from this Court.
Held:
This is premised on Sec. 2, Rule 3, of the Rules of Court which provides that every action must be
prosecuted and defended in the name of the real party-in-interest, and that all persons having interest in
the subject of the action and in obtaining the relief demanded shall be joined as plaintiffs. The Court will
exercise its power of judicial review only if the case is brought before it by a party who has the legal
standing to raise the constitutional or legal question. "Legal standing" means a personal and substantial
interest in the case such that the party has sustained or will sustain direct injury as a result of the
governmental act that is being challenged. The term "interest" is material interest, an interest in issue and
to be affected by the decree, as distinguished from mere interest in the question involved, or a mere
incidental interest. Moreover, the interest of the party plaintiff must be personal and not one based on a
desire to vindicate the constitutional right of some third and related party.

Page
Section 1-C, SY 06-07

12

San Beda College of Law Alabang


Constitutional Law 2 Case Digests

There are certain instances however when this Court has allowed exceptions to the rule on legal
standing, as when a citizen brings a case for mandamus to procure the enforcement of a public duty for
the fulfillment of a public right recognized by the Constitution, and when a taxpayer questions the validity
of a governmental act authorizing the disbursement of public funds.
Petitioners' arguments are devoid of merit. They lack basis in fact and in law. The ownership of these
paintings legally belongs to the foundation or corporation or the members thereof, although the public has
been given the opportunity to view and appreciate these paintings when they were placed on exhibit.
The confiscation of these properties by the Aquino administration however should not be understood to
mean that the ownership of these paintings has automatically passed on the government without
complying with constitutional and statutory requirements of due process and just compensation. If these
properties were already acquired by the government, any constitutional or statutory defect in their
acquisition and their subsequent disposition must be raised only by the proper parties the true owners
thereof whose authority to recover emanates from their proprietary rights which are protected by statutes
and the Constitution. Having failed to show that they are the legal owners of the artworks or that the
valued pieces have become publicly owned, petitioners do not possess any clear legal right whatsoever
to question their alleged unauthorized disposition.
Neither can this petition be allowed as a taxpayer's suit. Obviously, petitioners are not challenging any
expenditure involving public funds but the disposition of what they allege to be public properties. It is
worthy to note that petitioners admit that the paintings and antique silverware were acquired from private
sources and not with public money.
Anent the second requisite of actual controversy, petitioners argue that this case should be resolved by
this Court as an exception to the rule on moot and academic cases; that although the sale of the paintings
and silver has long been consummated and the possibility of retrieving the treasure trove is nil, yet the
novelty and importance of the Issue raised by the petition deserve this Court's attention. They submit that
the resolution by the Court of the Issue in this case will establish future guiding principles and doctrines
on the preservation of the nation's priceless artistic and cultural possessions for the benefit of the public
as a whole.
For a court to exercise its power of adjudication, there must be an actual case of controversy one
which involves a conflict of legal rights, an assertion of opposite legal claims susceptible of judicial
resolution; the case must not be moot or academic or based on extra-legal or other similar considerations
not cognizable by a court of justice. A case becomes moot and academic when its purpose has become
stale, such as the case before us. Since the purpose of this petition for prohibition is to enjoin respondent
public officials from holding the auction sale of the artworks on a particular date 11 January 1991
which is long past, the Issue raised in the petition have become moot and academic.
The cultural properties of the nation which shall be under the protection of the state are classified as the
"important cultural properties" and the "national cultural treasures." On the other hand, a "national cultural
treasures" is a unique object found locally, possessing outstanding historical, cultural, artistic and/or
scientific value which is highly significant and important to this country and nation. This Court takes note
of the certification issued by the Director of the Museum that the Italian paintings and silverware subject of
this petition do not constitute protected cultural properties and are not among those listed in the Cultural
Properties Register of the National Museum.
WHEREFORE, for lack of merit, the petition for prohibition and mandamus is DISMISSED.

OPOSA VS. FACTORAN, JR.


[224 SCRA 792; G.R. No. 101083; 30 Jul 1993]
Facts:
Principal petitioners, are all minors duly represented and joined by their respective parents. Impleaded as
an additional plaintiff is the Philippine Ecological Network, Inc. (PENI), a domestic, non-stock and nonprofit corporation organized for the purpose of, inter alia, engaging in concerted action geared for the
protection of our environment and natural resources. The original defendant was the Honorable Fulgencio
S. Factoran, Jr., then Secretary of the Department of Environment and Natural Resources (DENR). His
substitution in this petition by the new Secretary, the Honorable Angel C. Alcala, was subsequently
ordered upon proper motion by the petitioners. The complaint was instituted as a taxpayers' class suit and
alleges that the plaintiffs "are all citizens of the Republic of the Philippines, taxpayers, and entitled to the
full benefit, use and enjoyment of the natural resource treasure that is the country's virgin tropical forests."

Page
Section 1-C, SY 06-07

13

The same was filed for themselves and others who are equally concerned about the preservation of said
resource but are "so numerous that it is impracticable to bring them all before the Court."
On 22 June 1990, the original defendant, Secretary Factoran, Jr., filed a Motion to Dismiss the complaint
based on two grounds, namely: the plaintiffs have no cause of action against him and, the issue raised
by the plaintiffs is a political question which properly pertains to the legislative or executive branches of
Government. In their 12 July 1990 Opposition to the Motion, the petitioners maintain that, the complaint
shows a clear and unmistakable cause of action, the motion is dilatory and the action presents a
justiciable question as it involves the defendant's abuse of discretion.
On 18 July 1991, respondent Judge issued an order granting the aforementioned motion to dismiss. In
the said order, not only was the defendant's claim that the complaint states no cause of action against him
and that it raises a political question sustained, the respondent Judge further ruled that the granting of the
relief prayed for would result in the impairment of contracts which is prohibited by the fundamental law of
the land.
Plaintiffs thus filed the instant special civil action for certiorari under Rule 65 of the Revised Rules of Court
and ask this Court to rescind and set aside the dismissal order on the ground that the respondent Judge
gravely abused his discretion in dismissing the action. Again, the parents of the plaintiffs-minors not only
represent their children, but have also joined the latter in this case.
Petitioners contend that the complaint clearly and unmistakably states a cause of action as it contains
sufficient allegations concerning their right to a sound environment based on Articles 19, 20 and 21 of the
Civil Code (Human Relations), Section 4 of Executive Order (E.O.) No. 192 creating the DENR, Section 3
of Presidential Decree (P.D.) No. 1151 (Philippine Environmental Policy), Section 16, Article II of the 1987
Constitution recognizing the right of the people to a balanced and healthful ecology, the concept of
generational genocide in Criminal Law and the concept of man's inalienable right to self-preservation and
self-perpetuation embodied in natural law. Petitioners likewise rely on the respondent's correlative
obligation per Section 4 of E.O. No. 192, to safeguard the people's right to a healthful environment.
It is further claimed that the issue of the respondent Secretary's alleged grave abuse of discretion in
granting Timber License Agreements (TLAs) to cover more areas for logging than what is available
involves a judicial question.
Anent the invocation by the respondent Judge of the Constitution's non-impairment clause, petitioners
maintain that the same does not apply in this case because TLAs are not contracts. They likewise submit
that even if TLAs may be considered protected by the said clause, it is well settled that they may still be
revoked by the State when the public interest so requires.
Issue:
Whether or not the petitioners have locus standi.
Whether or not the petiton is in a form of a class suit.
Whether or not the TLAs can be out rightly cancelled.
Whether or not the petition should be dismissed.
Held:
As to the matter of the cancellation of the TLAs, respondents submit that the same cannot be done by the
State without due process of law. Once issued, a TLA remains effective for a certain period of time
usually for twenty-five (25) years. During its effectivity, the same can neither be revised nor cancelled
unless the holder has been found, after due notice and hearing, to have violated the terms of the
agreement or other forestry laws and regulations. Petitioners' proposition to have all the TLAs
indiscriminately cancelled without the requisite hearing would be violative of the requirements of due
process.
The subject matter of the complaint is of common and general interest not just to several, but to all
citizens of the Philippines. Consequently, since the parties are so numerous, it, becomes impracticable, if
not totally impossible, to bring all of them before the court. The plaintiffs therein are numerous and
representative enough to ensure the full protection of all concerned interests. Hence, all the requisites for
the filing of a valid class suit under Section 12, Rule 3 of the Revised Rules of Court are present both in
the said civil case and in the instant petition, the latter being but an incident to the former.
Petitioners minors assert that they represent their generation as well as generations yet unborn. Their
personality to sue in behalf of the succeeding generations can only be based on the concept of
intergenerational responsibility insofar as the right to a balanced and healthful ecology is concerned.

Page
Section 1-C, SY 06-07

14

San Beda College of Law Alabang


Constitutional Law 2 Case Digests
Nature means the created world in its entirety. Every generation has a responsibility to the next to
preserve that rhythm and harmony for the full enjoyment of a balanced and healthful ecology. The minors'
assertion of their right to a sound environment constitutes, at the same time, the performance of their
obligation to ensure the protection of that right for the generations to come.
The complaint focuses on one specific fundamental legal right the right to a balanced and healthful
ecology which, for the first time in our nation's constitutional history, is solemnly incorporated in the
fundamental law. Section 16, Article II of the 1987 Constitution.
While the right to a balanced and healthful ecology is to be found under the Declaration of Principles and
State Policies and not under the Bill of Rights, it does not follow that it is less important than any of the
civil and political rights enumerated in the latter. Such a right belongs to a different category of rights
altogether for it concerns nothing less than self-preservation and self-perpetuation aptly and fittingly
stressed by the petitioners the advancement of which may even be said to predate all governments and
constitutions. As a matter of fact, these basic rights need not even be written in the Constitution for they
are assumed to exist from the inception of humankind. If they are now explicitly mentioned in the
fundamental charter, it is because of the well-founded fear of its framers that unless the rights to a
balanced and healthful ecology and to health are mandated as state policies by the Constitution itself,
thereby highlighting their continuing importance and imposing upon the state a solemn obligation to
preserve the first and protect and advance the second, the day would not be too far when all else would
be lost not only for the present generation, but also for those to come generations which stand to inherit
nothing but parched earth incapable of sustaining life.
Conformably with the enunciated right to a balanced and healthful ecology and the right to health, as well
as the other related provisions of the Constitution concerning the conservation, development and
utilization of the country's natural resources, then President Corazon C. Aquino promulgated on 10 June
1987 E.O. No. 192, Section 4 of which expressly mandates that the Department of Environment and
Natural Resources "shall be the primary government agency responsible for the conservation,
management, development and proper use of the country's environment and natural resources,
specifically forest and grazing lands, mineral, resources, including those in reservation and watershed
areas, and lands of the public domain, as well as the licensing and regulation of all natural resources as
may be provided for by law in order to ensure equitable sharing of the benefits derived therefrom for the
welfare of the present and future generations of Filipinos." Section 3 thereof makes the following
statement of policy:
The above provision stresses "the necessity of maintaining a sound ecological balance and protecting
and enhancing the quality of the environment." Section 2 of the same Title, on the other hand, specifically
speaks of the mandate of the DENR; however, it makes particular reference to the fact of the agency's
being subject to law and higher authority.
It may, however, be recalled that even before the ratification of the 1987 Constitution, specific statutes
already paid special attention to the "environmental right" of the present and future generations. On 6
June 1977, P.D. No. 1151 and P.D. No. 1152 were issued. Thus, the right of the petitioners to a balanced
and healthful ecology is as clear as the DENR's duty under its mandate and by virtue of its powers and
functions under E.O. No. 192 and the Administrative Code of 1987 to protect and advance the said right.
A denial or violation of that right by the other who has the correlative duty or obligation to respect or
protect the same gives rise to a cause of action. Petitioners maintain that the granting of the TLAs, which
they claim was done with grave abuse of discretion, violated their right to a balanced and healthful
ecology; hence, the full protection thereof requires that no further TLAs should be renewed or granted.
It is settled in this jurisdiction that in a motion to dismiss based on the ground that the complaint fails to
state a cause of action; the question submitted to the court for resolution involves the sufficiency of the
facts alleged in the complaint itself. No other matter should be considered; furthermore, the truth of falsity
of the said allegations is beside the point for the truth thereof is deemed hypothetically admitted. Policy
formulation or determination by the executive or legislative branches of Government is not squarely put in
issue. What is principally involved is the enforcement of a right vis-a-vis policies already formulated and
expressed in legislation. It must, nonetheless, be emphasized that the political question doctrine is no
longer, the insurmountable obstacle to the exercise of judicial power or the impenetrable shield that
protects executive and legislative actions from judicial inquiry or review.
In the second place, even if it is to be assumed that the same are contracts, the instant case does not
involve a law or even an executive issuance declaring the cancellation or modification of existing timber
licenses. Hence, the non-impairment clause cannot as yet be invoked. Nevertheless, granting further that
a law has actually been passed mandating cancellations or modifications, the same cannot still be
stigmatized as a violation of the non-impairment clause. This is because by its very nature and purpose,
such as law could have only been passed in the exercise of the police power of the state for the purpose

Page
Section 1-C, SY 06-07

15

of advancing the right of the people to a balanced and healthful ecology, promoting their health and
enhancing the general welfare.
Finally, it is difficult to imagine, as the trial court did, how the non-impairment clause could apply with
respect to the prayer to enjoin the respondent Secretary from receiving, accepting, processing, renewing
or approving new timber licenses for, save in cases of renewal, no contract would have as of yet existed
in the other instances. Moreover, with respect to renewal, the holder is not entitled to it as a matter of
right.
Petition is hereby GRANTED, and the challenged Order of respondent Judge of 18 July 1991 dismissing
Civil Case No. 90-777 is hereby set aside. The petitioners may therefore amend their complaint to
implead as defendants the holders or grantees of the questioned timber license agreements.
AGAN JR. VS. PIATCO
[402 SCRA 612; G.R. No. 155001; 5 May 2003]
Facts:
Some time in 1993, six business leaders, explored the possibility of investing in the new NAIA airport
terminal, so they formed Asians Emerging Dragon Corp. They submitted proposals to the government for
the development of NAIA Intl. Passenger Terminal III (NAIA IPT III). The NEDA approved the NAIA IPT III
project. Bidders were invited, and among the proposal Peoples Air Cargo (Paircargo) was chosen. AEDC
protested alleging that preference was given to Paircargo, but still the project was awarded to Paircargo.
Because of that, it incorporated into, Phil. Intl. Airport Terminals Co. (PIATCO). The DOTC and PIATCO
entered into a concession agreement in 1997 to franchise and operate the said terminal for 21years. In
Nov. 1998 it was amended in the matters of pertaining to the definition of the obligations given to the
concessionaire, development of facilities and proceeds, fees and charges, and the termination of contract.
Since MIAA is charged with the maintenance and operations of NAIA terminals I and II, it has a contract
with several service providers. The workers filed the petition for prohibition claiming that they would lose
their job, and the service providers joined them, filed a motion for intervention. Likewise several
employees of the MIAA filed a petition assailing the legality of arrangements. A group of congressmen
filed similar petitions. Pres. Arroyo declared in her speech that she will not honor PIATCO contracts which
the Exec. Branch's legal office concluded null and void.
Issue:
Whether or Not the 1997 concession agreement is void, together with its amendments for being contrary
to the constitution.
Held:
The 1997 concession agreement is void for being contrary to public policy. The amendments have the
effect of changing it into and entirely different agreement from the contract bidded upon. The amendments
present new terms and conditions which provide financial benefit to PIATCO which may have the altered
the technical and financial parameters of other bidders had they know that such terms were available.
The 1997 concession agreement, the amendments and supplements thereto are set aside for being null
and void.
The petitioners have local standi. They are prejudiced by the concession agreement as their livelihood is
to be taken away from them.
UMALI VS. GUINGONA
[305 SCRA 533; G.R. No. 131124; 21 Mar 1999]
Facts:
Osmundo Umali the petitioner was appointed Regional Director of the Bureau of Internal Revenue by
Pres Fidel V. Ramos. He assigned him in Manila, November 29, 1993 to March 15, 1994 and Makati,
March 16, 1994 to August 4, 1994. On August 1, 1994, President Ramos received a confidential
memorandum against the petitioner for alleged violations of internal revenue laws, rules and regulations
during his incumbency as Regional Director, more particularly the following malfeasance, misfeasance
and nonfeasance. upon receipt of the said confidential memorandum, former President authorized the
issuance of an Order for the preventive suspension of the petitioner and immediately referred the
Complaint against the latter to the Presidential Commission on Anti-Graft and Corruption (PCAGC), for
investigation. Petitioner was duly informed of the charges against him. And was directed him to send in

Page
Section 1-C, SY 06-07

16

San Beda College of Law Alabang


Constitutional Law 2 Case Digests
his answer, copies of his Statement of Assets, and Liabilities for the past three years (3), and Personal
Data Sheet. Initial hearing was set on August 25, 1994, at 2:00 p.m., at the PCAGC Office. On August 23,
the petitioner filed his required answer. After evaluating the evidence on record, the PCAGC issued its
Resolution of September 23, 1994, finding a prima facie evidence to support six (6) of the twelve (12)
charges against petitioner. On October 6, 1994, acting upon the recommendation of the PCAGC, then
President Ramos issued Administrative Order No. 152 dismissing petitioner from the service, with
forfeiture of retirement and all benefits under the law.
Issue:
Whether or Not AO No. 152 violated petitioner's Right to Security of Tenure.
Whether or Not Petitioner was denied due process of law
Whether or Not the PCAGC is a validly Constituted government agency and whether the petitioner can
raise the issue of constitutionality belatedly in its motion for reconsideration of the trial courts decision.
Whether or Not the ombudsman's resolution dismissing the charges against the petitioner is still basis for
the petitioner's dismissal with forfeiture of benefits as ruled in AO No. 152
Held:
Petitioner maintains that as a career executive service officer, he can only be removed for cause and
under the Administrative Code of 1987, 6 loss of confidence is not one of the legal causes or grounds for
removal. Consequently, his dismissal from office on the ground of loss confidence violated his right to
security of tenure, petitioner theorized. After a careful study, we are of the irresistible conclusion that the
Court of Appeals ruled correctly on the first three Issue. To be sure, petitioner was not denied the right to
due process before the PCAGC. Records show that the petitioner filed his answer and other pleadings
with respect to his alleged violation of internal revenue laws and regulations, and he attended the
hearings before the investigatory body. It is thus decisively clear that his protestation of non-observance
of due process is devoid of any factual or legal basis. Neither can it be said that there was a violation of
what petitioner asserts as his security of tenure. According to petitioner, as a Regional Director of Bureau
of Internal Revenue, he is CESO eligible entitled to security of tenure. However, petitioner's claim of
CESO eligibility is anemic of evidentiary support. It was incumbent upon him to prove that he is a CESO
eligible but unfortunately, he failed to adduce sufficient evidence on the matter. His failure to do so is fatal.
As regards the issue of constitutionality of the PCAGC, it was only posed by petitioner in his motion for
reconsideration before the Regional Trial Court of Makati. It was certainly too late to raise for the first time
at such late stage of the proceedings. As to last issue, It is worthy to note that in the case under
consideration, the administrative action against the petitioner was taken prior to the institution of the
criminal case. The charges included in Administrative Order No. 152 were based on the results of
investigation conducted by the PCAGC and not on the criminal charges before the Ombudsman. In sum,
the petition is dismissable on the ground that the Issue posited by the petitioner do not constitute a valid
legal basis for overturning the finding and conclusion arrived at by the Court of Appeals. However, taking
into account the antecedent facts and circumstances aforementioned, the Court, in the exercise of its
equity powers, has decided to consider the dismissal of the charges against petitioner before the
Ombudsman, the succinct and unmistakable manifestation by the Commissioner of the Bureau of Internal
Revenue that his office is no longer interested in pursuing the case, and the position taken by the Solicitor
General, that there is no more basis for Administrative Order No. 152, as effective and substantive
supervening events that cannot be overlooked.
IN RE CUNANAN
[94 Phil 534; Resolution; 18 Mar 1954]
Facts:
Congress passed Republic Act Number 972, commonly known as the Bar Flunkers Act of 1953. In
accordance with the said law, the Supreme Court then passed and admitted to the bar those candidates
who had obtained an average of 72 per cent by raising it to 75 percent.
After its approval, many of the unsuccessful postwar candidates filed petitions for admission to the bar
invoking its provisions, while other motions for the revision of their examination papers were still pending
also invoked the aforesaid law as an additional ground for admission. There are also others who have
sought simply the reconsideration of their grades without, however, invoking the law in question. To avoid
injustice to individual petitioners, the court first reviewed the motions for reconsideration, irrespective of
whether or not they had invoked Republic Act No. 972.

Page
Section 1-C, SY 06-07

17

Issue:
Whether or Not RA No. 972 is constitutional and valid.
Held:
RA No. 972 has for its object, according to its author, to admit to the Bar, those candidates who suffered
from insufficiency of reading materials and inadequate preparation.
In the judicial system from which ours has been evolved, the admission, suspension, disbarment and
reinstatement of attorneys at law in the practice of the profession and their supervision have been
indisputably a judicial function and responsibility. We have said that in the judicial system from which ours
has been derived, the admission, suspension, disbarment or reinstatement of attorneys at law in the
practice of the profession is concededly judicial.
On this matter, there is certainly a clear distinction between the functions of the judicial and legislative
departments of the government.
It is obvious, therefore, that the ultimate power to grant license for the practice of law belongs exclusively
to this Court, and the law passed by Congress on the matter is of permissive character, or as other
authorities may say, merely to fix the minimum conditions for the license.
Republic Act Number 972 is held to be unconstitutional.
REPUBLIC ACT 6735, INITIATIVE AND REFERENDUM ACT
R.A. No. 6735 was, as its history reveals, intended to cover initiative to propose amendments to the
Constitution. The Act is a consolidation of House Bill No. 21505 and Senate Bill No. 17. The former was
prepared by the committee on Suffrage and Electoral Reforms of Representatives on the basis of two
House Bills referred to it, viz., (a) House Bill No. 497, which dealt with the initiative and referendum
mentioned in Sections 1 and 32 of Article VI of the Constitution; and (b) House Bill No. 988, which dealt
with the subject matter of House Bill No. 497, as well as with initiative and referendum under Section 3 of
Article XVII of the Constitution. Senate Bill No. 17 solely, dealt with initiative and referendum concerning
ordinances or resolutions of local government units. The Bicameral Conference Committee consolidated
Senate Bill No. 17 and House Bill No. 21505 into a draft bill, which was subsequently approved on 8 June
1989 by the Senate and by the House of Representatives. This approved bill is now R.A. No. 6735.

Page
Section 1-C, SY 06-07

18

San Beda College of Law Alabang


Constitutional Law 2 Case Digests

THE FUNDAMENTAL POWERS OF


THE STATE
THE POLICE POWER
AGUSTIN VS. EDU
[88 SCRA 195; G.R. No. L-49112; 2 Feb 1979]
Facts:
President Marcos issued the Letter of Instruction No. 229 which states that all owners, users or drivers
shall have at all times one pair of early warning devise (EWD) in their cars acquire from any source
depending on the owners choice. The Letter of Instruction was assailed by petitioner Leovillo Agustin to
have violated the constitution guarantee of due process against Hon Edu, Land Transportation
Commissioner, Hon. Juan Ponce Enrile, Minister of national Defense, Hon. Juinio, Minister of Public
Works, Transportation and Communication and Hon. Aquino, Minister of Public Highways. Because of
such contentions, the Implementing Rules and Regulation was ordered to be suspended for a period of 6
months. Petitioner alleges that EWD are not necessary because vehicles already have hazard lights
(blinking lights) that can be use as a warning device. Also petitioner contest that the letter of instruction
violates the delegation of police power because it is deemed harsh, oppressive and unreasonable for the
motorists and those dealers of EWD will become instant millionaires because of such law.
Issue:
Whether or not Petitioners contentions possess merit.
Held:
Petitioners contentions are without merit because the exercise of police power may interfere with
personal liberty or property to ensure and promote the safety, health and prosperity of the State. Also,
such letter of instruction is intended to promote public safety and it is indeed a rare occurrence that such
contention was alleged in a instruction with such noble purpose. Petitioner also failed to present the
factual foundation that is necessary to invalidate the said letter of instruction. In cases where there is
absence in the factual foundation, it should be presumed that constitutionality shall prevail. Pres. Marcos
on the other hand possesses vital statistics that will justify the need for the implementation of this
instruction. As signatory to the 1968 Vienna Conventions on Road Signs and Signals, our country must
abide with the standards given as stated in our Constitution that the Philippines adopts the generally
accepted principles of International Law as part of the law of the land. In the case at bar, the Vienna
Convention also requires the use of EWD. Vehicle owners are not obliged to buy an EDW. They can
personally create a EWD provided that it is in accordance to the specifications provided by law.
Petitioners allegation against the manufacturers of EDW being millionaires is deemed to be an
unfounded speculation. Wherefore, the petition is dismissed. The restraining order regarding the
implementation of the Reflector Law is lifted making the said law immediately executory.

ICHONG VS. HERNANDEZ


[101 Phil 1117; G.R. No. L-7995; 31 May 1957]
Facts:
Republic Act 1180 or commonly known as An Act to Regulate the Retail Business was passed. The said
law provides for a prohibition against foreigners as well as corporations owned by foreigners from
engaging from retail trade in our country. This was protested by the petitioner in this case. According to
him, the said law violates the international and treaty of the Philippines therefore it is unconstitutional.
Specifically, the Treaty of Amity between the Philippines and China was violated according to him.
Issue:
Whether or Not Republic Act 1180 is a valid exercise of police power.

Page
Section 1-C, SY 06-07

19

Held:
According to the Court, RA 1180 is a valid exercise of police power. It was also then provided that police
power can not be bargained away through the medium of a treaty or a contract. The Court also provided
that RA 1180 was enacted to remedy a real and actual danger to national economy posed by alien
dominance and control. If ever the law infringes upon the said treaty, the latter is always subject to
qualification or amendment by a subsequent law and the same may never curtain or restrict the scope of
the police power of the state.
LUTZ VS. ARANETA
[98 Phil 148; G.R. No. L-7859; 22 Dec 1955]
Facts:
Walter Lutz, as the Judicial Administrator of the Intestate Estate of Antonio Jayme Ledesma, seeks to
recover from J. Antonio Araneta, the Collector of Internal Revenue, the sum of money paid by the estate
as taxes, pursuant to the Sugar Adjustment Act. Under Section 3 of said Act, taxes are levied on the
owners or persons in control of the lands devoted to the cultivation of sugar cane. Furthermore, Section 6
states all the collections made under said Act shall be for aid and support of the sugar industry
exclusively. Lutz contends that such purpose is not a matter of public concern hence making the tax
levied for that cause unconstitutional and void. The Court of First Instance dismissed his petition, thus
this appeal before the Supreme Court.
Issue:
Whether or Not the tax levied under the Sugar Adjustment Act ( Commonwealth Act 567) is
unconstitutional.

Held:
The tax levied under the Sugar Adjustment Act is constitutional. The tax under said Act is levied with a
regulatory purpose, to provide means for the rehabilitation and stabilization of the threatened sugar
industry. Since sugar production is one of the great industries of our nation, its promotion, protection, and
advancement, therefore redounds greatly to the general welfare. Hence, said objectives of the Act is a
public concern and is therefore constitutional. It follows that the Legislature may determine within
reasonable bounds what is necessary for its protection and expedient for its promotion. If objectives and
methods are alike constitutionally valid, no reason is seen why the state may not levy taxes to raise funds
for their prosecution and attainment. Taxation may be made with the implement of the states police
power. In addition, it is only rational that the taxes be obtained from those that will directly benefit from it.
Therefore, the tax levied under the Sugar Adjustment Act is held to be constitutional.
TIO VS. VIDEOGRAM REGULATORY BOARD
[151 SCRA 208; G.R. No. L-75697; 18 Jun 1987]
Facts:
The case is a petition filed by petitioner on behalf of videogram operators adversely affected by
Presidential Decree No. 1987, An Act Creating the Videogram Regulatory Board" with broad powers to
regulate and supervise the videogram industry.
A month after the promulgation of the said
Revenue Code provided that:

Presidential Decree, the amended the National Internal

"SEC. 134.
Video Tapes. There shall be collected on each processed video-tape
cassette, ready for playback, regardless of length, an annual tax of five pesos; Provided,
That locally manufactured or imported blank video tapes shall be subject to sales tax."
"Section 10.
Tax on Sale, Lease or Disposition of Videograms. Notwithstanding
any provision of law to the contrary, the province shall collect a tax of thirty percent (30%)
of the purchase price or rental rate, as the case may be, for every sale, lease or
disposition of a videogram containing a reproduction of any motion picture or audiovisual
program.

Page
Section 1-C, SY 06-07

20

San Beda College of Law Alabang


Constitutional Law 2 Case Digests
Fifty percent (50%) of the proceeds of the tax collected shall accrue to the province, and
the other fifty percent (50%) shall accrue to the municipality where the tax is collected;
PROVIDED, That in Metropolitan Manila, the tax shall be shared equally by the
City/Municipality and the Metropolitan Manila Commission.
The rationale behind the tax provision is to curb the proliferation and unregulated circulation of
videograms including, among others, videotapes, discs, cassettes or any technical improvement or
variation thereof, have greatly prejudiced the operations of movie houses and theaters. Such unregulated
circulation have caused a sharp decline in theatrical attendance by at least forty percent (40%) and a
tremendous drop in the collection of sales, contractor's specific, amusement and other taxes, thereby
resulting in substantial losses estimated at P450 Million annually in government revenues.
Videogram(s) establishments collectively earn around P600 Million per annum from rentals, sales and
disposition of videograms, and these earnings have not been subjected to tax, thereby depriving the
Government of approximately P180 Million in taxes each year.
The unregulated activities of videogram establishments have also affected the viability of the movie
industry.
Issue:
Whether or not tax imposed by the DECREE is a valid exercise of police power.
Whether or nor the DECREE is constitutional .
Held:
Taxation has been made the implement of the state's police power. The levy of the 30% tax is for a public
purpose. It was imposed primarily to answer the need for regulating the video industry, particularly
because of the rampant film piracy, the flagrant violation of intellectual property rights, and the
proliferation of pornographic video tapes. And while it was also an objective of the DECREE to protect the
movie industry, the tax remains a valid imposition.
We find no clear violation of the Constitution which would justify us in pronouncing Presidential Decree
No. 1987 as unconstitutional and void. While the underlying objective of the DECREE is to protect the
moribund movie industry, there is no question that public welfare is at bottom of its enactment,
considering "the unfair competition posed by rampant film piracy; the erosion of the moral fiber of the
viewing public brought about by the availability of unclassified and unreviewed video tapes containing
pornographic films and films with brutally violent sequences; and losses in government revenues due to
the drop in theatrical attendance, not to mention the fact that the activities of video establishments are
virtually untaxed since mere payment of Mayor's permit and municipal license fees are required to
engage in business."
WHEREFORE, the instant Petition is hereby dismissed. No costs.
ASSO. OF SMALL LANDOWNERS VS. SEC. OF DAR
[175 SCRA 343; G.R. NO. L-78742; 14 JUL 1989]
Facts:
Several petitions are the root of the case:
a. A petition alleging the constitutionality of PD No. 27, EO 228 and 229 and RA 6657. Subjects
of the petition are a 9-hectare and 5 hectare Riceland worked by four tenants. Tenants were
declared full owners by EO 228 as qualified farmers under PD 27. The petitioners now
contend that President Aquino usurped the legislatures power.
b. A petition by landowners and sugarplanters in Victorias Mill Negros Occidental against
Proclamation 131 and EO 229. Proclamation 131 is the creation of Agrarian Reform Fund
with initial fund of P50Billion.
c. A petition by owners of land which was placed by the DAR under the coverage of Operation
Land Transfer.
d. A petition invoking the right of retention under PD 27 to owners of rice and corn lands not
exceeding seven hectares.
Issue:

Page
Section 1-C, SY 06-07

21

Whether or Not the aforementioned EOs, PD, and RA were constitutional.


Held:
The promulgation of PD 27 by President Marcos was valid in exercise of Police power and eminent
domain.
The power of President Aquino to promulgate Proc. 131 and EO 228 and 229 was authorized under Sec.
6 of the Transitory Provisions of the 1987 Constitution. Therefore it is a valid exercise of Police Power and
Eminent Domain.
RA 6657 is likewise valid. The carrying out of the regulation under CARP becomes necessary to deprive
owners of whatever lands they may own in excess of the maximum area allowed, there is definitely a
taking under the power of eminent domain for which payment of just compensation is imperative. The
taking contemplated is not a mere limitation of the use of the land. What is required is the surrender of the
title and the physical possession of said excess and all beneficial rights accruing to the owner in favour of
the farmer.
A statute may be sustained under the police power only if there is concurrence of the lawful subject and
the method.
Subject and purpose of the Agrarian Reform Law is valid, however what is to be determined is the method
employed to achieve it.
LOZANO VS. MARTINEZ
[146 SCRA 323; G.R. No. L-63419; 18 Dec 1986]
Facts:
A motion to quash the charge against the petitioners for violation of the BP 22 was made, contending that
no offense was committed, as the statute is unconstitutional. Such motion was denied by the RTC. The
petitioners thus elevate the case to the Supreme Court for relief. The Solicitor General, commented that it
was premature for the accused to elevate to the Supreme Court the orders denying their motions to
quash. However, the Supreme Court finds it justifiable to intervene for the review of lower court's denial of
a motion to quash.
Issue:
Whether or not BP 22 is constitutional as it is a proper exercise of police power of the State.
Held:
The enactment of BP 22 a valid exercise of the police power and is not repugnant to the constitutional
inhibition against imprisonment for debt.
The offense punished by BP 22 is the act of making and issuing a worthless check or a check that is
dishonored upon its presentation for payment. It is not the non-payment of an obligation which the law
punishes. The law is not intended or designed to coerce a debtor to pay his debt.
The law punishes the act not as an offense against property, but an offense against public order. The
thrust of the law is to prohibit, under pain of penal sanctions, the making of worthless checks and putting
them in circulation. An act may not be considered by society as inherently wrong, hence, not malum in se
but because of the harm that it inflicts on the community, it can be outlawed and criminally punished as
malum prohibitum. The state can do this in the exercise of its police power.
KWONG SING VS. CITY OF MANILA
[41 Phil 103; G.R. No. 15972; 11 Oct 1920]
Facts:
Kwong Sing, in his own behalf and of other Chinese laundrymen who has general and the same interest,
filed a complaint for a preliminary injunction. The Plaintiffs also questioned the validity of enforcing
Ordinance No. 532 by the city of Manila. Ordinance No. 532 requires that the receipt be in duplicate in
English and Spanish duly signed showing the kind and number of articles delivered by laundries and
dyeing and cleaning establishments. The permanent injunction was denied by the trial court. The

Page
Section 1-C, SY 06-07

22

San Beda College of Law Alabang


Constitutional Law 2 Case Digests
appellants claim is that Ordinance No. 532 savors of class legislation; putting in mind that they are
Chinese nationals. It unjustly discriminates between persons in similar circumstances; and that it
constitutes an arbitrary infringement of property rights. They also contest that the enforcement of the
legislation is an act beyond the scope of their police power. In view of the foregoing, this is an appeal with
the Supreme Court.
Issue:
Whether or Not the enforcement of Ordinance no, 532 is an act beyond the scope of police power
Whether or Not the enforcement of the same is a class legislation that infringes property rights.
Held:
Reasonable restraints of a lawful business for such purposes are permissible under the police power. The
police power of the City of Manila to enact Ordinance No. 532 is based on Section 2444, paragraphs (l)
and (ee) of the Administrative Code, as amended by Act No. 2744, authorizes the municipal board of the
city of Manila, with the approval of the mayor of the city:
(l) To regulate and fix the amount of the license fees for the following: xxxx xxxxxlaundries
xxxx.
(ee) To enact all ordinances it may deem necessary and proper for the sanitation and
safety, the furtherance of the prosperity, and the promotion of the morality, peace, good
order, comfort, convenience, and general welfare of the city and its inhabitants.
The court held that the obvious purpose of Ordinance No. 532 was to avoid disputes between laundrymen
and their patrons and to protect customers of laundries who are not able to decipher Chinese characters
from being defrauded. (Considering that in the year 1920s, people of Manila are more familiar with
Spanish and maybe English.)
In whether the ordinance is class legislation, the court held that the ordinance invades no fundamental
right, and impairs no personal privilege. Under the guise of police regulation, an attempt is not made to
violate personal property rights. The ordinance is neither discriminatory nor unreasonable in its operation.
It applies to all public laundries without distinction, whether they belong to Americans, Filipinos, Chinese,
or any other nationality. All, without exception, and each every one of them without distinction, must
comply with the ordinance. The obvious objection for the implementation of the ordinance is based in
sec2444 (ee) of the Administrative Code. Although, an additional burden will be imposed on the business
and occupation affected by the ordinance such as that of the appellant by learning even a few words in
Spanish or English, but mostly Arabic numbers in order to properly issue a receipt, it seems that the same
burdens are cast upon the them. Yet, even if private rights of person or property are subjected to restraint,
and even if loss will result to individuals from the enforcement of the ordinance, this is not sufficient
ground for failing to uphold the power of the legislative body. The very foundation of the police power is
the control of private interests for the public welfare.
Finding that the ordinance is valid, judgment is affirmed, and the petition for a preliminary injunction is
denied, with costs against the appellants.

TABLARIN VS. GUTIERREZ


[152 SCRA 730; G.R. No. 78164; 31 July 1987]
Facts:
The petitioners sought to enjoin the Secretary of Education, Culture and Sports, the Board of Medical
Education and the Center for Educational Measurement from enforcing Section 5 (a) and (f) of Republic
Act No. 2382, as amended, and MECS Order No. 52, series of 1985, dated 23 August 1985 and from
requiring the taking and passing of the NMAT as a condition for securing certificates of eligibility for
admission, from proceeding with accepting applications for taking the NMAT and from administering the
NMAT as scheduled on 26 April 1987 and in the future. The trial court denied said petition on 20 April
1987. The NMAT was conducted and administered as previously scheduled.

Page
Section 1-C, SY 06-07

23

Republic Act 2382, as amended by Republic Acts Nos. 4224 and 5946, known as the "Medical Act of
1959" defines its basic objectives in the following manner:
"SECTION 1. Objectives. This Act provides for and shall govern (a) the
standardization and regulation of medical education; (b) the examination for registration
of physicians; and (c) the supervision, control and regulation of the practice of medicine
in the Philippines."
The statute, among other things, created a Board of Medical Education. Its functions as specified in
Section 5 of the statute include the following:
"(a)
To determine and prescribe requirements for admission into a recognized college
of medicine;
x x x
(f)
To accept applications for certification for admission to a medical school and
keep a register of those issued said certificate; and to collect from said applicants the
amount of twenty-five pesos each which shall accrue to the operating fund of the Board
of Medical Education;
Section 7 prescribes certain minimum requirements for applicants to medical schools:
"Admission requirements. The medical college may admit any student who has not
been convicted by any court of competent jurisdiction of any offense involving moral
turpitude and who presents (a) a record of completion of a bachelor's degree in science
or arts; (b) a certificate of eligibility for entrance to a medical school from the Board of
Medical Education; (c) a certificate of good moral character issued by two former
professors in the college of liberal arts; and (d) birth certificate. Nothing in this act shall be
construed to inhibit any college of medicine from establishing, in addition to the
preceding, other entrance requirements that may be deemed admissible.
MECS Order No. 52, s. 1985, issued by the then Minister of Education, Culture and Sports and dated 23
August 1985, established a uniform admission test called the National Medical Admission Test (NMAT) as
an additional requirement for issuance of a certificate of eligibility for admission into medical schools of
the Philippines, beginning with the school year 1986-1987. This Order goes on to state that: "2. The
NMAT, an aptitude test, is considered as an instrument toward upgrading the selection of applicants for
admission into the medical schools and its calculated to improve the quality of medical education in the
country. The cutoff score for the successful applicants, based on the scores on the NMAT, shall be
determined every year by the Board of Medical Education after consultation with the Association of
Philippine Medical Colleges. The NMAT rating of each applicant, together with the other admission
requirements as presently called for under existing rules, shall serve as a basis for the issuance of the
prescribed certificate of eligibility for admission into the medical colleges.
Issue:
Whether or not Section 5 (a) and (f) of Republic Act No. 2382, as amended, and MECS Order No. 52, s.
1985 are constitutional.

Held:
Yes. We conclude that prescribing the NMAT and requiring certain minimum scores therein as a condition
for admission to medical schools in the Philippines, do not constitute an unconstitutional imposition.
The police power, it is commonplace learning, is the pervasive and non-waivable power and authority of
the sovereign to secure and promote all the important interests and needs in a word, the public order
of the general community. An important component of that public order is the health and physical
safety and well being of the population, the securing of which no one can deny is a legitimate objective of
governmental effort and regulation. Perhaps the only issue that needs some consideration is whether
there is some reasonable relation between the prescribing of passing the NMAT as a condition for
admission to medical school on the one hand, and the securing of the health and safety of the general
community, on the other hand. This question is perhaps most usefully approached by recalling that the
regulation of the practice of medicine in all its branches has long been recognized as a reasonable
method of protecting the health and safety of the public.

Page
Section 1-C, SY 06-07

24

San Beda College of Law Alabang


Constitutional Law 2 Case Digests
MECS Order No. 52, s. 1985 articulates the rationale of regulation of this type: the improvement of the
professional and technical quality of the graduates of medical schools, by upgrading the quality of those
admitted to the student body of the medical schools. That upgrading is sought by selectivity in the process
of admission, selectivity consisting, among other things, of limiting admission to those who exhibit in the
required degree the aptitude for medical studies and eventually for medical practice. The need to
maintain, and the difficulties of maintaining, high standards in our professional schools in general, and
medical schools in particular, in the current stage of our social and economic development, are widely
known. We believe that the government is entitled to prescribe an admission test like the NMAT as a
means for achieving its stated objective of "upgrading the selection of applicants into [our] medical
schools" and of "improv[ing] the quality of medical education in the country. We are entitled to hold that
the NMAT is reasonably related to the securing of the ultimate end of legislation and regulation in this
area. That end, it is useful to recall, is the protection of the public from the potentially deadly effects of
incompetence and ignorance in those who would undertake to treat our bodies and minds for disease or
trauma.
WHEREFORE, the Petition for Certiorari is DISMISSED and the Order of the respondent trial court
denying the petition for a writ of preliminary injunction is AFFIRMED. Costs against petitioners.

CITY GOVERNMENT OF QUEZON CITY VS. ERICTA


[122 SCRA 759; G.R. No. L-34915; 24 Jun 1983]
Facts:
Section 9 of Ordinance No. 6118, S-64, entitled "Ordinance Regulating The Establishment, Maintenance
And Operation Of Private Memorial Type Cemetery Or Burial Ground Within The Jurisdiction Of Quezon
City And Providing Penalties For The Violation Thereof" provides:
Sec. 9. At least six (6) percent of the total area of the memorial park cemetery shall be
set aside for charity burial of deceased persons who are paupers and have been
residents of Quezon City for at least 5 years prior to their death, to be determined by
competent City Authorities. The area so designated shall immediately be developed and
should be open for operation not later than six months from the date of approval of the
application.
For several years, the aforequoted section of the Ordinance was not enforced but seven years after the
enactment of the ordinance, the Quezon City Council passed a resolution to request the City Engineer,
Quezon City, to stop any further selling and/or transaction of memorial park lots in Quezon City where the
owners thereof have failed to donate the required 6% space intended for paupers burial.
The Quezon City Engineer then notified respondent Himlayang Pilipino, Inc. in writing that Section 9 of
the ordinance would be enforced.
Respondent Himlayang Pilipino reacted by filing a petition for declaratory relief, prohibition and
mandamus with preliminary injunction seeking to annul Section 9 of the Ordinance in question.
Respondent alleged that the same is contrary to the Constitution, the Quezon City Charter, the Local
Autonomy Act, and the Revised Administrative Code.
Issue:
Whether or Not Section 9 of the ordinance in question is a valid exercise of police power.
Held:
Section 9 of the City ordinance in question is not a valid exercise of police power. Section 9 cannot be
justified under the power granted to Quezon City to tax, fix the license fee, and regulate such other
business, trades, and occupation as may be established or practiced in the City.
Bill of rights states that 'no person shall be deprived of life, liberty or property without due process of law'
(Art. Ill, Section 1 subparagraph 1, Constitution). On the other hand, there are three inherent powers of
government by which the state interferes with the property rights, namely-. (1) police power, (2) eminent
domain, (3) taxation.
The police power of Quezon City is defined in sub-section 00, Sec. 12, Rep. Act 537 that reads as follows:

Page
Section 1-C, SY 06-07

25

To make such further ordinance and regulations not repugnant to law as may be
necessary to carry into effect and discharge the powers and duties conferred by this act
and such as it shall deem necessary and proper to provide for the health and safety, ,
and for the protection of property therein; and enforce obedience thereto with such lawful
fines or penalties as the City Council may prescribe under the provisions of subsection (jj)
of this section.
The power to regulate does not include the power to prohibit. The power to regulate does not include the
power to confiscate. The ordinance in question not only confiscates but also prohibits the operation of a
memorial park cemetery, because under Section 13 of said ordinance, 'Violation of the provision thereof is
punishable with a fine and/or imprisonment and that upon conviction thereof the permit to operate and
maintain a private cemetery shall be revoked or cancelled. The confiscatory clause and the penal
provision in effect deter one from operating a memorial park cemetery.
Moreover, police power is defined by Freund as 'the power of promoting the public welfare by restraining
and regulating the use of liberty and property'. It is usually exerted in order to merely regulate the use and
enjoyment of property of the owner. If he is deprived of his property outright, it is not taken for public use
but rather to destroy in order to promote the general welfare.
It seems to the court that Section 9 of Ordinance No. 6118, Series of 1964 of Quezon City is not a mere
police regulation but an outright confiscation. It deprives a person of his private property without due
process of law, nay, even without compensation.
MMDA Vs. Bel-Air Village
[328 SCRA 836; G.R. No. 135962; 27 Mar 2000]
Facts:
Metropolitan Manila Development Authority (MMDA), petitioner herein, is a Government Agency tasked
with the delivery of basic services in Metro Manila. Bel-Air Village Association (BAVA), respondent herein,
received a letter of request from the petitioner to open Neptune Street of Bel-Air Village for the use of the
public. The said opening of Neptune Street will be for the safe and convenient movement of persons and
to regulate the flow of traffic in Makati City. This was pursuant to MMDA law or Republic Act No. 7924. On
the same day, the respondent was appraised that the perimeter wall separating the subdivision and
Kalayaan Avenue would be demolished.
The respondent, to stop the opening of the said street and demolition of the wall, filed a preliminary
injunction and a temporary restraining order. Respondent claimed that the MMDA had no authority to do
so and the lower court decided in favor of the Respondent. Petitioner appealed the decision of the lower
courts and claimed that it has the authority to open Neptune Street to public traffic because it is an agent
of the State that can practice police power in the delivery of basic services in Metro Manila.
Issue:
Whether or not the MMDA has the mandate to open Neptune Street to public traffic pursuant to its
regulatory and police powers.
Held:
The Court held that the MMDA does not have the capacity to exercise police power. Police power is
primarily lodged in the National Legislature. However, police power may be delegated to government
units. Petitioner herein is a development authority and not a political government unit. Therefore, the
MMDA cannot exercise police power because it cannot be delegated to them. It is not a legislative unit of
the government. Republic Act No. 7924 does not empower the MMDA to enact ordinances, approve
resolutions and appropriate funds for the general welfare of the inhabitants of Manila. There is no syllable
in the said act that grants MMDA police power.
It is an agency created for the purpose of laying down policies and coordinating with various national
government agencies, peoples organizations, non-governmental organizations and the private sector for
the efficient and expeditious delivery of basic services in the vast metropolitan area.
TATEL VS. MUNICIPALITY OF VIRAC
[207 SCRA 157; G.R. No. 40243; 11 Mar 1992]

Page
Section 1-C, SY 06-07

26

San Beda College of Law Alabang


Constitutional Law 2 Case Digests
Facts:
Petitioner Celestino Tatel owns a warehouse in barrio Sta. Elena, Municipality of Virac. Complaints were
received by the municipality concerning the disturbance caused by the operation of the abaca bailing
machine inside petitioners warehouse. A committee was then appointed by the municipal council, and it
noted from its investigation on the matter that an accidental fire within the warehouse of the petitioner
created a danger to the lives and properties of the people in the neighborhood. Resolution No. 29 was
then passed by the Municipal council declaring said warehouse as a public nuisance within a purview of
Article 694 of the New Civil Code. According to respondent municipal officials, petitioners warehouse was
constructed in violation of Ordinance No. 13, series of 1952, prohibiting the construction of warehouses
near a block of houses either in the poblacion or barrios without maintaining the necessary distance of
200 meters from said block of houses to avoid loss of lives and properties by accidental fire. On the other
hand, petitioner contends that Ordinance No. 13 is unconstitutional.
Issue:
Whether or not petitioners warehouse is a nuisance within the meaning Article 694 of the Civil Code
Whether or not Ordinance No. 13, series of 1952 of the Municipality of Virac is unconstitutional and void.
Held:
The storage of abaca and copra in petitioners warehouse is a nuisance under the provisions of Article
694 of the Civil Code. At the same time, Ordinance No. 13 was passed by the Municipal Council of Virac
in the exercise of its police power. It is valid because it meets the criteria for a valid municipal ordinance:
1) must not contravene the Constitution or any statute, 2) must not be unfair or oppressive, 3) must not be
partial or discriminatory, 4) must not prohibit but may regulate trade, 5) must be general and consistent
with public policy, and 6) must not be unreasonable. The purpose of the said ordinance is to avoid the
loss of property and life in case of fire which is one of the primordial obligation of government. The lower
court did not err in its decision.

Page
Section 1-C, SY 06-07

27

THE POWER OF EMINENT DOMAIN


REPUBLIC VS. TAGLE
[299 SCRA 549; G.R. No. 129079; 2 Dec 1998]
Facts:
Private respondent Helena Z. Benitez is the registered owner of two (2) parcels of land located in
Barangay Salawag, Dasmarias, Cavite containing an area of 483,331 square meters more or less.
The Philippine Government, through the Philippine Human Resources Development Center (PHRDC),
negotiated with the Japanese International Cooperation Agency (JICA) Survey Team on the technicalities
of the establishment of the ASEAN Human Resources Development Project in the Philippines. Among
the five (5) main programs of the proposed project was Program III (Construction Manpower
Development) which involved the establishment of a Construction Manpower Development Center
(CMDC). PHRDC and private respondent Helena Z. Benitez, signed a Memorandum of Agreement which
provides, among others, that Benitez undertakes to lease within the period of twenty (20) years and/or sell
a portion of that property (which is no less than ten-hectares) in favor of PHRDC which likewise agrees to
lease within a period of twenty (20) years and/or buy said property site.
The Philippine Womens University (PWU) and Benitez granted a permit to PHRDC to occupy and use
the land in question and to undertake land development, electrical and road network installations and
other related works necessary to attain its objectives. Pursuant thereto, the CMDC took possession of the
property and erected buildings and other related facilities necessary for its operations. A deposit made by
the plaintiff with the Philippine National Bank (PNB) in the amount of P708,490.00 which is equivalent to
the assessed value of the property subject matter hereof based on defendants 1990 tax declaration, was
made.
In view of the agreement on the sale of the land in question, PHRDC prepared a Deed of Absolute Sale
with Benitez, as vendor, and PHRDC and CMDC, as vendees, duly represented by then Undersecretary
Gloria M. Arroyo, for the signature of Benitez. Benitez in her own capacity did not sign the deed of
absolute sale.
Failing to acquire the property involved through negotiated sale, petitioner, through the Department of
Trade and Industry, to which CMDC is attached, instituted a complaint for Eminent Domain, pursuant to
the provisions of Executive Order No. 1035, dated June 25, 1985.
A Motion for Issuance of Writ of Possession was granted by the court but quashed it subsequently.
Issue:
Whether or Not the respondent judge may quash a writ of possession on the ground that the expropriating
government agency is already occupying the property sought to be expropriated.
Held:
No. Under Section 7 of EO 1035, when the government or its authorized agent makes the required
deposit, the trial court has a ministerial duty to issue a writ of possession. The expropriation of real
property does not include mere physical entry or occupation of land. Although eminent domain usually
involves a taking of title, there may also be compensable taking of only some, not all, of the property
interests in the bundle of rights that constitute ownership.
In the instant case, it is manifest that the petitioner, in pursuit of an objective beneficial to public interest,
seeks to realize the same through its power of eminent domain. In exercising this power, petitioner
intended to acquire not only physical possession but also the legal right to possess and ultimately to own
the subject property. Hence, its mere physical entry and occupation of the property fall short of the taking
of title, which includes all the rights that may be exercised by an owner over the subject property.

CITY OF MANILA VS. CHINESE COMMUNITY


[40 Phil 349; No. 14355; 31 Oct 1919]

Page
Section 1-C, SY 06-07

28

San Beda College of Law Alabang


Constitutional Law 2 Case Digests
Facts: The City of Manila, plaintiff herein, prayed for the expropriation of a portion private cemetery for
the conversion into an extension of Rizal Avenue. Plaintiff claims that it is necessary that such public
improvement be made in the said portion of the private cemetery and that the said lands are within their
jurisdiction.
Defendants herein answered that the said expropriation was not necessary because other routes were
available. They further claimed that the expropriation of the cemetery would create irreparable loss and
injury to them and to all those persons owing and interested in the graves and monuments that would
have to be destroyed.
The lower court ruled that the said public improvement was not necessary on the particular-strip of land in
question. Plaintiff herein assailed that they have the right to exercise the power of eminent domain and
that the courts have no right to inquire and determine the necessity of the expropriation. Thus, the same
filed an appeal.
Issue:
Whether or not the courts may inquire into, and hear proof of the necessity of the expropriation.
Held:
The courts have the power of restricting the exercise of eminent domain to the actual reasonable
necessities of the case and for the purposes designated by the law. The moment the municipal
corporation or entity attempts to exercise the authority conferred, it must comply with the conditions
accompanying the authority. The necessity for conferring the authority upon a municipal corporation to
exercise the right of eminent domain is admittedly within the power of the legislature. But whether or not
the municipal corporation or entity is exercising the right in a particular case under the conditions imposed
by the general authority, is a question that the courts have the right to inquire to.
REPUBLIC VS. PLDT
[26 SCRA 320; G.R. No. L-18841; 27 Jan 1969]
Facts:
The plaintiff Republic of the Philippines is a political entity exercising government powers through one of
its branches, the Bureau of Telecommunication. Herein defendant, PLDT is a public service corporation
holding a franchise to install operates and maintains a telephone system. After its creation, the BOT set
up its own government telephone system by utilizing its own appropriations and other equipment and by
renting trunk lines of the PLDT to enable the govt offices to call privately. BOT entered into an
agreement with the RCA communications for joint overseas telephone service whereby BOT would
convey overseas calls received by RCA to local residents. PLDT complained to the BOT that it was a
violation of the condition of their agreement since the BOT had used trunk lines only for the use of
government offices but even to serve private persons or the general public in competition with the
business of PLDT. Subsequently, the plaintiff commenced suit against PLDT asking the court judgment be
rendered ordering the PLDT to execute a contract with the plaintiff, through the BOT for the use of the
facilities of PLDT's telephone system throughout the country under such conditions as the court may
consider reasonable. The CFI rendered judgment stating that it could not compel PLDT to enter into such
agreement. Hence this petition.
Issue:
Whether or Not PLDT may be compelled to enter into such agreement.
Held:
Yes, the state, may, in the interest of national welfare transfer utilities to public ownership upon payment
of just compensation, there is no reason why the state ma not require a public utility to render services in
the general interest provided just compensation is paid.

PEOPLE VS. FAJARDO


[104 Phil 443; G.R. No. L-12172; 29 Aug 1958]
Facts:

Page
Section 1-C, SY 06-07

29

The municipal council of baao, camarines sur stating among others that construction of a building, which
will destroy the view of the plaza, shall not be allowed and therefore be destroyed at the expense of the
owner, enacted an ordinance. Herein appellant filed a written request with the incumbent municipal mayor
for a permit to construct a building adjacent to their gasoline station on a parcel of land registered in
Fajardo's name, located along the national highway and separated from the public plaza by a creek. The
request was denied, for the reason among others that the proposed building would destroy the view or
beauty of the public plaza. Defendants reiterated their request for a building permit, but again the mayor
turned down the request. Whereupon, appellants proceeded with the construction of the building without a
permit, because they needed a place of residence very badly, their former house having been destroyed
by a typhoon and hitherto they had been living on leased property. Thereafter, defendants were charged
in violation of the ordinance and subsequently convicted. Hence this appeal.
Issue:
Whether or Not the ordinance is a valid exercise of police power.
Held:
No. It is not a valid exercise of police power. The ordinance is unreasonable and oppressive, in that it
operates to permanently deprive appellants of the right to use their own property; hence, it oversteps the
bounds of police power, and amounts to a taking of appellants property without just compensation. We do
not overlook that the modern tendency is to regard the beautification of neighborhoods as conducive to
the comfort and happiness of residents.
As the case now stands, every structure that may be erected on appellants' land, regardless of its own
beauty, stands condemned under the ordinance in question, because it would interfere with the view of
the public plaza from the highway. The appellants would, in effect, be constrained to let their land remain
idle and unused for the obvious purpose for which it is best suited, being urban in character. To legally
achieve that result, the municipality must give appellants just compensation and an opportunity to be
heard.
CITY OF BAGUIO V. NAWASA
[106 Phil; G.R. No. L-12032; 31 Aug 1959]
Facts:
Plaintiff a municipal corporation filed a complaint against defendant a public corporation, created under
Act.1383. It contends that the said act does not include within its purview the Baguio Water Works
system, assuming that it does, is unconstitutional because it deprives the plaintiff ownership, control and
operation of said water works without just compensation and due process of law. The defendant filed a
motion to dismiss ion the ground that it is not a proper exercise of police power and eminent domain. The
court denied the motion and ordered the defendants to file an answer. The court holds that the water
works system of Baguio belongs to private property and cannot be expropriated without just
compensation. Sec. 8 of R.A.1383 provides for the exchange of the NAWASA assets for the value of the
water works system of Baguio is unconstitutional for this is not just compensation. Defendants motion for
reconsideration was denied hence this appeal.
Issue:
Whether or Not there is a valid exercise of police power of eminent domain.
Held:
R.A. 1383 does not constitute a valid exercise of police power. The act does not confiscate, destroy or
appropriate property belonging to a municipal corporation. It merely directs that all water works belonging
to cities, municipalities and municipal districts in the Philippines to be transferred to the NAWASA. The
purpose is placing them under the control and supervision of an agency with a view to promoting their
efficient management, but in so doing does not confiscate them because it directs that they be paid with
equal value of the assets of NAWASA.
The Baguio water works system is not like a public road, the park, street other public property held in trust
by a municipal corporation for the benefit of the public. But it is a property of a municipal corporation,
water works cannot be taken away except for public use and upon payment of just compensation.
Judgment affirmed.

Page
Section 1-C, SY 06-07

30

San Beda College of Law Alabang


Constitutional Law 2 Case Digests
NATIONAL POWER CORP. VS. GUTIERREZ
[193 SCRA 1; G.R. No. 60077; 18 Jan 1991]
Facts:
Petitioner filed an action to acquire a right of way over the land of Respondents for the construction of
transmission lines. Petitioner was adjudged to pay the full market value of land traversed by the
transmission lines. Petitioner argued that it was only asking for a right of way.
Issue:
Whether or Not the acquisition of the right of way constitutes "taking" and such the case will be entitled
just compensation.
Held:
The acquisition of the right of way constitutes taking. It perpetually deprives Respondents of their
proprietary rights. No plant higher than three meters is allowed below the transmission lines. Because of
high tension current conveyed through the transmission lines, danger to life and limbs cannot be
discounted. The owner of the property is entitled to just compensation.
REPUBLIC VS. CASTELVI
[58 SCRA 336; G.R. No. L-20620; 15 Aug 1974]
Facts:
In 1947, the republic, through the Armed Forces of the Philippines (AFP), entered into a lease agreement
with Castelvi on a year-to-year basis. When Castelvi gave notice to terminate the lease in 1956, the AFP
refused. She then instituted an ejectment proceeding against the AFP. In 1959, however, the republic
commenced the expropriation proceedings for the land in question.
Issue:
Whether or Not the compensation should be determined as of 1947 or 1959.
Held:
The Supreme Court ruled that the taking should not be reckoned as of 1947, and that just compensation
should not be determined on the basis of the value of the property as of that year.
The requisites for taking are: 1) the expropriator must enter a private property, 2) the entry must be for
more than a momentary period, 3) it must be under warrant or color of authorities, 4) the property must be
devoted for public use or otherwise informally appropriated or injuriously affected, and 5) the utilization of
the property for public use must be such a way as to oust the owner and deprive him of beneficial
enjoyment of the property. Under Sec. 4 Rule 67 of the Rules of Court, just compensation is to be
determined as of the date of the filing of the complaint. The Supreme Court has ruled that when the taking
of the property sought to be expropriated coincides with the commencement of the expropriation
proceedings, or takes place subsequent to the filing of the complaint for eminent domain, the just
compensation should be determined as of the date of the filing of the complaint. In the instant case, it is
undisputed that the Republic was placed in possession of the Castelvi property, by authority of court, on
August 10, 1959. The taking of the Castelvi property for the purposes of determining the just
compensation to be paid must, therefore, be reckoned as of June 26, 1959 when the complaint for
eminent domain was filed. There is no basis to the contention of the Republic that a lease on a year-toyear basis can give rise to permanent right to occupy since by express provision a lease made for a
determinate time, as was the lease of Castelvi land in the instant case, ceases upon the day fixed, without
need of a demand (Art. 1669, New Civil Code). The Supreme Court, however, did not apply Art. 1250 of
the New Civil Code for the adjustment of the peso rate in times of extraordinary inflation or deflation
because in eminent domain cases the obligation to pay arises from law independent of contract.
EPZA VS. DULAY
[148 SCRA 305; G.R. No. L-59603; 29 Apr 1987]
Facts:

Page
Section 1-C, SY 06-07

31

The four parcels of land which are the subject of this case is where the Mactan Export Processing Zone
Authority in Cebu (EPZA) is to be constructed. Private respondent San Antonio Development Corporation
(San Antonio, for brevity), in which these lands are registered under, claimed that the lands were
expropriated to the government without them reaching the agreement as to the compensation.
Respondent Judge Dulay then issued an order for the appointment of the commissioners to determine the
just compensation. It was later found out that the payment of the government to San Antonio would be
P15 per square meter, which was objected to by the latter contending that under PD 1533, the basis of
just compensation shall be fair and according to the fair market value declared by the owner of the
property sought to be expropriated, or by the assessor, whichever is lower. Such objection and the
subsequent Motion for Reconsideration were denied and hearing was set for the reception of the
commissioners report. EPZA then filed this petition for certiorari and mandamus enjoining the respondent
from further hearing the case.
Issue:
Whether or Not the exclusive and mandatory mode of determining just compensation in PD 1533 is
unconstitutional.
Held:
The Supreme Court ruled that the mode of determination of just compensation in PD 1533 is
unconstitutional.
The method of ascertaining just compensation constitutes impermissible encroachment to judicial
prerogatives. It tends to render the courts inutile in a matter in which under the Constitution is reserved to
it for financial determination. The valuation in the decree may only serve as guiding principle or one of the
factors in determining just compensation, but it may not substitute the courts own judgment as to what
amount should be awarded and how to arrive at such amount. The determination of just compensation is
a judicial function. The executive department or the legislature may make the initial determination but
when a party claims a violation of the guarantee in the Bill of Rights that the private party may not be
taken for public use without just compensation, no statute, decree, or executive order can mandate that
its own determination shall prevail over the courts findings. Much less can the courts be precluded from
looking into the justness of the decreed compensation.
AMIGABLE VS. CUENCA
[43 SCRA 360; G.R. No. L-26400; 29 Feb. 1972]
Facts:
Victoria Amigable is the registered owner of a particular lot. At the back of her Transfer Certificate of Title
(1924), there was no annotation in favor of the government of any right or interest in the property. Without
prior expropriation or negotiated sale, the government used a portion of the lot for the construction of the
Mango and Gorordo Avenues. On 1958, Amigables counsel wrote the President of the Philippines,
requesting payment of the portion of the said lot. It was disallowed by the Auditor General in his 9 th
Endorsement. Petitioner then filed in the court a quo a complaint against the Republic of the Philippines
and Nicolas Cuenca, in his capacity as Commissioner of Public Highways for the recovery of ownership
and possession of the lot. According to the defendants, the action was premature because it was not filed
first at the Office of the Auditor General. According to them, the right of action for the recovery of any
amount had already prescribed, that the Government had not given its consent to be sued, and that
plaintiff had no cause of action against the defendants.
Issue:
Whether or Not, under the facts of the case, appellant may properly sue the government.
Held:
In the case of Ministerio v. Court of First Instance of Cebu, it was held that when the government takes
away property from a private landowner for public use without going through the legal process of
expropriation or negotiated sale, the aggrieved party may properly maintain a suit against the government
without violating the doctrine of governmental immunity from suit without its consent. In the case at bar,
since no annotation in favor of the government appears at the back of the certificate of title and plaintiff
has not executed any deed of conveyance of any portion of the lot to the government, then she remains
the owner of the lot. She could then bring an action to recover possession of the land anytime, because
possession is one of the attributes of ownership. However, since such action is not feasible at this time

Page
Section 1-C, SY 06-07

32

San Beda College of Law Alabang


Constitutional Law 2 Case Digests
since the lot has been used for other purposes, the only relief left is for the government to make due
compensationprice or value of the lot at the time of the taking.
PHILIPPINE PRESS INSTITUTE VS. COMELEC
[244 SCRA 272; G.R. No. 119694; 22 May 1995]
Facts:
Respondent Comelec promulgated Resolution No. 2772 directing newspapers to provide free Comelec
space of not less than one-half page for the common use of political parties and candidates. The Comelec
space shall be allocated by the Commission, free of charge, among all candidates to enable them to
make known their qualifications, their stand on public Issue and their platforms of government. The
Comelec space shall also be used by the Commission for dissemination of vital election information.
Petitioner Philippine Press Institute, Inc. (PPI), a non-profit organization of newspaper and magazine
publishers, asks the Supreme Court to declare Comelec Resolution No. 2772 unconstitutional and void on
the ground that it violates the prohibition imposed by the Constitution upon the government against the
taking of private property for public use without just compensation. On behalf of the respondent Comelec,
the Solicitor General claimed that the Resolution is a permissible exercise of the power of supervision
(police power) of the Comelec over the information operations of print media enterprises during the
election period to safeguard and ensure a fair, impartial and credible election.
Issue:
Whether or not Comelec Resolution No. 2772 is unconstitutional.

Held:
The Supreme Court declared the Resolution as unconstitutional. It held that to compel print media
companies to donate Comelec space amounts to taking of private personal property without payment
of the just compensation required in expropriation cases. Moreover, the element of necessity for the
taking has not been established by respondent Comelec, considering that the newspapers were not
unwilling to sell advertising space. The taking of private property for public use is authorized by the
constitution, but not without payment of just compensation. Also Resolution No. 2772 does not constitute
a valid exercise of the police power of the state. In the case at bench, there is no showing of existence of
a national emergency to take private property of newspaper or magazine publishers.
REYES VS. NATIONAL HOUSING AUTHORITY
[395 SCRA 494; GR NO. 147511; 20 JAN 2003]
Facts:
Respondent National Housing Authority (NHA) filed complaints for the expropriation of sugarcane lands
belonging to the petitioners. The stated public purpose of the expropriation was the expansion of the
Dasmarias Resettlement Project to accommodate the squatters who were relocated from the
Metropolitan Manila area. The trial court rendered judgment ordering the expropriation of these lots and
the payment of just compensation. The Supreme Court affirmed the judgment of the lower court.
A few years later, petitioners contended that respondent NHA violated the stated public purpose for the
expansion of the Dasmarias Resettlement Project when it failed to relocate the squatters from the Metro
Manila area, as borne out by the ocular inspection conducted by the trial court which showed that most of
the expropriated properties remain unoccupied. Petitioners likewise question the public nature of the use
by respondent NHA when it entered into a contract for the construction of low cost housing units, which is
allegedly different from the stated public purpose in the expropriation proceedings. Hence, it is claimed
that respondent NHA has forfeited its rights and interests by virtue of the expropriation judgment and the
expropriated properties should now be returned to herein petitioners.
Issue:
Whether or not the judgment of expropriation was forfeited in the light of the failure of respondent NHA to
use the expropriated property for the intended purpose but for a totally different purpose.

Page
Section 1-C, SY 06-07

33

Held:
The Supreme Court held in favor of the respondent NHA. Accordingly, petitioners cannot insist on a
restrictive view of the eminent domain provision of the Constitution by contending that the contract for low
cost housing is a deviation from the stated public use. It is now settled doctrine that the concept of public
use is no longer limited to traditional purposes. The term "public use" has now been held to be
synonymous with "public interest," "public benefit," "public welfare," and "public convenience." Thus,
whatever may be beneficially employed for the general welfare satisfies the requirement of public use."
In addition, the expropriation of private land for slum clearance and urban development is for a public
purpose even if the developed area is later sold to private homeowners, commercials firms, entertainment
and service companies, and other private concerns. Moreover, the Constitution itself allows the State to
undertake, for the common good and in cooperation with the private sector, a continuing program of urban
land reform and housing which will make at affordable cost decent housing and basic services to
underprivileged and homeless citizens in urban centers and resettlement areas. The expropriation of
private property for the purpose of socialized housing for the marginalized sector is in furtherance of
social justice.
MUNICIPALITY OF PARAAQUE VS. VM REALTY CORPORATION
[292 SCRA 676; G. R. NO. 127820; 20 JUL 1998]
Facts:
Petitioner sought to exercise its power of eminent domain based on a resolution by the municipal council.
Petitioner cites a previous case wherein a resolution gave authority to exercise eminent domain.
Petitioner also relies on the Implementing Rules, which provides that a resolution authorizes a Local
Government Unit to exercise eminent domain.
Issue:
Whether or Not an LGU can exercise its power of eminent domain pursuant to a resolution by its lawmaking body.
Held:
Under Section 19, of the present Local Government Code (RA 7160), it is stated as the first requisite that
LGUs can exercise its power of eminent domain if there is an ordinance enacted by its legislative body
enabling the municipal chief executive. A resolution is not an ordinance, the former is only an opinion of a
law-making body, the latter is a law. The case cited by Petitioner involves BP 337, which was the
previous Local Government Code, which is obviously no longer in effect. RA 7160 prevails over the
Implementing Rules, the former being the law itself and the latter only an administrative rule which cannot
amend the former.

ASLP VS. SEC. OF AGRARIAN REFORM


[175 SCRA 343; G.R. NO. 78742; 14 JUL 1989]
Facts:
Several petitions are the root of the case:
e. A petition alleging the constitutionality of PD No. 27, EO 228 and 229 and RA 6657. Subjects
of the petition are a 9-hectare and 5 hectare Riceland worked by four tenants. Tenants were
declared full owners by EO 228 as qualified farmers under PD 27. The petitioners now
contend that President Aquino usurped the legislatures power.
f. A petition by landowners and sugarplanters in Victorias Mill Negros Occidental against
Proclamation 131 and EO 229. Proclamation 131 is the creation of Agrarian Reform Fund
with initial fund of P50Billion.
g. A petition by owners of land which was placed by the DAR under the coverage of Operation
Land Transfer.
h. A petition invoking the right of retention under PD 27 to owners of rice and corn lands not
exceeding seven hectares.

Page
Section 1-C, SY 06-07

34

San Beda College of Law Alabang


Constitutional Law 2 Case Digests

Issue:
Whether or Not the aforementioned EOs, PD, and RA were constitutional.
Held:
The promulgation of PD 27 by President Marcos was valid in exercise of Police power and eminent
domain.
The power of President Aquino to promulgate Proc. 131 and EO 228 and 229 was authorized under Sec.
6 of the Transitory Provisions of the 1987 Constitution. Therefore it is a valid exercise of Police Power and
Eminent Domain.
RA 6657 is likewise valid. The carrying out of the regulation under CARP becomes necessary to deprive
owners of whatever lands they may own in excess of the maximum area allowed, there is definitely a
taking under the power of eminent domain for which payment of just compensation is imperative. The
taking contemplated is not a mere limitation of the use of the land. What is required is the surrender of the
title and the physical possession of said excess and all beneficial rights accruing to the owner in favour of
the farmer.
A statute may be sustained under the police power only if there is concurrence of the lawful subject and
the method.
Subject and purpose of the Agrarian Reform Law is valid, however what is to be determined is the method
employed to achieve it.
ESLABAN VS. ONORIO
[360 SCRA 230; G.R. NO. 146062; 28 JUN 2001]
Facts:
Clarita Vda. De Onorio is the owner of the land in Barangay M. Roxas, Sto. Nino, South Cotabato. Such
land is the subject for the construction of an irrigation canal of the National Irrigation Administration (NIA).
Mr. Santiago Eslaban Jr. is the project manager of NIA. The parties agreed to the construction of the
canal provided that the government will pay for the area that has been taken. A right-of-way agreement
was entered into by the parties in which respondent was paid the amount of P4, 180.00 as right of way
damages. Subsequently, respondent executed an Affidavit of Waiver of Rights and Fees which waives her
rights for the damage to the crops due to construction of the right of way. After which, respondent
demands that petitioner pay P111, 299.55 for taking her property but the petitioner refused. Petitioner
states that the government had not consented to be sued and that the respondent is not entitled for
compensation by virtue of the homestead patent under CA no. 141. The RTC held that the NIA should pay
respondent the amount of P107, 517.60 as just compensation for the 24,660 sq meters that have been
used for the construction of the canal. The Court of Appeals also affirmed the decision of the RTC.
Issue:
Whether or Not the CA erred in affirming the decision of the RTC.
Held:
The CA is correct in affirming the decision of the RTC but modifications shall be made regarding the value
of the just compensation. The following are the points to be considered in arriving in this decision.
First, Rule 7 par 5 of the Rule of Civil Procedure provides that the certification against forum shopping
should only be executed by the plaintiff or the principal. The petition for review was filed by Mr. Eslaban jr.
while the verification or certification were signed by Mr. Cesar Gonzales, an administrator of the agency.
Neither of the two has the authority to sign such certificate for they are not the plaintiff or principal. Such
case is a sufficient ground for dismissing this petition.
Second, PD NO. 1529 provides that the owner is required to recognize in favor of the government the
easement of a public highway, way, private way established by law, or any government canal where the
certificate of title does not state that the boundaries thereof have been pre-determined. In the case at bar,
the irrigation canal was constructed on Oct 1981 after the property had been registered in May of 1976. In
this case, prior expropriation proceedings must be filed and just compensation shall be paid to the owner
before the land could be taken for public use.

Page
Section 1-C, SY 06-07

35

Third, In this case, just compensation is defined as not only the correct amount to be paid but the
reasonable time for the Government to pay the owner. The CA erred in this point by stating that the
market value (just compensation) of the land is determined in the filing of the complaint in 1991.The
determination of such value should be from the time of its taking by the NIA in 1981.
Lastly, the petitioner cannot argue that the Affidavit of waiver of rights and fees executed by the
respondent pertains to the payment of the value of the land therefore exempting NIA to pay the value of
the land taken. Such waiver pertains only to the crops and improvements that were damage due to the
construction of the right-of-way not the value of the land.
Wherefore, decision of CA affirmed with modification regarding the just compensation in the amount of
P16, 047.61 per hectare.
KNECHT VS. COURT OF APPEALS
[290 SCRA 223; G.R. NO. 108015, 20 MAY 1998]
Facts:
The instant case is an unending sequel to several suits commenced almost twenty years ago involving a
parcel of land located at the corner of the south end of EDSA and F.B. Harrison in Pasay City. The land
was owned by petitioners Cristina de Knecht and her son, Rene Knecht. On the land, the Knechts
constructed eight houses, leased out the seven and occupied one of them as their residence. In 1979, the
government filed for the expropriation of Knechts property. The government wanted to use the land for
the completion of the Manila Flood Control and Drainage Project and the extension of the EDSA towards
Roxas Boulevard. In 1982, the City Treasurer of Pasay discovered that the Knechts failed to pay real
estate taxes on the property from 1980 to 1982. As a consequence of this deficiency, the City Treasurer
sold the property at public auction for the same amount of their deficiency taxes. The highest bidders
were respondent Spouses Anastacio and Felisa Babiera (the Babieras) and respondent Spouses
Alejandro and Flor Sangalang (the Sangalangs). Subsequently, Sangalang and Babiera sold the land to
respondent Salem Investment Corporation. On February 17, 1983, the Batasang Pambansa passed B.P.
Blg. 340 authorizing the national government to expropriate certain properties in Pasay City for the EDSA
Extension. The property of the Knechts was part of those expropriated under B.P. Blg. 340. The
government gave out just compensation for the lands expropriated under B.P. Blg. 340. Salem was
included and received partial payment. Seven of the eight houses of the Knechts were demolished and
the government took possession of the portion of land on which the houses stood. Since the Knechts
refused to vacate their one remaining house, Salem filed a case against them for unlawful detainer. As
defense, the Knechts claimed ownership of the land and building. The Municipal Trial Court however
ordered the Knechts' ejectment thus their residence was demolished.
The Knechts continuously claimed ownership of the property and allege that they must be given just
compensation.
Issue:
Whether or not Knechts are the lawful owners of the land at subject.
Held:
The Supreme Court held that the Knechts were not the owners anymore of the said land. The Knechts'
right to the land had been foreclosed after they failed to redeem it one year after the sale at public
auction. Since the petitions questioning the order of dismissal were likewise dismissed by the Court of
Appeals and this Court, the order of dismissal became final and res judicata on the issue of ownership of
the land. Petitioners contended that they did not receive notice of their tax delinquency. Neither did they
receive notice of the auction sale. However, this question has been previously raised in the cases which
have been already set aside. The court is not a trier of facts. Res judicata has already set it. The Knechts
therefore are not the lawful owners of the land and are not any longer accountable for just compensation
given by the government.
Note: Res judicata is a ground for dismissal of an action. It is a rule that precludes parties from relitigating
Issue actually litigated and determined by a prior and final judgment. It pervades every well-regulated
system of jurisprudence, and is based upon two grounds embodied in various maxims of the common law
one, public policy and necessity, that there should be a limit to litigation; and another, the individual
should not be vexed twice for the same cause. When a right of fact has been judicially tried and
determined by a court of competent jurisdiction, or an opportunity for such trial has been given, the
judgment of the court, so long as it remains unreversed, should be conclusive upon the parties and those

Page
Section 1-C, SY 06-07

36

San Beda College of Law Alabang


Constitutional Law 2 Case Digests
in privity with them in law or estate. To follow a contrary doctrine would subject the public peace and quiet
to the will and neglect of individuals and prefer the gratification of the litigious disposition of the parties to
the preservation of the public tranquility.
Res judicata applies when: (1) the former judgment or order is final; (2) the judgment or order is one on
the merits; (3) it was rendered by a court having jurisdiction over the subject matter and the parties; (4)
there is between the first and second actions, identity of parties, of subject matter and of cause of action.
REPUBLIC VS. KER
[383 SCRA 584; G.R. NO. 136171, 2 JULY 2002]
Facts:
Petitioner filed before the Regional Trial Court of Davao City a petition for expropriation of portions of two
parcels of land owned by respondent. Petitioner needed the parcels of land for the widening of the road
component of J.P. Laurel-Buhangin Interchange in Davao City. The Regional trial court rendered decision
of a fair just compensation for defendant Ker Corporation. However, it was challenged by Petitioner
Republic of the Philippines, represented by the Department of Public Works and Highways alleging that
just compensation for site must be reduced. Petitioner alleged that when the petition for expropriation was
filed, the tax declaration of the property indicated its assessed value at a lower price.
Issue:
Whether or not respondent Ker Company was given a decision for fair just compensation.
Held:
The Supreme Court held that the valuation for the lot Sites are excessive and unreasonable. Just
compensation cannot be measured by the assessed value of the property as stated in the tax declaration
and schedule of market values. For the purpose of appraisal, the fair market value of the property is taken
into account and such value refers to the highest price in terms of money which a property will bring if
exposed for sale in the public market.
In computing just compensation for expropriation proceedings, it is the value of the land at the time of the
taking or at the time of the filing of the complaint not at the time of the rendition of judgment which should
be taken into consideration. 4 Section 4, Rule 67 of the 1997 Rules of Civil Procedure provides that just
compensation is to be determined as of the date of the taking or the filing of the complaint whichever
came first. On this matter, the appellate court is correct in disregarding petitioner's claim.
MANOSCA VS. COURT OF APPEALS
[252 SCRA 412; G.R. NO. 106440, 29 JAN. 1996]
Facts:
The National Historical Institute declared the parcel of land owned by Petitioners as a national historical
landmark, because it was the site of the birth of Felix Manalo, the founder of Iglesia ni Cristo. The
Republic of the Philippines filed an action to appropriate the land. Petitioners argued that the
expropriation was not for a public purpose.
Issue:
Whether or Not the taking or exercise of eminent domain may be granted.
Held:
Public use should not be restricted to the traditional uses. The taking is for a public use because of the
contribution of Felix Manalo to the culture and history of the Philippines.

Page
Section 1-C, SY 06-07

37

THE POWER OF TAXATION


PASCUAL VS. SEC. OF PUBLIC WORKS
[110 PHIL 331; G.R. NO.L-10405; 29 DEC 1960]
Facts:
Petitioner, the governor of the Province of Rizal, filed an action for declaratory relief with injunction on the
ground that RA 920, Act appropriating funds for public works, providing P85,000 for the construction,
reconstruction, repair, extension and improvement of Pasig feeder road terminals, were nothing but
projected and planned subdivision roads within Antonio Subdivision. Antonio Subdivision is owned by the
respondent, Jose Zulueta, a member of the Senate of the Philippines. Respondent offered to donate the
said feeder roads to the municipality of Pasig and the offer was accepted by the council, subject to a
condition that the donor would submit plan of the roads and an agreement to change the names of two of
the street. However, the donation was not executed, which prompted Zuleta to write a letter to the district
engineer calling attention the approval of RA 920. The district engineer, on the other hand, did not
endorse the letter that inasmuch the feeder roads in question were private property at the time of passage
and approval of RA 920, the appropriation for the construction was illegal and therefore, void ab initio.
Petitioner, prayed for RA 920 be declared null and void and the alleged deed of donation be declared
unconstitutional. Lower court dismissed the case and dissolved the writ of preliminary injunction.
Issue:
Whether or Not the deed of donation and the appropriation of funds stipulated in RA 920 are
constitutional.
Held:
The ruling case law rules that the legislature is without power to appropriate public revenue for anything
but public purpose. The taxing power must be exercised for public purposes only and the money raised
by taxation can be expended only for public purposes and not for the advantage of private individuals.
In the case at bar, the legality of the appropriation of the feeder roads depend upon whether the said
roads were public or private property when the bill was passed by congress or when it became effective.
The land which was owned by Zulueta, the appropriation sought a private purpose and hence, null and
void. The donation did not cure the nullity of the appropriation; therefore a judicial nullification of a said
donation need not precede the declaration of unconstitutionality of the said appropriation.
The decision appealed from is reversed.

PUNSALAN VS. MUNICIPAL BOARD OF MANILA


[95 PHIL 46; NO.L-4817; 26 MAY 1954]
Facts:
Petitioners, who are professionals in the city, assail Ordinance No. 3398 together with the law authorizing
it (Section 18 of the Revised Charter of the City of Manila). The ordinance imposes a municipal
occupation tax on persons exercising various professions in the city and penalizes non-payment of the
same. The law authorizing said ordinance empowers the Municipal Board of the city to impose a
municipal occupation tax on persons engaged in various professions. Petitioners, having already paid
their occupation tax under section 201 of the National Internal Revenue Code, paid the tax under protest
as imposed by Ordinance No. 3398. The lower court declared the ordinance invalid and affirmed the
validity of the law authorizing it.
Issue:
Whether or Not the ordinance and law authorizing it constitute class legislation, and authorize what
amounts to double taxation.
Held:

Page
Section 1-C, SY 06-07

38

San Beda College of Law Alabang


Constitutional Law 2 Case Digests
The Legislature may, in its discretion, select what occupations shall be taxed, and in its discretion may tax
all, or select classes of occupation for taxation, and leave others untaxed. It is not for the courts to judge
which cities or municipalities should be empowered to impose occupation taxes aside from that imposed
by the National Government. That matter is within the domain of political departments. The argument
against double taxation may not be invoked if one tax is imposed by the state and the other is imposed by
the city. It is widely recognized that there is nothing inherently terrible in the requirement that taxes be
exacted with respect to the same occupation by both the state and the political subdivisions thereof.
Judgment of the lower court is reversed with regards to the ordinance and affirmed as to the law
authorizing it.
OSMEA VS. ORBOS
[220 SCRA 703; G.R. NO. 99886; 31 MAR 1993]
Facts:
On October 10, 1984, Pres. Marcos issued P.D. 1956 creating a Special Account in the General Fund,
designated as the Oil Price Stabilization Fund (OPSF). The OPSF was designed to reimburse oil
companies for cost increases in crude oil and imported petroleum products resulting from exchange rate
adjustments and from increases in the world market prices of crude oil.
Subsequently, the OPSF was reclassified into a "trust liability account," in virtue of E.O. 1024, and ordered
released from the National Treasury to the Ministry of Energy.
Pres. Aquino, amended P.D. 1956. She promulgated Executive Order No. 137 on February 27, 1987,
expanding the grounds for reimbursement to oil companies for possible cost underrecovery incurred as a
result of the reduction of domestic prices of petroleum products, the amount of the underrecovery being
left for determination by the Ministry of Finance.
The
petition
avers
that
the
creation
29(3), Article VI of the Constitution, reading as follows:

of

the

trust

fund

violates

(3) All money collected on any tax levied for a special purpose shall be treated as a
special fund and paid out for such purposes only. If the purpose for which a special fund
was created has been fulfilled or abandoned, the balance, if any, shall be transferred to
the general funds of the Government.
The petitioner argues that "the monies collected pursuant to . . P.D. 1956, as amended, must be treated
as a 'SPECIAL FUND,' not as a 'trust account' or a 'trust fund,' and that "if a special tax is collected for a
specific purpose, the revenue generated therefrom shall 'be treated as a special fund' to be used only for
the purpose indicated, and not channeled to another government objective." Petitioner further points out
that since "a 'special fund' consists of monies collected through the taxing power of a State, such amounts
belong to the State, although the use thereof is limited to the special purpose/objective for which it was
created."
He also contends that the "delegation of legislative authority" to the ERB violates 28 (2). Article VI of the
Constitution, viz.:
(2) The Congress may, by law, authorize the President to fix, within specified limits, and
subject to such limitations and restrictions as it may impose, tariff rates, import and export
quotas, tonnage and wharfage dues, and other duties or imposts within the framework of
the national development program of the Government;
and, inasmuch as the delegation relates to the exercise of the power of taxation, " the limits, limitations
and restrictions must be quantitative, that is, the law must not only specify how to tax, who (shall) be
taxed (and) what the tax is for, but also impose a specific limit on how much to tax." 12
Issue:
Whether or Not the invalidity of the "TRUST ACCOUNT" in the books of account of the Ministry of Energy
(now, the Office of Energy Affairs), created pursuant to 8, paragraph 1, of P.D. No. 1956, as amended,
"said creation of a trust fund being contrary to Section 29 (3), Article VI of the Constitution.
Whether or Not the unconstitutionality of 8, paragraph 1 (c) of P.D. No. 1956, as amended by Executive
Order No. 137, for "being an undue and invalid delegation of legislative power to the Energy Regulatory
Board.

Page
Section 1-C, SY 06-07

39

Held:
The OPSF is a "Trust Account" which was established "for the purpose of minimizing the frequent price
changes brought about by exchange rate adjustment and/or changes in world market prices of crude oil
and imported petroleum products." Under P.D. No. 1956, as amended by Executive Order No. 137 dated
27 February 1987, this Trust Account may be funded from any of the following sources:
a) Any increase in the tax collection from ad valorem tax or customs duty imposed on petroleum
products subject to tax under this Decree arising from exchange rate adjustment, as may be
determined by the Minister of Finance in consultation with the Board of Energy;
b) Any increase in the tax collection as a result of the lifting of tax exemptions of government
corporations, as may be determined by the Minister of Finance in consultation with the Board of
Energy;
c) Any additional amount to be imposed on petroleum products to augment the resources of the
Fund through an appropriate Order that may be issued by the Board of Energy requiring payment
of persons or companies engaged in the business of importing, manufacturing and/or marketing
petroleum products;
d) Any resulting peso cost differentials in case the actual peso costs paid by oil companies in the
importation of crude oil and petroleum products is less than the peso costs computed using the
reference foreign exchange rate as fixed by the Board of Energy.
Hence, it seems clear that while the funds collected may be referred to as taxes, they are exacted in the
exercise of the police power of the State. Moreover, that the OPSF is a special fund is plain from the
special treatment given it by E.O. 137. It is segregated from the general fund; and while it is placed in
what the law refers to as a "trust liability account," the fund nonetheless remains subject to the scrutiny
and review of the COA. The Court is satisfied that these measures comply with the constitutional
description of a "special fund." Indeed, the practice is not without precedent.
With regard to the alleged undue delegation of legislative power, the Court finds that the provision
conferring the authority upon the ERB to impose additional amounts on petroleum products provides a
sufficient standard by which the authority must be exercised. In addition to the general policy of the law to
protect the local consumer by stabilizing and subsidizing domestic pump rates, 8(c) of P.D. 1956
expressly authorizes the ERB to impose additional amounts to augment the resources of the Fund.
What petitioner would wish is the fixing of some definite, quantitative restriction, or "a specific limit on how
much to tax." The Court is cited to this requirement by the petitioner on the premise that what is involved
here is the power of taxation; but as already discussed, this is not the case. What is here involved is not
so much the power of taxation as police power. Although the provision authorizing the ERB to impose
additional amounts could be construed to refer to the power of taxation, it cannot be overlooked that the
overriding consideration is to enable the delegate to act with expediency in carrying out the objectives of
the law which are embraced by the police power of the State.
The interplay and constant fluctuation of the various factors involved in the determination of the price of oil
and petroleum products, and the frequently shifting need to either augment or exhaust the Fund, do not
conveniently permit the setting of fixed or rigid parameters in the law as proposed by the petitioner. To do
so would render the ERB unable to respond effectively so as to mitigate or avoid the undesirable
consequences of such fluidity. As such, the standard as it is expressed suffices to guide the delegate in
the exercise of the delegated power, taking account of the circumstances under which it is to be
exercised.
LLADOC VS. COMMISSIONER OF INTERNAL REVENUE
[14 SCRA 292; NO.L-19201; 16 JUN 1965]
Facts:
Sometime in 1957, M.B. Estate Inc., of Bacolod City, donated 10,000.00 pesos in cash to Fr. Crispin Ruiz,
the parish priest of Victorias, Negros Occidental, and predecessor of Fr. Lladoc, for the construction of a
new Catholic church in the locality. The donated amount was spent for such purpose.

Page
Section 1-C, SY 06-07

40

San Beda College of Law Alabang


Constitutional Law 2 Case Digests
On March 3, 1958, the donor M.B. Estate filed the donor's gift tax return. Under date of April 29, 1960.
Commissioner of Internal Revenue issued an assessment for the donee's gift tax against the Catholic
Parish of Victorias of which petitioner was the parish priest.
Issue:
Whether or not the imposition of gift tax despite the fact the Fr. Lladoc was not the Parish priest at the
time of donation, Catholic Parish priest of Victorias did not have juridical personality as the constitutional
exemption for religious purpose is valid.
Held:
Yes, imposition of the gift tax was valid, under Section 22(3) Article VI of the Constitution contemplates
exemption only from payment of taxes assessed on such properties as Property taxes contra
distinguished from Excise taxes The imposition of the gift tax on the property used for religious purpose is
not a violation of the Constitution. A gift tax is not a property by way of gift inter vivos.
The head of the Diocese and not the parish priest is the real party in interest in the imposition of the
donee's tax on the property donated to the church for religious purpose.

CASSANOVAS VS. HORD


[8 Phil 125; No. 3473; 22 Mar 1907]
Facts:
The Spanish Govt. by virtue of a royal decree granted the plaintiff certain mines. The plaintiff is now the
owner of those mines. The Collector of Internal Revenue imposed tax on the properties, contending that
they were valid perfected mine concessions and it falls within the provisions of sec.134 of Act No. 1189
known as Internal Revenue Act. The plaintiff paid under protest. He brought an action against the
defendant Collector of Internal Revenue to recover the sum of Php. 9, 600 paid by him as taxes.
Judgment was rendered in favor of the defendant, so the plaintiff appealed.
Issue:
Whether or Not Sec. 164 is void or valid.
Held:
The deed constituted a contract between the Spanish Government and the plaintiff. The obligation of
which contract was impaired by the enactment of sec. 134 of the Internal Revenue Law infringing sec. 5
of the Act of Congress which provides that no law impairing the obligation of contracts shall be enacted.
Sec. 134 of the Internal Revenue Law of 1904 is void because it impairs the obligation of contracts
contained in the concessions of mine made by the Spanish Government. Judgment reversed.

Page
Section 1-C, SY 06-07

41

THE BILL
OF RIGHTS

Page
Section 1-C, SY 06-07

42

San Beda College of Law Alabang


Constitutional Law 2 Case Digests
DUE PROCESS OF LAW
Art 3, Sec. 1.

No person shall be deprived of life, liberty, or property without due process of law
ERMITA-MALATE HOTEL AND MOTEL OPERATORS ASSO. VS. MAYOR OF MANILA
[20 SCRA 849; G.R. NO.L-24693; 31 JULY 1967]

Facts:
Petitioners Ermita-Malate Hotel and Motel Operators Association with one of its members, Hotel del Mar
Inc., and Go Chiu, the president and general manager of the second petitioner, filed a petition for
prohibition against Ordinance No. 4760 against the respondent Mayor of the City of Manila who was
sued in his capacity as such charged with the general power and duty to enforce ordinances of the City of
Manila and to give the necessary orders for the execution and enforcement of such ordinances. It was
alleged that the petitioner non-stock corporation is dedicated to the promotion and protection of the
interest of its eighteen members operating hotels and motels, characterized as legitimate businesses duly
licensed by both national and city authorities and regularly paying taxes. It was alleged that on June 13,
1963, the Municipal Board of the City of Manila enacted Ordinance No. 4760, approved on June 14, 1963
by the then acting City Mayor, Vice-Mayor Herminio Astorga. After which the alleged grievances against
the ordinance were set forth in detail. There was the assertion of its being beyond the powers of the
Municipal Board of the City of Manila to enact insofar as it regulate motels, on the ground that in the
revised charter of the City of Manila or in any other law, no reference is made to motels. it also being
provided that the premises and facilities of such hotels, motels and lodging houses would be open for
inspection either by the City Mayor, or the Chief of Police, or their duly authorized representatives. The
lower court on July 6, 1963 issued a writ of preliminary injunction ordering respondent Mayor to refrain
from enforcing said Ordinance No. 4760 from and after July 8, 1963.
Issue:
Whether or Not Ordinance No. 4760 of the City of Manila is unconstitutional, therefore, null and void.
Held:
A decent regard for constitutional doctrines of a fundamental character ought to have admonished the
lower court against such a sweeping condemnation of the challenged ordinance. Its decision cannot be
allowed to stand, consistently with what has been the accepted standards of constitutional adjudication, in
both procedural and substantive aspects.
Primarily what calls for a reversal of such a decision is the absence of any evidence to offset the
presumption of validity that attaches to a challenged statute or ordinance. As was expressed categorically
by Justice Malcolm: "The presumption is all in favor of validity x x x . The action of the elected
representatives of the people cannot be lightly set aside. The councilors must, in the very nature of things,
be familiar with the necessities of their particular municipality and with all the facts and circumstances
which surround the subject and necessitate action. The local legislative body, by enacting the ordinance,
has in effect given notice that the regulations are essential to the well being of the people x x x . The
Judiciary should not lightly set aside legislative action when there is not a clear invasion of personal or
property rights under the guise of police regulation.
It admits of no doubt therefore that there being a presumption of validity, the necessity for evidence to
rebut it is unavoidable, unless the statute or ordinance is void on its face which is not the case here. The
principle has been nowhere better expressed than in the leading case of O'Gorman & Young v. Hartford
Fire Insurance Co. where the American Supreme Court through Justice Brandeis tersely and succinctly
summed up the matter thus: The statute here questioned deals with a subject clearly within the scope of
the police power. We are asked to declare it void on the ground that the specific method of regulation
prescribed is unreasonable and hence deprives the plaintiff of due process of law. As underlying
questions of fact may condition the constitutionality of legislation of this character, the resumption of
constitutionality must prevail in the absence of some factual foundation of record for overthrowing the
statute." No such factual foundation being laid in the present case, the lower court deciding the matter on
the pleadings and the stipulation of facts, the presumption of validity must prevail and the judgment
against the ordinance set aside.
VILLEGAS VS. HIU CHIONG
[86 SCRA 270; NO.L-29646; 10 NOV 1978]

Page
Section 1-C, SY 06-07

43

Facts:
The controverted Ordinance no. 6537 was passed by the Municipal Board of Manila on February 22, 1968
and signed by Mayor Villegas. It is an ordinance making it unlawful for any person not a citizen of the
Philippines to be employed in any place of employment or to be engaged in any kind of trade business
or occupation within the city of Manila without securing an employment permit from the Mayor of Manila
and for other purposes.
Hiu Chiong Tsai Pao Ho, who was employed in Manila filed a petition praying for the writ of preliminary
injunction and restraining order to stop the enforcement of said ordinance.
Issue:
Whether or Not Ordinance no.6537 violates the due process and equal protection clauses of the
Constitution.
Held:
It is a revenue measure. The city ordinance which imposes a fee of 50.00 pesos to enable aliens
generally to be employed in the city of Manila is not only for the purpose of regulation.
While it is true that the first part which requires the alien to secure an employment permit from
the
Mayor involves the exercise of discretion and judgment in processing and approval or disapproval of
application is regulatory in character, the second part which requires the payment
of a sum of
50.00 pesos is not a regulatory but a revenue measure.
Ordinance no. 6537 is void and unconstitutional. This is tantamount to denial of the basic human right of
the people in the Philippines to engaged in a means of livelihood. While it is true that the Philippines as a
state is not obliged to admit aliens within it's territory, once an alien is admitted he cannot be deprived of
life without due process of law. This guarantee includes the means of livelihood. Also it does not lay down
any standard to guide the City Mayor in the issuance or denial of an alien employment permit fee.
NAMIL VS. COMELEC
[414 SCRA 553; G.R. NO. 150540; 28 OCT 2003]
Facts:
On May 20, 2001, the Municipal Board of Canvassers of Palimbang, Sultan Kudarat proclaimed the
petitioners as winning candidates for their Sangguniang Bayan. The following day, herein private
respondents were proclaimed winners as well. Private respondents claimed that they should be
recognized as the winners, and not the petitioners. Upon receipt of such letter, the Commissioner-incharge for Region XII asked the Law Department, the Regional Election Registrar and the Provincial
Elections Supervisor to submit their reports on the matter. All of them found the second proclamation
valid. Hence, the COMELEC issued a Resolution ordering the immediate installation of the private
respondents as the newly elected members of the Sangguniang Bayan, even though petitioners herein
have already taken their oath and have assumed office. Petitioners contend that such Resolution is null
and void because they were not accorded due notice and hearing, hence constituting a violation of the
due process principle.
Issue:
Whether or Not due the COMELEC has the power to suspend a proclamation or the effects thereof
without notice and hearing.
Held:
No. The COMELEC is without power to partially or totally annul a proclamation or suspend the effects of a
proclamation without notice and hearing. The proclamation on May 20, 2001 enjoys the presumption of
regularity and validity since no contest or protest was even filed assailing the same. The petitioners
cannot be removed from office without due process of law. Due process in quasi-judicial proceedings
before the COMELEC requires due notice and hearing. Furthermore, the proclamation of a winning
candidate cannot be annulled if he has not been notified of any motion to set aside his proclamation.
Hence, as ruled in Farias vs. COMELEC, Reyes vs. COMELEC and Gallardo vs. COMELEC, the
COMELEC is without power to partially or totally annul a proclamation or suspend the effects of a
proclamation without notice and hearing.
ICHONG VS. HERNANDEZ

Page
Section 1-C, SY 06-07

44

San Beda College of Law Alabang


Constitutional Law 2 Case Digests
[101 PHIL 1155; L-7995; 31 MAY 1957]
Facts:
Republic Act 1180 or commonly known as An Act to Regulate the Retail Business was passed. The said
law provides for a prohibition against foreigners as well as corporations owned by foreigners from
engaging from retail trade in our country. This was protested by the petitioner in this case. According to
him, the said law violates the international and treaty of the Philippines therefore it is unconstitutional.
Specifically, the Treaty of Amity between the Philippines and China was violated according to him.
Issue:
Whether or Not Republic Act 1180 is a valid exercise of police power.
Held:
According to the Court, RA 1180 is a valid exercise of police power. It was also then provided that police
power can not be bargained away through the medium of a treaty or a contract. The Court also provided
that RA 1180 was enacted to remedy a real and actual danger to national economy posed by alien
dominance and control. If ever the law infringes upon the said treaty, the latter is always subject to
qualification or amendment by a subsequent law and the same may never curtain or restrict the scope of
the police power of the state.
PHIL. PHOSPHATE FERTILIZER CORP. VS. TORRES
[231 SCRA 335; G.R. NO.98050; 17 MAR 1994]
Facts:
Philphos Movement for Progress, Inc. (PMPI for brevity), filed with the Department of Labor and
Employment a petition for certification election among the supervisory employees of petitioner, alleging
that as a supervisory union duly registered with the Department of Labor and Employment it was seeking
to represent the supervisory employees of Philippine Phosphate Fertilizer Corporation. Mediator-Arbiter
Rodolfo S. Milado issued an order directing the holding of a certification election among the supervisory
employees of petitioner, excluding therefrom the superintendents and the professional and technical
employees. However, the PMPI filed an amended petition with the Mediator-Arbiter wherein it sought to
represent not only the supervisory employees of petitioner but also its professional/technical and
confidential employees. The parties therein agreed to submit their respective position papers and to
consider the amended petition submitted for decision on the basis thereof and related documents.
Mediator-Arbiter Milado issued an order granting the petition and directing the holding of a certification
election among the "supervisory, professional (engineers, analysts, mechanics, accountants, nurses,
midwives, etc.), technical, and confidential employees. PHILPHOS appealed the order to the Secretary of
Labor and Employment who rendered a decision through Undersecretary Bienvenido Laguesma
dismissing the appeal. PHILPHOS moved for reconsideration but the same was denied; hence, the
instant petition alleging denial of due process on the part of the DOLE to which the mediator-arbiter was
under.
Issue:
Whether or Not there was denial of due process.
Held:
There was no denial of due process. The essence of due process is simply an opportunity to be heard or,
as applied to administrative proceedings, an opportunity to explain one's side or an opportunity to seek a
reconsideration of the action or ruling complained of petitioner PHILPHOS agreed to file its position paper
with the Mediator-Arbiter and to consider the case submitted for decision on the basis of the position
papers filed by the parties, there was sufficient compliance with the requirement of due process, as
petitioner was afforded reasonable opportunity to present its side. Moreover, petitioner could have, if it so
desired, insisted on a hearing to confront and examine the witnesses of the other party. But it did not;
instead it opted to submit its position paper with the Mediator-Arbiter. Besides, petitioner had all the
opportunity to ventilate its arguments in its appeal to the Secretary of Labor.
RUBI VS. PROVINCIAL BOARD OF MINDORO
[39 PHIL 660; G.R. NO. 14078; 7 MAR 1919]

Page
Section 1-C, SY 06-07

45

Facts:
This is an application for habeas corpus in favor of Rubi and other Manguianes of the Province of
Mindoro.
The provincial board of Mindoro adopted resolution No. 25 which states that provincial governor of any
province in which non-Christian inhabitants (uncivilized tribes) are found is authorized, when such a
course is deemed necessary in the interest of law and order, to direct such inhabitants to take up their
habitation on sites on unoccupied public lands to be selected by him and approved by the provincial
board. It is resolved that under section 2077 of the Administrative Code, 800 hectares of public land in
the sitio of Tigbao on Naujan Lake be selected as a site for the permanent settlement of Mangyanes in
Mindoro. Further, Mangyans may only solicit homesteads on this reservation providing that said
homestead applications are previously recommended by the provincial governor.
Thereafter, the provincial governor of Mindoro issued executive order No. 2, which says that the provincial
governor has selected a site in the sitio of Tigbao on Naujan Lake for the permanent settlement of
Mangyanes in Mindoro. In that case, pursuant to Section 2145 of the Revised Administrative Code, all the
Mangyans in the townships of Naujan and Pola and the Mangyans east of the Baco River including those
in the districts of Dulangan and Rubi's place in Calapan, were ordered to take up their habitation on the
site of Tigbao, Naujan Lake. Also, that any Mangyan who shall refuse to comply with this order shall upon
conviction be imprisoned not exceed in sixty days, in accordance with section 2759 of the revised
Administrative Code.
Said resolution of the provincial board of Mindoro were claimed as necessary measures for the protection
of the Mangyanes of Mindoro as well as the protection of public forests in which they roam, and to
introduce civilized customs among them.
It appeared that Rubi and those living in his rancheria have not fixed their dwelling within the reservation
of Tigbao and are liable to be punished.
It is alleged that the Manguianes are being illegally deprived of their liberty by the provincial officials of
that province. Rubi and his companions are said to be held on the reservation established at Tigbao,
Mindoro, against their will, and one Dabalos is said to be held under the custody of the provincial sheriff in
the prison at Calapan for having run away form the reservation.
Issue:
Whether or Not Section 2145 of the Administrative Code deprive a person of his liberty without due
process of law.
Whether or Not Section 2145 of the Administrative Code of 1917 is constitutional.
Held:
The Court held that section 2145 of the Administrative Code does not deprive a person of his liberty
without due process of law and does not deny to him the equal protection of the laws, and that
confinement in reservations in accordance with said section does not constitute slavery and involuntary
servitude. The Court is further of the opinion that section 2145 of the Administrative Code is a legitimate
exertion of the police power, somewhat analogous to the Indian policy of the United States. Section 2145
of the Administrative Code of 1917 is constitutional.
The preamble of the resolution of the provincial board of Mindoro which set apart the Tigbao reservation,
it will be read, assigned as reasons fort the action, the following: (1) The failure of former attempts for the
advancement of the non-Christian people of the province; and (2) the only successfully method for
educating the Manguianes was to oblige them to live in a permanent settlement. The Solicitor-General
adds the following; (3) The protection of the Manguianes; (4) the protection of the public forests in which
they roam; (5) the necessity of introducing civilized customs among the Manguianes.
Considered purely as an exercise of the police power, the courts cannot fairly say that the Legislature has
exceeded its rightful authority. It is, indeed, an unusual exercise of that power. But a great malady
requires an equally drastic remedy. One cannot hold that the liberty of the citizen is unduly interfered
without when the degree of civilization of the Manguianes is considered. They are restrained for their own
good and the general good of the Philippines. Nor can one say that due process of law has not been
followed.
None of the rights of the citizen can be taken away except by due process of law. To constitute "due
process of law," as has been often held, a judicial proceeding is not always necessary. In some instances,

Page
Section 1-C, SY 06-07

46

San Beda College of Law Alabang


Constitutional Law 2 Case Digests
even a hearing and notice are not requisite a rule which is especially true where much must be left to the
discretion of the administrative officers in applying a law to particular cases.
The idea of the provision in question is to unify the people of the Philippines so that they may approach
the highest conception of nationality. The public policy of the Government of the Philippine Islands is
shaped with a view to benefit the Filipino people as a whole. The Manguianes, in order to fulfill this
governmental policy, must be confined for a time, as we have said, for their own good and the good of the
country.
Therefore, petitioners are not unlawfully imprisoned or restrained of their liberty. Habeas corpus can,
therefore, not issue.
KWONG SING VS. CITY OF MANILA
[41 PHIL 103; G.R. NO. 15972; 11 OCT 1920]
Facts:
Kwong Sing, in his own behalf and of other Chinese laundrymen who has general and the same interest,
filed a complaint for a preliminary injunction. The Plaintiffs also questioned the validity of enforcing
Ordinance No. 532 by the city of Manila. Ordinance No. 532 requires that the receipt be in duplicate in
English and Spanish duly signed showing the kind and number of articles delivered by laundries and
dyeing and cleaning establishments. The permanent injunction was denied by the trial court. The
appellants claim is that Ordinance No. 532 savors of class legislation; putting in mind that they are
Chinese nationals. It unjustly discriminates between persons in similar circumstances; and that it
constitutes an arbitrary infringement of property rights. They also contest that the enforcement of the
legislation is an act beyond the scope of their police power. In view of the foregoing, this is an appeal with
the Supreme Court.
Issue:
Whether or Not the enforcement of Ordinance no, 532 is an act beyond the scope of police power
Whether or not the enforcement of the same is a class legislation that infringes property rights.
Held:
Reasonable restraints of a lawful business for such purposes are permissible under the police power. The
police power of the City of Manila to enact Ordinance No. 532 is based on Section 2444, paragraphs (l)
and (ee) of the Administrative Code, as amended by Act No. 2744, authorizes the municipal board of the
city of Manila, with the approval of the mayor of the city:
(l) To regulate and fix the amount of the license fees for the following: xxxx xxxxxlaundries xxxx.
(ee) To enact all ordinances it may deem necessary and proper for the sanitation and safety, the
furtherance of the prosperity, and the promotion of the morality, peace, good order, comfort,
convenience, and general welfare of the city and its inhabitants.
The court held that the obvious purpose of Ordinance No. 532 was to avoid disputes between laundrymen
and their patrons and to protect customers of laundries who are not able to decipher Chinese characters
from being defrauded. (Considering that in the year 1920s, people of Manila are more familiar with
Spanish and maybe English.)
In whether the ordinance is class legislation, the court held that the ordinance invades no fundamental
right, and impairs no personal privilege. Under the guise of police regulation, an attempt is not made to
violate personal property rights. The ordinance is neither discriminatory nor unreasonable in its operation.
It applies to all public laundries without distinction, whether they belong to Americans, Filipinos, Chinese,
or any other nationality. All, without exception, and each every one of them without distinction, must
comply with the ordinance. The obvious objection for the implementation of the ordinance is based in
sec2444 (ee) of the Administrative Code. Although, an additional burden will be imposed on the business
and occupation affected by the ordinance such as that of the appellant by learning even a few words in
Spanish or English, but mostly Arabic numbers in order to properly issue a receipt, it seems that the same
burdens are cast upon the them. Yet, even if private rights of person or property are subjected to restraint,
and even if loss will result to individuals from the enforcement of the ordinance, this is not sufficient
ground for failing to uphold the power of the legislative body. The very foundation of the police power is
the control of private interests for the public welfare.

Page
Section 1-C, SY 06-07

47

Finding that the ordinance is valid, judgment is affirmed, and the petition for a preliminary injunction is
denied, with costs against the appellants.
YU CONG ENG VS. TRINIDAD
[47 PHIL 385; G.R. NO. 20479; 6 FEB 1925]
Facts:
The petitioner, Yu Cong Eng, was charged by information in the court of first instance of Manila, with a
violation of Act 2972, which provides that (Section 1) it shall be unlawful for any person, company, or
partnership or corporation engaged in commerce, industry or any other activity for the purpose of profit in
the Philippine Islands, in accordance with existing law, to keep its account books in any language other
than English, Spanish or any local dialect. He was arrested, his books were seized, and the trial was
about to proceed, when he and the other petitioner, Co Liam, on their own behalf, and on behalf of all the
other Chinese merchants in the Philippines, filed the petition against the fiscal, or prosecuting attorney of
Manila, and the collector of internal revenue engaged in the prosecution, and against the judge presiding.
Issue:
Whether or Not Act 2972 is unconstitutional.
Held:
Yes. The Philippine government may make every reasonable requirement of its taxpayers to keep proper
records of their business transactions in English or Spanish or Filipino dialect by which an adequate
measure of what is due from them in meeting the cost of government can be had. But we are clearly of
opinion that it is not within the police power of the Philippine Legislature, because it would be oppressive
and arbitrary, to prohibit all Chinese merchants from maintaining a set of books in the Chinese language,
and in the Chinese characters, and thus prevent them from keeping advised of the status of their
business and directing its conduct.
ANIAG VS. COMELEC
[237 SCRA 194; G.R. NO. 104961; 7 OCT 1994]
Facts:
In preparation for the synchronized national and local elections, the COMELEC issued Resolution No.
2323, Gun Ban, promulgating rules and regulations on bearing, carrying and transporting of firearm or
other deadly weapons on security personnel or bodyguards, on bearing arms by members of security
agencies or police organizations, and organization or maintenance of reaction forces during the election
period. COMELEC also issued Resolution No. 2327 providing for the summary disqualification of
candidates engaged in gunrunning, using and transporting of firearms, organizing special strike forces,
and establishing spot checkpoints. Pursuant to the Gun Ban, Mr. Serrapio Taccad, Sergeant at Arms of
the House of Representatives, wrote petitioner for the return of the two firearms issued to him by the
House of Representatives. Petitioner then instructed his driver, Arellano, to pick up the firearms from
petitioners house and return them to Congress. The PNP set up a checkpoint. When the car driven by
Arellano approached the checkpoint, the PNP searched the car and found the firearms. Arellano was
apprehended and detained. He then explained the order of petitioner. Petitioner also explained that
Arellano was only complying with the firearms ban, and that he was not a security officer or a bodyguard.
Later, COMELEC issued Resolution No.92-0829 directing the filing of information against petitioner and
Arellano for violation of the Omnibus Election Code, and for petitioner to show cause why he should not
be disqualified from running for an elective position. Petitioner then questions the constitutionality of
Resolution No. 2327. He argues that gunrunning, using or transporting firearms or similar weapons and
other acts mentioned in the resolution are not within the provisions of the Omnibus Election Code. Thus,
according to petitioner, Resolution No. 2327 is unconstitutional. The issue on the disqualification of
petitioner from running in the elections was rendered moot when he lost his bid for a seat in Congress in
the elections.
Issue:
Whether or Not petitioner can be validly prosecuted for instructing his driver to return the firearms issued
to him on the basis of the evidence gathered from the warrant less search of his car

Page
Section 1-C, SY 06-07

48

San Beda College of Law Alabang


Constitutional Law 2 Case Digests

Held:
A valid search must be authorized by a search warrant issued by an appropriate authority. However, a
warrantless search is not violative of the Constitution for as long as the vehicle is neither searched nor its
occupants subjected to a body search, and the inspection of the vehicle is merely limited to a visual
search. In the case at bar, the guns were not tucked in Arellanos waist nor placed within his reach, as
they were neatly packed in gun cases and placed inside a bag at the back of the car. Given these
circumstances, the PNP could not have thoroughly searched the car lawfully as well as the package
without violating the constitutional injunction. Absent any justifying circumstance specifically pointing to
the culpability of petitioner and Arellano, the search could not have been valid. Consequently, the firearms
obtained from the warrantless search cannot be admitted for any purpose in any proceeding. It was also
shown in the facts that the PNP had not informed the public of the purpose of setting up the checkpoint.
Petitioner was also not among those charged by the PNP with violation of the Omnibus Election Code. He
was not informed by the City Prosecutor that he was a respondent in the preliminary investigation. Such
constituted a violation of his right to due process. Hence, it cannot be contended that petitioner was fully
given the opportunity to meet the accusation against him as he was not informed that he was himself a
respondent in the case. Thus, the warrantless search conducted by the PNP is declared illegal and the
firearms seized during the search cannot be used as evidence in any proceeding against the petitioner.
Resolution No. 92-0829 is unconstitutional, and therefore, set aside.
JAVIER VS. COMELEC
[144 SCRA 194; G.R. NOS. L-68379-81; 22 SEPT 1986]
Facts:
The petitioner and the private respondent were candidates in Antique for the Batasang Pambansa in the
May 1984 elections. The former appeared to enjoy more popular support but the latter had the advantage
of being the nominee of the KBL with all its perquisites of power. On May 13, 1984, the eve of the
elections, the bitter contest between the two came to a head when several followers of the petitioner were
ambushed and killed, allegedly by the latter's men. Seven suspects, including respondent Pacificador, are
now facing trial for these murders. Owing to what he claimed were attempts to railroad the private
respondent's proclamation, the petitioner went to the Commission on Elections to question the canvass of
the election returns. His complaints were dismissed and the private respondent was proclaimed winner by
the Second Division of the said body. The petitioner thereupon came to this Court, arguing that the
proclamation was void because made only by a division and not by the Commission on Elections en banc
as required by the Constitution. Meanwhile, on the strength of his proclamation, the private respondent
took his oath as a member of the Batasang Pambansa.

Issue:
Whether or Not the Second Division of the Commission on Elections authorized to promulgate its decision
of July 23, 1984, proclaiming the private respondent the winner in the election.
Held:
This Court has repeatedly and consistently demanded "the cold neutrality of an impartial judge" as the
indispensable imperative of due process. To bolster that requirement, we have held that the judge must
not only be impartial but must also appear to be impartial as an added assurance to the parties that his
decision will be just. The litigants are entitled to no less than that. They should be sure that when their
rights are violated they can go to a judge who shall give them justice. They must trust the judge,
otherwise they will not go to him at all. They must believe in his sense of fairness, otherwise they will not
seek his judgment. Without such confidence, there would be no point in invoking his action for the justice
they expect.
Due process is intended to insure that confidence by requiring compliance with what Justice Frankfurter
calls the rudiments of fair play. Fair play cans for equal justice. There cannot be equal justice where a
suitor approaches a court already committed to the other party and with a judgment already made and
waiting only to be formalized after the litigants shall have undergone the charade of a formal hearing.
Judicial (and also extra-judicial) proceedings are not orchestrated plays in which the parties are supposed
to make the motions and reach the denouement according to a prepared script. There is no writer to
foreordain the ending. The judge will reach his conclusions only after all the evidence is in and all the
arguments are filed, on the basis of the established facts and the pertinent law.

Page
Section 1-C, SY 06-07

49

YNOT VS. IAC


[148 SCRA 659; G.R. NO. 74457; 20 MAR 1987]
Facts:
Executive Order No. 626-A prohibited the transportation of carabaos and carabeef from one province to
another. The carabaos of petitioner were confiscated for violation of Executive Order No 626-A while he
was transporting them from Masbate to Iloilo. Petitioner challenged the constitutionality of Executive
Order No. 626-A. The government argued that Executive Order No. 626-A was issued in the exercise of
police power to conserve the carabaos that were still fit for farm work or breeding.
Issue:
Whether or Not EO No. 626-A is a violation of Substantive Due Process.
Held:
The challenged measure is an invalid exercise of police power, because it is not reasonably necessary for
the purpose of the law and is unduly oppressive. It is difficult to see how prohibiting the transfer of
carabaos from one province to another can prevent their indiscriminate killing. Retaining the carabaos in
one province will not prevent their slaughter there. Prohibiting the transfer of carabeef, after the slaughter
of the carabaos, will not prevent the slaughter either.
PHILCOMSAT VS. ALCUAZ
[180 SCRA 218; G.R. NO.84818; 18 DEC 1989]
Facts:
Herein petitioner is engaged in providing for services involving telecommunications. Charging rates for
certain specified lines that were reduced by order of herein respondent Jose Alcuaz Commissioner of the
National Telecommunications Commission. The rates were ordered to be reduced by fifteen percent
(15%) due to Executive Order No. 546 which granted the NTC the power to fix rates. Said order was
issued without prior notice and hearing.
Issue:
Whether or Not E.O. 546 is unconstitutional.
Held:
Yes. Respondents admitted that the application of a policy like the fixing of rates as exercised by
administrative bodies is quasi-judicial rather than quasi-legislative. But respondents contention that notice
and hearing are not required since the assailed order is merely incidental to the entire proceedings and
temporary in nature is erroneous. Section 16(c) of the Public Service Act, providing for the proceedings of
the Commission, upon notice and hearing, dictates that a Commission has power to fix rates, upon proper
notice and hearing, and, if not subject to the exceptions, limitations or saving provisions.
It is thus clear that with regard to rate-fixing, respondent has no authority to make such order without first
giving petitioner a hearing, whether the order be temporary or permanent, and it is immaterial whether the
same is made upon a complaint, a summary investigation, or upon the commission's own motion as in the
present case.
WHEREFORE, the writ prayed for is GRANTED and the order of respondents is hereby SET ASIDE.
EASTERN BROADCASTING CORP (DYRE) V. DANS JR.
[137 SCRA 628; L-59329; 19 JUL 1985]
Facts:
A petition was filed to reopen the Radio Station DYRE. DYRE was summarily closed on grounds of
national security. The radio station was allegedly used to incite people to sedition. Petitioner, DYRE
contends that they were denied due process. There was no hearing to establish factual evidence for the
closure. Furthermore, the closure of the radio station violates freedom of expression. Before the court

Page
Section 1-C, SY 06-07

50

San Beda College of Law Alabang


Constitutional Law 2 Case Digests
could even promulgate a decision upon the Issue raised, Petitioner, through its president Mr. Rene
Espina, filed a motion to withdraw the petition. The rights of the station were sold to a new owner, Manuel
Pastrana; who is no longer interested in pursuing the case. Despite the case becoming moot and
academic, (because there are no longer interested parties, thus the dismissal of the case) the Supreme
Court still finds that there is need to pass a RESOLUTION for the guidance of inferior courts and
administrative tribunals in matters as this case.
Issue:
Whether or not due process was exercised in the case of DYRE.
Whether or not the closure of DYRE is a violation of the Constitutional Right of Freedom of Expression.
Held:
The court finds that the closure of the Radio Station in 1980 as null and void. The absence of a hearing is
a violation of Constitutional Rights. The primary requirements in administrative proceedings are laid down
in the case of Ang Tibay v. Court of Industrial Relation (69 Phil.635). The Ang Tibay Doctrine should be
followed before any broadcast station may be closed. The Ang Tibay Doctrine provides the following
requirements:
(1)
(2)
(3)
(4)
(5)
(6)
(7)

The right to hearing, includes the right to present ones case and submit evidence
presented.
The tribunal must consider the evidence presented
The decision must have something to support itself.
Evidence must be substantial (reasonable evidence that is adequate to support
conclusion)
Decision must be based on the evidence presented at hearing
The tribunal body must act on its own independent consideration of law and facts
and not simply accept subordinates views
Court must render decision in such a manner that the proceeding can know the
various issued involved and reasons for decisions rendered.

The court stresses that while there is no controlling and precise definition of Due Process, it gives an
unavoidable standard that government actions must conform in order that deprivation of life, liberty and
property is valid.
The closure of the radio station is like wise a violation of the constitutional right of freedom of speech and
expression. The court stresses that all forms of media, whether print or broadcast are entitled to this
constitutional right. Although the government still has the right to be protected against broadcasts which
incite the listeners to violently overthrow it. The test for the limitation of freedom of expression is the
clear and present danger rule. If in the circumstances that the media is used in such nature as to create
this danger that will bring in such evils, then the law has the right to prevent it. However, Radio and
television may not be used to organize a rebellion or signal a start of widespread uprising. The freedom
to comment on public affairs is essential to the vitality of a representative democracy. The people
continues to have the right to be informed on public affairs and broadcast media continues to have the
pervasive influence to the people being the most accessible form of media. Therefore, broadcast stations
deserve the the special protection given to all forms of media by the due process and freedom of
expression clauses of the Constitution.
ANG TIBAY VS. COURT OF INDUSTRIAL RELATIONS (CIR)
[69 PHIL 635; G.R. NO. 46496; 27 FEB 1940]
Facts:
There was agreement between Ang Tibay and the National Labor Union, Inc (NLU). The NLU alleged that
the supposed lack of leather material claimed by Toribio Teodoro was but a scheme adopted to
systematically discharge all the members of the NLU, from work. And this averment is desired to be
proved by the petitioner with the records of the Bureau of Customs and Books of Accounts of native
dealers in leather. That National Worker's Brotherhood Union of Ang Tibay is a company or employer
union dominated by Toribio Teodoro, which was alleged by the NLU as an illegal one. The CIR, decided
the case and elevated it to the Supreme Court, but a motion for new trial was raised by the NLU. But the
Ang Tibay filed a motion for opposing the said motion.
Issue:

Page
Section 1-C, SY 06-07

51

Whether or Not, the motion for new trial is meritorious to be granted.


Held:
To begin with the issue before us is to realize the functions of the CIR. The CIR is a special court whose
functions are specifically stated in the law of its creation which is the Commonwealth Act No. 103). It is
more an administrative board than a part of the integrated judicial system of the nation. It is not intended
to be a mere receptive organ of the government. Unlike a court of justice which is essentially passive,
acting only when its jurisdiction is invoked and deciding only cases that are presented to it by the parties
litigant, the function of the CIR, as will appear from perusal of its organic law is more active, affirmative
and dynamic. It not only exercises judicial or quasi-judicial functions in the determination of disputes
between employers and employees but its functions are far more comprehensive and extensive. It has
jurisdiction over the entire Philippines, to consider, investigate, decide, and settle any question, matter
controversy or disputes arising between, and/ or affecting employers and employees or laborers, and
landlords and tenants or farm-laborers, and regulates the relations between them, subject to, and in
accordance with, the provisions of CA 103.
As laid down in the case of Goseco v. CIR, the SC had the occasion to point out that the CIR is not
narrowly constrained by technical rules of procedure, and equity and substantial merits of the case,
without regard to technicalities or legal forms and shall not be bound by any technical rules of legal
evidence but may inform its mind in such manner as it may deem just and equitable.

The fact, however, that the CIR may be said to be free from rigidity of certain procedural requirements
does not mean that it can in justiciable cases coming before it, entirely ignore or disregard the
fundamental and essential requirements of due process in trials and investigations of an administrative
character. There cardinal primary rights which must be respected even in proceedings of this character:
(1)
the right to a hearing, which includes the right to present one's cause and submit
evidence in support thereof;
(2)
The tribunal must consider the evidence presented;
(3)
The decision must have something to support itself;
(4)
The evidence must be substantial;
(5)
The decision must be based on the evidence presented at the hearing; or at least
contained in the record and disclosed to the parties affected;
(6)
The tribunal or body or any of its judges must act on its own independent
consideration of the law and facts of the controversy, and not simply accept the views of a
subordinate;
(7)
The Board or body should, in all controversial questions, render its decision in such
manner that the parties to the proceeding can know the various Issue involved, and the reason
for the decision rendered.
The failure to grasp the fundamental issue involved is not entirely attributable to the parties adversely
affected by the result. Accordingly, the motion for a new trial should be, and the same is hereby granted,
and the entire record of this case shall be remanded to the CIR, with instruction that it reopen the case
receive all such evidence as may be relevant, and otherwise proceed in accordance with the
requirements set forth. So ordered.
ATENEO DE MANILA UNIVERSITY VS. HON. JUDGE IGNACIO CAPULONG
[222 SCRA 644; G.R. 99327; 27 MAY 1993]
Facts:
Leonardo H. Villa, a first year law student of Petitioner University, died of serious physical injuries at
Chinese General Hospital after the initiation rites of Aquila Legis. Bienvenido Marquez was also
hospitalized at the Capitol Medical Center for acute renal failure occasioned by the serious physical
injuries inflicted upon him on the same occasion. Petitioner Dean Cynthia del Castillo created a Joint
Administration-Faculty-Student Investigating Committee which was tasked to investigate and submit a
report within 72 hours on the circumstances surrounding the death of Lennie Villa. Said notice also
required respondent students to submit their written statements within twenty-four (24) hours from receipt.
Although respondent students received a copy of the written notice, they failed to file a reply. In the
meantime, they were placed on preventive suspension. The Joint Administration-Faculty-Student
Investigating Committee, after receiving the written statements and hearing the testimonies of several
witness, found a prima facie case against respondent students for violation of Rule 3 of the Law School
Catalogue entitled "Discipline." Respondent students were then required to file their written answers to

Page
Section 1-C, SY 06-07

52

San Beda College of Law Alabang


Constitutional Law 2 Case Digests
the formal charge. Petitioner Dean created a Disciplinary Board to hear the charges against respondent
students. The Board found respondent students guilty of violating Rule No. 3 of the Ateneo Law School
Rules on Discipline which prohibits participation in hazing activities. However, in view of the lack of
unanimity among the members of the Board on the penalty of dismissal, the Board left the imposition of
the penalty to the University Administration. Accordingly, Fr. Bernas imposed the penalty of dismissal on
all respondent students. Respondent students filed with RTC Makati a TRO since they are currently
enrolled. This was granted. A TRO was also issued enjoining petitioners from dismissing the respondents.
A day after the expiration of the temporary restraining order, Dean del Castillo created a Special Board to
investigate the charges of hazing against respondent students Abas and Mendoza. This was requested to
be stricken out by the respondents and argued that the creation of the Special Board was totally unrelated
to the original petition which alleged lack of due process. This was granted and reinstatement of the
students was ordered.
Issue:
Was there denial of due process against the respondent students.
Held:
There was no denial of due process, more particularly procedural due process. Dean of the Ateneo Law
School, notified and required respondent students to submit their written statement on the incident.
Instead of filing a reply, respondent students requested through their counsel, copies of the charges. The
nature and cause of the accusation were adequately spelled out in petitioners' notices. Present is the twin
elements of notice and hearing.
Respondent students argue that petitioners are not in a position to file the instant petition under Rule 65
considering that they failed to file a motion for reconsideration first before the trial court, thereby by
passing the latter and the Court of Appeals. It is accepted legal doctrine that an exception to the doctrine
of exhaustion of remedies is when the case involves a question of law, as in this case, where the issue is
whether or not respondent students have been afforded procedural due process prior to their dismissal
from Petitioner University.
Minimum standards to be satisfied in the imposition of disciplinary sanctions in academic institutions, such
as petitioner university herein, thus:

(1) the students must be informed in writing of the nature and cause of any accusation
against them;
(2) that they shall have the right to answer the charges against them with the assistance
of counsel, if desired:
(3) they shall be informed of the evidence against them
(4) they shall have the right to adduce evidence in their own behalf; and
(5) the evidence must be duly considered by the investigating committee or official
designated by the school authorities to hear and decide the case.

US GOVERNMENT VS. JUDGE PURUNGAN


[389 SCRA 623; G.R. NO. 148571, 24 SEPT 2002]
Facts:
The United States of America, pursuant to the existing RP-US extradition treaty, requested the extradition
of Mark B. Jimenez. Upon receipt of the request, the secretary of foreign affairs (SFA) transmitted them to
the secretary of justice (SOJ) for appropriate action. In such event, the RTC held that Jimenez shell be
deprived of the right to notice and hearing during the evaluation stage of the extradition process.
Thereafter the US government, through DOJ, filed Petition for Extradition and Jimenezs immediate
arrest, to avoid flight. Before the RTC could render its decision, Jimenez filed an "Urgent
Manifestation/Ex-Parte Motion," praying that his application for an arrest warrant be set for hearing, which
was granted. During which, the lower court issued its questioned July 3, 2001 Order, directing the
issuance of a warrant for his arrest and fixing bail for his temporary liberty at one million pesos in cash.
After Jimenez had surrendered his passport and posted the required cash bond, he was granted
provisional liberty via the challenged Order dated July 4, 2001. Thus, Petition prays for the lifting of the
bail Order, the cancellation of the bond, and the taking of Jimenez into legal custody.
Issue:
Whether or not Jimenez is entitled to notice and hearing before a warrant for his arrest can be issued.

Page
Section 1-C, SY 06-07

53

Whether or not he is entitled to bail and to provisional liberty while the extradition proceedings are
pending.
Held:
By nature, extradition proceedings are not equivalent to a criminal case in which guilt or innocence is
determined. Consequently, an extradition case is not one in which the constitutional rights of the accused
are necessarily available. Having once escaped the jurisdiction of the requesting state, the reasonable
prima facie presumption is that the person would escape again if given the opportunity. Hence, if the
judge is convinced that a prima facie case exists, he immediately Issue a warrant for the arrest of the
potential extraditee and summons him or her to answer and to appear at scheduled hearings on the
petition. Potential extraditees are entitled to the rights to due process and to fundamental fairness. Due
process does not always call for a prior opportunity to be heard. A subsequent opportunity is sufficient due
to the flight risk involved. Indeed, available during the hearings on the petition and the answer is the full
chance to be heard and to enjoy fundamental fairness that is compatible with the summary nature of
extradition.
After being taken into custody, potential extraditees may apply for bail. Since the applicants have a history
of absconding, they have the burden of showing that (a) there is no flight risk and no danger to the
community; and (b) there exist special, humanitarian or compelling circumstances. In extradition cases,
bail is not a matter of right; it is subject to judicial discretion in the context of the peculiar facts of each
case.

Page
Section 1-C, SY 06-07

54

San Beda College of Law Alabang


Constitutional Law 2 Case Digests

EQUAL PROTECTION
Art 3, Sec. 1.

nor shall any person be denied the equal protection of the laws.
PEOPLE VS. CAYAT
[68 PHIL 12; G.R. NO. 45987; 5 MAY 1939]

Facts:
Law prohibits any member of a non-Christian tribe to buy, receive, have in his possession, or drink, any
intoxicating liquors of any kind. The law, Act No. 1639, exempts only the so-called native wines or liquors
which the members of such tribes have been accustomed to take.
Issue:
Whether or Not the law denies equal protection to one prosecuted and sentenced for violation of said law.
Held:
No. It satisfies the requirements of a valid classification, one of which is that the classification under the
law must rest on real or substantial distinctions.
The distinction is reasonable. The classification between the members of the non- Christian and the
members of the Christian tribes is not based upon accident of birth or parentage but upon the degree of
civilization and culture. The term non-Christian tribes refers to a geographical area and more directly to
natives of the Philippines of a low grade civilization usually living in tribal relationship apart from settled
communities. The distinction is reasonable for the Act was intended to meet the peculiar conditions
existing in the non- Christian tribes
The prohibition is germane to the purposes of the law. It is designed to insure peace and order in and
among the non- Christian tribes has often resulted in lawlessness and crime thereby hampering the
efforts of the government to raise their standards of life and civilization. This law is not limited in its
application to conditions existing at the time of the enactment. It is intended to apply for all times as long
as those conditions exists. The Act applies equally to all members of the class. That it may be unfair in its
operation against a certain number of non- Christians by reason of their degree of culture is not an
argument against the equality of its operation nor affect the reasonableness of the classification thus
established.

PASEI VS. DRILON


[163 SCRA 386; L-81958; 30 JUN 1988]
Facts:
Petitioner, Phil association of Service Exporters, Inc., is engaged principally in the recruitment of Filipino
workers, male and female of overseas employment. It challenges the constitutional validity of Dept. Order
No. 1 (1998) of DOLE entitled Guidelines Governing the Temporary Suspension of Deployment of
Filipino Domestic and Household Workers. It claims that such order is a discrimination against males and
females. The Order does not apply to all Filipino workers but only to domestic helpers and females with
similar skills, and that it is in violation of the right to travel, it also being an invalid exercise of the
lawmaking power. Further, PASEI invokes Sec 3 of Art 13 of the Constitution, providing for worker
participation in policy and decision-making processes affecting their rights and benefits as may be
provided by law. Thereafter the Solicitor General on behalf of DOLE submitting to the validity of the
challenged guidelines involving the police power of the State and informed the court that the respondent
have lifted the deployment ban in some states where there exists bilateral agreement with the Philippines
and existing mechanism providing for sufficient safeguards to ensure the welfare and protection of the
Filipino workers.
Issue:
Whether or not there has been a valid classification in the challenged Department Order No. 1.

Page
Section 1-C, SY 06-07

55

Held:
SC in dismissing the petition ruled that there has been valid classification, the Filipino female domestics
working abroad were in a class by themselves, because of the special risk to which their class was
exposed. There is no question that Order No.1 applies only to female contract workers but it does not
thereby make an undue discrimination between sexes. It is well settled hat equality before the law under
the constitution does not import a perfect identity of rights among all men and women. It admits of
classification, provided that:
1.
2.
3.
4.

Such classification rests on substantial distinctions


That they are germane to the purpose of the law
They are not confined to existing conditions
They apply equally to al members of the same class

In the case at bar, the classifications made, rest on substantial distinctions.


Dept. Order No. 1 does not impair the right to travel. The consequence of the deployment ban has on the
right to travel does not impair the right, as the right to travel is subjects among other things, to the
requirements of public safety as may be provided by law. Deployment ban of female domestic helper is
a valid exercise of police power. Police power as been defined as the state authority to enact legislation
that may interfere with personal liberty or property in order to promote general welfare. Neither is there
merit in the contention that Department Order No. 1 constitutes an invalid exercise of legislative power as
the labor code vest the DOLE with rule making powers.
DUMLAO VS. COMELEC
[95 SCRA 392; L-52245; 22 JAN 1980]
Facts:
Petitioner questions the constitutionality of section 4 of Batas Pambansa Blg. 52 as discriminatory and
contrary to the equal protection and due process guarantees of the Constitution.
Section 4 provided that any retired municipal or provincial city official that already received retirement
benefits and is 65 years of age shall not be qualified to run for the same local elective office from which
he has retired.
Issue:
Whether or Not Sec. 4 of BP.52 is unconstitutional being contrary to the equal protection and due process
rights.
Held:
No. The guarantee of equal protection is subject to rational classification based on reasonable and real
differentiations. In the present case, employees 65 years of age have been classified differently from
younger employees. The former are subject to compulsory retirement while the latter are not.
Retirement is not a reasonable disqualification for elective local officials because there can be retirees
who are even younger and a 65 year old retiree could be as good as a 65 year old official who is not a
retiree. But there is reason to disqualify a 65 year old elective official who is trying to run for office
because there is the need for new blood to assume relevance. When an official has retired he has
already declared himself tired and unavailable for the same government work.
WHEREFORE, the first paragraph of section 4 of Batas pambansa Bilang 52 is hereby declared valid.
TELECOMMUNICATIONS AND BROADCAST ATTORNEYS OF THE PHILS. VS. COMELEC
[289 SCRA 337; G.R. NO. 132922; 21 APR 1998]
Facts:
Petitioner Telecommunications and Broadcast Attorneys of the Philippines, Inc. (TELEBAP) is an
organization of lawyers of radio and television broadcasting companies. It was declared to be without
legal standing to sue in this case as, among other reasons, it was not able to show that it was to suffer
from actual or threatened injury as a result of the subject law. Petitioner GMA Network, on the other

Page
Section 1-C, SY 06-07

56

San Beda College of Law Alabang


Constitutional Law 2 Case Digests
hand, had the requisite standing to bring the constitutional challenge. Petitioner operates radio and
television broadcast stations in the Philippines affected by the enforcement of Section 92, B.P. No. 881.
Petitioners challenge the validity of Section 92, B.P. No. 881 which provides:
Comelec Time- The Commission shall procure radio and television time to be known as
the Comelec Time which shall be allocated equally and impartially among the
candidates within the area of coverage of all radio and television stations. For this
purpose, the franchise of all radio broadcasting and television stations are hereby
amended so as to provide radio or television time, free of charge, during the period of
campaign.
Petitioner contends that while Section 90 of the same law requires COMELEC to procure print space in
newspapers and magazines with payment, Section 92 provides that air time shall be procured by
COMELEC free of charge. Thus it contends that Section 92 singles out radio and television stations to
provide free air time.
Petitioner claims that it suffered losses running to several million pesos in providing COMELEC Time in
connection with the 1992 presidential election and 1995 senatorial election and that it stands to suffer
even more should it be required to do so again this year. Petitioners claim that the primary source of
revenue of the radio and television stations is the sale of air time to advertisers and to require these
stations to provide free air time is to authorize unjust taking of private property. According to petitioners,
in 1992 it lost P22,498,560.00 in providing free air time for one hour each day and, in this years elections,
it stands to lost P58,980,850.00 in view of COMELECs requirement that it provide at least 30 minutes of
prime time daily for such.
Issue:
Whether of not Section 92 of B.P. No. 881 denies radio and television broadcast companies the equal
protection of the laws.
Whether or not Section 92 of B.P. No. 881 constitutes taking of property without due process of law and
without just compensation.
Held:
Petitioners argument is without merit. All broadcasting, whether radio or by television stations, is
licensed by the government. Airwave frequencies have to be allocated as there are more individuals who
want to broadcast that there are frequencies to assign. Radio and television broadcasting companies,
which are given franchises, do not own the airwaves and frequencies through which they transmit
broadcast signals and images. They are merely given the temporary privilege to use them. Thus, such
exercise of the privilege may reasonably be burdened with the performance by the grantee of some form
of public service. In granting the privilege to operate broadcast stations and supervising radio and
television stations, the state spends considerable public funds in licensing and supervising them.
The argument that the subject law singles out radio and television stations to provide free air time as
against newspapers and magazines which require payment of just compensation for the print space they
may provide is likewise without merit. Regulation of the broadcast industry requires spending of public
funds which it does not do in the case of print media. To require the broadcast industry to provide free air
time for COMELEC is a fair exchange for what the industry gets.
As radio and television broadcast stations do not own the airwaves, no private property is taken by the
requirement that they provide air time to the COMELEC.
LACSON VS. EXECUTIVE SECRETARY
[301 SCRA 298; G.R. NO. 128096; 20 JAN 1999]
Facts:
Eleven persons believed to be members of the Kuratong Baleleng gang, an organized crime syndicate
involved in bank robberies, were slain by elements of the Anti-Bank Robbery and Intelligence Task Group
(ABRITG). Among those included in the ABRITG were petitioners and petitioner-intervenors.
Acting on a media expose of SPO2 Eduardo delos Reyes, a member of the Criminal Investigation
Command, that what actually transpired was a summary execution and not a shoot-out between the
Kuratong Baleleng gang members and the ABRITG, Ombudsman Aniano Desierto formed a panel of

Page
Section 1-C, SY 06-07

57

investigators to investigate the said incident. Said panel found the incident as a legitimate police
operation. However, a review board modified the panels finding and recommended the indictment for
multiple murder against twenty-six respondents including herein petitioner, charged as principal, and
herein petitioner-intervenors, charged as accessories. After a reinvestigation, the Ombudsman filed
amended informations before the Sandiganbayan, where petitioner was charged only as an accessory.
The accused filed separate motions questioning the jurisdiction of the Sandiganbayan, asserting that
under the amended informations, the cases fall within the jurisdiction of the Regional Trial Court pursuant
to Section 2 of R.A. 7975. They contend that the said law limited the jurisdiction of the Sandiganbayan to
cases where one or ore of the principal accused are government officals with Salary Grade 27 or higher,
or PNP officials with rank of Chief Superintendent or higher. Thus, they did not qualify under said
requisites. However, pending resolution of their motions, R.A. 8249 was approved amending the
jurisdiction of the Sandiganbayan by deleting the word principal from the phrase principal accused in
Section 2 of R.A. 7975.
Petitioner questions the constitutionality of Section 4 of R.A. 8249, including Section 7 which provides that
the said law shall apply to all cases pending in any court over which trial has not begun as of the approval
hereof.
Issue:
Whether or not Sections 4 and 7 of R.A. 8249 violate the petitioners right to due process and the equal
protection clause of the Constitution as the provisions seemed to have been introduced for the
Sandiganbayan to continue to acquire jurisdiction over the Kuratong Baleleng case.
Whether or not said statute may be considered as an ex-post facto statute.
Whether or not the multiple murder of the alleged members of the Kuratong Baleleng was committed in
relation to the office of the accused PNP officers which is essential to the determination whether the case
falls within the Sandiganbayans or Regional Trial Courts jurisdiction.
Held:
Petitioner and intervenors posture that Sections 4 and 7 of R.A. 8249 violate their right to equal
protection of the law is too shallow to deserve merit. No concrete evidence and convincing argument
were presented to warrant such a declaration. Every classification made by the law is presumed
reasonable and the party who challenges the law must present proof of arbitrariness. The classification is
reasonable and not arbitrary when the following concur: (1) it must rest on substantial distinction; (2) it
must be germane to the purpose of the law; (3) must not be limited to existing conditions only, and (4)
must apply equally to all members of the same class; all of which are present in this case.
Paragraph a of Section 4 provides that it shall apply to all cases involving certain public officials and
under the transitory provision in Section 7, to all cases pending in any court. Contrary to petitioner and
intervenors argument, the law is not particularly directed only to the Kuratong Baleleng cases. The
transitory provision does not only cover cases which are in the Sandiganbayan but also in any court.
There is nothing ex post facto in R.A. 8249. Ex post facto law, generally, provides retroactive effect of
penal laws. R.A. 8249 is not a penal law. It is a substantive law on jurisdiction which is not penal in
character. Penal laws are those acts of the Legislature which prohibit certain acts and establish penalties
for their violations or those that define crimes and provide for their punishment. R.A. 7975, as regards the
Sandiganbayans jurisdiction, its mode of appeal and other procedural matters, has been declared by the
Court as not a penal law, but clearly a procedural statute, one which prescribes rules of procedure by
which courts applying laws of all kinds can properly administer justice. Not being a penal law, the
retroactive application of R.A. 8249 cannot be challenged as unconstitutional.
In People vs. Montejo, it was held that an offense is said to have been committed in relation to the office if
it is intimately connected with the office of the offender and perpetrated while he was in the performance
of his official functions. Such intimate relation must be alleged in the information which is essential in
determining the jurisdiction of the Sandiganbayan. However, upon examination of the amended
information, there was no specific allegation of facts that the shooting of the victim by the said principal
accused was intimately related to the discharge of their official duties as police officers. Likewise, the
amended information does not indicate that the said accused arrested and investigated the victim and
then killed the latter while in their custody. The stringent requirement that the charge set forth with such
particularity as will reasonably indicate the exact offense which the accused is alleged to have committed
in relation to his office was not established.

Page
Section 1-C, SY 06-07

58

San Beda College of Law Alabang


Constitutional Law 2 Case Digests
Consequently, for failure to show in the amended informations that the charge of murder was intimately
connected with the discharge of official functions of the accused PNP officers, the offense charged in the
subject criminal cases is plain murder and, therefore, within the exclusive original jurisdiction of the
Regional Trial Court and not the Sandiganbayan.
INT'L. SCHOOL ALLIANCE VS. QUISUMBING
[333 SCRA 13; G.R. NO. 128845; 1 JUN 2000]
Facts:
Receiving salaries less than their counterparts hired abroad, the local-hires of private respondent School,
mostly Filipinos, cry discrimination. We agree. That the local-hires are paid more than their colleagues in
other schools is, of course, beside the point. The point is that employees should be given equal pay for
work of equal value.
Private respondent International School, Inc. (the School, for short), pursuant to Presidential Decree 732,
is a domestic educational institution established primarily for dependents of foreign diplomatic personnel
and other temporary residents. To enable the School to continue carrying out its educational program and
improve its standard of instruction, Section 2(c) of the same decree authorizes the School to
employ its own teaching and management personnel selected by it either locally or abroad, from
Philippine or other nationalities, such personnel being exempt from otherwise applicable laws and
regulations attending their employment, except laws that have been or will be enacted for the protection
of employees.
Accordingly, the School hires both foreign and local teachers as members of its faculty, classifying the
same into two: (1) foreign-hires and (2) local-hires.
The School grants foreign-hires certain benefits not accorded local-hires. These include housing,
transportation, shipping costs, taxes, and home leave travel allowance. Foreign-hires are also paid a
salary rate twenty-five percent (25%) more than local-hires. The School justifies the difference on two
"significant economic disadvantages" foreign-hires have to endure, namely: (a) the "dislocation factor"
and (b) limited tenure.
Issue:
Whether or Not the grants provided by the school to foreign hires and not to local hires discriminative of
their
constitutional
right
to
the
equal
protection
clause.
Held:
The foregoing provisions impregnably institutionalize in this jurisdiction the long honored legal truism of
"equal pay for equal work." Persons who work with substantially equal qualifications, skill, effort and
responsibility, under similar conditions, should be paid similar salaries. This rule applies to the School, its
"international character" notwithstanding.
The School contends that petitioner has not adduced evidence that local-hires perform work equal to that
of foreign-hires. The Court finds this argument a little cavalier. If an employer accords employees the
same position and rank, the presumption is that these employees perform equal work. This presumption
is borne by logic and human experience. If the employer pays one employee less than the rest, it is not
for that employee to explain why he receives less or why the others receive more. That would be adding
insult to injury. The employer has discriminated against that employee; it is for the employer to explain
why the employee is treated unfairly.
While we recognize the need of the School to attract foreign-hires, salaries should not be used as an
enticement to the prejudice of local-hires. The local-hires perform the same services as foreign-hires and
they ought to be paid the same salaries as the latter. For the same reason, the "dislocation factor" and the
foreign-hires' limited tenure also cannot serve as valid bases for the distinction in salary rates.
The Constitution enjoins the State to "protect the rights of workers and promote their welfare," "to afford
labor full protection." The State, therefore, has the right and duty to regulate the relations between labor
and capital. These relations are not merely contractual but are so impressed with public interest that labor
contracts, collective bargaining agreements included, must yield to the common good. Should such
contracts contain stipulations that are contrary to public policy, courts will not hesitate to strike down these
stipulations.

Page
Section 1-C, SY 06-07

59

In this case, we find the point-of-hire classification employed by respondent School to justify the
distinction in the salary rates of foreign-hires and local hires to be an invalid classification. There is no
reasonable distinction between the services rendered by foreign-hires and local-hires.
Wherefore, the petition is given due course. The petition is hereby granted in part. The orders of the
secretary of labor and employment dated June 10, 1996 and march 19, 1997, are hereby reversed and
set aside insofar as they uphold the practice of respondent school of according foreign-hires higher
salaries than local-hires.
ORMOC SUGAR COMPANY VS. TREASURER OF ORMOC CITY
[22 SCRA 603; L-23794; 17 FEB 1968]
Facts:
On January 29, 1964, the Municipal Board of Ormoc City passed Ordinance No. 4, Series of 1964,
imposing "on any and all productions of centrifugal sugar milled at the Ormoc Sugar Company, Inc., in
Ormoc City a municipal tax equivalent to one per centum (1%) per export sale to the United States of
America and other foreign countries." Payments for said tax were made, under protest, by Ormoc Sugar
Company, Inc. on March 20, 1964 for P7, 087.50 and on April 20, 1964 for P5, 000, or a total of P12,
087.50.
On June 1, 1964, Ormoc Sugar Company, Inc. filed before the Court of First Instance of Leyte, with
service of a copy upon the Solicitor General, a complaint against the City of Ormoc as well as its
Treasurer, Municipal Board and Mayor, alleging that the afore-stated ordinance is unconstitutional for
being violative of the equal protection clause (Sec. 1[1], Art. III, Constitution) and the rule of uniformity of
taxation (Sec. 22[1]), Art. VI, Constitution).
Answering, the defendants asserted that the tax ordinance was within defendant city's power to enact
under the Local Autonomy Act and that the same did not violate the afore-cited constitutional limitations.
After pre-trial and submission of the case on memoranda, the Court of First Instance, on August 6, 1964,
rendered a decision that upheld the constitutionality of the ordinance and declared the taxing power of
defendant chartered city broadened by the Local Autonomy Act to include all other forms of taxes,
licenses or fees not excluded in its charter.
Issue:
Whether or Not the ordinance is unconstitutional for being violative of the equal protection clause under
Sec. 1[1], Art. III, Constitution.
Whether or not it was violative of the rule of uniformity of taxation under the Bill of Rights, Sec. 22[1], Art.
VI, Constitution.
Held:
The Constitution in the bill of rights provides: ". . . nor shall any person be denied the equal protection of
the laws." (Sec. 1 [1], Art. III) In Felwa vs. Salas, We ruled that the equal protection clause applies only to
persons or things identically situated and does not bar a reasonable classification of the subject of
legislation, and a classification is reasonable where (1) it is based on substantial distinctions which make
real differences; (2) these are germane to the purpose of the law; (3) the classification applies not only to
present conditions but also to future conditions which are substantially identical to those of the present;
(4) the classification applies only to those who belong to the same class.
A perusal of the requisites instantly shows that the questioned ordinance does not meet them, for it taxes
only centrifugal sugar produced and exported by the Ormoc Sugar Company, Inc. and none other. At the
time of the taxing ordinance's enactment, Ormoc Sugar Company, Inc., it is true, was the only sugar
central in the city of Ormoc. Still, the classification, to be reasonable, should be in terms applicable to
future conditions as well. The taxing ordinance should not be singular and exclusive as to exclude any
subsequently established sugar central, of the same class as plaintiff, for the coverage of the tax. As it is
now, even if later a similar company is set up, it cannot be subject to the tax because the ordinance
expressly points only to Ormoc City Sugar Company, Inc. as the entity to be levied upon.
Appellant, however, is not entitled to interest; on the refund because the taxes were not arbitrarily
collected (Collector of Internal Revenue v. Binalbagan). 6 At the time of collection, the ordinance provided
a sufficient basis to preclude arbitrariness, the same being then presumed constitutional until declared
otherwise.

Page
Section 1-C, SY 06-07

60

San Beda College of Law Alabang


Constitutional Law 2 Case Digests
Wherefore, the decision appealed from is hereby reversed, the challenged ordinance is declared
unconstitutional and the defendants-appellees are hereby ordered to refund the P12,087.50 plaintiffappellant paid under protest. No costs. So ordered.
PHILIPPINE JUDGES ASSO. VS. PRADO
[227 SCRA 703; G.R. NO. 105371; 11 NOV 1993]
Facts:
The Philippine Postal Corporation issued circular No. 92-28 to implement Section 35 of RA 7354
withdrawing the franking privilege from the SC, CA, RTCs, MeTCs, MTCs and Land Registration
Commission and with certain other government offices. It is alleged that RA 7354 is discriminatory
becasue while withdrawing the franking privilege from judiciary, it retains the same for the President &
Vice-President of the Philippines, Senator & members of the House of Representatives, COMELEC,
National Census & Statistics Office and the general public. The respondents counter that there is no
discrimination because the law is based on a valid classification in accordance with the equal protection
clause.
Issue:
Whether or Not Section 35 of RA 7354 is constitutional.
Held:
The equal protection of the laws is embraced in the concept of due process, as every unfair discrimination
offends the requirements of justice and fair play. It has nonetheless been embodied in a separate clause
in Article III Section 1 of the Constitution to provide for amore specific guarantee against any form of
undue favoritism or hostility from the government. Arbitrariness in general may be challenged on the
basis of the due process clause. But if the particular act assailed partakes of an unwarranted partiality or
prejudice, the sharper weapon to cut it down is the equal protection clause. Equal protection simply
requires that all persons or things similarly situated should be treated alike, both as to rights conferred
and responsibilities imposed. What the clause requires is equality among equals as determined
according to a valid classification. Section 35 of RA 7354 is declared unconstitutional. Circular No. 92-28
is set aside insofar

Page
Section 1-C, SY 06-07

61

SEARCHES AND SEIZURES


Art 3, Sec. 2. The right of the people to be secure in their persons, houses, papers, and effects
against unreasonable searches and seizures of whatever nature and for any purpose shall be inviolable,
and no search warrant or warrant of arrest shall issue except upon probable cause to be determined
personally by the judge after examination under oath or affirmation of the complainant and the witnesses
he may produce, and particularly describing the place to be searched and the persons or things to be
seized.
Art 3, Sec. 3. (1) The privacy of communication and correspondence shall be inviolable except upon
lawful order of the court, or when public safety or order requires otherwise as prescribed by law.
(2)
Any evidence obtained in violation of this or the preceding section shall be inadmissible for any
purpose in any proceeding.
PEOPLE VS. MARTI
[193 SCRA 57; G.R. NO. 81561; 18 JAN 1991]
Facts:
Accused-appellant went to a forwarding agency to send four packages to a friend in Zurich. Initially, the
accused was asked by the proprietress if the packages can be examined. However, he refused. Before
delivering said packages to the Bureau of Customs and the Bureau of Posts, the husband of the
proprietress opened said boxes for final inspection. From that inspection, included in the standard
operating procedure and out of curiosity, he took several grams of its contents.
He brought a letter and the said sample to the National Bureau of Investigation. When the NBI was
informed that the rest of the shipment was still in his office, three agents went back with him. In their
presence, the husband totally opened the packages. Afterwards, the NBI took custody of said packages.
The contents , after examination by forensic chemists, were found to be marijuana flowering tops.
The appellant, while claiming his mail at the Central Post Office, was invited by the agents for
questioning. Later on, the trial court found him guilty of violation of the Dangerous Drugs Act.
Issue:
Whether or Not the items admitted in the searched illegally searched and seized.
Whether or Not custodial investigation properly applied.
Whether or Not the trial court not give credence to the explanation of the appellant on how said packages
came to his possession.
Held:
No. The case at bar assumes a peculiar character since the evidence sought to be excluded was
primarily discovered and obtained by a private person, acting in a private capacity and without the
intervention and participation of State authorities. Under the circumstances, can accused/appellant validly
claim that his constitutional right against unreasonable searches and seizure has been violated. Stated
otherwise, may an act of a private individual, allegedly in violation of appellant's constitutional rights, be
invoked against the State. In the absence of governmental interference, the liberties guaranteed by the
Constitution cannot be invoked against the State. It was Mr. Job Reyes, the proprietor of the forwarding
agency, who made search/inspection of the packages. Said inspection was reasonable and a standard
operating procedure on the part of Mr. Reyes as a precautionary measure before delivery of packages to
the Bureau of Customs or the Bureau of Posts. Second, the mere presence of the NBI agents did not
convert the reasonable search effected by Reyes into a warrantless search and seizure proscribed by the
Constitution. Merely to observe and look at that which is in plain sight is not a search. Having observed
that which is open, where no trespass has been committed in aid thereof, is not search.
No. The law enforcers testified that accused/appellant was informed of his constitutional rights. It is
presumed that they have regularly performed their duties (See. 5(m), Rule 131) and their testimonies
should be given full faith and credence, there being no evidence to the contrary.
No. Appellant signed the contract as the owner and shipper thereof giving more weight to the
presumption that things which a person possesses, or exercises acts of ownership over, are owned by
him (Sec. 5 [j], Rule 131). At this point, appellant is therefore estopped to claim otherwise.

Page
Section 1-C, SY 06-07

62

San Beda College of Law Alabang


Constitutional Law 2 Case Digests

WATEROUS DRUG VS. NLRC


[280 SCRA 735 ; G.R.NO. 113271; 16 OCT 1997]
Facts:
Catolico was hired as a pharmacist by petitioner Waterous Drug Corporation on 15 August 1988. On 31
July 1989, Catolico received a memorandum from WATEROUS Vice President-General Manager Emma
R. Co warning her not to dispense medicine to employees chargeable to the latter's accounts because
the same was a prohibited practice. On the same date, Co issued another memorandum to Catolico
warning her not to negotiate with suppliers of medicine without consulting the Purchasing Department, as
this would impair the company's control of purchases and, besides she was not authorized to deal directly
with the suppliers.
As regards the first memorandum, Catolico did not deny her responsibility but explained that her act was
"due to negligence," since fellow employee Irene Soliven "obtained the medicines in bad faith and through
misrepresentation when she claimed that she was given a charge slip by the Admitting Dept." Catolico
then asked the company to look into the fraudulent activities of Soliven.
In a memorandum dated 21 November 1989, WATEROUS Supervisor Luzviminda E. Bautro warned
Catolico against the "rush delivery of medicines without the proper documents." On 29 January 1990,
WATEROUS Control Clerk Eugenio Valdez informed Co that he noticed an irregularity involving Catolico
and Yung Shin Pharmaceuticals, Inc.
Forthwith, in her memorandum dated 37 January 1990, Co asked Catolico to explain, within twenty-four
hours, her side of the reported irregularity. Catolico asked for additional time to give her explanation, and
she was granted a 48-hour extension from 1 to 3 February 1990. However, on 2 February 1990, she was
informed that effective 6 February 1990 to 7 March 1990, she would be placed on preventive suspension
to protect the interests of the company.
In a letter dated 2 February 1990, Catolico requested access to the file containing Sales Invoice No. 266
for her to be able to make a satisfactory explanation. In said letter she protested Saldaa's invasion of her
privacy when Saldaa opened an envelope addressed to Catolico.
In a letter to Co dated 10 February 1990, Catolico, through her counsel, explained that the check she
received from YSP was a Christmas gift and not a "refund of overprice." She also averred that the
preventive suspension was ill-motivated, as it sprang from an earlier incident between her and Co's
secretary, Irene Soliven.
On 5 March 1990, WATEROUS Supervisor Luzviminda Bautro, issued a memorandum notifying Catolico
of her termination. On 5 May 1990, Catolico filed before the Office of the Labor Arbiter a complaint for
unfair labor practice, illegal dismissal, and illegal suspension. In his decision of 10 May 1993, Labor
Arbiter Alex Arcadio Lopez found no proof of unfair labor practice against petitioners. Nevertheless, he
decided in favor of Catolico because petitioners failed to "prove what alleged as complainant's
dishonesty," and to show that any investigation was conducted. Hence, the dismissal was without just
cause and due process. He thus declared the dismissal and suspension illegal but disallowed
reinstatement.
Petitioners seasonably appealed from the decision and urged the NLRC to set it aside because the Labor
Arbiter erred in finding that Catolico was denied due process and that there was no just cause to
terminate her services.
In its decision of 30 September 1993, the NLRC affirmed the findings of the Labor Arbiter on the ground
that petitioners were not able to prove a just cause for Catolico's dismissal from her employment. It found
that petitioner's evidence consisted only of the check of P640.00 drawn by YSP in favor of complainant,
which her co-employee saw when the latter opened the envelope. But, it declared that the check was
inadmissible in evidence pursuant to Sections 2 and 3(1 and 2) of Article III of the Constitution. It
concluded:
With the smoking gun evidence of respondents being rendered inadmissible, by virtue of
the constitutional right invoked by complainants, respondents' case falls apart as it is
bereft of evidence which cannot be used as a legal basis for complainant's dismissal.
The NLRC then dismissed the appeal for lack of merit, but modified the dispositive portion of the
appealed decision by deleting the award for illegal suspension as the same was already included in the
computation of the aggregate of the awards in the amount of P35,401.86.
Issue:

Page
Section 1-C, SY 06-07

63

Whether or Not the dismissal of the private respondent is in violation of the Constitution, under the Bill of
Rights.
Held:
As to the first and second grounds, petitioners insist that Catolico had been receiving "commissions" from
YSP, or probably from other suppliers, and that the check issued to her on 9 November 1989 was not the
first or the last. They also maintained that Catolico occupied a confidential position and that Catolico's
receipt of YSP's check, aggravated by her "propensity to violate company rules," constituted breach of
confidence. And contrary to the findings of NLRC, Catolico was given ample opportunity to explain her
side of the controversy.
In her Comment, Catolico asserts that petitioners' evidence is too "flimsy" to justify her dismissal. The
check in issue was given to her, and she had no duty to turn it over to her employer. Company rules do
not prohibit an employee from accepting gifts from clients, and there is no indication in the contentious
check that it was meant as a refund for overpriced medicines. Besides, the check was discovered in
violation of the constitutional provision on the right to privacy and communication; hence, as correctly held
by the NLRC, it was inadmissible in evidence.
Catolico was denied due process. Procedural due process requires that an employee be apprised of the
charge against him, given reasonable time to answer the charge, allowed ample opportunity to be heard
and defend himself, and assisted by a representative if the employee so desires. Ample opportunity
connotes every kind of assistance that management must accord the employee to enable him to prepare
adequately for his defense, including legal representation. In the case at bar, although Catolico was given
an opportunity to explain her side, she was dismissed from the service in the memorandum of 5 March
1990 issued by her Supervisor after receipt of her letter and that of her counsel. No hearing was ever
conducted after the issues were joined through said letters.
Catolico was also unjustly dismissed. It is settled that the burden is on the employer to prove just and
valid cause for dismissing an employee, and its failure to discharge that burden would result in a finding
that the dismissal is unjustified. It clearly appears then that Catolico's dismissal was based on hearsay
information. Catolico's dismissal then was obviously grounded on mere suspicion, which in no case can
justify an employee's dismissal. Suspicion is not among the valid causes provided by the Labor Code for
the termination of employment; and even the dismissal of an employee for loss of trust and confidence
must rest on substantial grounds and not on the employer's arbitrariness, whims, caprices, or suspicion.
Besides, Catolico was not shown to be a managerial employee, to which class of employees the term
"trust and confidence" is restricted.
As regards the constitutional violation upon which the NLRC anchored its decision, that the Bill of Rights
does not protect citizens from unreasonable searches and seizures perpetrated by private individuals. It is
not true, as counsel for Catolico claims, that the citizens have no recourse against such assaults. On the
contrary, and as said counsel admits, such an invasion gives rise to both criminal and civil liabilities.
Finally, since it has been determined by the Labor Arbiter that Catolico's reinstatement would not be to the
best interest of the parties, he correctly awarded separation pay to Catolico. Separation pay in lieu of
reinstatement is computed at one month's salary for every year of service. In this case, however, Labor
Arbiter Lopez computed the separation pay at one-half month's salary for every year of service. Catolico
did not oppose or raise an objection. As such, we will uphold the award of separation pay as fixed by the
Labor Arbiter.
WHEREFORE, the instant petition is hereby DISMISSED and the challenged decision and resolution of
the National Labor Relations Commission dated 30 September 1993 and 2 December 1993, respectively,
in NLRC-NCR CA No. 005160-93 are AFFIRMED, except as to its reason for upholding the Labor
Arbiter's decision, viz., that the evidence against private respondent was inadmissible for having been
obtained in violation of her constitutional rights of privacy of communication and against unreasonable
searches and seizures which is hereby set aside.
STONEHILL VS. DIOKNO
[20 SCRA 383; L-19550; 19 JUN 1967]
Facts:
Upon application of the officers of the government named on the margin 1 hereinafter referred to as
Respondents-Prosecutors several judges2 hereinafter referred to as Respondents-Judges
issued, on different dates,3 a total of 42 search warrants against petitioners herein 4 and/or the
corporations of which they were officers,5 directed to the any peace officer, to search the persons above-

Page
Section 1-C, SY 06-07

64

San Beda College of Law Alabang


Constitutional Law 2 Case Digests
named and/or the premises of their offices, warehouses and/or residences, and to seize and take
possession of the following personal property to wit:
Books of accounts, financial records, vouchers, correspondence, receipts, ledgers, journals,
portfolios, credit journals, typewriters, and other documents and/or papers showing all business
transactions including disbursements receipts, balance sheets and profit and loss statements and
Bobbins (cigarette wrappers).
as "the subject of the offense; stolen or embezzled and proceeds or fruits of the offense," or "used or
intended to be used as the means of committing the offense," which is described in the applications
adverted to above as "violation of Central Bank Laws, Tariff and Customs Laws, Internal Revenue (Code)
and the Revised Penal Code."
Petitioners contentions are:
(1) they do not describe with particularity the documents, books and things to be seized;
(2) cash money, not mentioned in the warrants, were actually seized;
(3) the warrants were issued to fish evidence against the aforementioned petitioners in deportation
cases filed against them;
(4) the searches and seizures were made in an illegal manner; and
(5) the documents, papers and cash money seized were not delivered to the courts that issued the
warrants, to be disposed of in accordance with law
Respondents-prosecutors contentions
(1) that the contested search warrants are valid and have been issued in accordance with law;
(2) that the defects of said warrants, if any, were cured by petitioners' consent; and
(3) that, in any event, the effects seized are admissible in evidence against herein petitioners, regardless
of the alleged illegality of the aforementioned searches and seizures.
The documents, papers, and things seized under the alleged authority of the warrants in question may be
split into two (2) major groups, namely: (a) those found and seized in the offices of the aforementioned
corporations, and (b) those found and seized in the residences of petitioners herein.
Issue:
Whether or not those found and seized in the offices of the aforementioned corporations are obtained
legally.
Whether or not those found and seized in the residences of petitioners herein are obtained legally.
Held:
The petitioners have no cause of action to assail the legality of the contested warrants and of the seizures
made in pursuance thereof, for the simple reason that said corporations have their respective
personalities, separate and distinct from the personality of herein petitioners, regardless of the amount of
shares of stock or of the interest of each of them in said corporations, and whatever the offices they hold
therein may be. Indeed, it is well settled that the legality of a seizure can be contested only by the party
whose rights have been impaired thereby, and that the objection to an unlawful search and seizure is
purely personal and cannot be availed of by third parties.
With respect to the documents, papers and things seized in the residences of petitioners herein, the
aforementioned resolution of June 29, 1962, lifted the writ of preliminary injunction previously issued by
this Court, thereby, in effect, restraining herein Respondents-Prosecutors from using them in evidence
against petitioners herein.
Two points must be stressed in connection with this constitutional mandate, namely: (1) that no warrant
shall issue but upon probable cause, to be determined by the judge in the manner set forth in said
provision; and (2) that the warrant shall particularly describe the things to be seized.
None of these requirements has been complied with in the contested warrants. Indeed, the same were
issued upon applications stating that the natural and juridical person therein named had committed a
"violation of Central Ban Laws, Tariff and Customs Laws, Internal Revenue (Code) and Revised Penal
Code." In other words, no specific offense had been alleged in said applications. The averments thereof
with respect to the offense committed were abstract. As a consequence, it was impossible for the judges
who issued the warrants to have found the existence of probable cause, for the same presupposes the
introduction of competent proof that the party against whom it is sought has performed particular acts, or
committed specific omissions, violating a given provision of our criminal laws. As a matter of fact, the
applications involved in this case do not allege any specific acts performed by herein petitioners. It would
be the legal heresy, of the highest order, to convict anybody of a "violation of Central Bank Laws, Tariff

Page
Section 1-C, SY 06-07

65

and Customs Laws, Internal Revenue (Code) and Revised Penal Code," as alleged in the
aforementioned applications without reference to any determinate provision of said laws or
__________________________

Hon. Jose W. Diokno, in his capacity as Secretary of Justice, Jose Lukban, in his capacity as Acting
Director, National Bureau of Investigation, Special Prosecutors Pedro D. Cenzon, Efren I. Plana and
Manuel Villareal, Jr. and Assistant Fiscal Maneses G. Reyes, City of Manila.
2
Hon. Amado Roan, Judge of the Municipal (now City) Court of Manila, Hon. Roman Cansino, Judge of
the Municipal (now City) Court of Manila, Hon. Hermogenes Caluag, Judge of the Court of First Instance
of Rizal, Quezon City Branch, Hon. Eulogio Mencias, Judge of the Court of First Instance of Rizal, Pasig
Branch, and Hon. Damian Jimenez, Judge of the Municipal (now City) Court of Quezon City.
3
Covering the period from March 3 to March 9, 1962.
4
Harry S. Stonehill, Robert P. Brooks, John J. Brooks and Karl Beck.
5
U.S. Tobacco Corporation, Atlas Cement Corporation, Atlas Development Corporation, Far East
Publishing Corporation (Evening News), Investment Inc., Industrial Business Management Corporation,
General Agricultural Corporation, American Asiatic Oil Corporation, Investment Management Corporation,
Holiday Hills, Inc., Republic Glass Corporation, Industrial and Business Management Corporation, United
Housing Corporation, The Philippine Tobacco-Flue-Curing and Redrying Corporation, Republic Real
Estate Corporation and Merconsel Corporation.
BURGOS, SR. V. CHIEF OF STAFF, AFP
[133 SCRA 800; G.R. NO. 64261; 26 DEC 1984]
Facts:
Petitioners assail the validity of 2 search warrants issued on December 7, 1982 by respondent Judge
Cruz-Pano of the then Court of First Instance of Rizal, under which the premises known as No. 19, Road
3, Project 6, Quezon City, and 784 Units C & D, RMS Building, Quezon Avenue, Quezon City, business
addresses of the "Metropolitan Mail" and "We Forum" newspapers, respectively, were searched, and
office and printing machines, equipment, paraphernalia, motor vehicles and other articles used in the
printing, publication and distribution of the said newspapers, as well as numerous papers, documents,
books and other written literature alleged to be in the possession and control of petitioner Jose Burgos, Jr.
publisher-editor of the "We Forum" newspaper, were seized. As a consequence of the search and
seizure, these premises were padlocked and sealed, with the further result that the printing and
publication of said newspapers were discontinued. Respondents contend that petitioners should have
filed a motion to quash said warrants in the court that issued them before impugning the validity of the
same before this Court. Respondents also assail the petition on ground of laches (Failure or negligence
for an unreasonable and unexplained length of time to do that which, by exercising due diligence, could or
should have been done earlier. It is negligence or omission to assert a right within a reasonable time,
warranting a presumption that the party entitled to assert it either has abandoned it or declined to assert
it). Respondents further state that since petitioner had already used as evidence some of the documents
seized in a prior criminal case, he is stopped from challenging the validity of the search warrants.
Petitioners submit the following reasons to nullify the questioned warrants:
1. Respondent Judge failed to conduct an examination under oath or affirmation of the applicant and
his witnesses, as mandated by the above-quoted constitutional provision as well as Sec. 4, Rule
126 of the Rules of Court.
2. The search warrants pinpointed only one address which would be the former abovementioned
address.
3. Articles belonging to his co-petitioners were also seized although the warrants were only directed
against Jose Burgos, Jr.
4. Real properties were seized.
5. The application along with a joint affidavit, upon which the warrants were issued, from the
Metrocom Intelligence and Security Group could not have provided sufficient basis for the finding
of a probable cause upon which a warrant may be validly issued in accordance with Section 3,
Article IV of the 1973 Constitution.
Respondents justify the continued sealing of the printing machines on the ground that they have been
sequestered under Section 8 of Presidential Decree No. 885, as amended, which authorizes
sequestration of the property of any person engaged in subversive activities against the government in
accordance with implementing rules and regulations as may be issued by the Secretary of National
Defense.
Issue:
Whether or Not the 2 search warrants were validly issued and executed.

Page
Section 1-C, SY 06-07

66

San Beda College of Law Alabang


Constitutional Law 2 Case Digests

Held:
In regard to the quashal of warrants that petitioners should have initially filed to the lower court, this Court
takes cognizance of this petition in view of the seriousness and urgency of the constitutional Issue raised,
not to mention the public interest generated by the search of the "We Forum" offices which was televised
in Channel 7 and widely publicized in all metropolitan dailies. The existence of this special circumstance
justifies this Court to exercise its inherent power to suspend its rules. With the contention pertaining to
laches, the petitioners gave an explanation evidencing that they have exhausted other extra-judicial
efforts to remedy the situation, negating the presumption that they have abandoned their right to the
possession of the seized property.
On the enumerated reasons:
1. This objection may properly be considered moot and academic, as petitioners themselves
conceded during the hearing on August 9, 1983, that an examination had indeed been conducted
by respondent judge of Col. Abadilla and his witnesses.
2. The defect pointed out is obviously a typographical error. Precisely, two search warrants were
applied for and issued because the purpose and intent were to search two distinct premises. It
would be quite absurd and illogical for respondent judge to have issued two warrants intended for
one and the same place.
3. Section 2, Rule 126, of the Rules of Court, does not require that the property to be seized should
be owned by the person against whom the search warrant is directed. It may or may not be
owned by him.
4. Petitioners do not claim to be the owners of the land and/or building on which the machineries
were placed. This being the case, the machineries in question, while in fact bolted to the ground,
remain movable property susceptible to seizure under a search warrant.
5. The broad statements in the application and joint affidavit are mere conclusions of law and does
not satisfy the requirements of probable cause. Deficient of such particulars as would justify a
finding of the existence of probable cause, said allegation cannot serve as basis for the issuance
of a search warrant and it was a grave error for respondent judge to have done so. In Alvarez v.
Court of First Instance, this Court ruled that "the oath required must refer to the truth of the facts
within the personal knowledge of the petitioner or his witnesses, because the purpose thereof is
to convince the committing magistrate, not the individual making the affidavit and seeking the
issuance of the warrant, of the existence of probable cause." Another factor which makes the
search warrants under consideration constitutionally objectionable is that they are in the nature of
general warrants. The description of the articles sought to be seized under the search warrants
in question are too general.
With regard to the respondents invoking PD 885, there is an absence of any implementing rules and
regulations promulgated by the Minister of National Defense. Furthermore, President Marcos himself
denies the request of military authorities to sequester the property seized from petitioners. The closure of
the premises subjected to search and seizure is contrary to the freedom of the press as guaranteed in our
fundamental law. The search warrants are declared null and void.
TAMBASEN VS. PEOPLE
[246 SCRA 184; G.R. NO. 89103; 14 JUL 1995]
Facts:
In August 1988, P/Sgt. Natuel applied for issuance of search warrant alleging that he received information
that Petitioner had in his possession at his house M-16 Armalite rifles, hand grenades, .45 Cal. pistols,
dynamite sticks and subversive documents, which were used or intended to be used for illegal
purposes. The application was granted.
In September, a police team, searched the house of petitioner and seized 2 envelopes containing
P14000, handset with antennae, transceiver with antennae, regulator supply, academy notebook and
assorted papers and handset battery pack. In October, petitioner moved that the search and seizure be
declared illegal and that the seized articles be returned to him. In December, MTCC, in its order, directed
Lt. Col. Torres to return the money seized to petitioner ruling that any seizure should be limited to the
specified items covered thereby. SolGen petitioned with the RTC for the annulment of the order of MTCC
citing that pending the determination of legality of seizure of the articles, they should remain in custogia
legis. RTC granted the petition.
Issue:
Whether or Not the seizure of the articles which were not mentioned in the search warrant was legal.

Page
Section 1-C, SY 06-07

67

Held:
Section 2 Article III of the 1987 Constitution requires that a search warrant should particularly describe the
things to be seized. The police acts beyond the parameters of their authority if they seize articles not
described in the search warrants. The evident purpose and intent of the requirement is to limit the things
to be seized, to leave the officers of the law with no discretion; that unreasonable search and seizure may
not be made and that abuses may not be committed.
Petition granted. People of the Philippines is ordered to return the money seized.
PLACER VS. JUDGE VILLANUEVA
[126 SCRA 463; G.R. NOS. L-60349-62; 29 DEC 1983]
Facts:
Petitioners filed informations in the city court and they certified that Preliminary Investigation and
Examination had been conducted and that prima facie cases have been found. Upon receipt of said
informations, respondent judge set the hearing of the criminal cases to determine propriety of issuance of
warrants of arrest. After the hearing, respondent issued an order requiring petitioners to submit to the
court affidavits of prosecution witnesses and other documentary evidence in support of the informations to
aid him in the exercise of his power of judicial review of the findings of probable cause by petitioners.
Petitioners petitioned for certiorari and mandamus to compel respondent to issue warrants of arrest.
They contended that the fiscals certification in the informations of the existence of probable cause
constitutes sufficient justification for the judge to issue warrants of arrest.
Issue:
Whether or Not respondent city judge may, for the purpose of issuing warrants of arrest, compel the fiscal
to submit to the court the supporting affidavits and other documentary evidence presented during the
preliminary investigation.
Held:
Judge may rely upon the fiscals certification for the existence of probable cause and on the basis thereof,
issue a warrant of arrest. But, such certification does not bind the judge to come out with the warrant.
The issuance of a warrant is not a mere ministerial function; it calls for the exercise of judicial discretion
on the part of issuing magistrate. Under Section 6 Rule 112 of the Rules of Court, the judge must satisfy
himself of the existence of probable cause before issuing a warrant of arrest. If on the face of the
information, the judge finds no probable cause, he may disregard the fiscals certification and require
submission of the affidavits of witnesses to aid him in arriving at the conclusion as to existence of
probable cause.
Petition dismissed.
SOLIVEN VS. MAKASIAR
[167 SCRA 393; G.R. NO. 82585; 14 NOV 1988]
Facts:
In these consolidated cases, three principal issues were raised: (1) whether or not petitioners were denied
due process when informations for libel were filed against them although the finding of the existence of a
prima facie case was still under review by the Secretary of Justice and, subsequently, by the President;
and (2) whether or not the constitutional rights of Beltran were violated when respondent RTC judge
issued a warrant for his arrest without personally examining the complainant and the witnesses, if any, to
determine probable cause. Subsequent events have rendered the first issue moot and academic. On
March 30, 1988, the Secretary of Justice denied petitioners' motion for reconsideration and upheld the
resolution of the Undersecretary of Justice sustaining the City Fiscal's finding of a prima facie case
against petitioners. A second motion for reconsideration filed by petitioner Beltran was denied by the
Secretary of Justice on April 7, 1988. On appeal, the President, through the Executive Secretary, affirmed
the resolution of the Secretary of Justice on May 2, 1988. The motion for reconsideration was denied by
the Executive Secretary on May 16, 1988. With these developments, petitioners' contention that they
have been denied the administrative remedies available under the law has lost factual support.
Issue:

Page
Section 1-C, SY 06-07

68

San Beda College of Law Alabang


Constitutional Law 2 Case Digests

Whether or Not petitioners were denied due process when informations for libel were filed against them
although the finding of the existence of a prima facie case was still under review by the Secretary of
Justice and, subsequently, by the President.
Whether or Not the constitutional rights of Beltran were violated when respondent RTC judge issued a
warrant for his arrest without personally examining the complainant and the witnesses, if any, to
determine probable cause
Held:
With respect to petitioner Beltran, the allegation of denial of due process of law in the preliminary
investigation is negated by the fact that instead of submitting his counter- affidavits, he filed a "Motion to
Declare Proceedings Closed," in effect waiving his right to refute the complaint by filing counter-affidavits.
Due process of law does not require that the respondent in a criminal case actually file his counteraffidavits before the preliminary investigation is deemed completed. All that is required is that the
respondent be given the opportunity to submit counter-affidavits if he is so minded.

The second issue, raised by petitioner Beltran, calls for an interpretation of the constitutional provision on
the issuance of warrants of arrest. The pertinent provision reads:
Art. III, Sec. 2. The right of the people to be secure in their persons, houses, papers and
effects against unreasonable searches and seizures of whatever nature and for any
purpose shall be inviolable, and no search warrant or warrant of arrest shall issue except
upon probable cause to be determined personally by the judge after examination nder
oath or affirmation of the complainant and the witnesses he may produce, and particularly
describing the place to be searched and the persons or things to be seized.
The addition of the word "personally" after the word "determined" and the deletion of the grant of authority
by the 1973 Constitution to issue warrants to "other responsible officers as may be authorized by law,"
has apparently convinced petitioner Beltran that the Constitution now requires the judge to personally
examine the complainant and his witnesses in his determination of probable cause for the issuance of
warrants of arrest. This is not an accurate interpretation.
What the Constitution underscores is the exclusive and personal responsibility of the issuing judge to
satisfy himself of the existence of probable cause. In satisfying himself of the existence of probable cause
for the issuance of a warrant of arrest, the judge is not required to personally examine the complainant
and his witnesses. Following established doctrine and procedure, he shall: (1) personally evaluate the
report and the supporting documents submitted by the fiscal regarding the existence of probable cause
and, on the basis thereof, issue a warrant of arrest; or (2) if on the basis thereof he finds no probable
cause, he may disregard the fiscal's report and require the submission of supporting affidavits of
witnesses to aid him in arriving at a conclusion as to the existence of probable cause.
Sound policy dictates this procedure, otherwise judges would be unduly laden with the preliminary
examination and investigation of criminal complaints instead of concentrating on hearing and deciding
cases filed before their courts. It has not been shown that respondent judge has deviated from the
prescribed procedure. Thus, with regard to the issuance of the warrants of arrest, a finding of grave abuse
of discretion amounting to lack or excess of jurisdiction cannot be sustained. The petitions fail to establish
that public respondents, through their separate acts, gravely abused their discretion as to amount to lack
of jurisdiction. Hence, the writs of certiorari and prohibition prayed for cannot issue.
WHEREFORE, finding no grave abuse of discretion amounting to excess or lack of jurisdiction on the part
of the public respondents, the Court Resolved to DISMISS the petitions in G. R. Nos. 82585, 82827 and
83979. The Order to maintain the status quo contained in the Resolution of the Court en banc dated April
7, 1988 and reiterated in the Resolution dated April 26, 1988 is LIFTED.

SALAZAR VS. ACHACOSO


[183 SCRA 145; G.R. NO. 81510; 14 MAR 1990]
Facts:

Page
Section 1-C, SY 06-07

69

Rosalie Tesoro of Pasay City in a sworn statement filed with the POEA, charged petitioner with illegal
recruitment. Public respondent Atty. Ferdinand Marquez sent petitioner a telegram directing him to appear
to the POEA regarding the complaint against him. On the same day, after knowing that petitioner had no
license to operate a recruitment agency, public respondent Administrator Tomas Achacoso issued a
Closure and Seizure Order No. 1205 to petitioner. It stated that there will a seizure of the documents and
paraphernalia being used or intended to be used as the means of committing illegal recruitment, it having
verified that petitioner has (1) No valid license or authority from the Department of Labor and
Employment to recruit and deploy workers for overseas employment; (2) Committed/are committing acts
prohibited under Article 34 of the New Labor Code in relation to Article 38 of the same code. A team was
then tasked to implement the said Order. The group, accompanied by mediamen and Mandaluyong
policemen, went to petitioners residence. They served the order to a certain Mrs. For a Salazar, who let
them in. The team confiscated assorted costumes. Petitioner filed with POEA a letter requesting for the
return of the seized properties, because she was not given prior notice and hearing. The said Order
violated due process. She also alleged that it violated sec 2 of the Bill of Rights, and the properties were
confiscated against her will and were done with unreasonable force and intimidation.
Issue:
Whether or Not the Philippine Overseas Employment Administration (or the Secretary of Labor) can
validly issue warrants of search and seizure (or arrest) under Article 38 of the Labor Code
Held:
Under the new Constitution, . . . no search warrant or warrant of arrest shall issue except upon probable
cause to be determined personally by the judge after examination under oath or affirmation of the
complainant and the witnesses he may produce, and particularly describing the place to be searched and
the persons or things to be seized. Mayors and prosecuting officers cannot issue warrants of seizure or
arrest. The Closure and Seizure Order was based on Article 38 of the Labor Code. The Supreme Court
held, We reiterate that the Secretary of Labor, not being a judge, may no longer issue search or arrest
warrants. Hence, the authorities must go through the judicial process. To that extent, we declare Article
38, paragraph (c), of the Labor Code, unconstitutional and of no force and effect The power of the
President to order the arrest of aliens for deportation is, obviously, exceptional. It (the power to order
arrests) cannot be made to extend to other cases, like the one at bar. Under the Constitution, it is the sole
domain of the courts. Furthermore, the search and seizure order was in the nature of a general warrant.
The court held that the warrant is null and void, because it must identify specifically the things to be
seized.

WHEREFORE, the petition is GRANTED. Article 38, paragraph (c) of the Labor Code is declared
UNCONSTITUTIONAL and null and void. The respondents are ORDERED to return all materials seized
as a result of the implementation of Search and Seizure Order No. 1205.
MORANO VS. VIVO
[20 SCRA 562; G.R. L-22196; 30 JUN 1967]
Facts:
Chan Sau Wah, a Chinese citizen born in Fukien, China arrived in the Philippines on November 1961 to
visit her cousin, Samuel Lee Malaps. She left China and her children by a first marriage: Fu Tse Haw and
Fu Yan Kai both minors, in the care of neighbors in Fukien, China. Chan Sau wah arrived in the
Philippines with Fu Yan Fun, her minor son also by the first marriage. Chan Sau Wah and her minor son
Fu Yan Fun were permitted only into the Philippines under a temporary visitor's visa for two months and
after they posted a cash bond of 4,000 pesos. On January 1962, Chan Sau Wah married Esteban
Morano, a native-born Filipino citizen. Born to this union on September 1962 was Esteban Morano, Jr. To
prolong their stay in the Philippines, Chan Sau Wah and Fu Yan Fun obtained several extensions. The
last extension expired on September 10, 1962. In a letter dated August 31, 1962, the Commissioner of
Immigration ordered Chan Sau Wah and her son, Fu Yan Fun, to leave the country on or before
September 10, 1962 with a warning that upon failure so to do, he will issue a warrant for their arrest and
will cause the confiscation of their bond.
Issue:
Whether or Not the issuance of the warrant of arrest is unconstitutional.
Held:

Page
Section 1-C, SY 06-07

70

San Beda College of Law Alabang


Constitutional Law 2 Case Digests

Chan Sau Wah entered the Philippines on a tourist-temporary visitor's visa. She is a non-immigrant.
Under Section 13 just quoted, she may therefore be admitted if she were a qualified and desirable alien
and subject to the provisions of the last paragraph of Section 9. Therefore, first, she must depart
voluntarily to some foreign country; second, she must procure from the appropriate consul the proper
visa; and third, she must thereafter undergo examination by the officials of the Bureau of Immigration at
the port of entry for determination of her admissibility in accordance with the requirements of the
immigration Act. This Court in a number of cases has ruled, and consistently too, that an alien admitted as
a temporary visitor cannot change his or her status without first departing from the country and complying
with the requirements of Section 9 of the Immigration Act. The gravamen of petitioners' argument is that
Chan Sau Wah has, since her entry, married in Manila a native-born Filipino, Esteban Morano. It will not
particularly help analysis for petitioners to appeal to family solidarity in an effort to thwart her deportation.
Chan Sau Wah, seemingly is not one who has a high regard for such solidarity. Proof: She left two of her
children by the first marriage, both minors, in the care of neighbors in Fukien, China.Then, the wording of
the statute heretofore adverted to is a forbidding obstacle which will prevent this Court from writing into
the law an additional provision that marriage of a temporary alien visitor to a Filipino would ipso facto
make her a permanent resident in his country. This is a field closed to judicial action. No breadth of
discretion is allowed. We cannot insulate her from the State's power of deportation. it would be an easy
matter for an alien woman to enter the Philippines as a temporary visitor, go through a mock marriage, but
actually live with another man as husband and wife, and thereby skirt the provisions of our immigration
law. Also, a woman of undesirable character may enter this country, ply a pernicious trade, marry a
Filipino, and again throw overboard Sections 9 and 13 of the Act. Such a flanking movement, we are
confident, is impermissible.Recently we confirmed the rule that an alien wife of a Filipino may not stay
permanently without first departing from the Philippines. Reason: Discourage entry under false pretenses.
HARVEY V. DEFENSOR-SANTIAGO
[162 SCRA 840; G.R. NO. 82544; 28 JUN 1988]
Facts:
This is a petition for Habeas Corpus. Petitioners are the following: American nationals Andrew Harvey, 52
and Jonh Sherman 72. Dutch Citizen Adriaan Van Den Elshout, 58. All reside at Pagsanjan Laguna
respondent Commissioner Miriam Defensor Santiago issued Mission Orders to the Commission of
Immigration and Deportation (CID) to apprehended petitioners at their residences.
The Operation
Report read that Andrew Harvey was found together with two young boys. Richard Sherman was found
with two naked boys inside his room. While Van Den Elshout in the after Mission Report read that two
children of ages 14 and 16 has been under his care and subjects confirmed being live-in for sometime
now.
Seized during the petitioners apprehension were rolls of photo negatives and photos of suspected child
prostitutes shown in scandalous poses as well as boys and girls engaged in sex. Posters and other
literature advertising the child prostitutes were also found.
Petitioners were among the 22 suspected alien pedophiles. They were apprehended 17 February1988
after close surveillance for 3 month of the CID in Pagsanjan, Laguna. 17 of the arrested aliens opted for
self-deportation. One released for lack of evidence, another charged not for pedophile but working with
NO VISA, the 3 petitioners chose to face deportation proceedings. On 4 March1988, deportation
proceedings were instituted against aliens for being undesirable aliens under Sec.69 of Revised
Administrative Code.
Warrants of Arrest were issued 7March1988 against petitioners for violation of Sec37, 45 and 46 of
Immigration Act and sec69 of Revised Administrative Code. Trial by the Board of Special Inquiry III
commenced the same date. Petition for bail was filed 11March 1988 but was not granted by the
Commissioner of Immigration. 4 April1988 Petitioners filed a petition for Writ of Habeas Corpus. The
court heard the case on oral argument on 20 April 1988.

Issue:
Whether or Not the Commissioner has the power to arrest and detain petitioners pending determination of
existence of probable cause.
Whether or Not there was unreasonable searches and seizures by CID agents.

Page
Section 1-C, SY 06-07

71

Whether or Not the writ of Habeas Corpus may be granted to petitioners.


Held:
While pedophilia is not a crime under the Revised Penal Code, it violates the declared policy of the state
to promote and protect the physical, moral, spiritual and social well being of the youth. The arrest of
petitioners was based on the probable cause determined after close surveillance of 3 months. The
existence of probable cause justified the arrest and seizure of articles linked to the offense. The articles
were seized as an incident to a lawful arrest; therefore the articles are admissible evidences (Rule 126,
Section12 of Rules on Criminal Procedure).
The rule that search and seizures must be supported by a valid warrant of arrest is not an absolute rule.
There are at least three exceptions to this rule. 1.) Search is incidental to the arrest. 2.) Search in a
moving vehicle. 3.) Seizure of evidence in plain view. In view of the foregoing, the search done was
incidental to the arrest.
The filing of the petitioners for bail is considered as a waiver of any irregularity attending their arrest and
estops them from questioning its validity. Furthermore, the deportation charges and the hearing presently
conducted by the Board of Special Inquiry made their detention legal. It is a fundamental rule that habeas
corpus will not be granted when confinement is or has become legal, although such confinement was
illegal at the beginning.
The deportation charges instituted by the Commissioner of Immigration are in accordance with Sec37 (a)
of the Philippine Immigration Act of 1940 in relation to sec69 of the Revised Administrative code. Section
37 (a) provides that aliens shall be arrested and deported upon warrant of the Commissioner of
Immigration and Deportation after a determination by the Board of Commissioners of the existence of a
ground for deportation against them. Deportation proceedings are administrative in character and never
construed as a punishment but a preventive measure. Therefore, it need not be conducted strictly in
accordance with ordinary Court proceedings. What is essential is that there should be a specific charge
against the alien intended to be arrested and deported. A fair hearing must also be conducted with
assistance of a counsel if desired.
Lastly, the power to deport aliens is an act of the State and done under the authority of the sovereign
power. It a police measure against the undesirable aliens whose continued presence in the country is
found to be injurious to the public good and tranquility of the people.

SALES VS. SANDIGANBAYAN


[369 SCRA 293 G.R. NO. 143802; 16 NOV 2001]
Facts:
The petitioner, the incumbent mayor of Pagudpud Ilocos Norte, shot the former mayor and his political
rival Atty. Benemerito. After the shooting, he surrendered himself and hence the police inspector and wife
of the victim filed a criminal complaint for murder against him. The judge after conducting the preliminary
examination (p.e. for brevity) found probable cause and issued a warrant of arrest. Also after conducting
the preliminary investigation (p.i. for brevity), he issued a resolution forwarding the case to the prosecutor
for appropriate action. Petitioner received a subpoena directing him to file his counter affidavit, affidavit of
witnesses and other supporting documents. He did it the following day. While proceedings are ongoing,
he filed a petition for habeas corpus with the C.A alleging that: the warrant was null and void because the
judge who issued it was a relative by affinity of the private respondent and the p.e. and the p.i. were illegal
and irregular as the judge doesnt have jurisdiction on the case. The C.A. granted the petition holding that
the judge was a relative by affinity by 3 rd degree to the private respondent and the p.i. he conducted has 2
stages, the p.e. and the p.i. proper. The proceeding now consists only of one stage. He conducted the
requisite investigation prior to the issuance of warrant of arrest. Moreover he did not complete it. He only
examined the witness of the complainant. But the prosecution instead of conducting p.i. of his own
forwarded the records to the Ombudsman (OMB for brevity) for the latter to conduct the same. The OMB
directed the petitioner to submit his counter affidavit, but he did not comply with it finding the same
superfluous. The graft investigator recommended the filing of information for murder which the OMB
approved. Petitioner received a copy of the resolution but prevented seeking reconsideration thereof he
filed a motion to defer issuance of warrant of arrest pending the determination of probable cause. The
Sandiganbayan denied the motion. This is now a petition for review on the decision of the
Sandiganbayan,
Issue:

Page
Section 1-C, SY 06-07

72

San Beda College of Law Alabang


Constitutional Law 2 Case Digests
Whether or Not the OMB followed the procedure in conducting preliminary investigation.
Whether or Not petitioner was afforded an opportunity to be heard and to submit controverting evidence.
Held:
The proper procedure in the conduct of preliminary investigation was not followed because of the
following reasons. Firstly, the preliminary investigation was conducted by 3 different investigators, none of
whom completed the preliminary investigation There was not one continuous proceeding but rather, cases
of passing the buck, the last one being the OMB throwing the buck to the Sandiganbayan. Secondly, the
charge of murder is a non bailable offense. The gravity of the offense alone should have merited a deeper
and more thorough preliminary investigation. The OMB did nothing of the sort but wallowed the resolution
of the graft investigator. He did a worse job than the judge, by actually adopting the resolution of the graft
investigator without doing anything and threw everything to the Sandiganbayan for evaluation. Thirdly, a
person under preliminary investigation by the OMB is entitled to a motion for reconsideration, as
maintained by the Rules of Procedure by the OMB. The filing of the motion for reconsideration is an
integral part of the preliminary investigation proper. The denial thereof is tantamount to the denial of the
right itself to a preliminary investigation. This fact alone renders preliminary investigation conducted in this
case incomplete. And lastly, it was patent error for the Sandiganbayan to have relied purely on the OMBs
certification of probable cause given the prevailing facts of the case much more so in the face of the
latters flawed report and one side factual findings.
The court cannot accept the Sandiganbayans assertion of having found probable cause on its own,
considering the OMBs defective report and findings, which merely rekied on the testimonies of the
witnesses for the prosecution and disregarded the evidence for the defense.
Judgment is rendered setting aside the resolution of the Sandiganbayan, ordering the Sandiganbayan to
quash the warrant of arrest and remanding the OMB for completion of the preliminary investigation.
SILVA VS. PRESIDING JUDGE
[203 SCRA 140; G.R. No. 81756; 21 Oct 1991]
Facts:
Sgt. Villamor, chief of the PC Narcom Detachment in Dumaguete City filed an "application for search
warrant" and "Deposition of witness" against petitioner Nicomedes Silva and Martin Silva. Judge Nickarter
Ontal, then the presiding judge of RTC of Dumaguete issued Search Warrant No.1 pursuant to the said
applications for violation of RA 6425 Dangerous Drugs ACT of 1972. Such warrant states that there is a
probable cause to believe that Mr. Tama Silva has the possession and control of marijuana dried leaves,
cigarette and joint. The warrant authorizes Sgt. Villamor to make an immediate search at any time of the
room of Mr. Tama Silva at the residence of his father Comedes Silva and to open aparadors, lockers,
cabinets, cartons and containers to look for said illegal drugs. In the course of the search, the officers
seized money belonging to Antonieta Silva in the amount of P1,231.40. Petitioner filed a motion to quash
Search Warrant No.1 on the ground that 1) it was issued on the sole basis of mimeographed 2) the judge
failed to personally examine the complainant and witness by searching questions and answers.
Issue:
Whether or Not Search Warrant No.1 is invalid. WON the officers abused their authority in seizing the
money of Antonieta Silva.
Held:
Search Warrant No. 1 is invalid due to the failure of the judge to examine the witness in the form of
searching questions and answers. The questions asked were leading as they are answerable by mere
yes or no. Such questions are not sufficiently searching to establish probable cause. The questions were
already mimeographed and all the witness had to do was fill in their answers on the blanks provided.
Judge Ontal is guilty of grave abuse of discretion when he rejected the motion of Antonieta Silva seeking
the return of her money.
The officers who implemented the search warrant clearly abused their authority when they seized the
money of Antonieta Silva. The warrant did not indicate the seizure of money but only for marijuana leaves,
cigarettes..etc. Search Warrant No. 1 is declared null and void.
*** Sec 4 Rule 126 Rules of Court
Examination of the complainant, record -the judge before issuing the warrant, personally examine in the

Page
Section 1-C, SY 06-07

73

form of searching questions and answers, in writing and under oath the complainant and any witness he
may produce the facts personally known to them and attach to the record their sworn statements together
with their affidavits.
VEROY VS. LAYAGUE
[210 SCRA 97; G.R. No. 95630; 18 Jun 1992]
Facts:
Petitioners are husband and wife who owned and formerly resided at No. 13 Isidro St., Skyline Village.
Catalunan Grande, Davao City. When petitioner Leopoldo Veroy was promoted to the position of Assistant
Administrator of the Social Security System sometime in June, 1988, he and his family transferred to 130
K-8th St., East Kamias, Quezon City, where they are presently residing. The care and upkeep of their
residence in Davao City was left to two (2) houseboys, Jimmy Favia and Eric Burgos, who had their
assigned quarters at a portion of the premises. The Veroys would occasionally send money to Edna
Soguilon for the salary of the said houseboys and other expenses for the upkeep of their house. While the
Veroys had the keys to the interior of the house, only the key to the kitchen, where the circuit breakers
were located, was entrusted to Edna Soguilon to give her access in case of an emergency. Hence, since
1988, the key to the master's bedroom as well as the keys to the children's rooms were retained by herein
Petitioners so that neither Edna Soguilon nor the caretakers could enter the house.
Police Officers had an information that the petitioners residence was being used as a safehouse of rebel
soldiers. They were able to enter the yard with the help of the caretakers but did not enter the house since
the owner was not present and they did not have a search warrant. Petitioner Ma. Luisa was contacted by
telephone in her Quezon City residence by Capt. Obrero to ask permission to search the house in Davao
City as it was reportedly being used as a hideout and recruitment center of rebel soldiers. Petitioner Ma.
Luisa Veroy responded that she is flying to Davao City to witness the search but relented if the search
would not be conducted in the presence of Major Ernesto Macasaet, an officer of the PC/INP, Davao City
and a long time family friend of the Veroys.
The following day, Capt. Obrero and Major Macasaet met at the house of herein petitioners in Skyline
Village to conduct the search pursuant to the authority granted by petitioner Ma. Luisa Veroy. The
caretakers facilitated their entry into the yard, and using the key entrusted to Edna Soguilon, they were
able to gain entrance into the kitchen. However, a locksmith by the name of George Badiang had to be
employed to open the padlock of the door leading to the children's room. Capt. Obrero and Major
Macasaet then entered the children's room and conducted the search. Capt. Obrero recovered a .45 cal.
handgun with a magazine containing seven (7) live bullets in a black clutch bag inside an unlocked
drawer. Three (3) half-full jute sacks containing printed materials of RAM-SFP were also found in the
children's room. A search of the children's recreation and study area revealed a big travelling bag
containing assorted polo shirts, men's brief, two (2) pieces polo barong and short sleeve striped gray
polo. sweat shirt, two (2) pairs men's socks, a towel made in U.S.A., one blanket, a small black bag,
Gandhi brand, containing a book entitled "Islamic Revolution Future Path of the Nation", a road map of
the Philippines, a telescope, a plastic bag containing assorted medicines and religious pamphlets was
found in the master's bedroom. Sgt. Leo Justalero was instructed by Capt. Obrero to make an inventory
and receipt of the articles seized, in the house.
The case was referred for preliminary investigation to Quezon City Assistant Prosecutor Rodolfo
Ponferrada who was designated Acting Provincial Prosecutor for Davao City. In a resolution dated August
6, 1990, Fiscal Ponferrada recommended the filing of an information against herein petitioners for
Violation of Presidential Decree No. 1866 (Illegal Possession of Firearms and Ammunitions in
Furtherance of Rebellion). No bail was recommended.
Issue:
Whether or Not Presidential Decree No. 1866, or at least the third paragraph of Section 1 thereof, is
unconstitutional for being violative of the due process and equal protection clauses of the Constitution.
Held:
The issue of constitutionality of Presidential Decree No. 1866 has been laid to rest in the case of Misolas
v. Panga, G.R. No. 83341, January 30, 1990 (181 SCRA 648), where this Court held that the declaration
of unconstitutionality of the third paragraph of Section 1 of Presidential Decree No. 1866 is wanting in
legal basis since it is neither a bill of attainder nor does it provide a possibility of a double jeopardy.
Petitioners' contention that Republic Act 6968 has repealed Presidential Decree No. 1866 is bereft of
merit. It is a cardinal rule of statutory construction that where the words and phrases of a statute are not

Page
Section 1-C, SY 06-07

74

San Beda College of Law Alabang


Constitutional Law 2 Case Digests
obscure or ambiguous. its meaning and the intention of the legislature must be determined from the
language employed, and where there is no ambiguity in the words, there is no room for construction.
Petitioners contend that Section 1 of Presidential Decree No. 1866 is couched in general or vague terms.
The terms "deal in", "acquire", "dispose" or "possess" are capable of various interpretations such that
there is no definiteness as to whether or not the definition includes "constructive possession" or how the
concept of constructive possession should be applied. Petitioners were not found in actual possession of
the firearm and ammunitions. They were in Quezon City while the prohibited articles were found in Davao
City. Yet they were being charged under Presidential Decree No. 1866 upon the sole circumstance that
the house wherein the items were found belongs to them.

Petitioners question the admissibility in evidence of the articles seized in violation of their constitutional
right against unreasonable search and seizure. Petitioners aver that while they concede that Capt. Obrero
had permission from Ma. Luisa Veroy to break open the door of their residence, it was merely for the
purpose of ascertaining thereat the presence of the alleged "rebel" soldiers. The permission did not
include any authority to conduct a room to room search once inside the house. The items taken were,
therefore, products of an illegal search, violative of their constitutional rights As such, they are
inadmissible in evidence against them.
The Constitution guarantees the right of the people to be secure in their persons, houses, papers and
effects against unreasonable searches and seizures (Article III, Section 2 of the 1987 Constitution).
However, the rule that searches and seizures must be supported by a valid warrant is not an absolute
one. Among the recognized exceptions thereto are: (1) a search incidental to an arrest; (2) a search of a
moving vehicle; and (3) seizure of evidence in plain view (People v. Lo Ho Wing, G.R. No. 88017, January
21, 1991 [193 SCRA 122]).
None of these exceptions pertains to the case at bar. The reason for searching the house of herein
petitioners is that it was reportedly being used as a hideout and recruitment center for rebel soldiers.
While Capt. Obrero was able to enter the compound, he did not enter the house because he did not have
a search warrant and the owners were not present. This shows that he himself recognized the need for a
search warrant, hence, he did not persist in entering the house but rather contacted the Veroys to seek
permission to enter the same. Permission was indeed granted by Ma. Luisa Veroy to enter the house but
only to ascertain the presence of rebel soldiers. Under the circumstances it is undeniable that the police
officers had ample time to procure a search warrant but did not.
Undeniably, the offense of illegal possession of firearms is malum prohibitum but it does not follow that
the subject thereof is necessarily illegal per se. Motive is immaterial in mala prohibita but the subjects of
this kind of offense may not be summarily seized simply because they are prohibited. A search warrant is
still necessary. Hence, the rule having been violated and no exception being applicable, the articles
seized were confiscated illegally and are therefore protected by the exclusionary principle. They cannot
be used as evidence against the petitioners in the criminal action against them for illegal possession of
firearms. (Roan v. Gonzales, 145 SCRA 689-690 [1986]). Besides, assuming that there was indeed a
search warrant, still in mala prohibita, while there is no need of criminal intent, there must be knowledge
that the same existed. Without the knowledge or voluntariness there is no crime.
PREMISES CONSIDERED, the petition as granted and the criminal case against the petitioners for illegal
possession of firearms is DISMISSED.

PEOPLE VS. DEL ROSARIO


[234 SCRA 246; G.R. NO. 109633; 20 JUL 1994]
Facts:
Accused was charged and convicted by the trial court of illegal possession of firearms and illegal
possession and sale of drugs, particularly methamphetamine or shabu. After the issuance of the search
warrant, which authorized the search and seizure of an undetermined quantity of methamphetamine and
its paraphernalias, an entrapment was planned that led to the arrest of del Rosario and to the seizure of
the shabu, its paraphernalias and of a .22 caliber pistol with 3 live ammunition.
Issue:

Page
Section 1-C, SY 06-07

75

Whether or Not the seizure of the firearms was proper.


Held:
No. Sec 2 art. III of the constitution specifically provides that a search warrant must particularly describe
the things to be seized. In herein case, the only objects to be seized that the warrant determined was the
methamphetamine and the paraphernalias therein. The seizure of the firearms was unconstitutional.
Wherefore the decision is reversed and the accused is acquitted.
PEOPLE VS. GESMUNDO
[219 SCRA 743; G.R. NO. 89373; 19 MAR 1993]
Facts:
According to the prosecution, in the morning of Nov. 17, 1986, PO Jose Luciano gave money and
instructed his civilian informer to buy marijuana from the accused at the Cocoland Hotel. He actually saw
the accused selling marijuana to his civilian informer and that same day Luciano applied for a search
warrant.
About 2pm that day, a police raiding team armed with a search warrant went to the Brgy captain for them
to be accompanied in serving the said warrant at the residence of the accused. The police was allowed to
enter the house upon the strength of the warrant shown to the accused. The accused begged the police
not to search and to leave the house. The police still searched the house and was led to the kitchen. She
pointed a metal basin on top of a table as the hiding place of died marijuana flowering tops contained in a
plastic bag marked ISETANN. The police also recovered from a native uway cabinet dried marijuana
flowering tops wrapped in 3 pieces of komiks paper.
According to the accused, when the police arrived at her house, she saw Sgt. Yte and PFC Jose Luciano.
She invited Sgt. Yte to enter her house while Luciano was left in the jeep that was parked near the house.
While inside the house Yte showed the accused something he claimed as a search warrant, when
someone coming from the kitchen uttered eto na They proceeded to the kitchen and saw Luciano
holding a plastic bag with four other companions. They confronted the accused and insisted that the bags
belonged to her. Accused denied the accusation and told them that she doesnt know anything about it.
She was made to sign a prepared document. She was brought to the police station and was detained.
The court renders judgment finding the accused guilty.
Issue:
Whether or Not the evidence was properly obtained by the police.
Held:
In the investigation report prepared by Luciano stated that during the search they discovered a hole at the
backyard of the house of the suspect, there was a big biscuit can inside the hole and on top of the cover a
flower pot was placed wherein the marijuana was kept. However, there was no mention of any marijuana
obtained from a flower pot in any of their testimonies. There were inconsistencies insofar the prosecution
is concerned, as to what was recovered and where, the trial court concluded that these inconsistencies
are trivial. There must sufficient evidence that the marijuana was actually surrendered by the accused. As
held in PP vs. Remorosa, Irreconcilable and unexplained contradictions in the testimonies of the
prosecution witnesses cast doubt on the guilt of appellant and his culpability to the crime charged.
The claim that the marijuana was planted was strengthen as the police violated sec 7, rule 126 rules of
the court provides no search of a house, room or any other premise shall be made except in the presence
of the lawful occupant thereof or any member of his family or in the absence of the latter, in the presence
of two (2) witnesses of sufficient age and discretion residing in the same locality. This requirement is
mandatory to ensure regularity in the execution of the search warrant. Violation of said rule is in fact
punishable under Article 130 of the Revised Penal Code.

The document (PAGPAPATUNAY) was inadmissible to the court as the accused was not informed of her
right not to sign the document neither was she informed that she has the right to the assistance of a
counsel and the fact that it may be used as evidence against her. It was not proved that the marijuana
belonged to her. Not only does the law require the presence of witnesses when the search is conducted,

Page
Section 1-C, SY 06-07

76

San Beda College of Law Alabang


Constitutional Law 2 Case Digests
but it also imposes upon the person making the search the duty to issue a detailed receipt for the property
seized. He is likewise required to deliver the property seized to the judge who issued the warrant,
together with a true and accurate inventory thereof duly verified under oath. Again, these duties are
mandatory and are required to preclude substitution of the items seized by interested parties.
The guilt of the accused was has not been established. Judgment is reversed.

UMIL VS. RAMOS


[187 SCRA 311; G.R. NO. 81567; 3 OCT 1991]
Facts:
On 1 February 1988, military agents were dispatched to the St. Agnes Hospital, Roosevelt Avenue,
Quezon City, to verify a confidential information which was received by their office, about a "sparrow man"
(NPA member) who had been admitted to the said hospital with a gunshot wound. That the wounded man
in the said hospital was among the five (5) male "sparrows" who murdered two (2) Capcom mobile patrols
the day before, or on 31 January 1988 at about 12:00 o'clock noon, before a road hump along
Macanining St., Bagong Barrio, Caloocan City. The wounded man's name was listed by the hospital
management as "Ronnie Javellon," twenty-two (22) years old of Block 10, Lot 4, South City Homes,
Bian, Laguna however it was disclosed later that the true name of the wounded man was Rolando Dural.
In view of this verification, Rolando Dural was transferred to the Regional Medical Servicesof the
CAPCOM, for security reasons. While confined thereat, he was positively identified by the eyewitnesses
as the one who murdered the 2 CAPCOM mobile patrols.
Issue:
Whether or Not Rolando was lawfully arrested.
Held:
Rolando Dural was arrested for being a member of the NPA, an outlawed subversive organization.
Subversion being a continuing offense, the arrest without warrant is justified as it can be said that he was
committing as offense when arrested. The crimes rebellion, subversion, conspiracy or proposal to commit
such crimes, and crimes or offenses committed in furtherance therefore in connection therewith constitute
direct assaults against the state and are in the nature of continuing crimes.
PEOPLE VS. SUCRO
[195 SCRA 388; G.R. No. 93239; 18 Mar 1991]
Facts:
Pat. Fulgencio went to Arlie Regalados house at C. Quimpo to monitor activities of Edison SUCRO
(accused). Sucro was reported to be selling marijuana at a chapel 2 meters away from Regalados
house. Sucro was monitored to have talked and exchanged things three times. These activities are
reported through radio to P/Lt. Seraspi. A third buyer was transacting with appellant and was reported
and later identified as Ronnie Macabante. From that moment, P/Lt.Seraspi proceeded to the area. While
the police officers were at the Youth Hostel in Maagama St. Fulgencio told Lt. Seraspi to intercept.
Macabante was intercepted at Mabini and Maagama crossing in front of Aklan Medical center.
Macabante saw the police and threw a tea bag of marijuana on the ground. Macabante admitted buying
the marijuana from Sucro in front of the chapel.
The police team intercepted and arrested SUCRO at the corner of C. Quimpo and Veterans. Recovered
were 19 sticks and 4 teabags of marijuana from a cart inside the chapel and another teabag from
Macabante.
Issue:
Whether or Not arrest without warrant is lawful.
Whether or Not evidence from such arrest is admissible.
Held:

Page
Section 1-C, SY 06-07

77

Search and seizures supported by a valid warrant of arrest is not an absolute rule. Rule 126, Sec 12 of
Rules of Criminal Procedure provides that a person lawfully arrested may be searched for dangerous
weapons or anything, which may be used as proff of the commission of an offense, without a search
warrant.(People v. Castiller) The failure of the police officers to secure a warrant stems from the fact that
their knowledge required from the surveillance was insufficient to fulfill requirements for its issuance.
However, warantless search and seizures are legal as long as PROBABLE CAUSE existed. The police
officers have personal knowledge of the actual commission of the crime from the surveillance of the
activities of the accused. As police officers were the ones conducting the surveillance, it is presumed that
they are regularly in performance of their duties.
PEOPLE V. RODRIGUEZA
[205 SCRA 791; G.R. No. 95902; 4 Feb 1992]
Facts:
NARCOM agents staged a buy-bust operation, after gaining information that there was an ongoing illegal
traffic of prohibited drugs in Tagas, Albay. The participating agents were given money treated with
ultraviolet powder. One of the agents went to said location, asked for a certain Don. Thereafter, the Don,
herein accused, met with him and a certain object wrapped in a plastic later identified as marijuana was
given in exchange for P200. The agent went back to headquarters and made a report, based on which, a
team was subsequently organized and a raid was conducted in the house of the father of the accused.
During the raid, the NARCOM agents were able to confiscate dried marijuana leaves and a plastic syringe
among others. There was no authorization by any search warrant. The accused was found positive of
ultraviolet powder. The lower court, considering the evidences obtained and testimonies from the
prosecution, found him guilty of violating the Dangerous Drugs Act of 1972 and sentenced him to
reclusion perpetua.
Issue:
Whether or Not the lower court was correct in its judgment.

Held:
The NARCOM agents procedure in the entrapment of the accused failed to meet the qualification that the
suspected drug dealer must be caught red-handed in the act of selling marijuana to a person posing as a
buyer, since the operation was conducted after the actual exchange. Said raid also violated accused
right against unreasonable search and seizure, as the situation did not fall in the circumstances wherein a
search may be validly made even without a search warrant, i.e. when the search is incidental to a lawful
arrest; when it involves prohibited articles in plain view. The NARCOM agents could not have justified
their act by invoking the urgency and necessity of the situation because the testimonies of the prosecution
witnesses reveal that the place had already been put under surveillance for quite some time. Had it been
their intention to conduct the raid, then they should, because they easily could, have first secured a
search warrant during that time. The Court further notes the confusion and ambiguity in the identification
of the confiscated marijuana leaves and other prohibited drug paraphernalia presented as evidence
against appellant:
CIC Taduran, who acted as the poseur buyer, testified that appellant sold him 100 grams of dried
marijuana leaves wrapped in a plastic bag. Surprisingly, and no plausible explanation has been advanced
therefor, what were submitted to and examined by the PCCL and thereafter utilized as evidence against
the appellant were the following items:
One (1) red and white colored plastic bag containing the following:
Exh. "A"Thirty (30) grams of suspected dried marijuana fruiting tops contained inside a
transparent plastic bag.
Exh. "B" Fifty (50) grams of suspected dried marijuana leaves and seeds contained
inside a white colored plastic labelled "Robertson".
Exh. "C" Four (4) aluminum foils each containing suspected dried marijuana fruiting
tops having a total weight of seven grams then further wrapped
with a piece of aluminum foil.
Exh. "D" Five (5) small transparent plastic bags each containing suspected dried
marijuana fruiting tops having a total weight of seventeen grams.
Exh. "E" One plastic syringe.

Page
Section 1-C, SY 06-07

78

San Beda College of Law Alabang


Constitutional Law 2 Case Digests

Evidently, these prohibited articles were among those confiscated during the so-called follow-up raid in
the house of Rodriguezas father. The unanswered question then arises as to the identity of the marijuana
leaves that became the basis of appellant's conviction. In People vs. Rubio, this Court had the occasion
to rule that the plastic bag and the dried marijuana leaves contained therein constitute the corpus delicti of
the crime. As such, the existence thereof must be proved with certainty and conclusiveness. Failure to do
so would be fatal to the cause of the prosecution. Conviction is reversed and set aside and accused is
acquitted.

PEOPLE VS. SY CHUA


[396 SCRA 657; G.R. No.136066-67; 4 Feb 2003]
Facts:
Accused-appellant Binad Sy Chua was charged with violation of Section 16, Article III of R.A. 6425, as
amended by R.A. 7659, and for Illegal Possession of Ammunitions and Illegal Possession of Drugs in two
separate Informations.
SPO2 Nulud and PO2 Nunag received a report from their confidential informant that accused-appellant
was about to deliver drugs that night at the Thunder Inn Hotel in Balibago, Angeles City. So, the PNP
Chief formed a team of operatives. The group positioned themselves across McArthur Highway near Bali
Hai Restaurant, fronting the hotel. The other group acted as their back up.
Afterwards, their informer pointed to a car driven by accused-appellant which just arrived and parked near
the entrance of the hotel. After accused-appellant alighted from the car carrying a sealed Zest-O juice
box, SPO2 Nulud and PO2 Nunag hurriedly accosted him and introduced themselves as police officers.
As accused-appellant pulled out his wallet, a small transparent plastic bag with a crystalline substance
protruded from his right back pocket. Forthwith, SPO2 Nulud subjected him to a body search which
yielded twenty (20) pieces of live .22 caliber firearm bullets from his left back pocket. When SPO2 Nunag
peeked into the contents of the Zest-O box, he saw that it contained a crystalline substance. SPO2 Nulud
instantly confiscated the small transparent plastic bag, the Zest-O juice box, the twenty (20) pieces of .22
caliber firearm bullets and the car used by accused-appellant. SPO2 Nulud and the other police
operatives who arrived at the scene brought the confiscated items to the office of Col. Guttierez at the
PNP Headquarters in Camp Pepito, Angeles City.
Accused-appellant vehemently denied the accusation against him and narrated a different version of the
incident.
Accused-appellant alleged that he was driving the car of his wife to follow her and his son to Manila. He
felt sleepy, so he decided to take the old route along McArthur Highway. He stopped in front of a small
store near Thunder Inn Hotel to buy cigarettes and candies. While at the store, he noticed a man
approaches and examines the inside of his car. When he called the attention of the onlooker, the man
immediately pulled out a .45 caliber gun and made him face his car with raised hands. The man later on
identified himself as a policeman. During the course of the arrest, the policeman took out his wallet and
instructed him to open his car. He refused, so the policeman took his car keys and proceeded to search
his car. At this time, the police officers companions arrived at the scene in two cars. PO2 Nulud, who just
arrived at the scene, pulled him away from his car in a nearby bank, while the others searched his car.
Thereafter, he was brought to a police station and was held inside a bathroom for about fifteen minutes
until Col. Guttierez arrived, who ordered his men to call the media. In the presence of reporters, Col.
Guttierez opened the box and accused-appellant was made to hold the box while pictures were being
taken.
The lower court acquitted Sy Chua for the Illegal Possession of Ammunitions, yet convicted him for Illegal
Possession of 1,955.815 grams of shabu. Hence, this appeal to the Court.
Issue:
Whether or Not the arrest of accused-appellant was lawful; and (2) WON the search of his person and the
subsequent confiscation of shabu allegedly found on him were conducted in a lawful and valid manner.
Held:

Page
Section 1-C, SY 06-07

79

The lower court believed that since the police received information that the accused will distribute illegal
drugs that evening at the Thunder Inn Hotel and its vicinities. The police officer had to act quickly and
there was no more time to secure a search warrant. The search is valid being akin to a stop and frisk.
The trial court confused the concepts of a stop-and-frisk and of a search incidental to a lawful arrest.
These two types of warrantless searches differ in terms of the requisite quantum of proof before they may
be validly effected and in their allowable scope.
In a search incidental to a lawful arrest, as the precedent arrest determines the validity of the incidental
search, the legality of the arrest is questioned, e.g., whether an arrest was merely used as a pretext for
conducting a search. In this instance, the law requires that there first be arrest before a search can be
madethe process cannot be reversed. Accordingly, for this exception to apply, two elements must
concur: (1) the person to be arrested must execute an overt act indicating that he has just committed, is
actually committing, or is attempting to commit a crime; and (2) such overt act is done in the presence or
within the view of the arresting officer.
We find the two aforementioned elements lacking in the case at bar. Accused-appellant did not act in a
suspicious manner. For all intents and purposes, there was no overt manifestation that accused-appellant
has just committed, is actually committing, or is attempting to commit a crime. Reliable information
alone, absent any overt act indicative of a felonious enterprise in the presence and within the view of the
arresting officers, is not sufficient to constitute probable cause that would justify an in flagrante delicto
arrest.
With regard to the concept of stop-and frisk: mere suspicion or a hunch will not validate a stop-andfrisk. A genuine reason must exist, in light of the police officers experience and surrounding conditions,
to warrant the belief that the person detained has weapons concealed about him. Finally, a stop-andfrisk serves a two-fold interest: (1) the general interest of effective crime prevention and detection for
purposes of investigating possible criminal behavior even without probable cause; and (2) the interest of
safety and self-preservation which permit the police officer to take steps to assure himself that the person
with whom he deals is not armed with a deadly weapon that could unexpectedly and fatally be used
against the police officer.
A stop-and-frisk was defined as the act of a police officer to stop a citizen on the street, interrogate him,
and pat him for weapon(s) or contraband. It should also be emphasized that a search and seizure should
precede the arrest for this principle to apply. The foregoing circumstances do not obtain in the case at bar.
To reiterate, accused-appellant was first arrested before the search and seizure of the alleged illegal
items found in his possession. The apprehending police operative failed to make any initial inquiry into
accused-appellants business in the vicinity or the contents of the Zest-O juice box he was carrying. The
apprehending police officers only introduced themselves when they already had custody of accusedappellant.
In the case at bar, neither the in flagrante delicto nor the stop and frisk principles is applicable to justify
the warrantless arrest and consequent search and seizure made by the police operatives on accusedappellant.
Wherefore, accused-appellant Binad Sy Chua is hereby Acquitted.
GO VS. COURT OF APPEALS
[206 SCRA 138; G.R. NO. 101837; 11 FEB 1992]
Facts:
Petitioner, while traveling in the wrong direction on a one-way street, almost had a collision with another
vehicle. Petitioner thereafter got out of his car, shot the driver of the other vehicle, and drove off. An
eyewitness of the incident was able to take down petitioners plate number and reported the same to the
police, who subsequently ordered a manhunt for petitioner. 6 days after the shooting, petitioner
presented himself in the police station, accompanied by 2 lawyers, the police detained him. Subsequently
a criminal charge was brought against him. Petitioner posted bail, the prosecutor filed the case to the
lower court, setting and commencing trial without preliminary investigation. Prosecutor reasons that the
petitioner has waived his right to preliminary investigation as bail has been posted and that such situation,
that petitioner has been arrested without a warrant lawfully, falls under Section 5, Rule 113 and Section 7,
Rule 112 of The 1985 Rules of Criminal Procedure which provides for the rules and procedure pertaining
to situations of lawful warrantless arrests. Petitioner in his petition for certiorari assails such procedure
and actions undertaken and files for a preliminary investigation.
Issue:

Page
Section 1-C, SY 06-07

80

San Beda College of Law Alabang


Constitutional Law 2 Case Digests

Whether or Not warrantless arrest of petitioner was lawful.


Whether or Not petitioner effectively waived his right to preliminary investigation.
Held:
Petitioner and prosecutor err in relying on Umil v. Ramos, wherein the Court upheld the warrantless arrest
as valid effected 1 to 14 days from actual commission of the offenses, which however constituted
continuing crimes, i.e. subversion, membership in an outlawed organization, etc. There was no lawful
warrantless arrest under Section 5, Rule 113. This is because the arresting officers were not actually
there during the incident, thus they had no personal knowledge and their information regarding petitioner
were derived from other sources. Further, Section 7, Rule 112, does not apply.
Petitioner was not arrested at all, as when he walked in the police station, he neither expressed surrender
nor any statement that he was or was not guilty of any crime. When a complaint was filed to the
prosecutor, preliminary investigation should have been scheduled to determine probable cause.
Prosecutor made a substantive error, petitioner is entitled to preliminary investigation, necessarily in a
criminal charge, where the same is required appear thereat. Petition granted, prosecutor is ordered to
conduct preliminary investigation, trial for the criminal case is suspended pending result from preliminary
investigation, petitioner is ordered released upon posting a bail bond.
CALLANTA VS. VILLANUEVA
[77 SCRA 377; G.R. NOS. 24646 & L-24674; 20 JUN 1977]
Facts:
Two complaints for grave oral defamation were filed against Faustina Callanta. The City Judge of
Dagupan City, Felipe Villanueva, denied the motions to quash the complaints. Thus, petitioner Callanta
brought the suits for certiorari in the Supreme Court. Petitioner questions the validity of the issuance of
warrant of arrest by respondent, arguing that the City Fiscal should have conducted the preliminary
investigation. According to petitioners counsel, there was jurisdictional infirmity. After the issuance of the
warrants of arrest and the bail fixed at P600, petitioner posted the bail bond, thus obtaining her
provisional liberty. The City Fiscal in this case did not disagree with the judges investigation, and agreed
with the complaints filed.
Issue:
Whether or Not petitioners contentions are to be given merit.
Held:
Based on many precedent cases of the Supreme Court, where the accused has filed bail and waived the
preliminary investigation proper, he has waived whatever defect, if any, in the preliminary examination
conducted prior to the issuance of the warrant of arrest. In the case at bar, it is futile for the petitioner to
question the validity of the issuance of the warrant of arrest, because she posted the bail bond. Petitioner
also erred in arguing that only the City Fiscal can conduct a preliminary investigation. According to the
Charter of the City of Dagupan, the City Court of Dagupan City may also conduct preliminary
investigation for any offense, without regard to the limits of punishment, and may release, or commit and
bind over any person charged with such offense to secure his appearance before the proper court.
Petition for certiorari is denied. Restraining order issued by the Court is lifted and set aside.

POSADAS VS. COURT OF APPEALS


[188 SCRA 288; G.R. NO. 89139; 2 AUG 1990]
Facts:
Members of the Integrated National Police (INP) of the Davao Metrodiscom assigned with the Intelligence
Task Force, Pat. Ursicio Ungab and Pat. Umbra Umpar conducted surveillance along Magallanes Street,
Davao City. While in the vicinity of Rizal Memorial Colleges they spotted petitioner carrying a "buri" bag

Page
Section 1-C, SY 06-07

81

and they noticed him to be acting suspiciously. They approached the petitioner and identified themselves
as members of the INP. Petitioner attempted to flee but his attempt to get away was unsuccessful. They
then checked the "buri" bag of the petitioner where they found one (1) caliber .38 Smith & Wesson
revolver with Serial No. 770196, two (2) rounds of live ammunition for a .38 caliber gun, a smoke (tear
gas) grenade, and two (2) live ammunitions for a .22 caliber gun. They brought the petitioner to the police
station for further investigation. In the course of the same, the petitioner was asked to show the necessary
license or authority to possess firearms and ammunitions found in his possession but he failed to do so.
He was then taken to the Davao Metrodiscom office and the prohibited articles recovered from him were
indorsed to M/Sgt. Didoy the officer then on duty. He was prosecuted for illegal possession of firearms
and ammunitions in the Regional Trial Court of Davao City.
Issue:
Whether or Not the warantless search is valid.
Held:
In justifying the warrantless search of the buri bag then carried by the petitioner, argues that under
Section 12, Rule 136 of the Rules of Court a person lawfully arrested may be searched for dangerous
weapons or anything used as proof of a commission of an offense without a search warrant. It is further
alleged that the arrest without a warrant of the petitioner was lawful under the circumstances.
in the case at bar, there is no question that, indeed, it is reasonable considering that it was effected on the
basis of a probable cause. The probable cause is that when the petitioner acted suspiciously and
attempted to flee with the buri bag there was a probable cause that he was concealing something illegal
in the bag and it was the right and duty of the police officers to inspect the same.
It is too much indeed to require the police officers to search the bag in the possession of the petitioner
only after they shall have obtained a search warrant for the purpose. Such an exercise may prove to be
useless, futile and much too late.
Clearly, the search in the case at bar can be sustained under the exceptions heretofore discussed, and
hence, the constitutional guarantee against unreasonable searches and seizures has not been violated.

PEOPLE V. MENGOTE
[210 SCRA 174; G.R. NO. 87059; 22 JUN 1992]
Facts:
The Western Police District received a telephone call from an informer that there were three suspicious
looking persons at the corner of Juan Luna and North Bay Boulevard in Tondo, Manila. A surveillance
team of plainclothesmen was forthwith dispatched to the place. The patrolmen saw two men looking from
side to side, one of whom holding his abdomen. They approached the persons and identified themselves
as policemen, whereupon the two tried to run but unable to escape because the other lawmen
surrounded them. The suspects were then searched. One of them the accused-appellant was found with
a .38 caliber with live ammunitions in it, while his companion had a fan knife. The weapons were taken
from them and they were turned over to the police headquarters for investigation. An information was
filed before the RTC convicting the accused of illegal possession of firearm arm. A witness testified that
the weapon was among the articles stolen at his shop, which he reported to the police including the
revolver. For his part, Mengote made no effort to prove that he owned the fire arm or that he was licensed
to possess it but instead, he claimed that the weapon was planted on him at the time of his arrest. He was
convicted for violation of P.D.1866 and was sentenced to reclusion perpetua. In his appeal he pleads that
the weapon was not admissible as evidence against him because it had been illegally seized and
therefore the fruit of a poisonous tree.
Issue:
Whether or not the warrantless search and arrest was illegal.
Held:
An evidence obtained as a result of an illegal search and seizure inadmissible in any proceeding for any
purpose as provided by Art. III sec 32 of the Constitution. Rule 113 sec.5 of the Rules of Court, provides
arrest without warrant lawful when: (a) the person to be arrested has committed, is actually committing, or

Page
Section 1-C, SY 06-07

82

San Beda College of Law Alabang


Constitutional Law 2 Case Digests
is attempting to commit an offense, (b) when the offense in fact has just been committed, and he has
personal knowledge of the facts indicating the person arrested has committed it and (c) the person to be
arrested has escaped from a penal establishment or a place where he is serving final judgment or
temporarily confined while his case is pending, or has escaped while being transferred from one
confinement to another.
These requirements have not been established in the case at bar. At the time of the arrest in question, the
accused appellant was merely looking from side to side and holding his abdomen, according to the
arresting officers themselves. There was apparently no offense that has just been committed or was being
actually committed or at least being attempt by Mengote in their presence. Moreover a person may not be
stopped and frisked in a broad daylight or on a busy street on unexplained suspicion.
Judgment is reversed and set aside. Accused-appellant is acquitted.
PEOPLE VS. TANGLIBEN
[184 SCRA 220; G.R. No.L-63630; 6 Apr 1990]
Facts:
Patrolmen Silverio and Romeo Punzalan were conducting surveillance at the San Fernando Victory Liner
Terminal. At around 9:30pm they noticed a person, Medel Tangliben, carrying a traveling bag who acted
suspiciously. They confronted him, inspected his bag, and there they found marijuana leaves. The
accused was then taken to the Police Headquarters for further investigations. The TC found Tangliben
guilty of violating sec.4 art. 2 of the RA 6425 or the Dangerous Drugs Act of 1972.
Issue:
Whether or Not there was an unlawful search due to lack of search warrant.
Held;
No. Rule 113 sec. 5 provides the a peace officer or a private person may w/o a warrant arrest a person
when in his presence the person to be arrested has committed, is committing, or is attempting to commit
an offense.
In the present case, the accused was found to have been committing possession of marijuana and can be
therefore searched lawfully even without a search warrant. Another reason is that this case poses
urgency on the part of the arresting police officers. It was found out that an informer pointed to the
accused telling the policemen that the accused was carrying marijuana. The police officers had to act
quickly and there was not enough time to secure a search warrant.
PEOPLE VS. MALMSTEDT
[198 SCRA 401; G.R. No. 91107; 19 Jun 1991]
Facts:
In an information filed against the accused- appellant Mikael Malmstead was charged before the RTC of
La Trinidad, Benguet, for violation of Section 4, Art. II of Republic Act 6425, as amended, otherwise
known as the Dangerous Drugs Act of 1972, as amended.
Accused Mikael Malmstedt, a Swedish national, entered the Philippines for the third time in December
1988 as a tourist. He had visited the country sometime in 1982 and 1985.
In the evening of 7 May 1989, accused left for Baguio City. Upon his arrival thereat in the morning of the
following day, he took a bus to Sagada and stayed in that place for two (2) days. Then in the 7 in the
morning of May 11, 1989, the accused went to Nangonogan bus stop in Sagada.
At about 8: 00 o'clock in the morning of that same day (11 May 1989), Captain Alen Vasco, the
Commanding Officer of the First Regional Command (NARCOM) stationed at Camp Dangwa, ordered his
men to set up a temporary checkpoint at Kilometer 14, Acop, Tublay, Mountain Province, for the purpose
of checking all vehicles coming from the Cordillera Region. The order to establish a checkpoint in the said
area was prompted by persistent reports that vehicles coming from Sagada were transporting marijuana
and other prohibited drugs. Moreover, information was received by the Commanding Officer of NARCOM,
that same morning that a Caucasian coming from Sagada had in his possession prohibited drugs. The
group composed of seven (7) NARCOM officers, in coordination with Tublay Police Station, set up a

Page
Section 1-C, SY 06-07

83

checkpoint at the designated area at about 10:00 o'clock in the morning and inspected all vehicles coming
from the Cordillera Region.
The two (2) NARCOM officers started their inspection from the front going towards the rear of the bus.
Accused who was the sole foreigner riding the bus was seated at the rear thereof.
During the inspection, CIC Galutan noticed a bulge on accused's waist. Suspecting the bulge on
accused's waist to be a gun, the officer asked for accused's passport and other identification papers.
When accused failed to comply, the officer required him to bring out whatever it was that was bulging on
his waist. The bulging object turned out to be a pouch bag and when accused opened the same bag, as
ordered, the officer noticed four (4) suspicious-looking objects wrapped in brown packing tape, prompting
the officer to open one of the wrapped objects. The wrapped objects turned out to contain hashish, a
derivative of marijuana.
Thereafter, accused was invited outside the bus for questioning. But before he alighted from the bus,
accused stopped to get two (2) travelling bags from the luggage carrier. Upon stepping out of the bus, the
officers got the bags and opened them. A teddy bear was found in each bag. Feeling the teddy bears, the
officer noticed that there were bulges inside the same which did not feel like foam stuffing. It was only
after the officers had opened the bags that accused finally presented his passport.
Accused was then brought to the headquarters of the NARCOM at Camp Dangwa, La Trinidad, Benguet
for further investigation. At the investigation room, the officers opened the teddy bears and they were
found to also contain hashish. Representative samples were taken from the hashish found among the
personal effects of accused and the same were brought to the PC Crime Laboratory for chemical
analysis.
In the chemistry report, it was established that the objects examined were hashish. a prohibited drug
which is a derivative of marijuana. Thus, an information was filed against accused for violation of the
Dangerous Drugs Act.
ACCUSEDS DEFENSE
During the arraignment, accused entered a plea of "not guilty." For his defense, he raised the issue of
illegal search of his personal effects. He also claimed that the hashish was planted by the NARCOM
officers in his pouch bag and that the two (2) travelling bags were not owned by him, but were merely
entrusted to him by an Australian couple whom he met in Sagada. He further claimed that the Australian
couple intended to take the same bus with him but because there were no more seats available in said
bus, they decided to take the next ride and asked accused to take charge of the bags, and that they
would meet each other at the Dangwa Station.
The trial court found the guilt of the accused Mikael Malmstedt established beyond reasonable doubt.
Seeking the reversal of the decision of the trial court finding him guilty of the crime charged, accused
argues that the search of his personal effects was illegal because it was made without a search warrant
and, therefore, the prohibited drugs which were discovered during the illegal search are not admissible as
evidence against him.
Issue:
Whether or Not the contention of the accused is valid, and therefore the RTC ruling be reversed.
Held:
The Constitution guarantees the right of the people to be secure in their persons, houses, papers and
effects against unreasonable searches and seizures. However, where the search is made pursuant to a
lawful arrest, there is no need to obtain a search warrant. A lawful arrest without a warrant may be made
by a peace officer or a private person under the following circumstances.
Sec. 5 Arrest without warrant; when lawful. A peace officer or a private person may,
without a warrant, arrest a person:
(a)
When, in his presence, the person to be arrested has committed is
actually committing, or is attempting to commit an offense;
(b)
When an offense has in fact just been committed, and he has personal
knowledge of facts indicating that the person to be arrested has committed it; and
(c)
When the person to be arrested is a prisoner who has escaped from a
penal establishment or place where he is serving final judgment or temporarily
confined while his case is pending, or has escaped while being transferred from
one confinement to another.

Page
Section 1-C, SY 06-07

84

San Beda College of Law Alabang


Constitutional Law 2 Case Digests

Accused was searched and arrested while transporting prohibited drugs (hashish). A crime was actually
being committed by the accused and he was caught in flagrante delicto. Thus, the search made upon his
personal effects falls squarely under paragraph (1) of the foregoing provisions of law, which allow a
warrantless search incident to a lawful arrest. While it is true that the NARCOM officers were not armed
with a search warrant when the search was made over the personal effects of accused, however, under
the circumstances of the case, there was sufficient probable cause for said officers to believe that
accused was then and there committing a crime.
Probable cause has been defined as such facts and circumstances which could lead a reasonable,
discreet and prudent man to believe that an offense has been committed, and that the objects sought in
connection with the offense are in the place sought to be searched. Warrantless search of the personal
effects of an accused has been declared by this Court as valid, because of existence of probable cause,
where the smell of marijuana emanated from a plastic bag owned by the accused, 10 or where the
accused was acting suspiciously, 11 and attempted to flee.
The appealed judgment of conviction by the trial court is hereby affirmed. Costs against the accusedappellant.
PEOPLE VS. AMMINUDIN
[163 SCRA 402; G.R. L-74869; 6 Jul 1988]
Facts:
Idel Aminnudin, accused-appellant was arrested on June 25, 1984, shortly after disembarking from the
M/V Wilcon 9 at about 8:30 in the evening, in Iloilo City. The PC officers who were in fact waiting for him
because of a tip from one their informers simply accosted him, inspected his bag and finding what looked
liked marijuana leaves took him to their headquarters for investigation. The two bundles of suspect
articles were confiscated from him and later taken to the NBI laboratory for examination. It was found to
contain three kilos of what were later analyzed as marijuana leaves by an NBI forensic examiner. An
information for violation of the Dangerous Drugs Act was filed against him. Later, the information was
amended to include Farida Ali y Hassen, who had also been arrested with him that same evening and
likewise investigated. Both were arraigned and pleaded not guilty. Subsequently, the fiscal filed a motion
to dismiss the charge against Ali on the basis of a sworn statement of the arresting officers absolving her
after a 'thorough investigation." The motion was granted, and trial proceeded only against the accusedappellant, who was eventually convicted . In his defense, Aminnudin disclaimed the marijuana, averring
that all he had in his bag was his clothing consisting of a jacket, two shirts and two pairs of pants. He
alleged that he was arbitrarily arrested and immediately handcuffed. His bag was confiscated without a
search warrant. At the PC headquarters, he was manhandled to force him to admit he was carrying the
marijuana, the investigator hitting him with a piece of wood in the chest and arms even as he parried the
blows while he was still handcuffed. He insisted he did not even know what marijuana looked like and that
his business was selling watches and sometimes cigarettes. However the RTC rejected his allegations.
Saying that he only has two watches during that time and that he did not sufficiently proved the injuries
allegedly sustained.
Issue:
Whether or not search of defendants bag is legal.
Held:
The search was illegal. Defendant was not caught in flagrante delicto, which could allow warrantless
arrest or search. At the moment of his arrest, he was not committing a crime. Nor was he about to do so
or had just done so. To all appearances, he was like any of the other passengers innocently disembarking
from the vessel. The said marijuana therefore could not be appreciated as evidence against the
defendant, and furthermore he is acquitted of the crime as charged.

PEOPLE VS. SAYCON


[236 SCRA 325; G.R. NO. 110995; 5 SEPT 1994]
Facts:

Page
Section 1-C, SY 06-07

85

On or about 8 July 1992, at about 6:00 in the morning, the Coastguard personnel received information
from NARCOM agent Ruben Laddaran that a suspected "shabu" courier by the name of Alvaro Saycon
was on board the MV Doa Virginia, which was arriving at that moment in Dumaguete City. Upon receipt
of the information, the Coastguard chief officer CPO Tolin, instructed them to intercept the suspect. A
combined team of NARCOM agents and Philippine Coastguard personnel consisting of CPO Tolin, a
certain Miagme, and Senior Police Officers Ruben Laddaran and Winifredo Noble of NARCOM posted
themselves at the gate of Pier 1. The MV Doa Virginia docked at 6:00 a.m. that same morning at Pier 1
in Dumaguete City. Alvaro Saycon alighted from the boat carrying a black bag and went through the
checkpoint manned by the Philippine Coastguard where he was identified by police officer Winifredo
Noble of NARCOM. Saycon was then invited to the Coastguard Headquarters at the Pier area. He
willingly went with them. At the headquarters, the coastguard asked Saycon to open his bag, and the
latter willingly obliged. In it were personal belongings and a maong wallet. Inside that maong wallet, there
was a Marlboro pack containing the suspected "shabu". When police officer Winifredo Noble asked
Saycon whether the Marlboro pack containing the suspected "shabu" was his, Saycon merely bowed his
head. Then Saycon, his bag and the suspected "shabu" were brought to the NARCOM office for booking.
When Alvaro Saycon was arrested, the NARCOM agents did not have a warrant of arrest. The PNP's
Forensic Analyst declared in court that she had conducted an examination of the specimens and found
out that the specimens weighed 4.2 grams in total, consisted of methamphetamine hydrochloride, more
widely known as "shabu."
Issue:
Whether or Not the warrantless search was valid.
Held:
The warrantless search was valid, as the accused was a passenger of a motor vehicle. There was
probable cause to believe that the accused was carrying prohibited drugs. Three weeks earlier, agents of
the Narcotics Command bought methamine hydrochloride from him. An agent of the Narcotics Command
reported that the accused would be arriving on board the vessel and carrying methamphetamine
hydrochloride with him. Drug couriers do not go about their trade with some external sign indicating that
they are transporting prohibited drugs. This must be taken into account in determining probable cause.

PEOPLE VS. MUSA


[217 SCRA 597; G.,R. NO. 96177; 27 JAN 1993]
Facts:
A civilian informer gave the information that Mari Musa was engaged in selling marijuana in Suterville,
Zamboanga City. Sgt. Ani was ordered by NARCOM leader T/Sgt. Belarga, to conduct a surveillance and
test buy on Musa. The civilian informer guided Ani to Musas house and gave the description of Musa. Ani
was able to buy one newspaper-wrapped dried marijuana for P10.00.
The next day, a buy-bust was planned. Ani was to raise his right hand if he successfully buys marijuana
from Musa. As Ani proceeded to the house, the NARCOM team positioned themselves about 90 to 100
meters away. From his position, Belarga could see what was going on. Musa came out of the house and
asked Ani what he wanted. Ani said he wanted more marijuana and gave Musa the P20.00 marked
money. Musa went into the house and came back, giving Ani two newspaper wrappers containing dried
marijuana. Ani opened and inspected it. He raised his right hand as a signal to the other NARCOM
agents, and the latter moved in and arrested Musa inside the house. Belarga frisked Musa in the living
room but did not find the marked money (gave it to his wife who slipped away). T/Sgt. Belarga and Sgt.
Lego went to the kitchen and found a cellophane colored white and stripe hanging at the corner of the
kitchen. They asked Musa about its contents but failed to get a response. So they opened it and found
dried marijuana leaves inside. Musa was then placed under arrest.
Issue:
Whether or Not the seizure of the plastic bag and the marijuana inside it is unreasonable, hence,
inadmissible as evidence.
Held:

Page
Section 1-C, SY 06-07

86

San Beda College of Law Alabang


Constitutional Law 2 Case Digests

Yes. It constituted unreasonable search and seizure thus it may not be admitted as evidence. The
warrantless search and seizure, as an incident to a suspects lawful arrest, may extend beyond the
person of the one arrested to include the premises or surroundings under his immediate control. Objects
in the plain view of an officer who has the right to be in the position to have that view are subject to
seizure and may be presented as evidence. The plain view doctrine is usually applied where a police
officer is not searching for evidence against the accused, but nonetheless inadvertently comes across an
incriminating object. It will not justify the seizure of the object where the incriminating nature of the object
is not apparent from the plain view of the object.
In the case at bar, the plastic bag was not in the plain view of the police. They arrested the accused in
the living room and moved into the kitchen in search for other evidences where they found the plastic bag.
Furthermore, the marijuana inside the plastic bag was not immediately apparent from the plain view of
said object.
Therefore, the plain view does not apply. The plastic bag was seized illegally and cannot be presented in
evidence pursuant to Article III Section 3 (2) of the Constitution.
PITA VS. COURT OF APPEALS
[178 SCRA 362; G.R. NO. 80806; 5 OCT 1989]
Facts:
On December 1 and 3, 1983, pursuing an Anti-Smut Campaign initiated by the Mayor of the City of
Manila, Ramon D. Bagatsing, elements of the Special Anti-Narcotics Group, Auxilliary Services Bureau,
Western Police District, INP of the Metropolitan Police Force of Manila, seized and confiscated from
dealers, distributors, newsstand owners and peddlers along Manila sidewalks, magazines, publications
and other reading materials believed to be obscene, pornographic and indecent and later burned the
seized materials in public at the University belt along C.M. Recto Avenue, Manila, in the presence of
Mayor Bagatsing and several officers and members of various student organizations.
Among the publications seized, and later burned, was "Pinoy Playboy" magazines published and coedited by plaintiff Leo Pita.
Plaintiff filed a case for injunction with prayer for issuance of the writ of preliminary injunction against
Mayor Bagatsing and Narcisco Cabrera, as superintendent of Western Police District of the City of
Manila, seeking to enjoin said defendants and their agents from confiscating plaintiffs magazines or from
preventing the sale or circulation thereof claiming that the magazine is a decent, artistic and educational
magazine which is not per se obscene, and that the publication is protected by the Constitutional
guarantees of freedom of speech and of the press. Plaintiff also filed an Urgent Motion for issuance of a
temporary restraining order against indiscriminate seizure, confiscation and burning of plaintiff's "Pinoy
Playboy" Magazines, pending hearing on the petition for preliminary injunction. The Court granted the
temporary restraining order. The case was set for trial upon the lapse of the TRO. RTC ruled that the
seizure was valid. This was affirmed by the CA.
Issue:
Whether or Not the seizure was illegal.
Held:
The Court ruled that the government authorities have not shown the required proof to justify a ban and to
warrant confiscation of the literature. First of all, they were not possessed of a lawful court order: (1)
finding the said materials to be pornography, and (2) authorizing them to carry out a search and seizure,
by way of a search warrant. The court provides the following guidelines to be observed:
1. The authorities must apply for the issuance of a search warrant from a judge, if in their opinion an
obscenity seizure is in order;
2. The authorities must convince the court that the materials sought to be seized are obscene and
pose a clear and present danger of an evil substantive enough to warrant State interference and
action;
3. The judge must determine whether or not the same are indeed obscene. The question is to be
resolved on a case-to-case basis and on the judges sound discretion;
4. If in the opinion of the court, probable cause exists, it shall issue the search warrant prayed for;
5. The proper suit is then brought in the court under Article 201 of the RPC (Obscene publications).
6. Any conviction is subject to appeal. The appellate court may assess whether or not the properties
seized are indeed obscene.

Page
Section 1-C, SY 06-07

87

GUANZON VS. DE VILLA


[181 SCRA 623; G.R. 80508; 30 JAN 1990]
Facts:
The 41 petitioners alleged that the "saturation drive" or "aerial target zoning" that were conducted in their
place (Tondo Manila) were unconstitutional. They alleged that there is no specific target house to be
search and that there is no search warrant or warrant of arrest served. Most of the policemen are in their
civilian clothes and without nameplates or identification cards. The residents were rudely rouse from their
sleep by banging on the walls and windows of their houses. The residents were at the point of highpowered guns and herded like cows. Men were ordered to strip down to their briefs for the police to
examine their tattoo marks. The residents complained that they're homes were ransacked, tossing their
belongings and destroying their valuables. Some of their money and valuables had disappeared after the
operation. The residents also reported incidents of maulings, spot-beatings and maltreatment. Those who
were detained also suffered mental and physical torture to extract confessions and tactical informations.
The respondents said that such accusations were all lies. Respondents contends that the Constitution
grants to government the power to seek and cripple subversive movements for the maintenance of peace
in the state. The aerial target zoning were intended to flush out subversives and criminal elements
coddled by the communities were the said drives were conducted. They said that they have intelligently
and carefully planned months ahead for the actual operation and that local and foreign media joined the
operation to witness and record such event.
Issue:
Whether or Not the saturation drive committed consisted of violation of human rights.
Held:
It is not the police action per se which should be prohibited rather it is the procedure used or the methods
which "offend even hardened sensibilities" .Based on the facts stated by the parties, it appears to have
been no impediment to securing search warrants or warrants of arrest before any houses were searched
or individuals roused from sleep were arrested. There is no showing that the objectives sought to be
attained by the "aerial zoning" could not be achieved even as th rights of the squatters and low income
families are fully protected. However, the remedy should not be brought by a tazpaer suit where not one
victim complaints and not one violator is properly charged. In the circumstances of this taxpayers' suit,
there is no erring soldier or policeman whom the court can order prosecuted. In the absence of clear facts
no permanent relief can be given.
In the meantime where there is showing that some abuses were committed, the court temporary restraint
the alleged violations which are shocking to the senses. Petition is remanded to the RTC of Manila.
PEOPLE VS. ARUTA
[288 SCRA 626; G.R. NO. 120515; 13 APR 1998]
Facts:
On Dec. 13, 1988, P/Lt. Abello was tipped off by his informant that a certain Aling Rosa will be arriving
from Baguio City with a large volume of marijuana and assembled a team. The next day, at the Victory
Liner Bus terminal they waited for the bus coming from Baguio, when the informer pointed out who Aling
Rosa was, the team approached her and introduced themselves as NARCOM agents. When Abello
asked aling Rosa about the contents of her bag, the latter handed it out to the police. They found dried
marijuana leaves packed in a plastic bag marked cash katutak.
Instead of presenting its evidence, the defense filed a demurrer to evidence alleging the illegality of the
search and seizure of the items. In her testimony, the accused claimed that she had just come from
Choice theatre where she watched a movie Balweg. While about to cross the road an old woman asked
her for help in carrying a shoulder bag, when she was later on arrested by the police. She has no
knowledge of the identity of the old woman and the woman was nowhere to be found. Also, no search
warrant was presented.
The trial court convicted the accused in violation of the dangerous drugs of 1972
Issue:

Page
Section 1-C, SY 06-07

88

San Beda College of Law Alabang


Constitutional Law 2 Case Digests
Whether or Not the police correctly searched and seized the drugs from the accused.
Held:
The following cases are specifically provided or allowed by law:

1. Warrantless search incidental to a lawful arrest recognized under Section 12, Rule 126 of the
2.

3.

4.

5.
6.
7.

Rules of Court 8 and by prevailing jurisprudence


Seizure of evidence in "plain view," the elements of which are: (a) a prior valid intrusion based
on the valid warrantless arrest in which the police are legally present in the pursuit of their
official duties; (b) the evidence was inadvertently discovered by the police who had the right
to be where they are; (c) the evidence must be immediately apparent, and (d) "plain view"
justified mere seizure of evidence without further search;
Search of a moving vehicle. Highly regulated by the government, the vehicle's inherent
mobility reduces expectation of privacy especially when its transit in public thoroughfares
furnishes a highly reasonable suspicion amounting to probable cause that the occupant
committed a criminal activity;
Consented warrantless search;
Customs search;
Stop and Frisk;
Exigent and Emergency Circumstances.

The essential requisite of probable cause must still be satisfied before a warrantless search and seizure
can be lawfully conducted.
The accused cannot be said to be committing a crime, she was merely crossing the street and was not
acting suspiciously for the Narcom agents to conclude that she was committing a crime. There was no
legal basis to effect a warrantless arrest of the accuseds bag, there was no probable cause and the
accused was not lawfully arrested.
The police had more than 24 hours to procure a search warrant and they did not do so. The seized
marijuana was illegal and inadmissible evidence.
RULE 113, RULES OF COURT
Section 5. Arrest without warrant; when lawful. A peace officer or a private person may, without a
warrant, arrest a person:
(a) When, in his presence, the person to be arrested has committed, is actually committing, or is
attempting to commit an offense;
(b) When an offense has just been committed, and he has probable cause to believe based on
personal knowledge of facts or circumstances that the person to be arrested has committed
it; and
(c) When the person to be arrested is a prisoner who has escaped from a penal establishment or
place where he is serving final judgment or is temporarily confined while his case is pending,
or has escaped while being transferred from one confinement to another.
In cases falling under paragraph (a) and (b) above, the person arrested without a warrant shall be
forthwith delivered to the nearest police station or jail and shall be proceeded against in accordance with
section 7 of Rule 112.
RULE 126, RULES OF COURT
Section 2. Court where application for search warrant shall be filed. An application for search warrant
shall be filed with the following:
a) Any court within whose territorial jurisdiction a crime was committed.
b) For compelling reasons stated in the application, any court within the judicial region where the
crime was committed if the place of the commission of the crime is known, or any court within the
judicial region where the warrant shall be enforced.
However, if the criminal action has already been filed, the application shall only be made in the court
where the criminal action is pending.

Page
Section 1-C, SY 06-07

89

Section 7. Right to break door or window to effect search. The officer, if refused admittance to the
place of directed search after giving notice of his purpose and authority, may break open any outer or
inner door or window of a house or any part of a house or anything therein to execute the warrant or
liberate himself or any person lawfully aiding him when unlawfully detained therein.
Section 12. Delivery of property and inventory thereof to court; return and proceedings thereon.
(a) The officer must forthwith deliver the property seized to the judge who issued the warrant,
together with a true inventory thereof duly verified under oath.
(b) Ten (10) days after issuance of the search warrant, the issuing judge shall ascertain if the return
has been made, and if none, shall summon the person to whom the warrant was issued and
require him to explain why no return was made. If the return has been made, the judge shall
ascertain whether section 11 of this Rule has been complained with and shall require that the
property seized be delivered to him. The judge shall see to it that subsection (a) hereof has been
complied with.
(c) The return on the search warrant shall be filed and kept by the custodian of the log book on
search warrants who shall enter therein the date of the return, the result, and other actions of the
judge.
A violation of this section shall constitute contempt of court.

Page
Section 1-C, SY 06-07

90

San Beda College of Law Alabang


Constitutional Law 2 Case Digests

LIBERTY OF ABODE AND OF TRAVEL


Art 3, Sec. 6. The liberty of abode and of changing the same within the limits prescribed by law shall
not be impaired except upon lawful order of the court. Neither shall the right to travel be impaired except
in the interest of national security, public safety, or public health, as may be provided by law.
CAUNCA VS. SALAZAR
[82 PHIL 851; NO.L-2690; 1 JAN 1949]
Facts:
This is an action for habeas corpus brought by Bartolome Caunca in behalf of his cousin Estelita Flores
who was employed by the Far Eastern Employment Bureau, owned by Julia Salazar, respondent herein.
An advanced payment has already been given to Estelita by the employment agency, for her to work as a
maid. However, Estelita wanted to transfer to another residence, which was disallowed by the
employment agency. Further she was detained and her liberty was restrained. The employment agency
wanted that the advance payment, which was applied to her transportation expense from the province
should be paid by Estelita before she could be allowed to leave.
Issue:
Whether or Not an employment agency has the right to restrain and detain a maid without returning the
advance payment it gave?
Held:
An employment agency, regardless of the amount it may advance to a prospective employee or maid, has
absolutely no power to curtail her freedom of movement. The fact that no physical force has been exerted
to keep her in the house of the respondent does not make less real the deprivation of her personal
freedom of movement, freedom to transfer from one place to another, freedom to choose ones residence.
Freedom may be lost due to external moral compulsion, to founded or groundless fear, to erroneous belief
in the existence of an imaginary power of an impostor to cause harm if not blindly obeyed, to any other
psychological element that may curtail the mental faculty of choice or the unhampered exercise of the will.
If the actual effect of such psychological spell is to place a person at the mercy of another, the victim is
entitled to the protection of courts of justice as much as the individual who is illegally deprived of liberty by
duress or physical coercion.

MANOTOC VS. COURT OF APPEALS


[142 SCRA 149; G.R. NO. L-62100; 30 MAY 1986]
Facts:
Petitioner was charged with estafa. He posted bail. Petitioner filed before each of the trial courts a motion
entitled, "motion for permission to leave the country," stating as ground therefor his desire to go to the
United States, "relative to his business transactions and opportunities." The prosecution opposed said
motion and after due hearing, both trial judges denied the same. Petitioner thus filed a petition for
certiorari and mandamus before the then Court of Appeals seeking to annul the orders dated March 9 and
26, 1982, of Judges Camilon and Pronove, respectively, as well as the communication-request of the
Securities and Exchange Commission, denying his leave to travel abroad. He likewise prayed for the
issuance of the appropriate writ commanding the Immigration Commissioner and the Chief of the Aviation
Security Command (AVSECOM) to clear him for departure. The Court of Appeals denied the petition.
Petitioner contends that having been admitted to bail as a matter of right, neither the courts which granted
him bail nor the Securities and Exchange Commission which has no jurisdiction over his liberty could
prevent him from exercising his constitutional right to travel.
Issue:

Page
Section 1-C, SY 06-07

91

Whether or Not his constitutional right to travel has been violated.


Held:
A court has the power to prohibit a person admitted to bail from leaving the Philippines. This is a
necessary consequence of the nature and function of a bail bond. The condition imposed upon petitioner
to make himself available at all times whenever the court requires his presence operates as a valid
restriction on his right to travel. Indeed, if the accused were allowed to leave the Philippines without
sufficient reason, he may be placed beyond the reach of the courts. Petitioner has not shown the
necessity for his travel abroad. There is no indication that the business transactions cannot be undertaken
by any other person in his behalf.
MARCOS VS. MANGLAPUS
[177 SCRA 668; G.R. NO. 88211; 15 SEPT 1989]
Facts:
This case involves a petition of mandamus and prohibition asking the court to order the respondents
Secretary of Foreign Affairs, etc. To issue a travel documents to former Pres. Marcos and the immediate
members of his family and to enjoin the implementation of the President's decision to bar their return to
the Philippines. Petitioners assert that the right of the Marcoses to return in the Philippines is guaranteed
by the Bill of Rights, specifically Sections 1 and 6. They contended that Pres. Aquino is without power to
impair the liberty of abode of the Marcoses because only a court may do so within the limits prescribed by
law. Nor the President impair their right to travel because no law has authorized her to do so.
They further assert that under international law, their right to return to the Philippines is guaranteed
particularly by the Universal Declaration of Human Rights and the International Covenant on Civil and
Political Rights, which has been ratified by the Philippines.
Issue:
Whether or not, in the exercise of the powers granted by the constitution, the President (Aquino) may
prohibit the Marcoses from returning to the Philippines.
Held:
"It must be emphasized that the individual right involved is not the right to travel from the Philippines to
other countries or within the Philippines. These are what the right to travel would normally connote.
Essentially, the right involved in this case at bar is the right to return to one's country, a distinct right under
international law, independent from although related to the right to travel. Thus, the Universal Declaration
of Human Rights and the International Covenant on Civil and Political Rights treat the right to freedom of
movement and abode within the territory of a state, the right to leave the country, and the right to enter
one's country as separate and distinct rights. What the Declaration speaks of is the "right to freedom of
movement and residence within the borders of each state". On the other hand, the Covenant guarantees
the right to liberty of movement and freedom to choose his residence and the right to be free to leave any
country, including his own. Such rights may only be restricted by laws protecting the national security,
public order, public health or morals or the separate rights of others. However, right to enter one's country
cannot be arbitrarily deprived. It would be therefore inappropriate to construe the limitations to the right to
return to ones country in the same context as those pertaining to the liberty of abode and the right to
travel.
The Bill of rights treats only the liberty of abode and the right to travel, but it is a well considered view that
the right to return may be considered, as a generally accepted principle of International Law and under
our Constitution as part of the law of the land.
The court held that President did not act arbitrarily or with grave abuse of discretion in determining that
the return of the Former Pres. Marcos and his family poses a serious threat to national interest and
welfare. President Aquino has determined that the destabilization caused by the return of the Marcoses
would wipe away the gains achieved during the past few years after the Marcos regime.
The return of the Marcoses poses a serious threat and therefore prohibiting their return to the Philippines,
the instant petition is hereby DISMISSED.

Page
Section 1-C, SY 06-07

92

San Beda College of Law Alabang


Constitutional Law 2 Case Digests

SILVERIO VS. COURT OF APPEALS


[195 SCRA 760 ; G.R. 94284; 8 APR 1991]
Facts:
Petitioner was charged with violation of Section 2 (4) of the revised securities act. Respondent filed to
cancel the passport of the petitioner and to issue a hold departure order. The RTC ordered the DFA to
cancel petitioners passport, based on the finding that the petitioner has not been arraigned and there
was evidence to show that the accused has left the country with out the knowledge and the permission of
the court.
Issue:
Whether or Not the right to travel may be impaired by order of the court.
Held:
The bail bond posted by petitioner has been cancelled and warrant of arrest has been issued by reason
that he failed to appear at his arraignments. There is a valid restriction on the right to travel, it is imposed
that the accused must make himself available whenever the court requires his presence. A person facing
criminal charges may be restrained by the Court from leaving the country or, if abroad, compelled to
return (Constitutional Law, Cruz, Isagani A., 1987 Edition, p. 138). So it is also that "An accused released
on bail may be re-arrested without the necessity of a warrant if he attempts to depart from the Philippines
without prior permission of the Court where the case is pending (ibid., Sec. 20 [2nd
par. ]).
Article III, Section 6 of the 1987 Constitution should be interpreted to mean that while the liberty of travel
may be impaired even without Court Order, the appropriate executive officers or administrative authorities
are not armed with arbitrary discretion to impose limitations. They can impose limits only on the basis of
"national security, public safety, or public health" and "as may be provided by law," a limitive phrase which
did not appear in the 1973 text (The Constitution, Bernas, Joaquin G.,S.J., Vol. I, First Edition, 1987, p.
263). Apparently, the phraseology in the 1987 Constitution was a reaction to the ban on international
travel imposed under the previous regime when there was a Travel Processing Center, which issued
certificates of eligibility to travel upon application of an interested party (See Salonga vs. Hermoso &
Travel Processing Center, No. 53622, 25 April 1980, 97 SCRA 121).
Holding an accused in a criminal case within the reach of the Courts by preventing his departure from the
Philippines must be considered as a valid restriction on his right to travel so that he may be dealt with in
accordance with law. The offended party in any criminal proceeding is the People of the Philippines. It is
to their best interest that criminal prosecutions should run their course and proceed to finality without
undue delay, with an accused holding himself amenable at all times to Court Orders and processes

DEFENSOR-SANTIAGO VS. VASQUEZ


[217 SCRA 633; G.R. NOS. 99289-90; 27 JAN 1993]
Facts:
An information was filed against petitioner with the Sandiganbayan for violation of the Anti Graft and
Corrupt Practices Act. The order of arrest was issued with bail for release fixed at Php. 15,000 so she
filed a motion for acceptance of cash bail bond. On the same day the Sandiganbayan issued a resolution
authorizing the petitioner to post cash bond which the later filed in the amount of Php.15, 000. Her
arraignment was set, but petitioner asked for the cancellation of her bail bond and that she be allowed
provisional release on recognizance. The Sandiganbayan deferred it. The Sandiganbayan issued a hold
departure order against petitioner, by reason of the announcement she made that she would be leaving
for the U.S. to accept a fellowship a Harvard. In the instant motion she submitted before the S.C. she
argues that her right to travel is impaired.
Issue:
Whether or Not the petitioners right to travel is impaired.
Held:

Page
Section 1-C, SY 06-07

93

The petitioner does not deny and as a matter of fact even made a public statement, that she he every
intension of leaving the country to pursue higher studies abroad. The court upholds the course of action of
the Sandiganbayan in taking judicial notice of such fact of petitioners pal to go abroad and in thereafter
issuing a sua sponte the hold departure order is but an exercise of respondent courts inherent power to
preserve and to maintain effectiveness of its jurisdiction over the case and the person of the accused.
Also, the petitioner assumed obligations, when she posted bail bond. She holds herself amenable at all
times to the orders and process of eth court. She may legally be prohibited from leaving the country
during the pendency of the case. (Manotoc v. C.A.)
MARCOS VS. SANDIGANBAYAN
[247 SCRA 127; G.R. NO. 115132-34; 9 AUG 1995]
Facts:
This is a petition for certiorari to set aside as arbitrary and in grave abuse of discretion resolutions of the
Sandiganbayan's First Division denying petitioner's motion for leave to travel abroad for medical
treatment.
The former first lady Imelda Marcos was found guilty by the First Division of the Sandiganbayan of
violating 3 of the Anti Graft and Corrupt Practices Act. After conviction she filed a "Motion for Leave to
Travel Abroad" to seek diagnostic tests and treatment by practitioners of oriental medicine in China
allegedly because of "a serious and life threatening medical condition" requiring facilities not available in
the Philippines that was denied. Then she again filed an "Urgent Ex-Parte Motion for Permission to Travel
Abroad" to undergo diagnosis and treatment in China. This was supported by several medical reports that
were prepared by her doctor Roberto Anastacio.
Again another Motion to leave was filed by Mrs. Marcos to US and Europe for treatment of several Heart
diseases alleging that the tests were not available here.
The presiding justice, Garchitorena, contacted Dr. Gregorio B. Patacsil, Officer-in-Charge of the Philippine
Heart Center, and later wrote him a letter, asking for "expert opinion on coronary medicine". The court still
found no merit to allow the petitioners motion to leave and denied all of the motions.
Petitioner filed a motion for reconsideration and a "Motion to Admit Clinical Summary and to Resolve
Motion for Reconsideration." Attached was a recent medical report and letters of Vice President Joseph E.
Estrada offering to be guarantor for the return of petitioner and those of twenty four members of the
House of Representatives requesting the court to allow petitioner to travel abroad. This was also denied
by the Court also stating their express disapproval of the involvement of the VP and the Cabinet members
so as to influence the resolutions, decisions or orders or any judicial action of respondent court.
Issue:
Whether or Not the Sandiganbayan erred in disallowing the Motion for Leave to Travel Abroad because it
(1) disregarded the medical findings (2) it motu propio contacted a third party asking the latter to give an
opinion on petitioner's motion and medical findings (3) said that there was no necessity to get medical
treatment abroad.
Held:
No. The contention of the petitioner that was invalid to contact a third party asking the latter to give an
opinion on petitioner's motion and medical findings was erroneous. Respondent court had to seek expert
opinion because petitioner's motion was based on the advice of her physician. The court could not be
expected to just accept the opinion of petitioner's physician in resolving her request for permission to
travel. What would be objectionable would be if respondent court obtained information without disclosing
its source to the parties and used it in deciding a case against them.
In disregarding the medical reports, the petitioner failed to prove the necessity for a trip abroad. It should
be emphasized that considering the fact that she is facing charges before the courts in several cases, in
two of which she was convicted although the decision is still pending reconsideration, petitioner did not
have an absolute right to leave the country and the burden was on her to prove that because of danger to
health if not to her life there was necessity to seek medical treatment in foreign countries.
On the third issue, the Court ordered petitioner to undergo several tests which summarily states that the
required medical treatment was available here in the Philippines and that the expertise and facilities here

Page
Section 1-C, SY 06-07

94

San Beda College of Law Alabang


Constitutional Law 2 Case Digests
were more than adequate to cater to her medical treatment. The heart ailments of the petitioner were not
as severe as that was reported by Dr. Anastacio.
Wherefore, the petitioner is Dismissed without prejudice to the filling of another motion for leave to travel
abroad, should petitioner still desire, based on her heart condition. In such an event the determination of
her medical condition should be made by joint panel of medical specialists recommended by both the
accused and the prosecution.
RUBI VS. PROVINCIAL BOARD OF MINDORO
[39 PHIL 660; NO. 14078; 7 MAR 1919]
Facts:
The provincial board of Mindoro adopted resolution No. 25 wherein non-Christian inhabitants (uncivilized
tribes) will be directed to take up their habitation on sites on unoccupied public lands. It is resolved that
under section 2077 of the Administrative Code, 800 hectares of public land in the sitio of Tigbao on
Naujan Lake be selected as a site for the permanent settlement of Mangyanes in Mindoro. Further,
Mangyans may only solicit homesteads on this reservation providing that said homestead applications are
previously recommended by the provincial governor.
In that case, pursuant to Section 2145 of the Revised Administrative Code, all the Mangyans in the
townships of Naujan and Pola and the Mangyans east of the Baco River including those in the districts of
Dulangan and Rubi's place in Calapan, were ordered to take up their habitation on the site of Tigbao,
Naujan Lake. Also, that any Mangyan who shall refuse to comply with this order shall upon conviction be
imprisoned not exceed in sixty days, in accordance with section 2759 of the revised Administrative Code.
Said resolution of the provincial board of Mindoro were claimed as necessary measures for the protection
of the Mangyanes of Mindoro as well as the protection of public forests in which they roam, and to
introduce civilized customs among them.
It appeared that Rubi and those living in his rancheria have not fixed their dwelling within the reservation
of Tigbao and are liable to be punished.
It is alleged that the Manguianes are being illegally deprived of their liberty by the provincial officials of
that province. Rubi and his companions are said to be held on the reservation established at Tigbao,
Mindoro, against their will, and one Dabalos is said to be held under the custody of the provincial sheriff in
the prison at Calapan for having run away form the reservation.
Issue:
Whether or Not Section 2145 of the Administrative Code deprive a person of his liberty pf abode. Thus,
WON Section 2145 of the Administrative Code of 1917 is constitutional.
Held:
The Court held that section 2145 of the Administrative Code does not deprive a person of his liberty of
abode and does not deny to him the equal protection of the laws, and that confinement in reservations in
accordance with said section does not constitute slavery and involuntary servitude. The Court is further of
the opinion that section 2145 of the Administrative Code is a legitimate exertion of the police power.
Section 2145 of the Administrative Code of 1917 is constitutional.
Assigned as reasons for the action: (1) attempts for the advancement of the non-Christian people of the
province; and (2) the only successfully method for educating the Manguianes was to oblige them to live in
a permanent settlement. The Solicitor-General adds the following; (3) The protection of the Manguianes;
(4) the protection of the public forests in which they roam; (5) the necessity of introducing civilized
customs among the Manguianes.
One cannot hold that the liberty of the citizen is unduly interfered without when the degree of civilization of
the Manguianes is considered. They are restrained for their own good and the general good of the
Philippines.
Liberty regulated by law": Implied in the term is restraint by law for the good of the individual and for the
greater good of the peace and order of society and the general well-being. No man can do exactly as he
pleases.
None of the rights of the citizen can be taken away except by due process of law.

Page
Section 1-C, SY 06-07

95

Therefore, petitioners are not unlawfully imprisoned or restrained of their liberty. Habeas corpus can,
therefore, not issue.

Page
Section 1-C, SY 06-07

96

San Beda College of Law Alabang


Constitutional Law 2 Case Digests
FREEDOM OF RELIGION
Art 3, Sec. 5. No law shall be made respecting an establishment of religion, or prohibiting the free
exercise thereof. The free exercise and enjoyment of religious profession and worship, without
discrimination or preference, shall forever be allowed. No religious test shall be required for the exercise
of civil or political rights.
AGLIPAY VS. RUIZ
[64 PHIL 201; G.R. NO. 45459; 13 MAR 1937]
Facts:
Petitioner seeks the issuance of a writ of prohibition against respondent Director of Posts from issuing
and selling postage stamps commemorative of the 33 rd International Eucharistic Congress. Petitioner
contends that such act is a violation of the Constitutional provision stating that no public funds shall be
appropriated or used in the benefit of any church, system of religion, etc. This provision is a result of the
principle of the separation of church and state, for the purpose of avoiding the occasion wherein the state
will use the church, or vice versa, as a weapon to further their ends and aims. Respondent contends that
such issuance is in accordance to Act No. 4052, providing for the appropriation funds to respondent for
the production and issuance of postage stamps as would be advantageous to the government.
Issue:
Whether or Not there was a violation of the freedom to religion.
Held:
What is guaranteed by our Constitution is religious freedom and not mere religious toleration. It is
however not an inhibition of profound reverence for religion and is not a denial of its influence in human
affairs. Religion as a profession of faith to an active power that binds and elevates man to his Creator is
recognized. And in so far as it instills into the minds the purest principles of morality, its influence is
deeply felt and highly appreciated. The phrase in Act No. 4052 advantageous to the government does
not authorize violation of the Constitution. The issuance of the stamps was not inspired by any feeling to
favor a particular church or religious denomination. They were not sold for the benefit of the Roman
Catholic Church. The postage stamps, instead of showing a Catholic chalice as originally planned,
contains a map of the Philippines and the location of Manila, with the words Seat XXXIII International
Eucharistic Congress. The focus of the stamps was not the Eucharistic Congress but the city of Manila,
being the seat of that congress. This was to to advertise the Philippines and attract more tourists, the
officials merely took advantage of an event considered of international importance. Although such
issuance and sale may be inseparably linked with the Roman Catholic Church, any benefit and
propaganda incidentally resulting from it was no the aim or purpose of the Government.
GARCES VS. ESTENZO
[104 SCRA 510; G.R. L-53487; 25 MAY 1981]
Facts:
Two resolutions of the Barangay Council of Valencia, Ormoc City were passed:
a. Resolution No. 5- Reviving the traditional socio-religious celebration every fifth of April. This
provided for the acquisition of the image of San Vicente Ferrer and the construction of a
waiting shed. Funds for the said projects will be obtained through the selling of tickets and
cash donations.
b. Resolution No. 6- The chairman or hermano mayor of the fiesta would be the caretaker of the
image of San Vicente Ferrer and that the image would remain in his residence for one year
and until the election of his successor. The image would be made available to the Catholic
Church during the celebration of the saints feast day.
These resolutions have been ratified by 272 voters, and said projects were implemented. The image was
temporarily placed in the altar of the Catholic Church of the barangay. However, after a mass, Father
Sergio Marilao Osmea refused to return the image to the barangay council, as it was the churchs
property since church funds were used in its acquisition.
Resolution No. 10 was passed for the authorization of hiring a lawyer for the replevin case against the
priest for the recovery of the image. Resolution No. 12 appointed Brgy. Captain Veloso as a

Page
Section 1-C, SY 06-07

97

representative to the case. The priest, in his answer assailed the constitutionality of the said resolutions.
The priest with Andres Garces, a member of the Aglipayan Church, contends that Sec. 8 Article IV 1 and
Sec 18(2) Article VIII) 2 of the constitution was violated.
Issue:
Whether or Not any freedom of religion clause in the Constitution violated.
Held:
No. As said by the Court this case is a petty quarrel over the custody of the image. The image was
purchased in connection with the celebration of the barrio fiesta and not for the purpose of favoring any
religion nor interfering with religious matters or beliefs of the barrio residents. Any activity intended to
facilitate the worship of the patron saint(such as the acquisition) is not illegal. Practically, the image was
placed in a laymans custody so that it could easily be made available to any family desiring to borrow the
image in connection with prayers and novena. It was the councils funds that were used to buy the image,
therefore it is their property. Right of the determination of custody is their right, and even if they decided to
give it to the Church, there is no violation of the Constitution, since private funds were used. Not every
government activity which involves the expenditure of public funds and which has some religious tint is
violative of the constitutional provisions regarding separation of church and state, freedom of worship and
banning the use of public money or property.
AMERICAN BIBLE SOCIETY VS. CITY OF MANILA
[101PHIL 386; G.R. NO. 9637; 30 APR 1957]
Facts:
New Yorks Education Law requires local public school authorities to lend textbooks free of charge to all
students in grade 7 to 12, including those in private schools. The Board of Education contended that said
statute was invalid and violative of the State and Federal Constitutions. An order barring the
Commissioner of Education (Allen) from removing appellants members from office for failure to comply
with the requirement and an order preventing the use of state funds for the purchase of textbooks to be
lent to parochial schools were sought for. The trial court held the statute unconstitutional. The Appellate
Division reversed the decision and dismissed the complaint since the appellant have no standing. The
New York Court of Appeals, ruled that the appellants have standing but the law is not unconstitutional.
Issue:
Whether or Not the said ordinances are constitutional and valid (contention: it restrains the free exercise
and enjoyment of the religious profession and worship of appellant).
Held:
Section 1, subsection (7) of Article III of the Constitution, provides that:
(7) No law shall be made respecting an establishment of religion, or prohibiting the free exercise
thereof, and the free exercise and enjoyment of religious profession and worship, without
discrimination or preference, shall forever be allowed. No religion test shall be required for the
exercise of civil or political rights.
The provision aforequoted is a constitutional guaranty of the free exercise and enjoyment of religious
profession and worship, which carries with it the right to disseminate religious information.
It may be true that in the case at bar the price asked for the bibles and other religious pamphlets was in
some instances a little bit higher than the actual cost of the same but this cannot mean that appellant was
engaged in the business or occupation of selling said "merchandise" for profit. For this reason. The Court
believe that the provisions of City of Manila Ordinance No. 2529, as amended, cannot be applied to
appellant, for in doing so it would impair its free exercise and enjoyment of its religious profession and
worship as well as its rights of dissemination of religious beliefs.
With respect to Ordinance No. 3000, as amended, the Court do not find that it imposes any charge upon
the enjoyment of a right granted by the Constitution, nor tax the exercise of religious practices.
It seems clear, therefore, that Ordinance No. 3000 cannot be considered unconstitutional, however
inapplicable to said business, trade or occupation of the plaintiff. As to Ordinance No. 2529 of the City of
Manila, as amended, is also not applicable, so defendant is powerless to license or tax the business of
plaintiff Society.

Page
Section 1-C, SY 06-07

98

San Beda College of Law Alabang


Constitutional Law 2 Case Digests
WHEREFORE, defendant shall return to plaintiff the sum of P5,891.45 unduly collected from it.
GERMAN VS. BARANGAN
[135 SCRA 514; G.R. NO. 68828; 27 MAR 1985]
Facts:
Petitioners converged at J.P. Laurel Street to hear Mass at the St. Jude Chapel, which adjoined
Malacaang. Respondent barred them for security reasons. Petitioners filed a petition for mandamus.
Issue:
Whether or Not there was a violation of the constitutional freedom.
Held:
Petitioners' intention was not really to perform an act of religious worship but to conduct an antigovernment demonstration since they wore yellow T-shirts, raised their clenched fists and shouted antigovernment slogans. While every citizen has the right to religious freedom, the exercise must be done in
good faith. Besides, the restriction was reasonable as it was designed to protect the lives of the President
and his family, government officials and diplomatic and foreign guests transacting business with
Malacanang. The restriction was also intended to secure the executive offices within the Malacanang
grounds from possible external attacks and disturbances. (Minority opinion) The sole justification for a
prior restraint or limitation on the exercise of the freedom of religion is the existence of a grave and
imminent, of a serious evil to public safety, public morals, public health or any other legitimate public
interest that the State has a right to prevent. The burden to show the existence of grave and imminent
danger lies on the officials who would restrain petitioners. Respondents were in full control and had the
capability to stop any untoward move. There was no clear and present danger of any serious evil to public
safety or the security of Malacanang.
EBRALINAG VS. DIVISION SUPERINTENDENT OF CEBU
[219 SCRA 256 ; G.R. NO. 95770; 1 MAR 1993]
Facts:
Two special civil actions for certiorari, Mandamus and Prohibition were filed and consolidated for raising
same issue. Petitioners allege that the public respondents acted without or in excess of their jurisdiction
and with grave abuse of discretion. Respondents ordered expulsion of 68 HS and GS students of
Bantayan, Pinamungajan, Caracar, Taburan and Asturias in Cebu. Public school authorities expelled
these students for refusing to salute the flag, sing the national anthem and recite the Panatang
Makabayan required by RA1265. They are Jehovahs Witnesses believing that by doing these is
religious worship/devotion akin to idolatry against their teachings. They contend that to compel
transcends constitutional limits and invades protection against official control and religious freedom. The
respondents relied on the precedence of Gerona et al v. Secretary of Education. Gerona doctrine
provides that we are a system of separation of the church and state and the flag is devoid of religious
significance and it doesnt involve any religious ceremony. The freedom of religious belief guaranteed by
the Constitution does not mean exception from non-discriminatory laws like the saluting of flag and
singing national anthem. This exemption disrupts school discipline and demoralizes the teachings of civic
consciousness and duties of citizenship.
Issue:
Whether or Not religious freedom has been violated.
Held:
Religious freedom is a fundamental right of highest priority. The 2 fold aspect of right to religious worship
is: 1.) Freedom to believe which is an absolute act within the realm of thought. 2.) Freedom to act on
ones belief regulated and translated to external acts. The only limitation to religious freedom is the
existence of grave and present danger to public safety, morals, health and interests where State has right
to prevent. The expulsion of the petitioners from the school is not justified.
The 30 yr old previous GERONA decision of expelling and dismissing students and teachers who refuse
to obey RA1265 is violates exercise of freedom of speech and religious profession and worship.
Jehovahs Witnesses may be exempted from observing the flag ceremony but this right does not give

Page
Section 1-C, SY 06-07

99

them the right to disrupt such ceremonies. In the case at bar, the Students expelled were only standing
quietly during ceremonies. By observing the ceremonies quietly, it doesnt present any danger so evil and
imminent to justify their expulsion. What the petitioners request is exemption from flag ceremonies and
not exclusion from public schools. The expulsion of the students by reason of their religious beliefs is also
a violation of a citizens right to free education. The non-observance of the flag ceremony does not totally
constitute ignorance of patriotism and civic consciousness. Love for country and admiration for national
heroes, civic consciousness and form of government are part of the school curricula. Therefore,
expulsion due to religious beliefs is unjustified.
Petition for Certiorari and Prohibition is GRANTED. Expulsion is ANNULLED.
FONACIER VS. COURT OF APPEALS
[96 PHIL 417; G.R. L-5917; 28 JAN 1955]
Facts:
Case was filed by Iglesia Filipina Independiente (IFI), represented by its supreme bishop Gerardo
Bayaca, against Bishop Fonacier seeking to render an accounting of his administration of all the temporal
properties and to recover the same on the ground that he ceased to be the supreme bishop of IFI. Isabelo
De los Reyes Jr. had been elected as the Supreme Bishop.
Petitioner claims that he was not properly removed as Supreme Bishop and his legal successor was Juan
Jamias. He claims that the there was an accounting of his administration and was turned over to bishop
Jamias. Also, that Isabelo De los Reyes and Bayaca have abandoned their faith and formally joined the
Prostestant Episcopal Church of America.
CFI rendered judgment declaring Isabelo De Los Reyes, Jr. as the sole and legitimate Supreme Bishop of
IFI and ordered Fonacier to render an accounting of his admistration
CA affirmed the decision of the CFI
Issue:
Whether or not the petitioner should still be regarded as the legitimate supreme bishop of IFI.
Held:
Supreme Court affirmed CAs decision. The legitimate Supreme Bishop of IFI is Isabelo De los Reyes, Jr.
The Supreme Court affirms the validity of the election of Bishop Delos Reyes as the Supreme Bishop
based on their internal laws
To finally dispose of the property issue, the Court, citing Watson v. Jones,368 declared that the rule in
property controversies within religious congregations strictly independent of any other superior
ecclesiastical association (such as the Philippine Independent Church) is that the rules for resolving such
controversies should be those of any voluntary association. If the congregation adopts the majority rule
then the majority should prevail; if it adopts adherence to duly constituted authorities within the
congregation, then that should be followed.
PAMIL VS. TELECOM
[86 SCRA 413; G.R. 34854; 20 NOV 1978]
Facts:
Fr. Margarito Gonzaga was elected as Municipal Mayor in Alburquerque, Bohol. Petitioner, also an
aspirant for said office, then filed a suit for quo warranto for Gonzagas disqualification based on the
Administrative Code provision: In no case shall there be elected or appointed to a municipal office
ecclesiastics, soldiers in active service, persons receiving salaries or compensation from provincial or
national funds, or contractors for public works of the municipality." The respondent Judge, in sustaiing Fr.
Gonzagas right to the office, ruled that the provision had already been impliedly repealed by the Election
Code of 1971. Petitioner on the other hand argues that there was no implied repeal.
Issue:
Whether or Not Fr. Gonzaga is eligible for the position of municipal mayor, according to law.

Page
Section 1-C, SY 06-07

100

San Beda College of Law Alabang


Constitutional Law 2 Case Digests
Whether or Not the prohibition regarding elected or appointed ecclesiastics is constitutional.
Held:
The court was divided. Five voted that the prohibition was not unconstitutional. Seven others voted that
the provision was impliedly repealed. However, the minority vote overruled the seven. According to the
dissenting seven, there are three reasons for the said provision to be inoperative. First, the 1935
Constitution stated, No religious test shall be required for the exercise of civil or political rights. Second,
said section 2175 is superseded by the Constitution. Third, section 2175 has been repealed by Sec. 23 of
the Election Code (1971): Appointive public office holders and active members of the Armed Forces are
no longer disqualified from running for an elective office. Ecclesiastics were no longer included in the
enumeration of persons ineligible under the said Election Code. On the other hand, the controlling five
argued: Section 2175 of the Administrative Code deals with a matter different from that of section 23 of
the Election Code. Also, section 2175 of the Administrative Code did not violate the right to freedom of
religion because it did not give any requirement for a religious test.
The view of the dissenting seven failed to obtain a vote of eight members, so it was not controlling. The
provision of the Administrative Code remained operative.
ESTRADA VS. ESCRITOR
[492 SCRA 1 ; AM NO P-02-1651; 22 JUN 2006]
Facts:
Escritor is a court interpreter since 1999 in the RTC of Las Pinas City. She has been living with Quilapio,
a man who is not her husband, for more than twenty five years and had a son with him as well.
Respondents husband died a year before she entered into the judiciary while Quilapio is still legally
married to another woman.
Complainant Estrada requested the Judge of said RTC to investigate respondent. According to
complainant, respondent should not be allowed to remain employed therein for it will appear as if the
court allows such act.
Respondent claims that their conjugal arrangement is permitted by her religionthe Jehovahs Witnesses
and the Watch Tower and the Bible Trace Society. They allegedly have a Declaration of Pledging
Faithfulness under the approval of their congregation. Such a declaration is effective when legal
impediments render it impossible for a couple to legalize their union.
Issue:
Whether or Not the State could penalize respondent for such conjugal arrangement.

Held:
No. The State could not penalize respondent for she is exercising her right to freedom of religion. The
free exercise of religion is specifically articulated as one of the fundamental rights in our Constitution. As
Jefferson put it, it is the most inalienable and sacred of human rights. The States interest in enforcing its
prohibition cannot be merely abstract or symbolic in order to be sufficiently compelling to outweigh a free
exercise claim. In the case at bar, the State has not evinced any concrete interest in enforcing the
concubinage or bigamy charges against respondent or her partner. Thus the States interest only
amounts to the symbolic preservation of an unenforced prohibition.
Furthermore, a distinction between public and secular morality and religious morality should be kept in
mind. The jurisdiction of the Court extends only to public and secular morality.
The Court further states that our Constitution adheres the benevolent neutrality approach that gives room
for accommodation of religious exercises as required by the Free Exercise Clause. This benevolent
neutrality could allow for accommodation of morality based on religion, provided it does not offend
compelling state interests. Assuming arguendo that the OSG has proved a compelling state interest, it
has to further demonstrate that the state has used the least intrusive means possible so that the free
exercise is not infringed any more than necessary to achieve the legitimate goal of the state. Thus the
conjugal arrangement cannot be penalized for it constitutes an exemption to the law based on her right to
freedom of religion.

Page
Section 1-C, SY 06-07

101

ISLAMIC DA'WAH COUNCIL OF THE PHILIPPINES VS. EXECUTIVE SECRETARY


[405 SCRA 497;GR 153888; 9 JUL 2003]
Facts:
Petitioner Islamic Da'wah Council of the Philippines, Inc. (IDCP) is a corporation that operates under
Department of Social Welfare and Development, a non-governmental organization that extends voluntary
services to the Filipino people, especially to Muslim communities. It claims to be a federation of national
Islamic organizations and an active member of international organizations such as the Regional Islamic
Da'wah Council of Southeast Asia and the Pacific (RISEAP) and The World Assembly of Muslim Youth.
The RISEAP accredited petitioner to issue halal certifications in the Philippines. Thus, among the
functions petitioner carries out is to conduct seminars, orient manufacturers on halal food and issue halal
certifications to qualified products and manufacturers.
Petitioner alleges that, the actual need to certify food products as halal and also due to halal food
producers' request, petitioner formulated in 1995 internal rules and procedures based on the Qur'an and
the Sunnah for the analysis of food, inspection thereof and issuance of halal certifications. In that same
year, petitioner began to issue, for a fee, certifications to qualified products and food manufacturers.
Petitioner even adopted for use on its halal certificates a distinct sign or logo registered in the Philippine
Patent Office.
On 2001, respondent Office of the Executive Secretary issued EO 465 creating the Philippine Halal
Certification Scheme and designating respondent OMA to oversee its implementation. Under the EO,
respondent OMA has the exclusive authority to issue halal certificates and perform other related
regulatory activities.
Issue:
Whether or Not EO violates the constitutional provision on the separation of Church and State.
Held:
It is unconstitutional for the government to formulate policies and guidelines on the halal certification
scheme because said scheme is a function only religious organizations, entity or scholars can lawfully
and validly perform for the Muslims. According to petitioner, a food product becomes halal only after the
performance of Islamic religious ritual and prayer. Thus, only practicing Muslims are qualified to slaughter
animals for food. A government agency like herein respondent OMA cannot therefore perform a religious
function like certifying qualified food products as halal. Without doubt, classifying a food product as halal
is a religious function because the standards used are drawn from the Qur'an and Islamic beliefs. By
giving OMA the exclusive power to classify food products as halal, EO 46 encroached on the religious
freedom of Muslim organizations like herein petitioner to interpret for Filipino Muslims what food products
are fit for Muslim consumption. Also, by arrogating to itself the task of issuing halal certifications, the State
has in effect forced Muslims to accept its own interpretation of the Qur'an and Sunnah on halal food.
In the case at bar, we find no compelling justification for the government to deprive Muslim organizations,
like herein petitioner, of their religious right to classify a product as halal, even on the premise that the
health of Muslim Filipinos can be effectively protected by assigning to OMA the exclusive power to issue
halal certifications. The protection and promotion of the muslim Filipinos' right to health are already
provided for in existing laws and ministered to by government agencies charged with ensuring that food
products released in the market are fit for human consumption, properly labeled and safe. Unlike EO 46,
these laws do not encroach on the religious freedom of Muslims.

Page
Section 1-C, SY 06-07

102

San Beda College of Law Alabang


Constitutional Law 2 Case Digests
FREEDOM OF EXPRESSION
Art 3, Sec. 4. No law shall be passed abridging the freedom of speech, of expression, or of the press,
or the right of the people peaceably to assemble and petition the government for redress of grievances.
Art 3, Sec. 7. The right of the people to information on matters of public concern shall be recognized.
Access to official records, and to documents, and papers pertaining to official acts, transactions, or
decisions, as well as to government research data used as basis for policy development, shall be
afforded the citizen, subject to such limitations as may be provided by law.
Art 3, Sec. 8. The right of the people, including those employed in the public and private sectors, to
form unions, associations, or societies for purposes not contrary to law shall not be abridged.
Art 3, Sec. 18. (1) No person shall be detained solely by reason of his political beliefs and aspirations.
NEAR VS. MINNESOTA
[283 US 697]
Facts:
A complaint alleged that the defendants, on September 24, 1927, and on eight subsequent dates in
October and November, 1927, published and circulated editions of The Saturday Press(published in
Minneapolis) which were 'largely devoted to malicious, scandalous and defamatory articles'(based on
Session Laws of Minnesota). The articles charged, in substance, provides that a Jewish gangster was in
control of gambling, bootlegging, and racketeering in Minneapolis, and that law enforcing officers and
agencies were not energetically performing their duties. Most of the charges were directed against the
chief of police; he was charged with gross neglect of duty, illicit relations with gangsters, and with
participation in graft. The county attorney was charged with knowing the existing conditions and with
failure to take adequate measures to remedy them. The mayor was accused of inefficiency and
dereliction. One member of the grand jury was stated to be in sympathy with the gangsters. A special
grand jury and a special prosecutor were demanded to deal with the situation in general, and, in
particular, to investigate an attempt to assassinate one Guilford, one of the original defendants, who, it
appears from the articles, was shot by gangsters after the first issue of the periodical had been published.
Now defendants challenged the Minnesota statute which provides for the abatement, as a public
nuisance, of a malicious, scandalous and defamatory news paper, magazine or other periodical. The
District Court ruled against defendants. Hence the appeal.
Issue:
Whether or Not the proceeding authorized by the statute herein constitutes an infringement of the
freedom of the press.
Held:
Yes. The insistence that the statute is designed to prevent the circulation of scandal which tends to disturb
the public peace and to provoke assaults and the commission of crime is unavailing.
The reason for the enactment, as the state court has said, is that prosecutions to enforce penal statutes
for libel do not result in 'efficient repression or suppression of the evils of scandal.' In the present instance,
the proof was that nine editions of the newspaper or periodical in question were published on successive
dates, and that they were chiefly devoted to charges against public officers and in relation to the
prevalence and protection of crime. In such a case, these officers are not left to their ordinary remedy in a
suit for libel, or the authorities to a prosecution for criminal libel. The statute not only operates to suppress
the offending newspaper or periodical, but to put the publisher under an effective censorship.
Every freeman has an undoubted right to lay what sentiments he pleases before the public; to forbid this,
is to destroy the freedom of the press; but if he publishes what is improper, mischievous or illegal, he
must take the consequence of his own temerity.
The liberty of the press was to be unrestrained, but he who used it was to be responsible in case of its
abuse.' Public officers, whose character and conduct remain open to debate and free discussion in the
press, find their remedies for false accusations in actions under libel laws providing for redress and
punishment, and not in proceedings to restrain the publication of newspapers and periodicals.

Page
Section 1-C, SY 06-07

103

Characterizing the publication as a business, and the business as a nuisance, does not permit an
invasion of the constitutional immunity against restraint. Nor can it be said that the constitutional freedom
from previous restraint is lost because charges are made of derelictions which constitute crimes.
The preliminary freedom, by virtue of the very reason for its existence, does not depend, as this court has
said, on proof of truth.
GROSJEAN VS. AMERICAN PRESS CO.
[297 US 233]
Facts:
The nine publishers(corporations) who brought the suit publish thirteen newspapers and these thirteen
publications are the only ones within the state of Louisiana having each a circulation of more than 20,000
copies per week. The suit assailed Act No. 231 of the Louisiana Legislature, as their freedom of the press
was abridged in contravention to the due process clause.
Issue:
Whether or Not Act 23 unconstitutional.
Held:
Yes. Freedom of speech and of the press are rights of the same fundamental character, safeguarded by
the due process of law clause. The word 'liberty' contained in that amendment embraces not only the right
of a person to be free from physical restraint, but the right to be free in the enjoyment of all his faculties as
well.
The Act operates as a restraint in a double sense. First, its effect is to curtail the amount of revenue
realized from advertising; and, second, its direct tendency is to restrict circulation. This is plain enough
when we consider that, if it were increased to a high degree, as it could be if valid it well might result in
destroying both advertising and circulation.
Judge Cooley has laid down the test to be applied: The evils to be prevented were not the censorship of
the press merely, but any action of the government by means of which it might prevent such free and
general discussion of public matters as seems absolutely essential to prepare the people for an intelligent
exercise of their rights as citizens.
The tax here involved is bad not because it takes money from the pockets of the appellees. It is bad
because, it is seen to be a deliberate and calculated device in the guise of a tax to limit the circulation of
information to which the public is entitled in virtue of the constitutional guaranties. A free press stands as
one of the great interpreters between the government and the people.
The form in which the tax is imposed is in itself suspicious. It is not measured or limited by the volume of
advertisements. It is measured alone by the extent of the circulation of the publication in which the
advertisements are carried, with the plain purpose of penalizing the publishers and curtailing the
circulation of a selected group of newspapers.
NEW YORK TIMES VS. UNITED STATES
[403 US 713]
Facts:
The court granted certiorari in the cases in which the United States seeks to enjoin the New York Times
and the Washington Post from publishing the contents of a classified study entitled "History of U.S.
Decision-Making Process on Viet Nam Policy." Said articles reveal the workings of government that led to
the Vietnam war. The Government argues that "the authority of the Executive Department to protect the
nation against publication of information whose disclosure would endanger the national security stems
from two interrelated sources: the constitutional power of the President over the conduct of foreign affairs
and his authority as Commander-in-Chief. In such case the Executive Branch seeks judicial aid in
preventing publication. The court ruled in favor of the newspaper companies hence the appeal.
Issue:
Whether or not the freedom of the press was abridged.

Page
Section 1-C, SY 06-07

104

San Beda College of Law Alabang


Constitutional Law 2 Case Digests

Held:
Yes. To find that the President has "inherent power" to halt the publication of news by resort to the courts
would wipe out the First Amendment (Bill of Rights) and destroy the fundamental liberty and security of
the very people the Government hopes to make "secure."
No branch of government could abridge the people's rights granted by the Constitution including the
freedom of the press. The language of the First Amendment support the view that the press must be left
free to publish news, whatever the source, without censorship, injunctions, or prior restraints. The press
was protected so that it could bare the secrets of government and inform the people. Only a free and
unrestrained press can effectively expose deception in government. And paramount among the
responsibilities of a free press is the duty to prevent any part of the government from deceiving the people
and sending them off to distant lands to die of foreign fevers and foreign shot and shell.
GONZALES VS. COMELEC
[27 SCRA 835; G.R. L-27833; 18 APR 1969]
Facts:
RA 4880 which took effect on June 17, 1967, prohibiting the too early nomination of candidates and
limiting the period of election campaign or partisan political activity was challenged on constitutional
grounds. More precisely, the basic liberties of free speech and free press, freedom of assembly and
freedom of association are invoked to nullify the act. Petitioner Cabigao was, at the time of the filing the
petition, an incumbent councilor in the 4th District of Manila and the Nacionalista Party official candidate
for Vice-Mayor of Manila to which he was subsequently elected on November 11, 1967; petitioner
Gonzales, on the other hand, is a private individual, a registered voter in the City of Manila and a political
leader of his co-petitioner. There was the further allegation that the nomination of a candidate and the
fixing of period of election campaign are matters of political expediency and convenience which only
political parties can regulate or curtail by and among themselves through self-restraint or mutual
understanding or agreement and that the regulation and limitation of these political matters invoking the
police power, in the absence of clear and present danger to the state, would render the constitutional
rights of petitioners meaningless and without effect. Senator Lorenzo M. Taada was asked to appear as
amicus curiae, and elucidated that Act No. 4880 could indeed be looked upon as a limitation on the
preferred rights of speech and press, of assembly and of association. He did justify its enactment
however under the clear and present danger doctrine, there being the substantive evil of elections,
whether for national or local officials, being debased and degraded by unrestricted campaigning, excess
of partisanship and undue concentration in politics with the loss not only of efficiency in government but of
lives as well. The Philippine Bar Association, the Civil Liberties Union, the U.P. Law Center and the U.P.
Women Lawyers' Circle were requested to give their opinions. Respondents contend that the act was
based on the police power of the state.
Issue:
Whether or Not RA 4880 unconstitutional.
Held:
Yes. As held in Cabansag v. Fernandez there are two tests that may supply an acceptable criterion for
permissible restriction on freedom of speech. These are the clear and present danger rule and the
'dangerous tendency' rule. The first, means that the evil consequence of the comment or utterance must
be extremely serious and the degree of imminence extremely high before the utterance can be punished.
The danger to be guarded against is the 'substantive evil' sought to be prevented. It has the advantage of
establishing according to the above decision a definite rule in constitutional law. It provides the criterion as
to what words may be publicly established. The "dangerous tendency rule" is such that If the words
uttered create a dangerous tendency which the state has a right to prevent, then such words are
punishable. It is not necessary that some definite or immediate acts of force, violence, or unlawfulness
be advocated. It is sufficient that such acts be advocated in general terms. Nor is it necessary that the
language used be reasonably calculated to incite persons to acts of force, violence, or unlawfulness. It is
sufficient if the natural tendency and probable effect of the utterance be to bring about the substantive evil
which the legislative body seeks to prevent.
The challenged statute could have been more narrowly drawn and the practices prohibited more precisely
delineated to satisfy the constitutional requirements as to a valid limitation under the clear and present
danger doctrine. As the author Taada clearly explained, such provisions were deemed by the legislative

Page
Section 1-C, SY 06-07

105

body to be part and parcel of the necessary and appropriate response not merely to a clear and present
danger but to the actual existence of a grave and substantive evil of excessive partisanship, dishonesty
and corruption as well as violence that of late has invariably marred election campaigns and partisan
political activities in this country.
The very idea of a government, republican in form, implies a right on the part of its citizens to meet
peaceably for consultation in respect to public affairs and to petition for redress of grievances. As in the
case of freedom of expression, this right is not to be limited, much less denied, except on a showing of a
clear and present danger of a substantive evil that Congress has a right to prevent.
The prohibition of any speeches, announcements or commentaries, or the holding of interviews for
against the election of any party or candidate for public office and the prohibition of the publication
distribution of campaign literature or materials, against the solicitation of votes whether directly
indirectly, or the undertaking of any campaign literature or propaganda for or against any candidate
party is repugnant to a constitutional command.

or
or
or
or

IGLESIA NI CRISTO VS. COURT OF APPEALS


[259 SCRA 529; G.R. NO. 119673; 26 JUL 1996]
Facts:
Petitioner has a television program entitled "Ang Iglesia ni Cristo" aired on Channel 2 every Saturday and
on Channel 13 every Sunday. The program presents and propagates petitioner's religious beliefs,
doctrines and practices often times in comparative studies with other religions. Petitioner submitted to the
respondent Board of Review for Moving Pictures and Television the VTR tapes of its TV program Series
Nos. 116, 119, 121 and 128. The Board classified the series as "X" or not for public viewing on the ground
that they "offend and constitute an attack against other religions which is expressly prohibited by law." On
November 28, 1992, it appealed to the Office of the President the classification of its TV Series No. 128
which allowed it through a letter of former Executive Secretary Edelmiro A. Amante, Sr., addressed for
Henrietta S. Mendez reversing the decision of the respondent Board. According to the letter the episode
in is protected by the constitutional guarantee of free speech and expression and no indication that the
episode poses any clear and present danger. Petitioner also filed Civil Case. Petitioner alleged that the
respondent Board acted without jurisdiction or with grave abuse of discretion in requiring petitioner to
submit the VTR tapes of its TV program and in x-rating them. It cited its TV Program Series Nos. 115,
119, 121 and 128. In their Answer, respondent Board invoked its power under PD No. 1986 1 in relation to
Article 201 of the Revised Penal Code. The Iglesia ni Cristo insists on the literal translation of the bible
and says that our (Catholic) veneration of the Virgin Mary is not to be condoned because nowhere it is
found in the bible. The board contended that it outrages Catholic and Protestant's beliefs. RTC ruled in
favor of petitioners. CA however reversed it hence this petition.
Issue:
Whether or Not the "ang iglesia ni cristo" program is not constitutionally protected as a form of religious
exercise and expression.
Held:
Yes. Any act that restrains speech is accompanied with presumption of invalidity. It is the burden of the
respondent Board to overthrow this presumption. If it fails to discharge this burden, its act of censorship
will be struck down. This is true in this case. So-called "attacks" are mere criticisms of some of the deeply
held dogmas and tenets of other religions. RTCs ruling clearly suppresses petitioner's freedom of speech
and interferes with its right to free exercise of religion. attack is different from offend any race or
religion. The respondent Board may disagree with the criticisms of other religions by petitioner but that
gives it no excuse to interdict such criticisms, however, unclean they may be. Under our constitutional
scheme, it is not the task of the State to favor any religion by protecting it against an attack by another
religion. Religious dogmas and beliefs are often at war and to preserve peace among their followers,
especially the fanatics, the establishment clause of freedom of religion prohibits the State from leaning
towards any religion. Respondent board cannot censor the speech of petitioner Iglesia ni Cristo simply
because it attacks other religions, even if said religion happens to be the most numerous church in our
country. The basis of freedom of religion is freedom of thought and it is best served by encouraging the
marketplace of dueling ideas. It is only where it is unavoidably necessary to prevent an immediate and
grave danger to the security and welfare of the community that infringement of religious freedom may be
justified, and only to the smallest extent necessary to avoid the danger. There is no showing whatsoever
of the type of harm the tapes will bring about especially the gravity and imminence of the threatened

Page
Section 1-C, SY 06-07

106

San Beda College of Law Alabang


Constitutional Law 2 Case Digests
harm. Prior restraint on speech, including religious speech, cannot be justified by hypothetical fears but
only by the showing of a substantive and imminent evil. It is inappropriate to apply the clear and present
danger test to the case at bar because the issue involves the content of speech and not the time, place or
manner of speech. Allegedly, unless the speech is first allowed, its impact cannot be measured, and the
causal connection between the speech and the evil apprehended cannot be established. The
determination of the question as to whether or not such vilification, exaggeration or fabrication falls within
or lies outside the boundaries of protected speech or expression is a judicial function which cannot be
arrogated by an administrative body such as a Board of Censors." A system of prior restraint may only be
validly administered by judges and not left to administrative agencies.
ADIONG VS. COMELEC
[207 SCRA 712; G.R. NO. 103956; 31 MAR 1992]
Facts:
COMELEC promulgated Resolution No. 2347 which provides that decals and stickers may be posted only
in any of the authorized posting areas, prohibiting posting in "mobile" places, public or private. Petitioner
Blo Umpar Adiong, a senatorial candidate in the May 11, 1992 elections now assails the Resolution. In
addition, the petitioner believes that with the ban on radio, television and print political advertisements, he,
being a neophyte in the field of politics stands to suffer grave and irreparable injury with this prohibition.
Issue:
Whether or Not the COMELECs prohibition unconstitutional.
Held:
The prohibition unduly infringes on the citizen's fundamental right of free speech. The preferred freedom
of expression calls all the more for the utmost respect when what may be curtailed is the dissemination of
information to make more meaningful the equally vital right of suffrage. The so-called balancing of
interests individual freedom on one hand and substantial public interests on the other is made even
more difficult in election campaign cases because the Constitution also gives specific authority to the
Commission on Elections to supervise the conduct of free, honest, and orderly elections. When faced with
border line situations where freedom to speak by a candidate or party and freedom to know on the part of
the electorate are invoked against actions intended for maintaining clean and free elections, the police,
local officials and COMELEC, should lean in favor of freedom. The regulation of election campaign
activity may not pass the test of validity if it is too general in its terms or not limited in time and scope in its
application, if it restricts one's expression of belief in a candidate or one's opinion of his or her
qualifications, if it cuts off the flow of media reporting, and if the regulatory measure bears no clear and
reasonable nexus with the constitutionally sanctioned objective.
The posting of decals and stickers in mobile places like cars and other moving vehicles does not
endanger any substantial government interest. There is no clear public interest threatened by such
activity so as to justify the curtailment of the cherished citizen's right of free speech and expression.
Under the clear and present danger rule not only must the danger be patently clear and pressingly
present but the evil sought to be avoided must be so substantive as to justify a clamp over one's mouth or
a writing instrument to be stilled. The regulation strikes at the freedom of an individual to express his
preference and, by displaying it on his car, to convince others to agree with him. A sticker may be
furnished by a candidate but once the car owner agrees to have it placed on his private vehicle, the
expression becomes a statement by the owner, primarily his own and not of anybody else. The restriction
as to where the decals and stickers should be posted is so broad that it encompasses even the citizen's
private property, which in this case is a privately-owned vehicle. In consequence of this prohibition,
another cardinal rule prescribed by the Constitution would be violated. Section 1, Article III of the Bill of
Rights provides that no person shall be deprived of his property without due process of law.
The prohibition on posting of decals and stickers on "mobile" places whether public or private except in
the authorized areas designated by the COMELEC becomes censorship.
NATIONAL PRESS CLUB VS. COMELEC
[201 SCRA 1; G.R. NO. 1026653; 5 MAR 1992]
Facts:
Petitioners in these cases consist of representatives of the mass media which are prevented from selling
or donating space and time for political advertisements; two (2) individuals who are candidates for office

Page
Section 1-C, SY 06-07

107

(one for national and the other for provincial office) in the coming May 1992 elections; and taxpayers and
voters who claim that their right to be informed of election Issue and of credentials of the candidates is
being curtailed. It is principally argued by petitioners that Section 11 (b) of Republic Act No. 6646 1 invades
and violates the constitutional guarantees comprising freedom of expression. Petitioners maintain that the
prohibition imposed by Section 11 (b) amounts to censorship, because it selects and singles out for
suppression and repression with criminal sanctions, only publications of a particular content, namely,
media-based election or political propaganda during the election period of 1992. It is asserted that the
prohibition is in derogation of media's role, function and duty to provide adequate channels of public
information and public opinion relevant to election Issue. Further, petitioners contend that Section 11 (b)
abridges the freedom of speech of candidates, and that the suppression of media-based campaign or
political propaganda except those appearing in the Comelec space of the newspapers and on Comelec
time of radio and television broadcasts, would bring about a substantial reduction in the quantity or
volume of information concerning candidates and Issue in the election thereby curtailing and limiting the
right of voters to information and opinion.
Issue:
Whether or Not Section 11 (b) of Republic Act No. 6646 constitutional.
Held:
Yes. It seems a modest proposition that the provision of the Bill of Rights which enshrines freedom of
speech, freedom of expression and freedom of the press has to be taken in conjunction with Article IX (C)
(4) which may be seen to be a special provision applicable during a specific limited period i.e., "during
the election period." In our own society, equality of opportunity to proffer oneself for public office, without
regard to the level of financial resources that one may have at one's disposal, is clearly an important
value. One of the basic state policies given constitutional rank by Article II, Section 26 of the Constitution
is the egalitarian demand that "the State shall guarantee equal access to opportunities for public service
and prohibit political dynasties as may be defined by law." The essential question is whether or not the
assailed legislative or administrative provisions constitute a permissible exercise of the power of
supervision or regulation of the operations of communication and information enterprises during an
election period, or whether such act has gone beyond permissible supervision or regulation of media
operations so as to constitute unconstitutional repression of freedom of speech and freedom of the press.
The Court considers that Section 11 (b) has not gone outside the permissible bounds of supervision or
regulation of media operations during election periods.
Section 11 (b) is limited in the duration of its applicability and enforceability. By virtue of the operation of
Article IX (C) (4) of the Constitution, Section 11 (b) is limited in its applicability in time to election periods.
Section 11 (b) does not purport in any way to restrict the reporting by newspapers or radio or television
stations of news or news-worthy events relating to candidates, their qualifications, political parties and
programs of government. Moreover, Section 11 (b) does not reach commentaries and expressions of
belief or opinion by reporters or broadcasters or editors or commentators or columnists in respect of
candidates, their qualifications, and programs and so forth, so long at least as such comments, opinions
and beliefs are not in fact advertisements for particular candidates covertly paid for. In sum, Section 11 (b)
is not to be read as reaching any report or commentary other coverage that, in responsible media, is not
paid for by candidates for political office. Section 11 (b) as designed to cover only paid political
advertisements of particular candidates.
The limiting impact of Section 11 (b) upon the right to free speech of the candidates themselves is not
unduly repressive or unreasonable.
US VS. BUSTOS
[37 PHIL. 731; G.R. L-12592; 8 MAR 1918]
Facts:
In the latter part of 1915, numerous citizens of the Province of Pampanga assembled, and prepared and
signed a petition to the Executive Secretary(privileged communication) through the law office of Crossfield
and O'Brien, and five individuals signed affidavits, charging Roman Punsalan, justice of the peace of
Macabebe and Masantol, Pampanga, with malfeasance in office and asking for his removal. The specific
charges against the justice of the peace include the solicitation of money from persons who have pending
cases before the judge. Now, Punsalan alleged that accused published a writing which was false,
scandalous, malicious, defamatory, and libelous against him.
Issue:

Page
Section 1-C, SY 06-07

108

San Beda College of Law Alabang


Constitutional Law 2 Case Digests
Whether or Not accused is entitled to constitutional protection by virtue of his right to free speech and free
press.
Held:
Yes. The guaranties of a free speech and a free press include the right to criticize judicial conduct. The
administration of the law is a matter of vital public concern. Whether the law is wisely or badly enforced is,
therefore, a fit subject for proper comment. If the people cannot criticize a justice of the peace or a judge
the same as any other public officer, public opinion will be effectively suppressed. It is a duty which every
one owes to society or to the State to assist in the investigation of any alleged misconduct. It is further the
duty of all who know of any official dereliction on the part of a magistrate or the wrongful act of any public
officer to bring the facts to the notice of those whose duty it is to inquire into and punish them.
The right to assemble and petition is the necessary consequence of republican institutions and the
complement of the part of free speech. Assembly means a right on the part of citizens to meet peaceably
for consultation in respect to public affairs. Petition means that any person or group of persons can apply,
without fear of penalty, to the appropriate branch or office of the government for a redress of grievances.
The persons assembling and petitioning must, of course, assume responsibility for the charges made. All
persons have an interest in the pure and efficient administration of justice and of public affairs.
Public policy, the welfare of society, and the orderly administration of government have demanded
protection for public opinion. The inevitable and incontestable result has been the development and
adoption of the doctrine of privilege. All persons have an interest in the pure and efficient administration of
justice and of public affairs. The duty under which a party is privileged is sufficient if it is social or moral in
its nature and this person in good faith believes he is acting in pursuance thereof although in fact he is
mistaken. Although the charges are probably not true as to the justice of the peace, they were believed to
be true by the petitioners. Good faith surrounded their action. Probable cause for them to think that
malfeasance or misfeasance in office existed is apparent. The ends and the motives of these citizens to
secure the removal from office of a person thought to be venal were justifiable. In no way did they
abuse the privilege.
In the usual case malice can be presumed from defamatory words. Privilege destroys that presumption. A
privileged communication should not be subjected to microscopic examination to discover grounds of
malice or falsity.
PITA VS. COURT OF APPEALS
[178 SCRA 362; G.R. NO.80806; 5 OCT 1989]
Facts:
On December 1 and 3, 1983, pursuing an Anti-Smut Campaign initiated by the Mayor of the City of
Manila, Ramon D. Bagatsing, elements of the Special Anti-Narcotics Group, Auxilliary Services Bureau,
Western Police District, INP of the Metropolitan Police Force of Manila, seized and confiscated from
dealers, distributors, newsstand owners and peddlers along Manila sidewalks, magazines, publications
and other reading materials believed to be obscene, pornographic and indecent and later burned the
seized materials in public at the University belt along C.M. Recto Avenue, Manila, in the presence of
Mayor Bagatsing and several officers and members of various student organizations.
Among the publications seized, and later burned, was "Pinoy Playboy" magazines published and coedited by plaintiff Leo Pita.
Plaintiff filed a case for injunction with prayer for issuance of the writ of preliminary injunction against
Mayor Bagatsing and Narcisco Cabrera, as superintendent of Western Police District of the City of
Manila, seeking to enjoin said defendants and their agents from confiscating plaintiffs magazines or from
preventing the sale or circulation thereof claiming that the magazine is a decent, artistic and educational
magazine which is not per se obscene, and that the publication is protected by the Constitutional
guarantees of freedom of speech and of the press. Plaintiff also filed an Urgent Motion for issuance of a
temporary restraining order against indiscriminate seizure, confiscation and burning of plaintiff's "Pinoy
Playboy" Magazines, pending hearing on the petition for preliminary injunction. The Court granted the
temporary restraining order. The case was set for trial upon the lapse of the TRO. RTC ruled that the
seizure was valid. This was affirmed by the CA.
Issue:
Whether or Not the seizure violative of the freedom of expression of the petitioner.

Page
Section 1-C, SY 06-07

109

Held:
Freedom of the press is not without restraint as the state has the right to protect society from
pornographic literature that is offensive to public morals, as indeed we have laws punishing the author,
publishers and sellers of obscene publications. However, It is easier said than done to say, that if the
pictures here in question were used not exactly for art's sake but rather for commercial purposes, the
pictures are not entitled to any constitutional protection. Using the Kottinger rule: the test of obscenity is
"whether the tendency of the matter charged as obscene, is to deprave or corrupt those whose minds are
open to such immoral influences and into whose hands a publication or other article charged as being
obscene may fall." Another is whether it shocks the ordinary and common sense of men as an indecency.
Ultimately "whether a picture is obscene or indecent must depend upon the circumstances of the case
and that the question is to be decided by the "judgment of the aggregate sense of the community reached
by it." The government authorities in the instant case have not shown the required proof to justify a ban
and to warrant confiscation of the literature First of all, they were not possessed of a lawful court order: (1)
finding the said materials to be pornography, and (2) authorizing them to carry out a search and seizure,
by way of a search warrant. The court provides that the authorities must apply for the issuance of a
search warrant from a judge, if in their opinion an obscenity seizure is in order and that;
1. The authorities must convince the court that the materials sought to be seized are obscene and
pose a clear and present danger of an evil substantive enough to warrant State interference and
action;
2. The judge must determine whether or not the same are indeed obscene. The question is to be
resolved on a case-to-case basis and on the judges sound discretion;
AYER PRODUCTIONS VS. CAPULONG
[160 SCRA 861; G.R. NO. L-82380; 29 APR 1988]
Facts:
Petitioner McElroy an Australian film maker, and his movie production company, Ayer Productions,
envisioned, sometime in 1987, for commercial viewing and for Philippine and international release, the
historic peaceful struggle of the Filipinos at EDSA. The proposed motion picture entitled "The Four Day
Revolution" was endorsed by the MTRCB as and other government agencies consulted. Ramos also
signified his approval of the intended film production.
It is designed to be viewed in a six-hour mini-series television play, presented in a "docu-drama" style,
creating four fictional characters interwoven with real events, and utilizing actual documentary footage as
background. David Williamson is Australia's leading playwright and Professor McCoy (University of New
South Wales) is an American historian have developed a script.
Enrile declared that he will not approve the use, appropriation, reproduction and/or exhibition of his name,
or picture, or that of any member of his family in any cinema or television production, film or other medium
for advertising or commercial exploitation. petitioners acceded to this demand and the name of Enrile was
deleted from the movie script, and petitioners proceeded to film the projected motion picture. However, a
complaint was filed by Enrile invoking his right to privacy. RTC ordered for the desistance of the movie
production and making of any reference to plaintiff or his family and from creating any fictitious character
in lieu of plaintiff which nevertheless is based on, or bears substantial or marked resemblance to Enrile.
Hence the appeal.
Issue:
Whether or Not freedom of expression was violated.

Held:
Yes. Freedom of speech and of expression includes the freedom to film and produce motion pictures and
exhibit such motion pictures in theaters or to diffuse them through television. Furthermore the
circumstance that the production of motion picture films is a commercial activity expected to yield
monetary profit, is not a disqualification for availing of freedom of speech and of expression.
The projected motion picture was as yet uncompleted and hence not exhibited to any audience. Neither
private respondent nor the respondent trial Judge knew what the completed film would precisely look like.

Page
Section 1-C, SY 06-07

110

San Beda College of Law Alabang


Constitutional Law 2 Case Digests
There was, in other words, no "clear and present danger" of any violation of any right to privacy. Subject
matter is one of public interest and concern. The subject thus relates to a highly critical stage in the
history of the country.
At all relevant times, during which the momentous events, clearly of public concern, that petitioners
propose to film were taking place, Enrile was a "public figure:" Such public figures were held to have lost,
to some extent at least, their right to privacy.
The line of equilibrium in the specific context of the instant case between the constitutional freedom of
speech and of expression and the right of privacy, may be marked out in terms of a requirement that the
proposed motion picture must be fairly truthful and historical in its presentation of events.
LOPEZ VS. SANDIGANBAYAN
[34 SCRA 116; L-26549; 31 JUL 1970]
Facts:
In the early part of January, 1956, there appeared on the front page of The Manila Chronicle, of which
petitioner Lopez was the publisher, as well as on other dailies, a news story of a sanitary inspector
assigned to the Babuyan Islands, Fidel Cruz, sending a distress signal to a passing United States Airforce
plane which in turn relayed the message to Manila. An American Army plane dropping on the beach of an
island an emergency-sustenance kit containing, among other things, a two-way radio set. He utilized it to
inform authorities in Manila that the people in the place were living in terror, due to a series of killings
committed since Christmas of 1955. Losing no time, the Philippines defense establishment rushed to the
island a platoon of scout rangers. Upon arriving Major Encarnacion and his men found, instead of the
alleged killers, a man named Fidel Cruz who merely wanted transportation home to Manila. In view of this
finding, Major Encarnacion branded as a "hoax," the report of respondent.
This Week Magazine of the Manila Chronicle, then edited by Gatbonton, devoted a pictorial article to it in
its issue of January 15, 1956. Mention was made that while Fidel Cruz story turned out to be false it
brought attention to the government that people in that most people in the area are sick sick, only two
individuals able to read and write, food and clothing being scarce.
The magazine carried photographs of the person purporting to be Fidel Cruz. Unfortunately, the pictures
that were published were that of private respondent Fidel G. Cruz, a businessman contractor from Santa
Maria, Bulacan. It turned out that the photographs of respondent Cruz and that of Fidel Cruz, sanitary
inspector, were on file in the library of the Manila Chronicle but when the news quiz format was prepared,
the two photographs were in advertently switched. However a correction was published immediately.
Respondent sued petitioners in the Court of First Instance of Manila for the recovery of damages alleging
the defamatory character of the above publication of his picture. Defense interposed that they are beating
the deadline. The court ruled in his favor. Hence the appeal.
Issue:
Whether or Not petitioners abused the freedom of the press.
Held:
No. The SC, quoting Quisumbing v. Lopez, found for plaintiff, but with reduced damages, since the error
in this case could have been checked considering that this was a weekly magazine and not a daily. The
ruling: "there is no evidence in the record to prove that the publication of the news item under
consideration was prompted by personal ill will or spite, or that there was intention to do harm,' and that
on the other hand there was 'an honest and high sense of duty to serve the best interests of the public,
without self-seeking motive and with malice towards none.' Every citizen of course has the right to enjoy a
good name and reputation, but we do not consider that the respondents, under the circumstances of this
case, had violated said right or abused the freedom of the press. The newspapers should be given such
leeway and tolerance as to enable them to courageously and effectively perform their important role in our
democracy. In the preparation of stories, press reporters and editors usually have to race with their
deadlines; and consistently with good faith and reasonable care, they should not be held to account, to a
point of suppression, for honest mistakes or imperfection in the choice of words.
No inroads on press freedom should be allowed in the guise of punitive action visited on what otherwise
could be characterized as libel whether in the form of printed words or a defamatory imputation resulting
from the publication of respondent's picture with the offensive caption as in the case here complained of.
This is merely to underscore the primacy that freedom of the press enjoys.

Page
Section 1-C, SY 06-07

111

PRIMICIAS VS. FUGOSO


[80 PHIL 71; L-1800; 27 JAN 1948]
Facts:
An action was instituted by the petitioner for the refusal of the respondent to issue a permit to them to
hold a public meeting in Plaza Miranda for redress of grievances to the government. The reason alleged
by the respondent in his defense for refusing the permit is, "that there is a reasonable ground to believe,
basing upon previous utterances and upon the fact that passions, specially on the part of the losing
groups, remains bitter and high, that similar speeches will be delivered tending to undermine the faith and
confidence of the people in their government, and in the duly constituted authorities, which might threaten
breaches of the peace and a disruption of public order." Giving emphasis as well to the delegated police
power to local government. Stating as well Revised Ordinances of 1927 prohibiting as an offense against
public peace, and penalizes as a misdemeanor, "any act, in any public place, meeting, or procession,
tending to disturb the peace or excite a riot; or collect with other persons in a body or crowd for any
unlawful purpose; or disturb or disquiet any congregation engaged in any lawful assembly." Included
herein is Sec. 1119, Free use of Public Place.1
Issue:
Whether or Not the freedom of speech was violated.
Held:
Yes. Dealing with the ordinance, specifically, Sec. 1119, said section provides for two constructions: (1)
the Mayor of the City of Manila is vested with unregulated discretion to grant or refuse, to grant permit for
the holding of a lawful assembly or meeting, parade, or procession in the streets and other public places
of the City of Manila; (2) The right of the Mayor is subject to reasonable discretion to determine or specify
the streets or public places to be used with the view to prevent confusion by overlapping, to secure
convenient use of the streets and public places by others, and to provide adequate and proper policing to
minimize the risk of disorder. The court favored the second construction. First construction tantamount to
authorizing the Mayor to prohibit the use of the streets. Under our democratic system of government no
such unlimited power may be validly granted to any officer of the government, except perhaps in cases of
national emergency.
The Mayors first defense is untenable. Fear of serious injury cannot alone justify suppression of free
speech and assembly. It is the function of speech to free men from the bondage of irrational fears. To
justify suppression of free speech there must be reasonable ground to fear that serious evil will result if
free speech is practiced. There must be reasonable ground to believe that the danger apprehended is
imminent. There must be reasonable ground to believe that the evil to be prevented is a serious one . The
fact that speech is likely to result in some violence or in destruction of property is not enough to justify its
suppression. There must be the probability of serious injury to the state.
ZALDIVAR VS. SANDIGANBAYAN
[170 SCRA 1; G.R. NO. 79690-707; 1 FEB 1989]
Facts:
The case stemmed from the resolution of the Supreme Court stopping the respondent from investigating
graft cases involving Antique Gov. Enrique Zaldivar. The Court ruled that since the adoption of the 1987
Constitution, respondents powers as Tanodbayan have been superseded by the creation of the Office of
the Ombudsman, he however becomes the Special Prosecutor of the State, and can only conduct an
investigation and file cases only when so authorized by the Ombudsman. A motion for reconsideration
was filed by the respondent wherein he included statements which were unrelated in the Issue raised in
the Court. This include: (a)That he had been approached twice by a leading member of the court and he
was asked to 'go slow on Zaldivar and 'not to be too hard on him; (b) That he "was approached and asked
to refrain from investigating the COA report on illegal disbursements in the Supreme Court because 'it will
embarass the Court;" and (c) that in several instances, the undersigned respondent was called over the
phone by a leading member of the Court and was asked to dismiss the cases against two Members of the
Court." Statements of the respondent saying that the SCs order '"heightens the people's apprehension
over the justice system in this country, especially because the people have been thinking that only the
small fly can get it while big fishes go scot-free was publicized in leading newspapers.

Page
Section 1-C, SY 06-07

112

San Beda College of Law Alabang


Constitutional Law 2 Case Digests
Now, the Court Resolved to require respondent to explain in writing why he should not be punished for
contempt of court for making such public statements reported in the media. Respondent then sought to
get some members of the Court to inhibit themselves in the resolution of the Zaldivar case for alleged bias
and prejudice against him. A little later, he in effect asked the whole Court to inhibit itself from passing
upon the Issue involved in proceeding and to pass on responsibility for this matter to the Integrated Bar of
the Philippines, upon the ground that respondent cannot expect due process from this Court, that the
Court has become incapable of judging him impartially and fairly. The Court found respondent guilty of
contempt of court and indefinitely suspended from the practice of law. Now, he assails said conviction,
invoking his freedom of speech. Counsel for respondent urges that it is error "for this Court to apply the
"visible tendency" rule rather than the "clear and present danger" rule in disciplinary and contempt
charges."
Issue:
Whether or Not there was a violation of the freedom of speech/expression.
Held:
There was no violation. The Court did not purport to announce a new doctrine of "visible tendency," it was
simply paraphrasing Section 3 (d) of Rule 71 of the Revised Rules of Court which penalizes a variety of
contumacious conduct including: "any improper conduct tending, directly or indirectly, to impede, obstruct
or degrade the administration of justice."
Under either the "clear and present danger" test or the "balancing-of-interest test," the Court held that the
statements made by respondent Gonzalez are of such a nature and were made in such a manner and
under such circumstances, as to transcend the permissible limits of free speech. What is here at stake is
the authority of the Supreme Court to confront and prevent a "substantive evil" consisting not only of the
obstruction of a free and fair hearing of a particular case but also the avoidance of the broader evil of the
degradation of the judicial system of a country and the destruction of the standards of professional
conduct required from members of the bar and officers of the courts, which has some implications to the
society.
REYES VS. BAGATSING
[125 SCRA 553; L-65366; 9 NOV 1983]
Facts:
Petitioner sought a permit from the City of Manila to hold a peaceful march and rally on October 26, 1983
from 2:00 to 5:00 in the afternoon, starting from the Luneta to the gates of the United States Embassy.
Once there, and in an open space of public property, a short program would be held. The march would be
attended by the local and foreign participants of such conference. That would be followed by the handing
over of a petition based on the resolution adopted at the closing session of the Anti-Bases Coalition.
There was likewise an assurance in the petition that in the exercise of the constitutional rights to free
speech and assembly, all the necessary steps would be taken by it "to ensure a peaceful march and rally.
However the request was denied. Reference was made to persistent intelligence reports affirming the
plans of subversive/criminal elements to infiltrate or disrupt any assembly or congregations where a large
number of people is expected to attend. Respondent suggested that a permit may be issued if it is to be
held at the Rizal Coliseum or any other enclosed area where the safety of the participants themselves
and the general public may be ensured. An oral argument was heard and the mandatory injunction was
granted on the ground that there was no showing of the existence of a clear and present danger of a
substantive evil that could justify the denial of a permit. However Justice Aquino dissented that the rally is
violative of Ordinance No. 7295 of the City of Manila prohibiting the holding of rallies within a radius of five
hundred (500) feet from any foreign mission or chancery and for other purposes. Hence the Court
resolves.
Issue:
Whether or Not the freedom of expression and the right to peaceably assemble violated.
Held:
Yes. The invocation of the right to freedom of peaceable assembly carries with it the implication that the
right to free speech has likewise been disregarded. It is settled law that as to public places, especially so
as to parks and streets, there is freedom of access. Nor is their use dependent on who is the applicant for
the permit, whether an individual or a group. There can be no legal objection, absent the existence of a
clear and present danger of a substantive evil, on the choice of Luneta as the place where the peace rally
would start. Time immemorial Luneta has been used for purposes of assembly, communicating thoughts
between citizens, and discussing public questions.

Page
Section 1-C, SY 06-07

113

Such use of the public places has from ancient times, been a part of the privileges, immunities, rights, and
liberties of citizens.
With regard to the ordinance, there was no showing that there was violation and even if it could be shown
that such a condition is satisfied it does not follow that respondent could legally act the way he did. The
validity of his denial of the permit sought could still be challenged.
A summary of the application for permit for rally: The applicants for a permit to hold an assembly should
inform the licensing authority of the date, the public place where and the time when it will take place. If it
were a private place, only the consent of the owner or the one entitled to its legal possession is required.
Such application should be filed well ahead in time to enable the public official concerned to appraise
whether there may be valid objections to the grant of the permit or to its grant but at another public place.
It is an indispensable condition to such refusal or modification that the clear and present danger test be
the standard for the decision reached. Notice is given to applicants for the denial.
BAYAN VS. EXECUTIVE SECRETARY ERMITA
[488 SCRA 226; G.R. NO. 169838; 25 APR 2006]
Facts:
Rallies of September 20, October 4, 5 and 6, 2005 is at issue. BAYANs rally was violently dispersed. 26
petitioners were injured, arrested and detained when a peaceful mass action they was preempted and
violently dispersed by the police. KMU asserts that the right to peaceful assembly, are affected by Batas
Pambansa No. 880 and the policy of Calibrated Preemptive Response (CPR) being followed to
implement it. KMU, et al., claim that on October 4, 2005, a rally KMU co-sponsored was to be conducted
at the Mendiola bridge but police blocked them along C.M. Recto and Lepanto Streets and forcibly
dispersed them, causing injuries to several of their members. They further allege that on October 6,
2005, a multi-sectoral rally which KMU also co-sponsored was scheduled to proceed along Espaa
Avenue in front of the UST and going towards Mendiola bridge. Police officers blocked them along
Morayta Street and prevented them from proceeding further. They were then forcibly dispersed, causing
injuries on one of them. Three other rallyists were arrested.
All petitioners assail Batas Pambansa No. 880 The Public Assembly Act of 1985, some of them in toto
and others only Sections 4, 5, 6, 12, 13(a), and 14(a), as well as the policy of CPR. They seek to stop
violent dispersals of rallies under the no permit, no rally policy and the CPR policy announced on Sept.
21, 2005.
Petitioners Bayan, et al., contend that BP 880 is clearly a violation of the Constitution and the
International Covenant on Civil and Political Rights and other human rights treaties of which the
Philippines is a signatory.
They argue that B.P. No. 880 requires a permit before one can stage a public assembly regardless of the
presence or absence of a clear and present danger. It also curtails the choice of venue and is thus
repugnant to the freedom of expression clause as the time and place of a public assembly form part of the
message for which the expression is sought.
Petitioners Jess del Prado, et al., in turn, argue that B.P. No. 880 is unconstitutional as it is a curtailment
of the right to peacefully assemble and petition for redress of grievances because it puts a condition for
the valid exercise of that right. It also characterizes public assemblies without a permit as illegal and
penalizes them and allows their dispersal. Thus, its provisions are not mere regulations but are actually
prohibitions. Regarding the CPR policy, it is void for being an ultra vires act that alters the standard of
maximum tolerance set forth in B.P. No. 880, aside from being void for being vague and for lack of
publication.
KMU, et al., argue that the Constitution sets no limits on the right to assembly and therefore B.P. No. 880
cannot put the prior requirement of securing a permit. And even assuming that the legislature can set
limits to this right, the limits provided are unreasonable: First, allowing the Mayor to deny the permit on
clear and convincing evidence of a clear and present danger is too comprehensive. Second, the five-day
requirement to apply for a permit is too long as certain events require instant public assembly, otherwise
interest on the issue would possibly wane.As to the CPR policy, they argue that it is preemptive, that the
government takes action even before the rallyists can perform their act, and that no law, ordinance or
executive order supports the policy. Furthermore, it contravenes the maximum tolerance policy of B.P.
No. 880 and violates the Constitution as it causes a chilling effect on the exercise by the people of the
right to peaceably assemble.
Respondents argued that petitioners have no standing. BP 880 entails traffic re-routing to prevent grave
public inconvenience and serious or undue interference in the free flow of commerce and trade. It is

Page
Section 1-C, SY 06-07

114

San Beda College of Law Alabang


Constitutional Law 2 Case Digests
content-neutral regulation of the time, place and manner of holding public assemblies. According to
Atienza RA. 7160 gives the Mayor power to deny a permit independently of B.P. No. 880. and that the
permit is for the use of a public place and not for the exercise of rights; and that B.P. No. 880 is not a
content-based regulation because it covers all rallies.
Issue:
Whether or Not BP 880 and the CPR Policy unconstitutional.
Held:
No question as to standing. Their right as citizens to engage in peaceful assembly and exercise the right
of petition, as guaranteed by the Constitution, is directly affected by B.P. No. 880. B.P. 880 is not an
absolute ban of public assemblies but a restriction that simply regulates the time, place and manner of the
assemblies. It refers to all kinds of public assemblies that would use public places. The reference to
lawful cause does not make it content-based because assemblies really have to be for lawful causes,
otherwise they would not be peaceable and entitled to protection. Maximum tolerance 1 is for the
protection and benefit of all rallyists and is independent of the content of the expressions in the rally.
There is, likewise, no prior restraint, since the content of the speech is not relevant to the regulation.
The so-called calibrated preemptive response policy has no place in our legal firmament and must be
struck down as a darkness that shrouds freedom. It merely confuses our people and is used by some
police agents to justify abuses. Insofar as it would purport to differ from or be in lieu of maximum
tolerance, this was declared null and void.
The Secretary of the Interior and Local Governments, are DIRECTED to take all necessary steps for the
immediate compliance with Section 15 of Batas Pambansa No. 880 through the establishment or
designation of at least one suitable freedom park or plaza in every city and municipality of the country.
After thirty (30) days from the finality of this Decision, subject to the giving of advance notices, no prior
permit shall be required to exercise the right to peaceably assemble and petition in the public parks or
plazas of a city or municipality that has not yet complied with Section 15 of the law.
FERNANDO VS. ESTORNINOS
[G.R. NO 159751; 6 DEC 2006]
Facts:
Acting on reports of sale and distribution of pornographic materials, officers of the PNP Criminal
Investigation and Detection Group in the National Capital Region conducted police surveillance on the
store Gaudencio E. Fernando Music Fair (Music Fair) in Quiapo. A Search Warrant for violation of Article
201 of RPC against petitioner and a certain Warren Tingchuy and the seizure of the following items:
a.
b.
c.
d.

Copies of New Rave Magazines with nude obscene pictures;


Copies of IOU Penthouse Magazine with nude obscene pictures;
Copies of Hustler International Magazine with nude obscene pictures; and
Copies of VHS tapes containing pornographic shows.

The police searched the premises and confiscated twenty-five VHS tapes(among of which is Kahit sa
Pangarap Lang with Myra Manibog as actress who is naked) and ten different magazines(Dalaga,
Penthouse, Swank, Erotic, Rave, Playhouse, Gallery, QUI), which they deemed pornographic. Petitioners
were charged and convicted. CA affirmed the decision hence this appeal.
Issue:
Whether or Not the CA erred in affirming RTCs decision.
Held:
No. As obscenity is an unprotected speech which the State has the right to regulate, the State in pursuing
its mandate to protect the public from obscene, immoral and indecent materials must justify the regulation
or limitation. (Kottinger Rule Applied).
MALABANAN VS. RAMENTO
[129 SCRA 359; G.R. NO.62270; 21 MAY 1984]

Page
Section 1-C, SY 06-07

115

Facts:
Petitioners were officers of the Supreme Student Council of respondent University. They sought and were
granted by the school authorities a permit to hold a meeting from 8:00 A.M. to 12:00 P.M, on August 27,
1982. Pursuant to such permit, along with other students, they held a general assembly at the Veterinary
Medicine and Animal Science basketball court (VMAS), the place indicated in such permit, not in the
basketball court as therein stated but at the second floor lobby. At such gathering they manifested in
vehement and vigorous language their opposition to the proposed merger of the Institute of Animal
Science with the Institute of Agriculture. The same day, they marched toward the Life Science Building
and continued their rally. It was outside the area covered by their permit. Even they rallied beyond the
period allowed. They were asked to explain on the same day why they should not be held liable for
holding an illegal assembly. Then on September 9, 1982, they were informed that they were under
preventive suspension for their failure to explain the holding of an illegal assembly. The validity thereof
was challenged by petitioners both before the Court of First Instance of Rizal against private respondents
and before the Ministry of Education, Culture, and Sports. Respondent Ramento found petitioners guilty
of the charge of illegal assembly which was characterized by the violation of the permit granted resulting
in the disturbance of classes and oral defamation. The penalty was suspension for one academic year.
Hence this petition.
Issue:
Whether on the facts as disclosed resulting in the disciplinary action and the penalty imposed, there was
an infringement of the right to peaceable assembly and its cognate right of free speech.
Held:
Yes. Student leaders are likely to be assertive and dogmatic. They would be ineffective if during a rally
they speak in the guarded and judicious language of the academe. But with the activity taking place in the
school premises and during the daytime, no clear and present danger of public disorder is discernible.
This is without prejudice to the taking of disciplinary action for conduct, "materially disrupts classwork or
involves substantial disorder or invasion of the rights of others."
The rights to peaceable assembly and free speech are guaranteed students of educational institutions.
Necessarily, their exercise to discuss matters affecting their welfare or involving public interest is not to be
subjected to previous restraint or subsequent punishment unless there be a showing of a clear and
present danger to a substantive evil that the state, has a right to present. As a corollary, the utmost
leeway and scope is accorded the content of the placards displayed or utterances made. The peaceable
character of an assembly could be lost, however, by an advocacy of disorder under the name of dissent,
whatever grievances that may be aired being susceptible to correction through the ways of the law. If the
assembly is to be held in school premises, permit must be sought from its school authorities, who are
devoid of the power to deny such request arbitrarily or unreasonably. In granting such permit, there may
be conditions as to the time and place of the assembly to avoid disruption of classes or stoppage of work
of the non-academic personnel. Even if, however, there be violations of its terms, the penalty incurred
should not be disproportionate to the offense.
NON VS. DAMES
[185 SCRA 523; G.R. NO. 89317; 20 MAY 1990]
Facts:
Petitioners, students in private respondent Mabini Colleges, Inc. in Daet, Camarines Norte, were not
allowed to re-enroll by the school for the academic year 1988-1989 for leading or participating in student
mass actions against the school in the preceding semester. The subject of the protests is not, however,
made clear in the pleadings.
Petitioners filed a petition in the court seeking their readmission or re-enrollment to the school, but the trial
court dismissed the petition. They now petition the court to reverse its ruling in Alcuaz vs. PSBA 1, which
was also applied in the case. The court said that petitioners waived their privilege to be admitted for reenrollment with respondent college when they adopted, signed, and used its enrollment form for the first
semester of school year 1988-89, which states that: The Mabini College reserves the right to deny
admission of students whose scholarship and attendance are unsatisfactory and to require withdrawal of
students whose conduct discredits the institution and/or whose activities unduly disrupts or interfere with
the efficient operation of the college. Students, therefore, are required to behave in accord with the Mabini
College code of conduct and discipline.
Issue:

Page
Section 1-C, SY 06-07

116

San Beda College of Law Alabang


Constitutional Law 2 Case Digests

Whether or Not the students right to freedom of speech and assembly infringed.
Held:
Yes. The protection to the cognate rights of speech and assembly guaranteed by the Constitution is
similarly available to students is well-settled in our jurisdiction. However there are limitations. The
permissible limitation on Student Exercise of Constitutional Rights within the school presupposes that
conduct by the student, in class or out of it, which for any reason whether it stems from time, place, or
type of behavior should not materially disrupt classwork or must not involve substantial disorder or
invasion of the rights of others.
IN RE: TULFO
[A.M. NO. 90-4-1545-0; 17 APR 1990]
Facts:
In Oct. 13, 1989, Tulfo wrote an article in his column in PDI 'On Target' stating that the Supreme Court
rendered an idiotic decision in legalizing checkpoints, and again on Oct. 16, 1989, where he called the
Supreme Court stupid and "sangkatutak na mga bobo justices of the Philippine Supreme Court". Tulfo
was required to show cause why he should not be punished for contempt. Tulfo said that he was just
reacting emotionally because he had been a victim of harassment in the checkpoints, and "idiotic" meant
illogical and unwise, and "bobo" was just quoted from other attorneys, and since the case had been
decided and terminated, there was not contempts. Lastly, the article does not pose any clear and present
danger to the Supreme court.
Issue:
Whether or Not Tulfo is in contempt.
Held:
Yes. At the time Tulfo wrote the article, the checkpoints case had not yet been decided upon, and the
Supreme Court was still acting on an MR filed from the CA. The power to punish is inherent as it is
essential for self-preservation. Contempt of court is defiance of the authority, justice and dignity of the
courts. It brings disrepute to the court. There are two kinds of publications which can be punished for
contempt:
a. those whose object is to affect the decision in a pending case.
b. those whose object is to bring courts to discredit. Tulfo's article constituted both.
It should have been okay to criticize if respectful language was used, but if its object is only to degrade
and ridicule, then it is clearly an obstruction of justice. Nothing constructive can be gained from them.
Being emotional is no excuse for being insulting. Quoting is not an excuse also, because at the end of his
article, Tulfo said, "So you bobo justices, watch out!" Also, he said he was not sorry for having written the
articles.
Tulfo is found in contempt of court and is gravely censured.
PBM EMPLOYEES VS. PBM
[51 SCRA 189; G.R. NO. L-31195; 5 JUN 1993]
Facts:
The petitioner Philippine Blooming Mills Employees Organization (PBMEO) is a legitimate labor union
composed of the employees of the respondent Philippine Blooming Mills Co., Inc., and petitioners.
Benjamin Pagcu and Rodulfo Munsod are officers and members of the petitioner Union. Petitioners claim
that on March 1, 1969, they decided to stage a mass demonstration at Malacaang on March 4, 1969, in
protest against alleged abuses of the Pasig police. PBMEO thru Pagcu confirmed the planned
demonstration and stated that the demonstration or rally cannot be cancelled because it has already been
agreed upon in the meeting. Pagcu explained further that the demonstration has nothing to do with the
Company because the union has no quarrel or dispute with Management. The Management, thru Atty.
C.S. de Leon, Company personnel manager, informed PBMEO that the demonstration is an inalienable
right of the union guaranteed by the Constitution but emphasized that any demonstration for that matter

Page
Section 1-C, SY 06-07

117

should not unduly prejudice the normal operation of the Company. Workers who without previous leave of
absence approved by the Company, particularly , the officers present who are the organizers of the
demonstration, who shall fail to report for work the following morning shall be dismissed, because such
failure is a violation of the existing CBA and, therefore, would be amounting to an illegal strike. Because
the petitioners and their members numbering about 400 proceeded with the demonstration despite the
pleas of the respondent Company that the first shift workers should not be required to participate in the
demonstration and that the workers in the second and third shifts should be utilized for the demonstration
from 6 A.M. to 2 P.M. on March 4, 1969, filed a charge against petitioners and other employees who
composed the first shift, for a violation of Republic Act No. 875(Industrial Peace Act), and of the CBA
providing for 'No Strike and No Lockout.' Petitioners were held guilty in by CIR for bargaining in bad faith,
hence this appeal.
Issue:
Whether or Not the petitioners right to freedom of speech and to peaceable assemble violated.
Held:
Yes. A constitutional or valid infringement of human rights requires a more stringent criterion, namely
existence of a grave and immediate danger of a substantive evil which the State has the right to prevent.
This is not present in the case. It was to the interest herein private respondent firm to rally to the defense
of, and take up the cudgels for, its employees, so that they can report to work free from harassment,
vexation or peril and as consequence perform more efficiently their respective tasks enhance its
productivity as well as profits. Herein respondent employer did not even offer to intercede for its
employees with the local police. In seeking sanctuary behind their freedom of expression well as their
right of assembly and of petition against alleged persecution of local officialdom, the employees and
laborers of herein private respondent firm were fighting for their very survival, utilizing only the weapons
afforded them by the Constitution the untrammelled enjoyment of their basic human rights. The
pretension of their employer that it would suffer loss or damage by reason of the absence of its
employees from 6 o'clock in the morning to 2 o'clock in the afternoon, is a plea for the preservation merely
of their property rights. The employees' pathetic situation was a stark reality abused, harassment and
persecuted as they believed they were by the peace officers of the municipality. As above intimated, the
condition in which the employees found themselves vis-a-vis the local police of Pasig, was a matter that
vitally affected their right to individual existence as well as that of their families. Material loss can be
repaired or adequately compensated. The debasement of the human being broken in morale and
brutalized in spirit-can never be fully evaluated in monetary terms. As heretofore stated, the primacy of
human rights freedom of expression, of peaceful assembly and of petition for redress of grievances
over property rights has been sustained. To regard the demonstration against police officers, not against
the employer, as evidence of bad faith in collective bargaining and hence a violation of the collective
bargaining agreement and a cause for the dismissal from employment of the demonstrating employees,
stretches unduly the compass of the collective bargaining agreement, is "a potent means of inhibiting
speech" and therefore inflicts a moral as well as mortal wound on the constitutional guarantees of free
expression, of peaceful assembly and of petition. Circulation is one of the aspects of freedom of
expression. If demonstrators are reduced by one-third, then by that much the circulation of the Issue
raised by the demonstration is diminished. The more the participants, the more persons can be apprised
of the purpose of the rally. Moreover, the absence of one-third of their members will be regarded as a
substantial indication of disunity in their ranks which will enervate their position and abet continued
alleged police persecution.

Page
Section 1-C, SY 06-07

118

San Beda College of Law Alabang


Constitutional Law 2 Case Digests
THE IMPAIRMENT CLAUSE
Art 3, Sec. 10. No law impairing the obligation of contracts shall be passed.
RUTTER VS. ESTEBAN
[93 PHIL 68; NO.L-3708; 18 MAY 1953]
Facts:
On August 20,1941 Rutter sold to Esteban two parcels of land situated in the Manila for P9,600 of which
P4,800 were paid outright, and the balance was made payable as follows: P2,400 on or before August 7,
1942, and P2,400 on or before August 27, 1943, with interest at the rate of 7 percent per annum. To
secure the payment of said balance of P4,800, a first mortgage has been constituted in favor of the
plaintiff. Esteban failed to pay the two installments as agreed upon, as well as the interest that had
accrued and so Rutter instituted an action to recover the balance due, the interest due and the attorney's
fees. The complaint also contains a prayer for sale of the properties mortgaged in accordance with law.
Esteban claims that this is a prewar obligation contracted and that he is a war sufferer, having filed his
claim with the Philippine War Damage Commission for the losses he had suffered as a consequence of
the last war; and that under section 2 of RA 342(moratorium law), payment of his obligation cannot be
enforced until after the lapse of eight years. The complaint was dismissed. A motion for recon was made
which assails the constitutionality of RA 342.
Issue:
Whether or Not RA 342 unconstitutional on non-impairment clause grounds.
Held:
Yes. The moratorium is postponement of fulfillment of obligations decreed by the state through the
medium of the courts or the legislature. Its essence is the application of police power. The economic
interests of the State may justify the exercise of its continuing and dominant protective power
notwithstanding interference with contracts. The question is not whether the legislative action affects
contracts incidentally, or directly or indirectly, but whether the legislation is addressed to a legitimate end
and the measures taken are reasonable and appropriate to that end.
However based on the Presidents general SONA and consistent with what the Court believes to be as
the only course dictated by justice, fairness and righteousness, declared that the continued operation and
enforcement of RA 342 at the present time is unreasonable and oppressive, and should not be prolonged
should be declared null and void and without effect. This holds true as regards Executive Orders Nos. 25
and 32, with greater force and reason considering that said Orders contain no limitation whatsoever in
point of time as regards the suspension of the enforcement and effectivity of monetary obligations.

ORTIGAS VS. FEATI BANK


[94 SCRA 533; NO.L-24670; 14 DEC 1979]
Facts:
Plaintiff is engaged in real estate business, developing and selling lots to the public, particularly the
Highway Hills Subdivision along EDSA. On March 4, 1952, plaintiff, as vendor, and Augusto Padilla and
Natividad Angeles, as vendees, entered into separate agreements of sale on installments over two
parcels of land of the Subdivision. On July 19, 1962, the said vendees transferred their rights and
interests over the aforesaid lots in favor of one Emma Chavez. Upon completion of payment of the
purchase price, the plaintiff executed the corresponding deeds of sale in favor of Emma Chavez. Both the
agreements (of sale on installment) and the deeds of sale contained the stipulations or restrictions that:
1. The parcel of land shall be used exclusively for residential purposes, and she shall not be entitled
to take or remove soil, stones or gravel from it or any other lots belonging to the Seller.
2. All buildings and other improvements (except the fence) which may be constructed at any time in
said lot must be, (a) of strong materials and properly painted, (b) provided with modern sanitary
installations connected either to the public sewer or to an approved septic tank, and (c) shall not
be at a distance of less than two (2) meters from its boundary lines.

Page
Section 1-C, SY 06-07

119

Eventually said lots were bought by defendant. Lot 5 directly from Chavez and Lot 6 from Republic Flour
Mills by deed of exchange, with same restrictions. Plaintiff claims that restriction is for the beautification of
the subdivision. Defendant claimed of the commercialization of western part of EDSA. Defendant began
constructing a commercial bank building. Plaintiff demand to stop it, which forced him to file a case, which
was later dismissed, upholding police power. Motion for recon was denied, hence the appeal.
Issue:
Whether or Not non-impairment clause violated.
Held:
No. Resolution is a valid exercise of police power. EDSA, a main traffic artery which runs through several
cities and municipalities in the Metro Manila area, supports an endless stream of traffic and the resulting
activity, noise and pollution are hardly conducive to the health, safety or welfare of the residents in its
route. Health, safety, peace, good order and general welfare of the people in the locality are justifications
for this. It should be stressed, that while non-impairment of contracts is constitutionally guaranteed, the
rule is not absolute, since it has to be reconciled with the legitimate exercise of police power.

LOZANO VS. MARTINEZ


[146 SCRA 323; NO.L-63419; 18 DEC 1986]
Facts:
A motion to quash the charge against the petitioners for violation of the BP 22 was made, contending that
no offense was committed, as the statute is unconstitutional. Such motion was denied by the RTC. The
petitioners thus elevate the case to the Supreme Court for relief. The Solicitor General, commented that it
was premature for the accused to elevate to the Supreme Court the orders denying their motions to
quash. However, the Supreme Court finds it justifiable to intervene for the review of lower court's denial of
a motion to quash.
Issue:
Whether or Not BP 22 impairs freedom of contract. Whether or not BP 22 transgresses the constitutional
inhibition against imprisonment for debt.
Held:
The freedom of contract which is constitutionally protected is freedom to enter into "lawful" contracts.
Contracts which contravene public policy are not lawful. Checks can not be categorized as mere
contracts. It is a commercial instrument which, in this modem day and age, has become a convenient
substitute for money; it forms part of the banking system and therefore not entirely free from the
regulatory power of the state.
The offense punished by BP 22 is the act of making and issuing a worthless check or a check that is
dishonored upon its presentation for payment. It is not the non-payment of an obligation which the law
punishes. The law is not intended or designed to coerce a debtor to pay his debt. The thrust of the law is
to prohibit, under pain of penal sanctions, the making of worthless checks and putting them in circulation.

Page
Section 1-C, SY 06-07

120

San Beda College of Law Alabang


Constitutional Law 2 Case Digests

EX POST FACTO LAWS


Art 3, Sec. 22. No ex post facto law or bill of attainder shall be enacted.
PEOPLE VS. FERRER
[48 SCRA 382; NOS.L-32613-14; 27 DEC 1972]
Facts:
Hon. Judge Simeon Ferrer is the Tarlac trial court judge that declared RA1700 or the Anti-Subversive Act
of 1957 as a bill of attainder. Thus, dismissing the information of subversion against the following: 1.)
Feliciano Co for being an officer/leader of the Communist Party of the Philippines (CPP) aggravated by
circumstances of contempt and insult to public officers, subversion by a band and aid of armed men to
afford impunity. 2.) Nilo Tayag and 5 others, for being members/leaders of the NPA, inciting, instigating
people to unite and overthrow the Philippine Government. Attended by Aggravating Circumstances of Aid
or Armed Men, Craft, and Fraud. The trial court is of opinion that 1.) The Congress usurped the powers of
the judge 2.) Assumed judicial magistracy by pronouncing the guilt of the CPP without any forms of
safeguard of a judicial trial. 3.) It created a presumption of organizational guilt by being members of the
CPP regardless of voluntariness.
The Anti Subversive Act of 1957 was approved 20June1957. It is an act to outlaw the CPP and similar
associations penalizing membership therein, and for other purposes. It defined the Communist Party
being although a political party is in fact an organized conspiracy to overthrow the Government, not only
by force and violence but also by deceit, subversion and other illegal means. It declares that the CPP is a
clear and present danger to the security of the Philippines. Section 4 provided that affiliation with full
knowledge of the illegal acts of the CPP is punishable. Section 5 states that due investigation by a
designated prosecutor by the Secretary of Justice be made prior to filing of information in court. Section 6
provides for penalty for furnishing false evidence. Section 7 provides for 2 witnesses in open court for
acts penalized by prision mayor to death. Section 8 allows the renunciation of membership to the CCP
through writing under oath. Section 9 declares the constitutionality of the statute and its valid exercise
under freedom if thought, assembly and association.
Issue:
Whether or not RA1700 is a bill of attainder/ ex post facto law.
Whether or Not RA1700 violates freedom of expression.
Held:
The court holds the VALIDITY Of the Anti-Subversion Act of 1957.
A bill of attainder is solely a legislative act. It punishes without the benefit of the trial. It is the substitution
of judicial determination to a legislative determination of guilt. In order for a statute be measured as a bill
of attainder, the following requisites must be present: 1.) The statute specifies persons, groups. 2.) the
statute is applied retroactively and reach past conduct. (A bill of attainder relatively is also an ex post
facto law.)
In the case at bar, the statute simply declares the CPP as an organized conspiracy for the overthrow of
the Government for purposes of example of SECTION 4 of the Act. The Act applies not only to the CPP
but also to other organizations having the same purpose and their successors. The Acts focus is on the
conduct
not
person.
Membership to this organizations, to be UNLAWFUL, it must be shown that membership was acquired
with the intent to further the goals of the organization by overt acts. This is the element of MEMBERSHIP
with KNOWLEDGE that is punishable. This is the required proof of a members direct participation. Why
is membership punished. Membership renders aid and encouragement to the organization. Membership
makes himself party to its unlawful acts.
Furthermore, the statute is PROSPECTIVE in nature. Section 4 prohibits acts committed after approval
of the act. The members of the subversive organizations before the passing of this Act is given an
opportunity to escape liability by renouncing membership in accordance with Section 8. The statute
applies the principle of mutatis mutandis or that the necessary changes having been made.

Page
Section 1-C, SY 06-07

121

The declaration of that the CPP is an organized conspiracy to overthrow the Philippine Government
should not be the basis of guilt. This declaration is only a basis of Section 4 of the Act. The EXISTENCE
OF SUBSTANTIVE EVIL justifies the limitation to the exercise of Freedom of Expression and
Association in this matter. Before the enactment of the statute and statements in the preamble, careful
investigations by the Congress were done. The court further stresses that whatever interest in freedom of
speech and association is excluded in the prohibition of membership in the CPP are weak considering
NATIONAL SECURITY and PRESERVATION of DEMOCRACY.
The court set basic guidelines to be observed in the prosecution under RA1700. In addition to proving
circumstances/ evidences of subversion, the following elements must also be established:
1. Subversive Organizations besides the CPP, it must be proven that the organization purpose
is to overthrow the present Government of the Philippines and establish a domination of a
FOREIGN POWER. Membership is willfully and knowingly done by overt acts.
2. In case of CPP, the continued pursuance of its subversive purpose. Membership is willfully
and knowingly done by overt acts.
The court did not make any judgment on the crimes of the accused under the Act. The Supreme Court
set aside the resolution of the TRIAL COURT.

BAYOT VS. SANDIGANBAYAN


[128 SCRA 383; NO.L-61776 TO NO.L-61861; 23 MAR 1984]
Facts:
Bayot is one of the several persons who was accused in more than 100 counts of estafa thru falsification
of Public documents before the Sandiganbayan. The said charges started from his alleged involvement
as a government auditor of the commission on audit assigned to the Ministry of education and culture,
with some other employees from the said ministry. The bureau of treasury and the teachers camp in
Baguio City for the preparation and encashment of fictitious TCAA checks for the nom-existent obligations
of the teachers camp resulting in damage to the government of several millions. The 1 st 32 cases were
filed on july 25, 1987, while Bayot ran for municipal mayor of Amadeo Cavite and was elected on January
1980. but on May 1980 Sandiganbayan promulgated a decision convicting the accused together with his
other co-accused in all but one of the thirty two cases filed against them.
On Mach 16, 1982 Batas Pambansa Blg 195 was passed amending RA 3019.
Issue:
Whether or Not it would be violative of the constitutional guarantee against an ex post facto law.
Held:
The court finds no merit in the petitioners contention that RA 3019 as amended by Batas Pambansa Blg
195, which includes the crime of estafa through falsification of Public Documents as among crimes
subjecting the public officer charged therewith with suspension from public office pending action in court,
is a penal provision which violates the constitutional prohibition against the enactment of ex post facto
law. Accdg to the RPC suspension from employment and public office during trial shall not be considered
as a penalty. It is not a penalty because it is not a result of a judicial proceeding. In fact, if acquitted the
official who is suspended shall be entitled to reinstatement and the salaries and benefits which he failed
to receive during suspension. And does not violate the constitutional provision against ex post facto law.
The claim of the petitioner that he cannot be suspended because he is currently occupying a position
diffren tfrom that under which he is charged is untenable. The amendatory provision clearly states that
any incumbent public officer against whom any criminal prosecution under a valid information under RA
3019 for any offense involving fraud upon the government or public funds or property or whatever stage
of execution and mode of participation shall be suspended from office. The use of the word office
applies to any office which the officer charged may be holding and not only the particular office under
which he was charged.

Page
Section 1-C, SY 06-07

122

San Beda College of Law Alabang


Constitutional Law 2 Case Digests

PEOPLE VS. SANDIGANBAYAN


[211 SCRA 241; G.R. NO. 101724; 3 JUL 1992]
Facts:
Two letter complaints were filed with the Tanodbayan by Teofilo Gelacio on October 28,1986 and
December 9, 1986, a political leader of Governor Valentina Plaza, wife of Congressman Democrito Plaza
of Agusan del Sur, shortly after private respondent had replaced Mrs. Plaza as OIC/provincial Governor of
Agusan del Sur on March 1986 The complaint questioned the issuance to Governor Paredes, when he
was still the provincial attorney in 1976 of a free patent title for a lot in the Rosario public land subdivision
in San Francisco, Agusan del Sur. He misrepresented to a Lands Inspector of the Bureau of Lands that
the lands subject herein are disposable lands, thereby inducing said inspector to recommend approval of
his application for free patent. On August 10, 1989 an information for violation of RA 3019 Anti-Graft and
Corrupt Practices Act was then filed in the Sandiganbayan after an ex parte preliminary investigation. A
motion to quash the information was filed by the private respondent contending among others that he is
charged for an offence which has prescribed. Said motion was granted. The crime was committed on
January 21, 1976, period of prescription was 10 years, therefore it has prescribed in 1986. Now the
motion to quash was being assailed.
Issue:
Whether or Not the motion to quash validly granted.
Held:
Yes. RA 3019, being a special law the computation of the period for the prescription of the crime is
governed by Sec. 29 of Act No. 3326, which begins to run from the day of the commission of the crime
and not the discovery of it. Additionally, BP 195 which was approved on March 16, 1982, amending Sec.
11 of RA 3019 by increasing ten to fifteen years of the period for the prescription or extinguishment of a
violation of RA 3019 may not be given retroactive application to the crime which was committed by
Paredes, as it is prejudicial to the accused. To apply BP 195 to Paredes would make it an ex post facto
law1 for it would alter his situation to his disadvantage by making him criminally liable for a crime that had
already been extinguished under the law existing when it was committed.

Page
Section 1-C, SY 06-07

123

NON-IMPRISONMENT FOR DEBT


Art 3, Sec. 20. No person shall be imprisoned for debt or non-payment of a poll tax.
SERAFIN VS. LINDAYAG
[67 SCRA 166; ADM. MATTER. NO. 297-MJ; 30 SEPT 1975]
Facts:
Plaintiff failed to pay a simple indebtedness for P1500 Carmelito Mendoza, then municipal secretary and
his wife Corazon Mendoza and therefore an estafa case was filed against her. Complainant admitted
complaint. Now complainant filed a case against respondent Judge for not dismissing the case and
issuing a warrant of arrest as it falls on the category of a simple indebtedness, since elements of estafa
are not present. Further she contended that no person should be imprisoned for non-payment of a loan of
a sum of money. Two months after respondent dismissed plaintiffs case. (Judge here committed gross
ignorance of law. Even if complainant desisted case was pursued.)
Issue:
Whether or Not there was a violation committed by the judge when it ordered the imprisonment of plaintiff
for non-payment of debt?
Held:
Yes. Since plaintiff did not commit any offense as, his debt is considered a simple loan granted by her
friends to her. There is no collateral or security because complainant was an old friend of the spouses
who lent the money and that when they wrote her a letter of demand she promised to pay them and said
that if she failed to keep her promise, they could get her valuable things at her home. Under the
Constitution she is protected. Judge therefore in admitting such a "criminal complaint" that was plainly
civil in aspects from the very face of the complaint and the "evidence" presented, and issuing on the same
day the warrant of arrest upon his utterly baseless finding "that the accused is probably guilty of the crime
charged," respondent grossly failed to perform his duties properly.
LOZANO VS. MARTINEZ
[146 SCRA 323; NO.L-63419; 18 DEC 1986]
Facts:
A motion to quash the charge against the petitioners for violation of the BP 22 was made, contending that
no offense was committed, as the statute is unconstitutional. Such motion was denied by the RTC. The
petitioners thus elevate the case to the Supreme Court for relief. The Solicitor General, commented that it
was premature for the accused to elevate to the Supreme Court the orders denying their motions to
quash. However, the Supreme Court finds it justifiable to intervene for the review of lower court's denial of
a motion to quash.
Issue:
Whether or not BP 22 is constitutional as it is a proper exercise of police power of the State.
Held:
The enactment of BP 22 a valid exercise of the police power and is not repugnant to the constitutional
inhibition against imprisonment for debt.
The offense punished by BP 22 is the act of making and issuing a worthless check or a check that is
dishonored upon its presentation for payment. It is not the non-payment of an obligation which the law
punishes. The law is not intended or designed to coerce a debtor to pay his debt.
The law punishes the act not as an offense against property, but an offense against public order. The
thrust of the law is to prohibit, under pain of penal sanctions, the making of worthless checks and putting
them in circulation. An act may not be considered by society as inherently wrong, hence, not malum in se
but because of the harm that it inflicts on the community, it can be outlawed and criminally punished as
malum prohibitum. The state can do this in the exercise of its police power.

Page
Section 1-C, SY 06-07

124

San Beda College of Law Alabang


Constitutional Law 2 Case Digests

Page
Section 1-C, SY 06-07

125

INVOLUNTARY SERVITUDE
Art 3, Sec. 18. (2)
No involuntary servitude in any form shall exist except as a punishment for a
crime whereof the party shall have been duly convicted.
CAUNCA VS. SALAZAR
[82 PHIL 851; NO.L-2690; 1 JAN 1949]
Facts:
This is an action for habeas corpus brought by Bartolome Caunca in behalf of his cousin Estelita Flores
who was employed by the Far Eastern Employment Bureau, owned by Julia Salazar, respondent herein.
An advanced payment has already been given to Estelita by the employment agency, for her to work as a
maid. However, Estelita wanted to transfer to another residence, which was disallowed by the
employment agency. Further she was detained and her liberty was restrained. The employment agency
wanted that the advance payment, which was applied to her transportation expense from the province
should be paid by Estelita before she could be allowed to leave.
Issue:
Whether or Not an employment agency has the right to restrain and detain a maid without returning the
advance payment it gave?
Held:
An employment agency, regardless of the amount it may advance to a prospective employee or maid, has
absolutely no power to curtail her freedom of movement. The fact that no physical force has been exerted
to keep her in the house of the respondent does not make less real the deprivation of her personal
freedom of movement, freedom to transfer from one place to another, freedom to choose ones residence.
Freedom may be lost due to external moral compulsion, to founded or groundless fear, to erroneous belief
in the existence of an imaginary power of an impostor to cause harm if not blindly obeyed, to any other
psychological element that may curtail the mental faculty of choice or the unhampered exercise of the will.
If the actual effect of such psychological spell is to place a person at the mercy of another, the victim is
entitled to the protection of courts of justice as much as the individual who is illegally deprived of liberty by
duress or physical coercion.

Page
Section 1-C, SY 06-07

126

San Beda College of Law Alabang


Constitutional Law 2 Case Digests
THE WRIT OF HABEAS CORPUS
Art 3, Sec. 15. The privilege of the writ of habeas corpus shall not be suspended except in cases of
invasion or rebellion when the public safety requires it.
LANSANG VS. GARCIA
[42 SCRA 448; L-33964; 11 Dec 1971]
Facts:
In the evening of August 21, 1971, at about 9 p.m., while the Liberal Party of the Philippines was holding a
public meeting at Plaza Miranda, Manila, for the presentation of its candidates in the general elections
scheduled for November 8, 1971, two hand grenades were thrown at the platform where said candidates
and other persons were. Eight persons were killed and many more injured. Proclamation 889 was issued
by the President suspending privilege of writ of habeas corpus stating that there is a conspiracy of
rebellion and insurrection in order to forcibly seize political power. Petitions for writ of habeas corpus were
filed by persons (13) who have been arrested without a warrant.
It was stated that one of the safeguards of the proclamation was that it is to be applied to persons caught
in flagrante delicto. Incidentally, Proc. 889-A was issued as an amendment, inserting the word actually
staging. Proc. 889-B was also issued lifting the suspension of privilege in 27 provinces, 3 sub-provinces
and 26 cities. Proc. 889-C was issued restoring the suspension in 13 provinces and cities(mostly in
Mindanao). Proc. 889-D further lifted the suspension in 7 provinces and 4 cities. Only 18 provinces and
sub-provinces and 2 cities whose privilege was suspended. Petitioners maintained that Proclamation No.
889 did not declare the existence of actual "invasion insurrection or rebellion or imminent danger thereof,
however it became moot and academic since it was amended. Petitioners further contend that public
safety did not require the issuance of proclamations stating: (a) that there is no rebellion; (b) that, prior to
and at the time of the suspension of the privilege, the Government was functioning normally, as were the
courts; (c) that no untoward incident, confirmatory of an alleged July-August Plan, has actually taken
place after August 21, 1971; (d) that the President's alleged apprehension, because of said plan, is nonexistent and unjustified; and (e) that the Communist forces in the Philippines are too small and weak to
jeopardize public safety to such extent as to require the suspension of the privilege of the writ of habeas
corpus.
A resolution was issued by majority of the Court having tentatively arrived at a consensus that it may
inquire in order to satisfy itself of the existence of the factual bases for the proclamations. Now the Court
resolves after conclusive decision reached by majority.
Issue:
Whether or Not the authority to decide whether the exigency has arisen requiring suspension (of the
privilege of the writ of habeas corpus) belongs to the President and his decision is final and conclusive
upon the courts and upon all other persons.
Whether or Not public safety require the suspension of the privilege of the writ of habeas corpus decreed
in Proclamation No. 889-A.
Held:
The President has authority however it is subject to judicial review. Two conditions must concur for the
valid exercise of the authority to suspend the privilege to the writ (a) there must be "invasion, insurrection,
or rebellion" or "imminent danger thereof," and (b) "public safety" must require the suspension of the
privilege. President has three (3) courses of action: (a) to call out the armed forces; (b) to suspend the
privilege of the writ of habeas corpus; and (c) to place the Philippines or any part thereof under martial
law. He had, already, called out the armed forces, proved inadequate. Of the two other alternatives, the
suspension of the privilege is the least harsh.
Petitioners contention that CPP-NPA has no ability, is negatived by the killing of 5 mayors, 20 barrio
captains and 3 chiefs of police; that there were fourteen (14) meaningful bombing incidents in the Greater
Manila Area in 1970. CPP has managed to infiltrate or establish and control nine major labor
organizations; has exploited the (11) major student or youth organizations; about thirty (30) mass
organizations actively advancing the CPP.

Page
Section 1-C, SY 06-07

127

RIGHTS OF THE ACCUSED


Art 3, Sec. 12. (1) Any person under investigation for the commission of an offense shall have the right
to be informed of his right to remain silent and to have competent and independent counsel preferably of
his own choice. If the person cannot afford the services of counsel, he must be provided with one. These
rights cannot be waived except in writing and in the presence of counsel.
(2)
No torture, force, violence, threat, intimidation, or any other means which vitiate the free will shall
be used against him. Secret detention places, solitary, incommunicado, or other similar forms of detention
are prohibited.
(3)
Any confession or admission obtained in violation of this or Section 17 hereof shall be
inadmissible in evidence against him.
(4)
The law shall provide for penal and civil sanctions for violations of this section as well as
compensation to and rehabilitation of victims of torture or similar practices, and their families.
Art 3, Sec. 14. (1) No person shall be held to answer for a criminal offense without due process of law.
(2) In all criminal prosecutions, the accused shall be presumed innocent until the contrary is proved, and
shall enjoy the right to be heard by himself and counsel, to be informed of the nature and cause of the
accusation against him, to have a speedy, impartial, and public trial, to meet the witnesses face to face,
and to have compulsory process to secure the attendance of witnesses and the production of evidence in
his behalf. However, after arraignment, trial may proceed notwithstanding the absence of the accused
provided that he has been duly notified and his failure to appear is unjustifiable.
Art 3, Sec. 11. Free access to the courts and quasi-judicial bodies and adequate legal assistance shall
not be denied to any person by reason of poverty.
Art 3, Sec. 16. All persons shall have the right to a speedy disposition of their cases before all judicial,
quasi-judicial, or administrative bodies.
Art 3, Sec. 17. No person shall be compelled to be a witness against himself.
Art 3, Sec. 19. (1) Excessive fines shall not be imposed, nor cruel, degrading or inhuman punishment
inflicted. Neither shall death penalty be imposed, unless, for compelling reasons involving heinous
crimes, the Congress hereafter provides for it. Any death penalty already imposed shall be reduced to
reclusion perpetua.
(2)
The employment of physical, psychological, or degrading punishment against any prisoner or
detainee or the use of substandard or inadequate penal facilities under subhuman conditions shall be
dealt with by law.
Art 3, Sec. 21. No person shall be twice put in jeopardy of punishment for the same offense. If an act is
punished by a law and an ordinance, conviction or acquittal under either shall constitute a bar to another
prosecution for the same act.
GAMBOA VS. CRUZ
[162 SCRA 642;L-56291; 27 JUN 1988]
Facts:
Petitioner was arrested for vagrancy without a warrant. During a line-up of 5 detainees including
petitioner, he was identified by a complainant to be a companion in a robbery, thereafter he was charged.
Petitioner filed a Motion to Acquit on the ground that the conduct of the line-up, without notice and in the
absence of his counsel violated his constitutional rights to counsel and to due process. The court denied
said motion. Hearing was set, hence the petition.
Issue:
Whether or Not petitioners right to counsel and due process violated.
Held:
No. The police line-up was not part of the custodial inquest, hence, petitioner was not yet entitled, at such
stage, to counsel. He had not been held yet to answer for a criminal offense. The moment there is a move
or even an urge of said investigators to elicit admissions or confessions or even plain information which
may appear innocent or innocuous at the time, from said suspect, he should then and there be assisted
by counsel, unless he waives the right, but the waiver shall be made in writing and in the presence of
counsel.

Page
Section 1-C, SY 06-07

128

San Beda College of Law Alabang


Constitutional Law 2 Case Digests

On the right to due process, petitioner was not, in any way, deprived of this substantive and constitutional
right, as he was duly represented by a counsel. He was accorded all the opportunities to be heard and to
present evidence to substantiate his defense; only that he chose not to, and instead opted to file a Motion
to Acquit after the prosecution had rested its case. What due process abhors is the absolute lack of
opportunity to be heard.
PEOPLE VS. JUDGE AYSON
[175 SCRA 216; G.R. NO. 85215; 7 JUL 1989]
Facts:
Felipe Ramos was a ticket freight clerk of the Philippine Airlines, assigned at its Baguio City station. It was
alleged that he was involved in irregularities in the sales of plane tickets, the PAL management notified
him of an investigation to be conducted. That investigation was scheduled in accordance with PAL's Code
of Conduct and Discipline, and the Collective Bargaining Agreement signed by it with the Philippine
Airlines Employees' Association (PALEA) to which Ramos pertained. A letter was sent by Ramos stating
his willingness to settle the amount of P76,000. The findings of the Audit team were given to him, and he
refuted that he misused proceeds of tickets also stating that he was prevented from settling said amounts.
He proffered a compromise however this did not ensue. Two months after a crime of estafa was charged
against Ramos. Ramos pleaded not guilty. Evidence by the prosecution contained Ramos written
admission and statement, to which defendants argued that the confession was taken without the accused
being represented by a lawyer. Respondent Judge did not admit those stating that accused was not
reminded of his constitutional rights to remain silent and to have counsel. A motion for reconsideration
filed by the prosecutors was denied. Hence this appeal.
Issue:
Whether or Not the respondent Judge correct in making inadmissible as evidence the admission and
statement of accused.
Held:
No. Section 20 of the 1987 constitution provides that the right against self-incrimination (only to witnesses
other than accused, unless what is asked is relating to a different crime charged- not present in case at
bar).
This is accorded to every person who gives evidence, whether voluntarily or under compulsion of
subpoena, in any civil, criminal, or administrative proceeding. The right is not to "be compelled to be a
witness against himself. It prescribes an "option of refusal to answer incriminating questions and not a
prohibition of inquiry." the right can be claimed only when the specific question, incriminatory in character,
is actually put to the witness. It cannot be claimed at any other time. It does not give a witness the right to
disregard a subpoena, to decline to appear before the court at the time appointed, or to refuse to testify
altogether. It is a right that a witness knows or should know. He must claim it and could be waived.
Rights in custodial interrogation as laid down in miranda v. Arizona: the rights of the accused include:
1) he shall have the right to remain silent and to counsel, and to be informed of such right.
2) nor force, violence, threat, intimidation, or any other means which vitiates the free will shall be
used against him.
3) any confession obtained in violation of these rights shall be inadmissible in evidence.
The individual may knowingly and intelligently waive these rights and agree to answer or make a
statement. But unless and until such rights and waivers are demonstrated by the prosecution at the trial,
no evidence obtained as a result of interrogation can be used against him.
PEOPLE VS. MAQUEDA
[242 SCRA 565; G.R. NO.112983; 22 MAR 1994]
Facts:
British Horace William Barker (consultant of WB) was slain inside his house in Tuba, Benguet while his
Filipino wife, Teresita Mendoza was badly battered with lead pipes on the occasion of a robbery. Two
household helpers of the victims identified Salvamante (a former houseboy of the victims) and Maqueda

Page
Section 1-C, SY 06-07

129

as the robbers. Mike Tabayan and his friend also saw the two accused a kilometer away from the house
of the victims that same morning, when the two accused asked them for directions.
Maqueda was then arrested in Guinyangan, Quezon. He was taken to Calauag, Quezon where he
signed a Sinumpaang Salaysay wherein he narrated his participation in the crime. According to SPO3
Molleno, he informed Maqueda of his constitutional rights before he signed such document. Afterwards he
was brought to the Benguet Provincial Jail. While he was under detention, Maqueda filed a Motion to
Grant Bail. He stated therein that "he is willing and volunteering to be a State witness in the above entitled
case, it appearing that he is the least guilty among the accused in this case."
Maqueda also admitted his involvement in the commission of the robbery to Prosecutor Zarate and to
Salvosa.
Issue:
Whether or Not the trial court was correct in holding that the Sinumpaan Salaysay is admissible as
evidence.
Held:
No. The Sinumpaang Salaysay is inadmissible because it was in clear violation of the constitutional rights
of the accused. First, he was not informed of his right to remain silent and his right to counsel. Second,
he cannot be compelled to be a witness against himself. At the time of the confession, the accused was
already facing charges in court. He no longer had the right to remain silent and to counsel but he had the
right to refuse to be a witness and not to have any prejudice whatsoever result to him by such refusal.
And yet, despite his knowing fully well that a case had already been filed in court, he still confessed when
he did not have to do so.
The contention of the trial court that the accused is not entitled to such rights anymore because the
information has been filed and a warrant of arrest has been issued already, is untenable. The exercise of
the rights to remain silent and to counsel and to be informed thereof under Section 12(1) of the Bill of
Rights are not confined to that period prior to the filing of a criminal complaint or information but are
available at that stage when a person is "under investigation for the commission of an offense."
Pursuant to Section 12(3) of the Bill of Rights therefore, such extra-judicial admission is inadmissible as
evidence.
As to the admissions made by Maqueda to Prosecutor Zarate and Ray Dean Salvosa, the trial court
admitted their testimony thereon only to prove the tenor of their conversation but not to prove the truth of
the admission because such testimony was objected to as hearsay. Maqueda voluntarily and freely made
them to Prosecutor Zarate not in the course of an investigation, but in connection with Maqueda's plea to
be utilized as a state witness; and as to the other admission (Salvosa), it was given to a private person
therefore admissible.
Note: a distinction between a confession and admission has been made by the SC:
Admission of a party. The act, declaration or omission of party as to a relevant fact may be given in
evidence against him.
Confession. The declaration of an accused acknowledging his guilt of the offense charged, or of any
offense necessarily included therein, may be given in evidence against him.
PEOPLE VS. BANDULA
[232 SCRA 566; G.R. NO. 89223; 27 MAY 1994]
Facts:
Six armed men barged into the compound of Polo Coconut Plantation in Tanjay, Negros Oriental. The
armed men were identified by Security Guard, including accused. Salva and Pastrano, security guards
were hogtied and accused proceeded to the Atty. Garay, counsel of plantation. They ransacked the place
and took with them money and other valuables. Atty. Garay was killed. Accused-appellant is charged with
robbery with homicide along with 3 others who were acquitted for insufficiency of evidence. Appellant
was convicted.
Now, appellant argues that the extrajudicial confessions he and accused Dionanao executed suffer from
constitutional infirmities, hence, inadmissible in evidence considering that they were extracted under
duress and intimidation, and were merely countersigned later by the municipal attorney who, by the

Page
Section 1-C, SY 06-07

130

San Beda College of Law Alabang


Constitutional Law 2 Case Digests
nature of his position, was not entirely an independent counsel nor counsel of their choice. Consequently,
without the extrajudicial confessions, the prosecution is left without sufficient evidence to convict him of
the crime charged.
Issue:
Whether or Not extrajudicial confessions of appellant is admissible as evidence against him.
Held:
No. When accused-appellant Bandula and accused Dionanao were investigated immediately after their
arrest, they had no counsel present. If at all, counsel came in only a day after the custodial investigation
with respect to accused Dionanao, and two weeks later with respect to appellant Bandula. And, counsel
who supposedly assisted both accused was Atty. Ruben Zerna, the Municipal Attorney of Tanjay. On top
of this, there are telltale signs that violence was used against the accused. Certainly, these are blatant
violations of the Constitution which mandates in
Sec. 12, Art. III. Irregularities present include:
1. The investigators did not inform the accused of their right to remain silent and to have competent
and independent counsel, preferably of their own choice, even before attempting to elicit
statements that would incriminate them.
2. Investigators continuously disregard the repeated requests of the accused for medical assistance.
Reason for Accused Sedigos "black eye" which even
Pat. Baldejera admitted is not established, as well as Bandulas fractured rib.
3. Counsel must be independent. He cannot be a special counsel, public or private prosecutor,
counsel of the police, or a municipal attorney whose interest is admittedly adverse to the
accused.
PEOPLE VS. LUCERO
[244 SCRA 425; G.R. NO.97936; 29 MAY 1995]
Facts:
Alejandro Lucero, Bienvenido Echavez, Balbino Echavez, Peter Doe, Richard Doe and John Doe were
charged with the crime of robbery with homicide.
The prosecution:
Accused-appellant (alighted from a gray-reddish car), armed with handgun, blocked the way of the said
complainant who was on board a Mercedes Benz passing along Road 14, Mindanao Avenue, Pag-asa,
QC, rob and carry away cash money; one gold necklace with cross pendant, 7 karat; one gold Rolex
watch; one 3 karat gold ring; one 2 karat gold ring, domino style; one solid gold bracelet; all worth
P363,600.00, belonging to DR. DEMETRIO Z. MADRID. Accused shot LORENZO BERNALES y ALERIA,
a driver of the said offended party, thus inflicting upon him mortal wounds, which resulted to the
instantaneous death of ALERIA.
Only the accused Echavez brothers and Alejandro Lucero were apprehended.
When Lucero told him that he had no lawyer, in due time, Atty. Diosdado Peralta conferred with Lucero.
He apprised Lucero of his constitutional rights. He observed no reaction from Lucero. Nonetheless, Atty.
Peralta gathered the impression that Lucero understood his advice.
When the investigator started asking the preliminary questions, Atty. Peralta left to attend the wake of his
friend. The next morning, Lucero was accompanied by CIS agents to Atty. Peralta's house. The
extrajudicial statement of Lucero was presented to Atty. Peralta. It was already signed by Lucero.
The three accused denied complicity in the crime charged.
Appellant Lucero's defense is alibi. He testified that he was at his house in Caloocan City.
He said he was surprised when several unidentified men accosted him while he was walking towards his
house. They chased him, handcuffed and blindfolded him and pushed him into a jeep. He was blindfolded
the whole night and did not know where he was taken. The men turned out to be police officers.
The next day, he learned he was in Camp Crame. He claimed that he was tortured. He was not informed
of the offense for which he was being investigated. Neither did they reveal the identity of the complainant.

Page
Section 1-C, SY 06-07

131

Lucero denied knowing Dr. Madrid, the Echavez brothers and the other accused in this case. He said he
only met Dr. Madrid at the CIS Office during the police line-up. He was made to line-up four (4) times
before Dr. Madrid finally identified him on the fourth time.
Lucero also claimed he signed the extrajudicial confession under duress. He denied engaging the
services of Atty, Peralta. He likewise confirmed that Atty. Peralta was not present during his actual
custodial interrogation.
After trial, the court a quo acquitted the Echavez brothers for insufficient evidence. The trial court,
however, convicted accused Lucero GUILTY as principal by direct participation of Robbery with Homicide
and sentenced to suffer an imprisonment term of RECLUSION PERPETUA.
Issue:
Whether or Not the lower court erred in convicting accused-appellant.
Held:
Appellant's conviction cannot be based on his extrajudicial confession.
Constitution requires that a person under investigation for the commission of a crime should be provided
with counsel. The Court have constitutionalized the right to counsel because of hostility against the use of
duress and other undue influence in extracting confessions from a suspect. Force and fraud tarnish
confessions can render them inadmissible.
The records show that Atty. Peralta, who was not the counsel of choice of appellant. Atty. Peralta himself
admitted he received no reaction from appellant although his impression was that appellant understood
him. More so, it was during his absence that appellant gave an uncounselled confession.
Constitution requires the right to counsel, it did not mean any kind of counsel but effective and vigilant
counsel. The circumstances clearly demonstrate that appellant received no effective counseling from Atty.
Peralta.
Whereof, Decision convicting appellant Alejandro Lucero y Cortel is hereby reversed.
PEOPLE VS. AGUSTIN
[240 SCRA 541; G.R. NO. 110290; 25 JAN 1995]
Facts:
Dr. Bayquen, a dentist, together with his son, Anthony; Anthony's girlfriend, Anna Theresa; his daughter,
Dominic; and Danny, a family friend, were on their way aboard their Brasilia to the doctor's residence at
Malvar Street, Baguio City. While they were cruising along Malvar Street and nearing the Baptist church,
a man came out from the right side of a car parked about two meters to the church. The man approached
the Brasilia, aimed his armalite rifle through its window, and fired at the passengers. The Brasilia swerved
and hit a fence. The gunman immediately returned to the parked car which then sped away. All those in
the car were hit and Dr. Bayquen and Anna Theresa died on the spot. Dominic was bale to get out of the
Brasilia to run to the Alabanza store where she telephoned her mother. Later, she and her mother brought
her father and Anthony to the hospital. Danny went home and was then brought to the Hospital for
treatment.
Accused Quiao, an alleged former military agent who had been picked up by the police authorities,
confessed during the investigation conducted by Baguio City Fiscal Erdolfo Balajadia in his office that he
was the triggerman. He implicated Abenoja, Jr., who engaged him to kill Dr. Bayquen for a fee, Cartel,
who provided the armalite, and a certain "Jimmy." During the investigation, Wilfredo Quiao was assisted
by Atty. Reynaldo Cajucom. Stenographic notes of the proceedings during the investigation as transcribed
with the sworn statement of Quiao was signed, with the assistance of Atty. Cajucom, and swore to
before City Fiscal Balajadia. The following day, Agustin was apprehended, and was investigated and was
afforded the privileges like that of Quijano. Agustins defense interpose that he was forced to admit
involvement at gunpoint in the Kennon Road. He further declared that although he was given a lawyer,
Cajucom (a law partner of the private prosecutor), he nevertheless, asked for his uncle Atty. Oliver Tabin,
and that Atty. Cajucom interviewed him from only two minutes in English and Tagalog but not in Ilocano,
the dialect he understands. The promise that he would be discharged as a witness did not push through
since Quijano escaped. However the RTC convicted him, since conspiracy was established. Hence the
appeal.

Page
Section 1-C, SY 06-07

132

San Beda College of Law Alabang


Constitutional Law 2 Case Digests
Issue:
Whether or Not accused-appellants extrajudicial statements admissible as evidence.
Held:
No. Extrajudicial statement is not extrajudicial confession. In a confession, there is an acknowledgment of
guilt of the accused, while an admission is a statement direct or implied of facts pertinent to the issue. The
rule on inadmissibility, however expressly includes admissions, not just confessions.The extrajudicial
admission of the appellant, contained in twenty-two pages appear to be signed by him and Atty. Cajucom
but for reasons not explained in the records, the transcript of the notes which consists of twelve pages
was not signed by the appellant. Since the court cannot even read or decipher the stenographic notes it
cannot be expected that appellant, who is a farmer and who reached only the fourth grade, to read or
decipher its contents. The appellant, therefore was deprived of his rights under Section 12(1), Article III of
the Constitution. Firstly, he was not fully and properly informed of his rights. The appellant was not
explicitly told of his right to have a competent and independent counsel of his choice, specifically asked if
he had in mind any such counsel and, if so, whether he could afford to hire his services, and, if he could
not, whether he would agree to be assisted by one to be provided for him. He was not categorically
informed that he could waive his rights to remain silent and to counsel and that this waiver must be in
writing and in the presence of his counsel. He had, in fact, waived his right to remain silent by agreeing to
be investigated. Yet, no written waiver of such right appears in the transcript and no other independent
evidence was offered to prove its existence. In short, after the appellant said that he wanted to be
assisted by counsel, the City fiscal, through suggestive language, immediately informed him that Atty.
Cajucom was ready to assist him. Moreso said counsel is not independent since he is an associate of the
private prosecutor.
PEOPLE VS. BOLANOS
[211 SCRA 262; G.R. NO. 101808; 3 JUL 1992]
Facts:
Oscar Pagdalian was murdered in Marble Supply, Balagtas Bulacan. According to Pat. Rolando Alcantara
and Francisco Dayao, deceased was with two companions on the previous night, one of whom the
accused who had a drinking spree with the deceased. When they apprehended the accused they found
the firearm of the deceased on the chair where the accused was allegedly seated. They boarded accused
along with Magtibay, other accused on the police vehicle and brought them to the police station. While in
the vehicle Bolanos admitted that he killed the deceased. RTC convicted him hence the appeal.
Issue:
Whether or Not accused-appellant deprived of his constitutional right to counsel.
Held:
Yes. Being already under custodial investigation while on board the police patrol jeep on the way to the
Police Station where formal investigation may have been conducted, appellant should have been
informed of his Constitutional rights under Article III, Section 12 of the 1987 Constitution, more particularly
par. 1 and par. 3.
PEOPLE VS. MACAM
[238 SCRA 306; G.R. NOS. 91011-12; 24 NOV 1994]
Facts:
Prosecutions version:
On Aug 18,1987, Eduardo Macam, Antonio Cedro, Eugenio Cawilan Jr., Danilo Roque and Ernesto
Roque went to the house of Benito Macam (uncle of Eduardo Macam) located at 43 Ferma Road QC.
Upon the arrival of the accused, Benito invited the former to have lunch. Benito asked his maid Salvacion
Enrera to call the companions of Eduardo who were waiting in a tricycle outside the house. A. Cedro, E.
Cawilan and D. Roque entered the house while E. Roque remained in the tricycle. After all the accused
had taken their lunch, Eduardo Macam grabbed the clutch bag of Benito Macam and pulled out his
uncles gun then declared a hold-up. They tied up the wife (Leticia Macam), children, maid (Salvacion)
and Nilo Alcantara and brought them to the room upstairs. After a while Leticia was brought to the

Page
Section 1-C, SY 06-07

133

bathroom and after she screamed she was stabbed and killed by A. Cedro. Benito, Nilo and Salvacion
was also stabbed but survived. The total value of the items taken was P536, 700.00.
Defenses version:
Danilo Roque stated that he being a tricycle driver drove the 4 accused to Benitos house for a fee of
P50.00. Instead of paying him, he was given a calling card by Eduardo Macam so that he can be paid the
following day. Upon arriving, he went with the accused inside the house to have lunch. Thereafter he
washed the dishes and swept the floor. When Eugenio Cawilan pulled a gun and announced the hold-up,
he was asked to gather some things and which he abided out of fear. While putting the said thins inside
the car of Benito (victim) he heard the accused saying kailangan patayin ang mga taong yan dahil kilala
ako ng mga yan. Upon hearing such phrase he escaped and went home using his tricycle. He also
testified that his brother Ernesto Roque has just arrived from the province and in no way can be involved
in the case at bar. On the following day, together with his brother, they went to the factory of the Zesto
Juice (owned by the father of Eduardo Macam) for him to get his payment (50.00) . He and his brother
was suddenly apprehended by the security guards and brought to the police headquarters in Q.C. They
were also forced to admit certain things.
After which, he together with all the accused, in handcuffs and bore contusions on their faces caused by
blows inflicted in their faces during investigation, was brought to the QC General Hospital before each
surviving victims and made to line-up for identification. Eugenio Cawilan was also charged with Antifencing Law but was acquitted in the said case.
Issue:
Whether or Not their right to counsel has been violated. WON the arrest was valid. WON the evidence
from the line-up is admissible.
Held:
It is appropriate to extend the counsel guarantee to critical stages of prosecution even before trial. A
police line-up is considered a critical stage of the proceedings. Any identification of an uncounseled
accused made in a police line-up is inadmissible. HOWEVER, the prosecution did not present evidence
regarding appellants identification at the line-up. The witnesses identified the accused again in open
court. Also, accused did not object to the in-court identification as being tainted by illegal line-up.
The arrest of the appellants was without a warrant. HOWEVER, they are estopped from questioning the
legality of such arrest because they have not moved to quash the said information and therefore
voluntarily submitted themselves to the jurisdiction of the trial court by entering a plea of not guilty and
participating in trial.
The court believed the version of the prosecution. Ernesto Roque, while remaining outside the house
served as a looked out.
Wherefore, decision of lower court is Affirmed. Danilo Roque and Ernesto Roque is guilty of the crime of
robbery with homicide as co-conspirators of the other accused to suffer reclusion perpetua.
Things taken: 2 toygun, airgun riffle, CO2 refiller, TV, betamax tapes, betamax rewinder, Samsonite
attache case, typewriter, chessboard, TOYOTA Crown Car Plate No. CAS-997, assorted jewelry. .22 gun
and money.
PEOPLE VS. DY
[158 SCRA 111; G.R. 74517; 23 FEB 1988]
Facts:
Pat. Padilla reported along with Benny Dy, with caliber .38 as suspect to the shooting incident at "Benny's
Bar," at Sitio Angol, Manoc-Manoc Malay, Aklan (Boracay) situated on the Island which caused the death
of Christian Langel Philippe, tourist, 24 years old and a Swiss nationale. He was charged with the Murder
With the Use of Unlicensed firearms. Appellant alleges that he carried the victim to the shore to be
brought to the hospital to save the latter, and who facilitated the surrender to Pat. Padilla a gun which his
helper found the following morning while cleaning the bar. Accused posted bail which was granted. The
accused denied having made any oral confession alleging that he went to Pat. Padilla not to report the
incident but to state that a boy helper in the bar had found a gun on the sand floor while cleaning and that
Pat. Padilla picked up the gun from the bar at his request. The Accused argues that even if he did make

Page
Section 1-C, SY 06-07

134

San Beda College of Law Alabang


Constitutional Law 2 Case Digests
such a confession, the same would be inadmissible in evidence. He was found guilty in the RTC. Hence
the appeal.
Issue:
Whether or Not the lower court correct in saying that the constitutional procedure on custodial
interrogation is not applicable in the instant case.
Held:
YES. Appellant's assertion that the gun he had surrendered was merely found by a boy helper while
cleaning the bar deserves no credence for, if it were so, it would have been absurd for him to have placed
himself under police custody in the early morning after the incident. Sworn Complaint for "Murder with
Use of Unlicensed Firearm" signed by the Chief of Police also attests to Appellant's oral confession. That
Complaint forms part of the record of the proceedings before the Municipal Circuit Trial Court of
Buruanga, Aklan, and is prima facie evidence of the facts therein stated. Appellant's voluntary surrender
implies no violation as "no warrant of arrest is issued for the apprehension of the accused for the reason
that he is already under police custody before the filing of the complaint." What was told by the Accused
to Pat, Padilla was a spontaneous statement not elicited through questioning, but given in ordinary
manner. No written confession was sought to be presented in evidence as a result of formal custodial
investigation.

NAVALLO VS. SANDIGANBAYAN


[234 SCRA 177; G.R. NO. 97214; 18 JUL 1994]
Facts:
Accused was the Collecting and Disbursing Officer of the Numancia National Vocational School, which
school is also located at del Carmen, Surigao del Norte. His duties included the collection of tuition fees,
preparation of vouchers for salaries of teachers and employees, and remittance of collections exceeding
P500.00 to the National Treasury. An information for malversation of public funds was filed. A warrant of
arrest was issued, but accused-petitioner could not be found. on 10 December 1978, Presidential Decree
No. 1606 took effect creating the Sandiganbayan and conferring on it original and exclusive jurisdiction
over crimes committed by public officers embraced in Title VII of the Revised Penal Code. On 15
November 1984, Navallo was finally arrested. He was released on provisional liberty upon the approval of
his property bail bond. When arraigned by the RTC on 18 July 1985, he pleaded not guilty. Upon motion
of the prosecution, the RTC transferred the case and transmitted its records to the Sandiganbayan.
Special Prosecutor Luz L. Quiones-Marcos opined that since Navallo had already been arraigned before
the case was transferred to the Sandiganbayan, the RTC should continue taking cognizance of the case.
The matter was referred to the Office of the Ombudsman which held otherwise. The information was then
docketed with the Sandiganbayan. A new order for Navallo's arrest was issued by the Sandiganbayan.
The warrant was returned with a certification by the RTC Clerk of Court that the accused had posted a
bail bond. Navallo filed a motion to quash, contending (1) that the Sandiganbayan had no jurisdiction over
the offense and the person of the accused and (2) that since the accused had already been arraigned by
the RTC, the attempt to prosecute him before the Sandiganbayan would constitute double jeopardy.
However this was denied and trial ensued and he was found guilty.
Issue:
Whether or Not the constitutional right against double jeopardy and in custodial investigations in favor of
the accused violated.
Held:
No. Double jeopardy requires the existence of the following requisites:
(1) The previous complaint or information or other formal charge is sufficient in form and substance
to sustain a conviction;
(2) The court has jurisdiction to try the case;
(3) The accused has been arraigned and has pleaded to the charge; and
(4) The accused is convicted or acquitted or the case is dismissed without his express consent.
The RTC was devoid of jurisdiction when it conducted an arraignment of the accused which by then had
already been conferred on the Sandiganbayan. Moreover, neither did the case there terminate with
conviction or acquittal nor was it dismissed.

Page
Section 1-C, SY 06-07

135

No. Appellant is not in custodial investigation. A person under a normal audit examination is not under
custodial investigation. An audit examiner himself can hardly be deemed to be the law enforcement officer
contemplated in the above rule. In any case, the allegation of his having been "pressured" to sign the
Examination Report prepared by Dulguime (examined cash, as ordered by Espino, the provincial auditor)
appears to be belied by his own testimony.
PEOPLE VS. ALICANDO
[251 SCRA 293; G.R. NO. 117487; 2 DEC 1995]
Facts:
Appellant was charged with the crime of rape with homicide of Khazie Mae Penecilla, a minor, four years
of age, choking her with his right hand. The incident happened after appellant drank liquor. A neighbor,
Leopoldo Santiago found the victims body and the parents and police were informed. Appellant was living
in his uncle's house some five arm's length from Penecilla's house. Appellant was arrested and
interrogated by PO3 Danilo Tan. He verbally confessed his guilt without the assistance of counsel. On the
basis of his uncounselled verbal confession and follow up interrogations, the police came to know and
recovered from appellant's house, Khazie Mae's green slippers, a pair of gold earrings, a buri mat, a
stained pillow and a stained T-shirt all of which were presented as evidence for the prosecution. He was
arraigned with the assistance of Atty. Rogelio Antiquiera of the PAO. Appellant pleaded guilty. The RTC
convicted him. Hence an automatic review for the imposition of death penalty.
Issue:
Whether or Not the death penalty proper.
Held:
No. The records do not reveal that the Information against the appellant was read in the language or
dialect known to him. The Information against the appellant is written in the English language. It is
unknown whether the appellant knows the English language. Neither is it known what dialect is
understood by the appellant. Nor is there any showing that the Information couched in English was
translated to the appellant in his own dialect before his plea of guilt. The RTC violated section 1(a) of Rule
116, the rule implementing the constitutional right of the appellant to be informed of the nature and cause
of the accusation against him. It also denied appellant his constitutional right to due process of law. It is
urged that we must presume that the arraignment of the appellant was regularly conducted. When life is
at stake, we cannot lean on this rebuttable presumption. There could be no presumption. The court must
be sure.
The trial court violated section 3 of Rule 116 when it accepted the plea of guilt of the appellant. Said
section requires that the court shall conduct a searching inquiry the voluntariness and full comprehension
of the consequences of his plea and require the prosecution to prove his guilt and the precise degree of
culpability. The accused may also present evidence in his behalf. The trial court simply inquired if
appellant had physical marks of maltreatment. It did not ask the appellant when he was arrested, who
arrested him, how and where he was interrogated, whether he was medically examined before and after
his interrogation, etc. It limited its efforts trying to discover late body marks of maltreatment as if
involuntariness is caused by physical abuse alone.
Further, there are physical evidence to prove Khazie was raped. These consists of a pillow with
bloodstains in its center 14 and the T-shirt 15 of the accused colored white with bloodstains on its bottom.
These physical evidence are evidence of the highest order. They strongly corroborate the testimony of
Luisa Rebada that the victim was raped.These are inadmissible evidence for they were gathered by PO3
Danilo Tan of the Iloilo City PNP as a result of custodial interrogation where appellant verbally confessed
to the crime without the benefit of counsel.
PEOPLE VS. DE GUZMAN
[224 SCRA 93; G.R. NOS. 98321-24; 30 JUN 1993]
Facts:
All the accused were charged before the Regional Trial Court of Cebu with three counts of murder and
one count of frustrated murder in four Informations. The victim Jose Bantug was found with gunshots in
the head, body, and skull. The other three informations charged them with the murder of Francisco
Carteciano y Sorilla and Antonio S. Carteciano, and the frustrated murder of Lorna V. Carteciano. The

Page
Section 1-C, SY 06-07

136

San Beda College of Law Alabang


Constitutional Law 2 Case Digests
other 8 accused were acquitted on the ground of reasonable doubt, while Victor Nuez was found guilty.
The facts shown by evidence are: One morning, Major Antonio Carteciano was driving his private jeep
Camp General Arcadio Maxilom in Lahug, Cebu City where he was stationed as medical officer of the
PC/INP Provincial Command. In the front seat with him is his wife Lorna, and at the backseat are his
mother in law, son, brother Francisco, neighbor Bantug, and Bantugs son. Near the intersection,
gunshots were heard from the left side of the street. Major Carteciano took his .45 cal pistol and fired.
However, gunshots were fired in succession, and Major Carteciano, his brother Francisco, Jose Bantug,
and his wife Lorna were hit. When the jeep stopped, several gunmen approached them. Nuez
demanded Lorna to give Nuez her husbands pistol. Lorna asked to take her valuables instead. Then,
Nuez shot Major Cartecianos head point blank. Then the gunmen hijacked another jeep and took off.
Lorna, her mother Juanita Ricaplaza, and her son Reiser Carteciano positively identified the accused.
Lorna identified Nuez as the one who shot her husband. Nuez claimed that his arrest was illegal and
that he was deprived of his right to counsel when he was subjected to a paraffin test without the
assistance of counsel.
Issue:
Whether or not the accused Nuezs constitutional right was violated
Held:
No. Nuez pleaded not guilty at the arraignment. Therefore, he is estopped from questioning the validity
of his arrest. Furthermore, the illegal arrest of an accused is not sufficient cause for setting aside a valid
judgment rendered upon a sufficient complaint after trial free from error. The witnesses also positively
identified the accused, so he cannot question the credibility of the witnesses. Regarding his right to
counsel, the Supreme Court held that-- the right to counsel attaches only upon the start of an
investigation, that is, when the investigating officer starts to ask questions to elicit information and/or
confessions or admissions from the accused. At such point or stage, the person being interrogated must
be assisted by counsel to avoid the pernicious practice of extorting false or coerced admissions or
confessions from the lips of the person undergoing interrogation. In the case at bar, when accused was
subjected to a paraffin test, he was not then under custodial investigation. Accused-appellant also argued
that since his co-accused were acquitted, then their acquittal negates conspiracy among them, and he
should not be convicted with the charges filed. However, the Court held that conspiracy was still proven
by the evidence, and the other co-accused were acquitted only because there was reasonable doubt.
Therefore, accused-appellant is still convicted of the four charges against him.
We, therefore, find that the conviction of accused-appellant for the crimes charged has been established
beyond reasonable doubt and the penalty imposed is in accordance with law. However, the civil indemnity
imposed by the trial court should be increased to P50,000 in conformity with our recent rulings on the
matter.
WHEREFORE, except for the modification that the civil indemnity to be paid by accused-appellant Victor
Nuez, Jr. to the heirs of each victim who died is hereby increased to P50,000, the appealed decision is
hereby affirmed in all other respects, with costs against accused-appellant
PEOPLE VS. JUDGE DONATO
[198 SCRA 130; G.R. NO.79269; 5 JUN 1991]
Facts:
Private respondent and his co-accused were charged of rebellion on October 2, 1986 for acts committed
before and after February 1986. Private respondent filed with a Motion to Quash alleging that: (a) the
facts alleged do not constitute an offense; (b) the Court has no jurisdiction over the offense charged; (c)
the Court has no jurisdiction over the persons of the defendants; and (d) the criminal action or liability has
been extinguished. This was denied. May 9, 1987 Respondent filed a petition for bail, which was opposed
that the respondent is not entitled to bail anymore since rebellion became a capital offense under PD
1996, 942 and 1834 amending ART. 135 of RPC. On 5 June 1987 the President issued Executive Order
No. 187 repealing, among others, P.D. Nos. 1996, 942 and 1834 and restoring to full force and effect
Article 135 of the Revised Penal Code as it existed before the amendatory decrees. Judge Donato now
granted the bail, which was fixed at P30,000.00 and imposed a condition that he shall report to the court
once every two months within the first ten days of every period thereof. Petitioner filed a supplemental
motion for reconsideration indirectly asking the court to deny bail to and to allow it to present evidence in
support thereof considering the "inevitable probability that the accused will not comply with this main
condition of his bail. It was contended that:

Page
Section 1-C, SY 06-07

137

1. The accused has evaded the authorities for thirteen years and was an escapee from detention
when arrested; (Chairman of CPP-NPA)
2. He was not arrested at his residence as he had no known address;
3. He was using the false name "Manuel Mercado Castro" at the time of his arrest and presented a
Driver's License to substantiate his false identity;
4. The address he gave "Panamitan, Kawit, Cavite," turned out to be also a false address;
5. He and his companions were on board a private vehicle with a declared owner whose identity and
address were also found to be false;
6. Pursuant to Ministry Order No. 1-A dated 11 January 1982 , a reward of P250,000.00 was offered
and paid for his arrest.
This however was denied. Hence the appeal.
Issue:
Whether or Not the private respondent has the right to bail.
Held:
Yes. Bail in the instant case is a matter of right. It is absolute since the crime is not a capital offense,
therefore prosecution has no right to present evidence. It is only when it is a capital offense that the right
becomes discretionary. However it was wrong for the Judge to change the amount of bail from 30K to 50K
without hearing the prosecution.
Republic Act No. 6968 approved on 24 October 1990, providing a penalty of reclusion perpetua to the
crime of rebellion, is not applicable to the accused as it is not favorable to him.
Accused validly waived his right to bail in another case(petition for habeas corpus). Agreements were
made therein: accused to remain under custody, whereas his co-detainees Josefina Cruz and Jose Milo
Concepcion will be released immediately, with a condition that they will submit themselves in the
jurisdiction of the court. Said petition for HC was dismissed. Bail is the security given for the release of a
person in custody of the law. Ergo, there was a waiver. We hereby rule that the right to bail is another of
the constitutional rights which can be waived. It is a right which is personal to the accused and whose
waiver would not be contrary to law, public order, public policy, morals, or good customs, or prejudicial to
a third person with a right recognized by law.
CARPIO VS. MAGLALANG
[196 SCRA 41; G.R. NO. 78162; 19 APR 1991]
Facts:
On January 8, 1987, information for the murder of Mayor Jose Payumo of Dinalupihan Bataan was filed
against Escao and ten other unindentified persons by the provincial fiscal in the RTC of Bataan at
Balanga. Four days later, the Acting Executive Judge of said court issued an order of arrest against
Escao recommending no bail for his provisional liberty. Pat. Cesar Diego who acted on the warrant
returned to the court with a certification issued by NBI agent Gonzales, stating therein that accused was
still under investigation.
Through counsel Rolando T. Cainoy, Escao filed in court an urgent ex-parte motion for his commitment
at the provincial jail of Bataan on the ground that he wanted to be where his family and counsel could
have easy access to him. He alleged therein that his detention at the NBI headquarters in Manila was
irregular and in defiance of the warrant of arrest issued by the court. This was granted.
A motion for reconsideration was filed by Director Carpio stating that the NBI needed physical custody of
Escao for the identification of the other accused in the case who were still the objects of a manhunt by
NBI agents; that in view of the finding of NBI agents that the other accused and suspects in the case were
subversive elements or members of the New People's Army, it was for the best interest of Escao that he
be detained at the NBI lock-up cell where security measures were adequate; and that the NBI would
produce the person of Escao before the court whenever required and every time that there would be a
hearing on the case. However another motion was executed by Escao stating that he now wants to be
detained in the NBI, alleging that he did not authorize his counsel to execute the first motion. Also,
Escao's counsel Rolando T. Cainoy filed an application for bail stating that Escao was arrested by NBI
agents on December 7, 1986 without a warrant having been presented to him and that since then he had
been detained in the lock-up cell of the NBI; that said agents, also without a warrant, searched his house
when he was arrested; that he was subjected to inhuman torture and forced to admit participation in the
killing of Mayor Payumo and to implicate other persons, and that during the custodial investigation, he

Page
Section 1-C, SY 06-07

138

San Beda College of Law Alabang


Constitutional Law 2 Case Digests
was not represented by counsel. In opposing said application, the public prosecutor averred that the
accused was charged with a capital offense for which no bail may be availed of, that the reasons
advanced in said application would be overcome by strong and sufficient evidence; and that during the
custodial investigation, he was represented by counsel. The court granted the application for bail fixing
the same at P30,000, having found no sufficient evidence against accused. Director Carpio was ordered
to justify his actions and so as not to be considered in contempt.
Issue:
Whether or Not the order granting right to bail was proper.
Held:
No. The order granting bail had been rendered moot not only by the fact that he had been released from
NBI custody, but also because Escao jumped bail and did not appear on the date set for his
arraignment. Notwithstanding, the Court resolved the issue of the legality of the order granting bail to
Escao. Although the right to bail is principally for the benefit of the accused, in the judicial determination
of the availability of said right, the prosecution should be afforded procedural due process. Thus, in the
summary proceeding on a motion praying for admission to bail, the prosecution should be given the
opportunity to present evidence and, thereafter, the court should spell out at least a resume of the
evidence on which its order granting or denying bail is based. Otherwise, the order is defective and
voidable. In the case at bar the RTC erred in not summarizing the factual basis of its order granting bail,
the court merely stated the number of prosecution witnesses but not their respective testimonies, and
concluded that the evidence presented by the prosecution was not "sufficiently strong" to deny bail to
Escao.
The facts, however, that Mayor Payumo was killed on August 20, 1986 when the 1973 Constitution
allowing the death penalty was still in force and that the application for bail was made on March 5, 1987
during the effectivity of the 1987 Constitution which abolished the death penalty, should not have gotten in
the way of resolving the application for bail in accordance with the Constitution and procedural rules.
Section 13, Article III of the Constitution explicitly provides that "(a)ll persons, except those charged with
offenses punishable by reclusion perpetua when evidence of guilt is strong, shall before conviction, be
bailable by sufficient sureties, or be released on recognizance as may be provided by law." As the phrase
"capital offenses" has been replaced by the phrase "offenses punishable by reclusion perpetua, 25 crimes
punishable by reclusion perpetua instead of those punishable by the death penalty, when evidence of guilt
is strong, are the exceptions to the rule that the right to bail should be made available to all accused. As
the court itself acknowledged in its order of April 2, 1987 that "capital punishment" in Section 4, Rule 114
has been amended to reclusion perpetua, the court should have proceeded accordingly: i.e., resolved the
application for bail pursuant to Section 13, Article III of the Constitution. It did not have to invoke the
abolition of the death penalty and the lack of legislative enactment restoring it in justifying the grant of bail.
All it had to do was to determine whether evidence of guilt is strong in the light of the provision of Section
13, Article III.
The RTC has the discretion in the consideration of the strength of the evidence at hand. However, in the
exercise of said discretion, the court is controlled by the following: first, the applicable provisions of the
Constitution and the statutes; second, by the rules which this Court may promulgate; and third, by those
principles of equity and justice that are deemed to be part of the laws of the land. 27 The lower court not
only failed to properly apply the pertinent provisions of the Constitution and the Rules but it also
disregarded equity and justice by its failure to take into account the factual milieu surrounding the
detention of Escao
PEOPLE VS. FORTES
[223 SCRA 619; G.R. NO. 90643; 25 JUN 1993]
Facts:
Agripino Gine of Barangay Naburacan, Municipality of Matnog, Province of Sorsogon, accompanied his
13-year old daughter, Merelyn, to the police station of the said municipality to report a rape committed
against the latter by the accused. Following this, the accused was apprehended and charged. A bond of
P25000 was granted for accuseds provisional release. The MCTC found him guilty. An appeal to RTC
was filed, the request for the fixing of bond was denied. Now accused assails denial of bail on the ground
that the same amounted to an undue denial of his constitutional right to bail.
Issue:
Whether or Not the accuseds right to bail violated.

Page
Section 1-C, SY 06-07

139

Held:
No. It is clear from Section 13, Article III of the 1987 Constitution and Section 3, Rule 114 of the Revised
Rules of Court, as amended, that before conviction bail is either a matter of right or of discretion. It is a
matter of right when the offense charged is punishable by any penalty lower than reclusion perpetua. To
that extent the right is absolute. If the offense charged is punishable by reclusion perpetua bail becomes a
matter of discretion. It shall be denied if the evidence of guilt is strong. The court's discretion is limited to
determining whether or not evidence of guilt is strong. But once it is determined that the evidence of guilt
is not strong, bail also becomes a matter of right. If an accused who is charged with a crime punishable
by reclusion perpetua is convicted by the trial court and sentenced to suffer such a penalty, bail is neither
a matter of right on the part of the accused nor of discretion on the part of the court.
COMMENDADOR VS. DE VILLA
[200 SCRA 80; G.R. NO. 93177; 2 AUG 1991]
Facts:
The petitioners in G.R. Nos. 93177 and 96948 who are officers of the AFP were directed to appear in
person before the Pre-Trial Investigating Officers for the alleged participation the failed coup on
December 1 to 9, 1989. Petitioners now claim that there was no pre-trial investigation of the charges as
mandated by Article of War 71. A motion for dismissal was denied. Now, their motion for reconsideration.
Alleging denial of due process.
In G.R. No. 95020, Ltc Jacinto Ligot applied for bail on June 5, 1990, but the application was denied by
GCM No.14. He filed with the RTC a petition for certiorari and mandamus with prayer for provisional
liberty and a writ of preliminary injunction. Judge of GCM then granted the provisional liberty. However he
was not released immediately. The RTC now declared that even military men facing court martial
proceedings can avail the right to bail.
The private respondents in G.R. No. 97454 filed with SC a petition for habeas corpus on the ground that
they were being detained in Camp Crame without charges. The petition was referred to RTC. Finding
after hearing that no formal charges had been filed against the petitioners after more than a year after
their arrest, the trial court ordered their release.
Issue:
Whether or Not there was a denial of due process.
Whether or not there was a violation of the accused right to bail.
Held:
NO denial of due process. Petitioners were given several opportunities to present their side at the pre-trial
investigation, first at the scheduled hearing of February 12, 1990, and then again after the denial of their
motion of February 21, 1990, when they were given until March 7, 1990, to submit their counter-affidavits.
On that date, they filed instead a verbal motion for reconsideration which they were again asked to submit
in writing. They had been expressly warned in the subpoena that "failure to submit counter-affidavits on
the date specified shall be deemed a waiver of their right to submit controverting evidence." Petitioners
have a right to pre-emptory challenge. (Right to challenge validity of members of G/SCM)
It is argued that since the private respondents are officers of the Armed Forces accused of violations of
the Articles of War, the respondent courts have no authority to order their release and otherwise interfere
with the court-martial proceedings. This is without merit. * The Regional Trial Court has concurrent
jurisdiction with the Court of Appeals and the Supreme Court over petitions for certiorari, prohibition or
mandamus against inferior courts and other bodies and on petitions for habeas corpus and quo warranto.
The right to bail invoked by the private respondents has traditionally not been recognized and is not
available in the military, as an exception to the general rule embodied in the Bill of Rights. The right to a
speedy trial is given more emphasis in the military where the right to bail does not exist.
On the contention that they had not been charged after more than one year from their arrest, there was
substantial compliance with the requirements of due process and the right to a speedy trial. The AFP
Special Investigating Committee was able to complete the pre-charge investigation only after one year
because hundreds of officers and thousands of enlisted men were involved in the failed coup.

Page
Section 1-C, SY 06-07

140

San Beda College of Law Alabang


Constitutional Law 2 Case Digests
Accordingly, in G.R. No. 93177, the petition is dismissed for lack of merit. In G.R. No. 96948, the petition
is granted, and the respondents are directed to allow the petitioners to exercise the right of peremptory
challenge under article 18 of the articles of war. In G.R. Nos. 95020 and 97454, the petitions are also
granted, and the orders of the respondent courts for the release of the private respondents are hereby
reversed and set aside. No costs.
MANOTOC VS. COURT OF APPEALS
[142 SCRA 149; G.R. NO. L-62100; 30 MAY 1986]
Facts:
Petitioner was charged with estafa. He posted bail. Petitioner filed before each of the trial courts a motion
entitled, "motion for permission to leave the country," stating as ground therefor his desire to go to the
United States, "relative to his business transactions and opportunities." The prosecution opposed said
motion and after due hearing, both trial judges denied the same. Petitioner thus filed a petition for
certiorari and mandamus before the then Court of Appeals seeking to annul the orders dated March 9 and
26, 1982, of Judges Camilon and Pronove, respectively, as well as the communication-request of the
Securities and Exchange Commission, denying his leave to travel abroad. He likewise prayed for the
issuance of the appropriate writ commanding the Immigration Commissioner and the Chief of the Aviation
Security Command (AVSECOM) to clear him for departure. The Court of Appeals denied the petition.
Petitioner contends that having been admitted to bail as a matter of right, neither the courts which granted
him bail nor the Securities and Exchange Commission which has no jurisdiction over his liberty could
prevent him from exercising his constitutional right to travel.
Issue:
Whether or Not the right to bail a matter of right.
Held:
The right to bail is a matter of right if the charge is not a capital offense or punishable by reclusion
perpetua to death.
A court has the power to prohibit a person admitted to bail from leaving the Philippines. This is a
necessary consequence of the nature and function of a bail bond.
The condition imposed upon petitioner to make himself available at all times whenever the court requires
his presence operates as a valid restriction on his right to travel.
Indeed, if the accused were allowed to leave the Philippines without sufficient reason, he may be placed
beyond the reach of the courts.
Petitioner has not shown the necessity for his travel abroad. There is no indication that the business
transactions cannot be undertaken by any other person in his behalf.
CALLANTA VS. VILLANUEVA
[77 SCRA 377; G.R. NOS. 24646 & L-24674; 20 JUN 1977]
Facts:
Two complaints for grave oral defamation were filed against Faustina Callanta. The City Judge of
Dagupan City, Felipe Villanueva, denied the motions to quash the complaints. Thus, petitioner Callanta
brought the suits for certiorari in the Supreme Court. Petitioner questions the validity of the issuance of
warrant of arrest by respondent, arguing that the City Fiscal should have conducted the preliminary
investigation. According to petitioners counsel, there was jurisdictional infirmity. After the issuance of the
warrants of arrest and the bail fixed at P600, petitioner posted the bail bond, thus obtaining her
provisional liberty. The City Fiscal in this case did not disagree with the judges investigation, and agreed
with the complaints filed.
Issue:
Whether or Not petitioners contentions are to be given merit.
Held:

Page
Section 1-C, SY 06-07

141

Based on many precedent cases of the Supreme Court, where the accused has filed bail and waived the
preliminary investigation proper, he has waived whatever defect, if any, in the preliminary examination
conducted prior to the issuance of the warrant of arrest. In the case at bar, it is futile for the petitioner to
question the validity of the issuance of the warrant of arrest, because she posted the bail bond. Petitioner
also erred in arguing that only the City Fiscal can conduct a preliminary investigation. According to the
Charter of the City of Dagupan, the City Court of Dagupan City may also conduct preliminary
investigation for any offense, without regard to the limits of punishment, and may release, or commit and
bind over any person charged with such offense to secure his appearance before the proper court.
Petition for certiorari is denied. Restraining order issued by the Court is lifted and set aside.
TATAD VS. SANDIGANBAYAN
[159 SCRA 70; G.R. NOS. L-72335-39; 21 MAR 1988]
Facts:
The complainant, Antonio de los Reyes, originally filed what he termed "a report" with the Legal Panel of
the Presidential Security Command (PSC) on October 1974, containing charges of alleged violations of
Rep. Act No. 3019 against then Secretary of Public Information Francisco S. Tatad. The "report" was
made to "sleep" in the office of the PSC until the end of 1979 when it became widely known that
Secretary (then Minister) Tatad had a falling out with President Marcos and had resigned from the
Cabinet. On December 12, 1979, the 1974 complaint was resurrected in the form of a formal complaint
filed with the Tanodbayan. The Tanodbayan acted on the complaint on April 1, 1980 which was around
two months after petitioner Tatad's resignation was accepted by Pres. Marcos by referring the complaint
to the CIS, Presidential Security Command, for investigation and report. On June 16, 1980, the CIS report
was submitted to the Tanodbayan, recommending the filing of charges for graft and corrupt practices
against former Minister Tatad and Antonio L. Cantero. By October 25, 1982, all affidavits and counteraffidavits were in the case was already for disposition by the Tanodbayan. However, it was only on June
5, 1985 that a resolution was approved by the Tanodbayan. Five criminal informations were filed with the
Sandiganbayan on June 12, 1985, all against petitioner Tatad alone. (1) Section 3, paragraph (e) of RA.
3019 for giving D' Group, a private corporation controlled by his brother-in-law, unwarranted benefits,
advantage or preference in the discharge of his official functions; (2) Violation of Section 3, paragraph (b)
for receiving a check of P125,000.00 from Roberto Vallar, President/General Manager of Amity Trading
Corporation as consideration for the release of a check of P588,000.00 to said corporation for printing
services rendered for the Constitutional Convention Referendum in 1973; (3) Violation of Section 7 on
three (3) counts for his failure to file his Statement of Assets and Liabilities for the calendar years 1973,
1976 and 1978. A motion to quash the information was made alleging that the prosecution deprived
accused of due process of law and of the right to a speedy disposition of the cases filed against him. It
was denied hence the appeal.
Issue:
Whether or not petitioner was deprived of his rights as an accused.

Held:
YES. Due process (Procedural) and right to speedy disposition of trial were violated. Firstly, the complaint
came to life, as it were, only after petitioner Tatad had a falling out with President Marcos. Secondly,
departing from established procedures prescribed by law for preliminary investigation, which require the
submission of affidavits and counter-affidavits by the complainant and the respondent and their
witnesses, the Tanodbayan referred the complaint to the Presidential Security Command for finding
investigation and report. The law (P.D. No. 911) prescribes a ten-day period for the prosecutor to resolve
a case under preliminary investigation by him from its termination. While we agree with the respondent
court that this period fixed by law is merely "directory," yet, on the other hand, it can not be disregarded or
ignored completely, with absolute impunity. A delay of close to three (3) years can not be deemed
reasonable or justifiable in the light of the circumstance obtaining in the case at bar.
GALMAN VS. SANDIGANBAYAN
[144 SCRA 43; G.R. NO.72670; 12 SEP 1986]
Facts:
Assassination of former Senator Benigno "Ninoy" Aquino, Jr. He was killed from his plane that had just
landed at the Manila International Airport. His brain was smashed by a bullet fired point-blank into the

Page
Section 1-C, SY 06-07

142

San Beda College of Law Alabang


Constitutional Law 2 Case Digests
back of his head by an assassin. The military investigators reported within a span of three hours that the
man who shot Aquino (whose identity was then supposed to be unknown and was revealed only days
later as Rolando Galman) was a communist-hired gunman, and that the military escorts gunned him down
in turn.
President was constrained to create a Fact Finding Board to investigate due to large masses of people
who joined in the ten-day period of national mourning yearning for the truth, justice and freedom.
The fact is that both majority and minority reports were one in rejecting the military version stating that
"the evidence shows to the contrary that Rolando Galman had no subversive affiliations. Only the soldiers
in the staircase with Sen. Aquino could have shot him; that Ninoy's assassination was the product of a
military conspiracy, not a communist plot. Only difference between the two reports is that the majority
report found all the twenty-six private respondents above-named in the title of the case involved in the
military conspiracy; " while the chairman's minority report would exclude nineteen of them.
Then Pres. Marcos stated that evidence shows that Galman was the killer.
Petitioners pray for issuance of a TRO enjoining respondent court from rendering a decision in the two
criminal cases before it, the Court resolved by nine-to-two votes 11 to issue the restraining order prayed
for. The Court also granted petitioners a five-day period to file a reply to respondents' separate comments
and respondent Tanodbayan a three-day period to submit a copy of his 84-page memorandum for the
prosecution.
But ten days later, the Court by the same nine-to-two-vote ratio in reverse, resolved to dismiss the petition
and to lift the TRO issued ten days earlier enjoining the Sandiganbayan from rendering its decision. The
same Court majority denied petitioners' motion for a new 5-day period counted from receipt of respondent
Tanodbayan's memorandum for the prosecution (which apparently was not served on them).
Thus, petitioners filed a motion for reconsideration, alleging that the dismissal did not indicate the legal
ground for such action and urging that the case be set for a full hearing on the merits that the people are
entitled to due process.
However, respondent Sandiganbayan issued its decision acquitting all the accused of the crime charged,
declaring them innocent and totally absolving them of any civil liability. Respondents submitted that with
the Sandiganbayan's verdict of acquittal, the instant case had become moot and academic. Thereafter,
same Court majority denied petitioners' motion for reconsideration for lack of merit.
Hence, petitioners filed their motion to admit their second motion for reconsideration alleging that
respondents committed serious irregularities constituting mistrial and resulting in miscarriage of justice
and gross violation of the constitutional rights of the petitioners and the sovereign people of the
Philippines to due process of law.
Issue:
Whether or not petitioner was deprived of his rights as an accused.
Whether or not there was a violation of the double jeopardy clause.
Held:
Petitioners' second motion for reconsideration is granted and ordering a re-trial of the said cases which
should be conducted with deliberate dispatch and with careful regard for the requirements of due process.
Deputy Tanodbayan Manuel Herrera (made his expose 15 months later when former Pres. was no longer
around) affirmed the allegations in the second motion for reconsideration that he revealed that the
Sandiganbayan Justices and Tanodbayan prosecutors were ordered by Marcos to whitewash the AquinoGalman murder case. Malacaang wanted dismissal to the extent that a prepared resolution was sent to
the Investigating Panel. Malacaang Conference planned a scenario of trial where the former President
ordered then that the resolution be revised by categorizing the participation of each respondent; decided
that the presiding justice, Justice Pamaran, (First Division) would personally handle the trial. A conference
was held in an inner room of the Palace. Only the First Lady and Presidential Legal Assistant Justice
Lazaro were with the President. The conferees were told to take the back door in going to the room where
the meeting was held, presumably to escape notice by the visitors in the reception hall waiting to see the
President. During the conference, and after an agreement was reached, Pres. Marcos told them 'Okay,
mag moro-moro na lamang kayo;' and that on their way out of the room Pres. Marcos expressed his
thanks to the group and uttered 'I know how to reciprocate'.

Page
Section 1-C, SY 06-07

143

The Court then said that the then President (code-named Olympus) had stage-managed in and from
Malacaang Palace "a scripted and predetermined manner of handling and disposing of the AquinoGalman murder case;" and that "the prosecution in the Aquino-Galman case and the Justices who tried
and decided the same acted under the compulsion of some pressure which proved to be beyond their
capacity to resist. Also predetermined the final outcome of the case" of total absolution of the twenty-six
respondents-accused of all criminal and civil liability. Pres. Marcos came up with a public statement aired
over television that Senator Aquino was killed not by his military escorts, but by a communist hired gun. It
was, therefore, not a source of wonder that President Marcos would want the case disposed of in a
manner consistent with his announced theory thereof which, at the same time, would clear his name and
his administration of any suspected guilty participation in the assassination. such a procedure would be a
better arrangement because, if the accused are charged in court and subsequently acquitted, they may
claim the benefit of the doctrine of double jeopardy and thereby avoid another prosecution if some other
witnesses shall appear when President Marcos is no longer in office.
More so was there suppression of vital evidence and harassment of witnesses. The disappearance of
witnesses two weeks after Ninoy's assassination. According to J. Herrera, "nobody was looking for these
persons because they said Marcos was in power. The assignment of the case to Presiding Justice
Pamaran; no evidence at all that the assignment was indeed by virtue of a regular raffle, except the
uncorroborated testimony of Justice Pamaran himself. The custody of the accused and their confinement
in a military camp, instead of in a civilian jail. The monitoring of proceedings and developments from
Malacaang and by Malacaang personnel. The partiality of Sandiganbayan betrayed by its decision:
That President Marcos had wanted all of the twenty-six accused to be acquitted may not be denied. In
rendering its decision, the Sandiganbayan overdid itself in favoring the presidential directive. Its bias and
partiality in favor of the accused was clearly obvious. The evidence presented by the prosecution was
totally ignored and disregarded.
The record shows that the then President misused the overwhelming resources of the government and
his authoritarian powers to corrupt and make a mockery of the judicial process in the Aquino-Galman
murder cases. "This is the evil of one-man rule at its very worst." Our Penal Code penalizes "any
executive officer who shall address any order or suggestion to any judicial authority with respect to any
case or business coming within the exclusive jurisdiction of the courts of justice."
Impartial court is the very essence of due process of law. This criminal collusion as to the handling and
treatment of the cases by public respondents at the secret Malacaang conference (and revealed only
after fifteen months by Justice Manuel Herrera) completely disqualified respondent Sandiganbayan and
voided ab initio its verdict. The courts would have no reason to exist if they were allowed to be used as
mere tools of injustice, deception and duplicity to subvert and suppress the truth. More so, in the case at
bar where the people and the world are entitled to know the truth, and the integrity of our judicial system
is at stake.
There was no double jeopardy. Courts' Resolution of acquittal was a void judgment for having been
issued without jurisdiction. No double jeopardy attaches, therefore. A void judgment is, in legal effect, no
judgment at all. By it no rights are divested. It neither binds nor bars anyone. All acts and all claims
flowing out of it are void.
Motion to Disqualify/Inhibit should have been resolved ahead. In this case, petitioners' motion for
reconsideration of the abrupt dismissal of their petition and lifting of the TRO enjoining the
Sandiganbayan from rendering its decision had been taken cognizance of by the Court which had
required the respondents', including the Sandiganbayan's, comments. Although no restraining order was
issued anew, respondent Sandiganbayan should not have precipitately issued its decision of total
absolution of all the accused pending the final action of this Court. All of the acts of the respondent judge
manifest grave abuse of discretion on his part amounting to lack of jurisdiction which substantively
prejudiced the petitioner.
With the declaration of nullity of the proceedings, the cases must now be tried before an impartial court
with an unbiased prosecutor. Respondents accused must now face trial for the crimes charged against
them before an impartial court with an unbiased prosecutor with all due process.
The function of the appointing authority with the mandate of the people, under our system of government,
is to fill the public posts. Justices and judges must ever realize that they have no constituency, serve no
majority nor minority but serve only the public interest as they see it in accordance with their oath of
office, guided only the Constitution and their own conscience and honor.
PEOPLE VS. DRAMAYO
[42 SCRA 60; G.R. L-21325; 29 OCT 1971]

Page
Section 1-C, SY 06-07

144

San Beda College of Law Alabang


Constitutional Law 2 Case Digests
Facts:
Dramayo brought up the idea of killing Estelito Nogaliza so that he could not testify in the robbery case
where he is an accused. The idea was for Dramayo and Ecubin to ambush Estelito, who was returning
from Sapao. The others were to station themselves nearby. Only Dramayo and Ecubin were convicted in
the RTC for murder. Hence the appeal
Issue:
Whether or not the accuseds criminal liability proved beyond reasonable doubt.
Held:
Yes. It is to be admitted that the starting point is the Presumption of innocence. So it must be, according
to the Constitution. That is a right safeguarded both appellants. Accusation is not, according to the
fundamental law, synonymous with guilt. It is incumbent on the prosecution demonstrate that culpability
lies. Appellants were not even called upon then to offer evidence on their behalf. Their freedom is forfeit
only if the requisite quantum of proof necessary for conviction be in existence. Their guilt be shown
beyond reasonable doubt. What is required then is moral certainty. "By reasonable doubt is meant that
which of possibility may arise, but it is doubt engendered by an investigation of the whole proof and an
inability, after such investigation, to let the mind rest easy upon the certainty of guilt. Absolute certain of
guilt is not demanded by the law to convict of any carnal charge but moral certainty is required, and this
certainty is required as to every proposition of proof regular to constitute the offense."
The judgment of conviction should not have occasioned any surprise on the part of the two appellants, as
from the evidence deserving of the fullest credence, their guilt had been more than amply demonstrated.
The presumption of innocence could not come to their rescue as it was more than sufficiently overcome
by the proof that was offered by the prosecution. The principal contention raised is thus clearly untenable.
It must be stated likewise that while squarely advanced for the first time, there had been cases where this
Court, notwithstanding a majority of the defendants being acquitted, the element of conspiracy likewise
being allegedly present, did hold the party or parties, responsible for the offense guilty of the crime
charged, a moral certainty having arisen as to their capability.
DUMLAO VS. COMELEC
[95 SCRA 392; L-52245; 22 JAN 1980]
Facts:
Petitioner Dumlao questions the constitutionality of Sec. 4 of Batas Pambansa Blg 52 as discriminatory
and contrary to equal protection and due process guarantees of the Constitution. Sec. 4 provides that any
retired elective provicial or municipal official who has received payments of retirement benefits and shall
have been 65 years of age at the commencement of the term of office to which he seeks to be elected,
shall not be qualified to run for the same elective local office from which he has retired. According to
Dumlao, the provision amounts to class legislation. Petitioners Igot and Salapantan Jr. also assail the
validity of Sec. 4 of Batas Pambansa Blg 52, which states that any person who has committed any act of
disloyalty to the State, including those amounting to subversion, insurrection, rebellion, or other similar
crimes, shall not be qualified for any of the offices covered by the act, or to participate in any partisan
activity therein: provided that a judgment of conviction of those crimes shall be conclusive evidence of
such fact and the filing of charges for the commission of such crimes before a civil court or military
tribunal after preliminary investigation shall be prima facie evidence of such fact.
Issue:
Whether or not the aforementioned statutory provisions violate the Constitution and thus, should be
declared null and void
Held:
In regards to the unconstitutionality of the provisions, Sec. 4 of BP Blg 52 remains constitutional and valid.
The constitutional guarantee of equal protection of the laws is subject to rational classification. One class
can be treated differently from another class. In this case, employees 65 years of age are classified
differently from younger employees. The purpose of the provision is to satisfy the need for new blood in
the workplace. In regards to the second paragraph of Sec. 4, it should be declared null and void for being
violative of the constitutional presumption of innocence guaranteed to an accused. Explicit is the
constitutional provision that, in all criminal prosecutions, the accused shall be presumed innocent until the
contrary is proved, and shall enjoy the right to be heard by himself and counsel (Article IV, section 19,
1973 Constitution). An accusation, according to the fundamental law, is not synonymous with guilt. The

Page
Section 1-C, SY 06-07

145

challenged proviso contravenes the constitutional presumption of innocence, as a candidate is


disqualified from running for public office on the ground alone that charges have been filed against him
before a civil or military tribunal. It condemns before one is fully heard. In ultimate effect, except as to the
degree of proof, no distinction is made between a person convicted of acts of dislotalty and one against
whom charges have been filed for such acts, as both of them would be ineligible to run for public office. A
person disqualified to run for public office on the ground that charges have been filed against him is
virtually placed in the same category as a person already convicted of a crime with the penalty of arresto,
which carries with it the accessory penalty of suspension of the right to hold office during the term of the
sentence (Art. 44, Revised Penal Code).
And although the filing of charges is considered as but prima facie evidence, and therefore, may be
rebutted, yet. there is "clear and present danger" that because of the proximity of the elections, time
constraints will prevent one charged with acts of disloyalty from offering contrary proof to overcome the
prima facie evidence against him.
Additionally, it is best that evidence pro and con of acts of disloyalty be aired before the Courts rather than
before an administrative body such as the COMELEC. A highly possible conflict of findings between two
government bodies, to the extreme detriment of a person charged, will thereby be avoided. Furthermore,
a legislative/administrative determination of guilt should not be allowed to be substituted for a judicial
determination.
Being infected with constitutional infirmity, a partial declaration of nullity of only that objectionable portion
is mandated. It is separable from the first portion of the second paragraph of section 4 of Batas
Pambansa Big. 52 which can stand by itself.
Wherefore, the first paragraph of section 4 of Batas pambansa Bilang 52 is hereby declared valid and that
portion of the second paragraph of section 4 of Batas Pambansa Bilang 52 is hereby declared null and
void, for being violative of the constitutional presumption of innocence guaranteed to an accused.
PEOPLE VS. ALCANTARA
[240 SCRA 122; G.R. NO. 91283; 17 JAN 1995]
Facts:
On July 19, 1988, Venancio Patricio, accompanied by Larry Salvador, drove a ten-wheeler truck a CocaCola plant in Antipolo to load cases of softdrinks. They were about to leave the plant at 10:00pm when
several men approached them to hitch for a ride. Ascertaining that Salvador knew appellant, Venancio
accommodated appellant's request. Appellant had four companions. At Ortigas Ave., one of them poked a
gun at Venancio and grabbed the steering wheel. At the North Diversion Road, Venancio and
Salvador(helper) were brought down from the vehicle and tied to the fence of the expressway, thereafter
they were stabbed and left bleeding to death. Venancio survived but Salvador did not.Appellant was
arrested in the vicinity of Otis Street in Pandacan, Manila. A few days later, he was turned over to the
Constabulary Highway Patrol Group. Sgt. Alberto Awanan brought the appellant to the MCU hospital and
was presented to Venancio for identification. Appellant was brought to the Headquarters at Camp Crame
where he confessed.
Appelants Defense: Denial and alibi. He said that he was just applying to be a driver and stayed there
even if he was told that no work was available, to confirm with the truck drivers. While he was applying for
CONCEPCION TRUCKING located across Otis street from the Coca-cola plant. He was arrested. He
denied any knowledge of the "hit" on the Coca-cola delivery truck. He remained in the custody of the
police for two days and two nights. On the third day of his detention, he was turned over to the
Constabulary Highway Patrol Group. Appellant was the brought to the MCU hospital. He was made to
confront Venancio whom he saw for the first time. CHPG Sgt. Awanan asked Venancio twice if appellant
was among those who hijacked the truck he was driving. On both times, Venancio did not respond.
Undaunted, Sgt. Awanan, called to a photographer present, forced appellant to stand about a foot from
Venancio, and told the latter to just point at the suspect. "Basta ituro mo lang," Sgt. Awanan directed.
Venancio obeyed, and pictures of him pointing to the suspect were taken. From the hospital, appellant
was brought to the Constabulary Highway Patrol Group headquarters at Camp Crame. Without being
apprised of his rights nor provided with counsel, he was interrogated and urged to confess his guilt. He
balked. At ten o'clock that night, hours after questioning began, appellant's interrogators started boxing
him and kicking him. He was also hit on the back with a chair, and electrocuted. Still, he refused to admit
to the crime. In the midst of his ordeal, appellant heard someone say, "Tubigan na iyan." He was then
blindfolded and brought to another room where he was made to lie down. Water was slowly and
continuously poured on his face, over his mouth. Appellant could no longer bear the pain caused by the
water treatment. Finally, he confessed to being one of the hijackers. He was led to another room, where
he was handcuffed and left until the following day. Later, he was made to sign prepared statements
containing his full confession.

Page
Section 1-C, SY 06-07

146

San Beda College of Law Alabang


Constitutional Law 2 Case Digests
Alcantara was arraigned under an information charging him and four others (at large) with the crime of
robbery with Homicide and Frustrated Homicide.
The trial court convicted the accused despite the following inconsistency between Venancios affidavit and
testimony:
Affidavit
1. mentioned 5 assailants
2. stabbing was preceded by a
3. conference by all assailants
4. claimed to have allowed assailants
5. to hitch a ride because Alcantara
6. was familiar to them

Testimony
- only Alcantara was identified
- only 3 assailants had a
- conference
- failed to identify Alcantara
- at the hospital and in open
- court (pointed to another person)

Issue:
Whether or not the rights of the accused was violated.

Held:
YES. The peoples evidence failed to meet the quantum required to overcome the presumption. The
second identification which correctly pointed to accused by Venancio should not be credited. There is no
reason for him to err as they know each other for 3 years. It was also incorrect to give too much weight to
Police Sgt. Awanans testimony as to the previous identification at the hospital. The testimony of Sgt.
Awanan was not corroborated by Venancio.
The identification procedure was irregular. Due process demands that the identification procedure of
criminal suspects must be free from impermissible suggestions as the influence of improper suggestion
probably accounts for more miscarriages of justice than any other single factor. Conviction must be based
on the strength of the prosecution and not the weakness of the defense. There was blatant violation of the
constitutional rights of appellant as an accused. Appellant belongs to the economically deprived in our
society. He is nearly illiterate(third grade education). Our Constitution and our laws strictly ordain their
protection following the Magsaysay desideratum that those who have less in life should have more in law.
CORPUZ VS. REPUBLIC
[194 SCRA 73; G.R. NO. 74259; 14 FEB 1991]
Facts:
Generoso Corpuz is the Supervising Accounting Clerk in the Office of the Provincial Treasurer of Nueva
Viscaya. He was designated Acting Supervising Cashier in the said office. In this capacity, he received
collections, disbursed funds and made bank deposits and withdrawals pertaining to government accounts.
On April 13, 1981 his designation as Acting Supervising Cashier was terminated and a transfer of
accountabilities was effected between him and his successor. The Certificate of turnover revealed a
shortage of P72,823.00. He was able to pay only P10,159.50. After a final demand letter for the total of
P50,596.07 which was not met, a case of malversation was filed against him. Corpuz did not deny such
facts but he insists that the shortage was malversed by other persons. He alleged that Paymaster
Diosdado Pineda through 1 of 4 separate checks (PNB) issued and encashed such checks while he was
of leave. Also, Acting Deputy Provincial Treasurer Bernardo Aluning made to post the amount on his
cashbook although he had not received the said amount. He was convicted in Sandiganbayan.
Issue:
Whether or Not Corpuz is guilty of malversation.
Held:
It is a subtle way of camouflaging the embezzlement of the money equivalent when 1 of the 4 checks
issued and encashed in the same day was entered in the accuseds cash book 3 months after such
encashments. Also, Corpuz claim that he was absent when Paymaster Diosdado Pineda through 1 of 4
separate checks (PNB) issued and encashed such checks, was not proven.

Page
Section 1-C, SY 06-07

147

Post-Audit is not a preliminary requirement to filing a malversation case. The failure of the public officer to
have duly forthcoming any public funds with which he is chargeable, upon demand by an authorized
officer shall be a prima facie evidence that he has put such missing funds to personal use.
The equipoise rule(balancing test) which is the presumption of innocence is applicable only where the
evidence of the parties is evenly balance, in which case the scale of justice should be tilt in favor of the
accused. There is no such balance in the case at bar. The evidence of the prosecution is overwhelming
and has not been overcome by the petitioner with his claims. The presumed innocence must yield to the
positive finding that he is guilty of malversation.
Wherefore his petition is denied. He is guilty as principal of Malversation of Public Funds.
PEOPLE VS. HOLGADO
[85 PHIL 752; G.R.L-2809; 22 MAR 1950]
Facts:
Appellant Frisco Holgado was charged in the court of First Instance of Romblon with slight illegal
detention because according to the information, being a private person, he did "feloniously and without
justifiable motive, kidnap and detain one Artemia Fabreag in the house of Antero Holgado for about eight
hours thereby depriving said Artemia Fabreag of her personal liberty. He pleaded guilty (without a
counsel) and said that he was just instructed by Mr. Ocampo, which no evidence was presented to indict
the latter.
Issue:
Whether or Not there was any irregularity in the proceedings in the trial court.
Held:
Yes. Rule 112, section 3 of ROC that : If the defendant appears without attorney, he must be informed by
the court that it is his right to have attorney being arraigned., and must be asked if he desires the aid of
attorney, the Court must assign attorney de oficio to defend him. A reasonable time must be allowed for
procuring attorney. This was violated. Moreso the guarantees of our Constitution that "no person shall be
held to answer for a criminal offense without due process of law", and that all accused "shall enjoy the
right to be heard by himself and counsel." In criminal cases there can be no fair hearing unless the
accused be given the opportunity to be heard by counsel.
The trial court failed to inquire as to the true import of the qualified plea of accused. The record does not
show whether the supposed instructions of Mr. Ocampo was real and whether it had reference to the
commission of the offense or to the making of the plea guilty. No investigation was opened by the court on
this matter in the presence of the accused and there is now no way of determining whether the supposed
instruction is a good defense or may vitiate the voluntariness of the confession. Apparently the court
became satisfied with the fiscal's information that he had investigated Mr. Ocampo and found that the
same had nothing to do with this case. Such attitude of the court was wrong for the simple reason that a
mere statement of the fiscal was not sufficient to overcome a qualified plea of the accused. But above all,
the court should have seen to it that the accused be assisted by counsel especially because of the
qualified plea given by him and the seriousness of the offense found to be capital by the court.
PEOPLE VS. MAGSI
[124 SCRA 64; G.R. NO.L-32888; 12 AUG 1983]
Facts:
Soon after appellant was apprehended on August 20, 1970, his arraignment was scheduled before the
Criminal Circuit Court of San Fernando, La Union. The case was actually set and rescheduled for six (6)
times, first of which was on August 1, 1970. On that date, despite appointment by the court of Atty. Mario
Rivera as de officio counsel for the accused, hearing was re-set to September 8, 1970 on motion of Atty.
Rivera, who was prompted to ask for it because of accused desire to be represented by a de parte
counsel. Prior to the next hearing, Atty. Rivera moved to withdraw as de officio counsel and it was
favorably acted on by the court on September 7, 1970. At the second hearing on September 8, 1970, for
failure of the de officio and de parte counsels to appear, despite a second call of the case, the hearing
was re-set for the next day and the court appointed Atty. Dominador Cariaso de officio counsel for the
accused. On the third hearing date, neither the de parte nor the de officio counsel was in Court, so Atty.
Rivera was reappointed that day as de officio counsel for arraignment purposes only. The accused del

Page
Section 1-C, SY 06-07

148

San Beda College of Law Alabang


Constitutional Law 2 Case Digests
Rosario entered a plea of guilty but qualified it with the allegation that he committed the crime out of fear
of his co-accused Eloy Magsi and the other coaccused. Appellant was found guilty of murder and made to
suffer the death penalty.
Issue:
Whether or not there was a violation of the rights of the accused.
Held:
YES. The desire to speed up the disposition of cases should not be effected at the sacrifice of the basic
rights of the accused. Citing People vs. Domingo (55 SCRA 243-244): the trial courts should exercise
solicitous care before sentencing the accused on a plea of guilty especially in capital offenses by first
insuring that the accused fully understands the gravity of the offense, the severity of the consequences
attached thereto as well as the meaning and significance of his plea of guilty; and that the prudent and
proper thing to do in capital cases is to take testimony, to assure the court that the accused has not
misunderstood the nature and effect of his plea of guilty. Mere pro-forma appointment of de officio
counsel, who fails to genuinely protect the interests of the accused, resetting of hearing by the court for
alleged reception of evidence when in fact none was conducted, perfunctory queries addressed to the
accused whether he understands the charges and the gravity of the penalty, are not sufficient compliance.

SORIANO VS. SANDIGANBAYAN


[131 SCRA 184; G.R. NO.L-65952; 31 JUL 1984]
Facts:
Tan was accused of qualified theft. The petitioner, who was an Asst. Fiscal, was assigned to investigate.
In the course of the investigation, petitioner demanded Php.4000 from Tan as price for dismissing the
case. Tan reported it to the NBI which set up an entrapment. Tan was given a Php.2000, marked bill, and
he had supplied the other half. The entrapment succeeded and an information was filed with the
Sandiganbayan. After trial, the Sandiganbayan rendered a decision finding the petitioner guilty as a
principal in violating the Anti Graft and Corrupt Practices Act (R.A.3019). A motion for reconsideration was
denied by the Sandiganbayan, hence this instant petition.
Issue:
Whether or Not the investigation conducted by the petitioner can be regarded as contract or transaction
within the purview of .RA.3019.
Held:
R.A. 3019 Sec.3. Corrupt practices of public officers - In addition to acts or omissions of public officers
already penalized by existing laws, the following shall constitute corrupt practices of any public officer and
are hereby declared to be unlawful: xxx b. Directly or indirectly requesting or receiving any gift, present,
share percentage or benefit, for himself or for other person, in connection with any contract or transaction
between the Govt. and any other party wherein the public officer in his official capacity has to intervene
under the law.
The petitioner stated that the facts make out a case of direct bribery under Art.210 of the RPC and not a
violation of R.A. 3019 sec.3 (b). The offense of direct bribery is not the offense charged and is not
included in the offense charged which is violation of R.A.3019 sec.3 (b).
The respondent claimed that, transaction as used hereof, is not limited to commercial or business
transaction, but includes all kinds of transaction whether commercial, civil, or administrative in nature.
The court agrees with the petitioner. It is obvious that the investigation conducted by the petitioner was
neither a contract nor transaction. A transaction like a contract is one which involves some consideration
as in credit transactions. And this element is absent in the investigation conducted by the petitioner.
Judgment modified. Petitioner is guilty of direct bribery under Art.210 of the RPC.

Page
Section 1-C, SY 06-07

149

BORJA VS. MENDOZA


[77 SCRA 422; G.R. NO.L-45667; 20 JUN 1977]
Facts:
Borja was accused of slight physical injuries in the City of Cebu. However, he was not arraigned. That not
withstanding, respondent Judge Senining proceeded with the trial in absentia and rendered a decision
finding petitioner guilty of the crime charged. The case was appealed to the Court o First Instance in Cebu
presided by respondent Judge Mendoza. It was alleged that the failure to arraign him is a violation of his
constitutional rights. It was also alleged that without any notice to petitioner and without requiring him to
submit his memorandum, a decision on the appealed case was rendered The Solicitor General
commented that the decision should be annulled because there was no arraignment.
Issue:
Whether or Not petitioners constitutional right was violated when he was not arraigned.
Held:
Yes. Procedural due process requires that the accused be arraigned so that he may be informed as to
why he was indicted and what penal offense he has to face, to be convicted only on a showing that his
guilt is shown beyond reasonable doubt with full opportunity to disprove the evidence against him. It is
also not just due process that requires an arraignment. It is required in the Rules that an accused, for the
first time, is granted the opportunity to know the precise charge that confronts him. It is imperative that he
is thus made fully aware of possible loss of freedom, even of his life, depending on the nature of the crime
imputed to him. At the very least then, he must be fully informed of why the prosecuting arm of the state is
mobilized against him. Being arraigned is thus a vital aspect of the constitutional rights guaranteed him.
Also, respondent Judge Senining convicted petitioner notwithstanding the absence of an arraignment.
With the violation of the constitutional right to be heard by himself and counsel being thus manifest, it is
correct that the Solicitor General agreed with petitioner that the sentence imposed on him should be set
aside for being null. The absence of an arraignment can be invoked at anytime in view of the
requirements of due process to ensure a fair and impartial trial.
Wherefore, the petition for certiorari is granted. The decision of respondent Judge Romulo R. Senining
dated December 28, 1973, finding the accused guilty of the crime of slight physical injuries, is nullified and
set aside. Likewise, the decision of respondent Judge Rafael T. Mendoza dated November 16, 1976,
affirming the aforesaid decision of Judge Senining, is nullified and set aside. The case is remanded to the
City Court of Cebu for the prosecution of the offense of slight physical injuries, with due respect and
observance of the provisions of the Rules of Court, starting with the arraignment of petitioner.

CONDE VS. RIVERA


[45 PHIL 650; G.R. NO. 21741; 25 JAN 1924]
Facts:
Aurelia Conde, formerly a municipal midwife in Lucena, Tayabas, has been forced to respond to no less
the five information for various crimes and misdemeanors, has appeared with her witnesses and counsel
at hearings no less than on eight different occasions only to see the cause postponed, has twice been
required to come to the Supreme Court for protection, and now, after the passage of more than one year
from the time when the first information was filed, seems as far away from a definite resolution of her
troubles as she was when originally charged.
Issue:
Whether or Not petitioner has been denied her right to a speedy and impartial trial.
Held:
Philippine organic and statutory law expressly guarantee that in all criminal prosecutions the accused
shall enjoy the right to have a speedy trial. Aurelia Conde, like all other accused persons, has a right to a
speedy trial in order that if innocent she may go free, and she has been deprived of that right in defiance
of law. We lay down the legal proposition that, where a prosecuting officer, without good cause, secures
postponements of the trial of a defendant against his protest beyond a reasonable period of time, as in
this instance for more than a year, the accused is entitled to relief by a proceeding in mandamus to

Page
Section 1-C, SY 06-07

150

San Beda College of Law Alabang


Constitutional Law 2 Case Digests
compel a dismissal of the information, or if he be restrained of his liberty, by habeas corpus to obtain his
freedom.
PEOPLE VS. TAMPAL
[244 SCRA 202; G.R. NO. 102485; 22 MAY 1995]
Facts:
Luis Tampal, Domingo Padumon, Arsenio Padumon, Samuel Padumon, Pablito Suco, Dario Suco and
Galvino Cadling were charged of robbery with homicide and multiple serious physical injuries in the
Regional Trial Court of Zamboanga with Hon. Wilfredo Ochotorena as presiding judge. However, only
private respondents, Luis Tampal, Domingo Padumon, Arsenio Padumon, and Samuel Padumon were
arrested, while the others remained at large.
The case was set for hearing on July 26, 1991, but Assistant Provincial Prosecutor Wilfredo Guantero
moved for postponement due to his failure to contact the material witnesses. The case was reset without
any objection from the defense counsel. The case was called on September 20, 1991 but the prosecutor
was not present. The respondent judge considered the absence of the prosecutor as unjustified, and
dismissed the criminal case for failure to prosecute. The prosecution filed a motion for reconsidereation,
claiming that his absence was because such date was a Muslim holiday and the office of the Provincial
prosecutor was closed on that day. The motion was denied by respondent judge.
Issue:
Whether or Not the postponement is a violation of the right of the accused to a speedy disposition of their
cases.
Whether or Not the dismissal serves as a bar to reinstatement of the case.
Held:
In determining the right of an accused to speedy disposition of their case, courts should do more than a
mathematical computation of the number of postponements of the scheduled hearings of the case. What
are violative of the right of the accused to speedy trial are unjustified postponements which prolong trial
for an unreasonable length of time. In the facts above, there was no showing that there was an unjust
delay caused by the prosecution, hence, the respondent judge should have given the prosecution a fair
opportunity to prosecute its case.
The private respondents cannot invoke their right against double jeopardy. In several cases it was held
that dismissal on the grounds of failure to prosecute is equivalent to an acquittal that would bar another
prosecution for the same offense, but in this case, this does not apply, considering that the rights of the
accused to a speedy trial was not violated by the State. Therefore, the order of dismissal is annulled and
the case is remanded to the court of origin for further proceedings.
REPUBLIC ACT NO. 8493 THE SPEEDY TRIAL ACT
The arraignment of an accused shall be held within 30 days from filing of the information, or from the date
the accused has appeared before the justice, judge or court in which the charge is pending, whichever
date last occurs. Thereafter, where a plea of not guilty is entered, the accused shall have at least 15 days
to prepare for trial. Trial shall commence within 30 days from arraignment as fixed by the court. In no case
shall the entire trial period exceed 180 days from the 1 st day of trial, except as otherwise authorized by the
Chief Justice of the Supreme Court.
RE: REQUEST FOR LIVE TV OF TRIAL OF JOSEPH ESTRADA
[360 SCRA 248; A.M. NO 01-4-03-SC; 29 JUN 2001]
Facts:
The Kapisanan ng mga Brodkaster ng Pilipinas (KBP) sent a letter requesting the Court to allow live
media coverage of the anticipated trial of the plunder and other criminal cases filed against former
President Joseph E. Estrada before the Sandiganbayan in order "to assure the public of full transparency
in the proceedings of an unprecedented case in our history." The request was seconded by Mr. Cesar N.
Sarino and, still later, by Senator Renato Cayetano and Attorney Ricardo Romulo.

Page
Section 1-C, SY 06-07

151

Issue:
Whether or Not live media coverage of the trial of the plunder and other criminal cases filed against
former President Joseph E. Estrada should be permitted by the court.
Held:
The propriety of granting or denying the instant petition involve the weighing out of the constitutional
guarantees of freedom of the press and the right to public information, on the other hand, along with the
constitutional power of a court to control its proceedings in ensuring a fair and impartial trial. When these
rights race against one another, jurisprudence tells us that the right of the accused must be preferred to
win.
Due process guarantees the accused a presumption of innocence until the contrary is proved in a trial
that is not lifted about its individual settings nor made an object of publics attention and where the
conclusions reached are induced not by any outside force or influence but only be evidence and
argument given in open court, where fitting dignity and calm ambiance is demanded.
An accused has a right to a public trial but it is a right that belongs to him, more than anyone else, where
his life or liberty can be held critically in balance. A public trial aims to ensure that he is fairly dealt with
and would not be unjustly condemned and that his rights are not compromised in secret conclaves of long
ago. A public trial is not synonymous with publicized trial, it only implies that the court doors must be open
to those who wish to come, sit in the available seats, conduct themselves with decorum and observe the
trial process.
The courts recognize the constitutionally embodied freedom of the press and the right to public
information. It also approves of media's exalted power to provide the most accurate and comprehensive
means of conveying the proceedings to the public. Nevertheless, within the courthouse, the overriding
consideration is still the paramount right of the accused to due process which must never be allowed to
suffer diminution in its constitutional proportions.
PEOPLE VS. SALAS
[143 SCRA 163; G.R. NO. L-66469; 29 JUL 1986]
Facts:
At about 6:00 o'clock in the morning of March 6, 1992, a 60 year old woman, identified as Virginia Talens
was found lying dead in a canal at Bo. San Nicolas, Mexico, Pampanga; she was last seen alive at about
3:00 o'clock early morning of March 6, 1992 by Orlando Pangan and Richard Pangan who were with her
going home coming from the wake of one Leonardo Flores; both Orlando and Richard Pangan testified
that accused was with them in going home at about 3:00 o'clock in the morning of March 6, 1992; Orlando
and Richard Pangan reached first their house and left the two on the way and that was the last time
Virginia was seen alive; just a few minutes after reaching his house and while inside his house, Orlando
Pangan heard a shout; another woman, one Serafia Gutierrez, testified that she likewise was awakened
by a shout at about 3:00 in the morning; Dr. Aguda who autopsied the victim found hematoma on the
head and chest, an abrasion on the left chin and stabwound on the neck which stabwound, the doctor
claims, was the cause of death of the victim; Police Investigator Gonzales who immediately responded
upon report, recovered at the scene a pin, the victim's wristwatch, earring, a ring and P135.00 money; he
likewise found on March 9, 1992 when he continued his investigation bloodstain on the front door of the
house of the accused which bloodstain when submitted for examination was found to be of human blood;
one Resultay was with Virginia Talens at about 5:00 afternoon of March 5, 1992 in going to the wake, who
claims that Virginia had money on a purse as while they were on the way Virginia bet on a jueteng she
saw Virginia got money from her purse a P500.00 bill but as she had no change she instead took P8.00
from her other pocket; one Ramil Talens, a son of the victim corroborated the claim of Resultay that
Virginia had with her at that time money worth P2,000.00 as in the morning of March 5, 1992 he gave her
mother for safekeeping the sum of P1,500.00 which he claims his mother placed in her purse and claims
further that at the wake, he asked and was given P50.00 by his mother as he also participated in the
gambling thereat, however, the purse of Virginia containing about P2,000.00 was no longer to be found
when she was found dead; Orlando Pangan saw the accused gambled in the wake; Virginia likewise
gambled at the wake; accused had been working for three days before March 6 at Sta. Ana, Pampanga
and up to March 5, 1992, but the following day, he did not anymore report for work at Sta. Ana,
Pampanga, was no longer to be found and was last seen at about 3:00 morning together with Virginia
Talens on their way home coming from the wake; the parents of [the] accused were informed by
Investigator Gonzales that their son was the suspect and adviced them to surrender him, but since March
6, 1992 when accused left Mexico, Pampanga, he returned only on September 19, 1992 at Arayat,
Pampanga, not at Mexico, Pampanga where he was ultimately apprehended by the Mexico Police on

Page
Section 1-C, SY 06-07

152

San Beda College of Law Alabang


Constitutional Law 2 Case Digests
September 22, 1992 after chancing on a radio message by the police of Arayat to their Provincial
commander that a vehicular incident occurred at Arayat, Pampanga where one Elmer Salas was the
victim and was hospitalized at the district hospital at Arayat, Pampanga where he used the name of
Rommel Salas and not Elmer Salas. The trial court rendered convicting Salas for Robbery with Homicide
Issue:
Whether or Not there is evidence sufficient to sustain a conviction of the appellant of the crime of Robbery
with Homicide.
Whether or Not the appellants crime homicide or robbery with homicide.
Held:
There was no eyewitness or direct evidence; either to the robbery or to the homicide and none of the
things allegedly stolen were ever recovered. However, direct evidence is not the only matrix from which
the trial court may draw its findings and conclusion of culpability. Resort to circumstantial evidence is
essential when to insist on direct testimony would result in setting felons free.
For circumstantial evidence to be sufficient to support a conviction, all the circumstances must be
consistent with each other, consistent with the theory that the accused is guilty of the offense charged,
and at the same time inconsistent with the hypothesis that he is innocent and with every other possible,
rational hypothesis excepting that of guilt. All the circumstances established must constitute an unbroken
chain which leads to one and fair and reasonable conclusion pointing solely to the accused, to the
exclusion of all other persons, as the author of the crime. The facts and circumstances consistent with the
guilt of the accused and inconsistent with his innocence can constitute evidence which, in weight and
probative value, may be deemed to surpass even direct evidence in its effect on the court.
The fatal stabbing of Virginia Talens occurred at around 3:00 a.m. of March 6, 1992. Appellant hastily
abandoned his house in Barrio San Nicolas, Mexico, Pampanga, his residence since childhood, on that
very date. Appellant was nowhere when his co-worker and barrio mate, Eduardo Bagtas, came to
appellant's house to fetch him for work at around 6:30 to 7:00 a.m. of March 6, 1992. Appellant also
abandoned his job as a painter in Sta. Ana, Pampanga, on March 6, 1992, the date of the crime, leaving
behind an unfinished painting project. He was not seen again from said date. Police investigators found
human bloodstains on the front door of appellant's house, on his clothing, and on his yellow slippers after
the victim was killed. Despite efforts of the police to find appellant as the principal suspect, a fact known
to appellant's family and neighbors, appellant did not present himself to the authorities. Appellant was
apprehended only a full six months after the date of the crime, following his confinement in a hospital in
Arayat, Pampanga because he was sideswiped by a Victory Liner bus in Arayat. When hospitalized,
appellant used the alias Rommel Salas, instead of his true name Elmer Salas. These circumstances
denote flight, which when unexplained, has always been considered by the courts as indicative of guilt.
Both appellant and victim gambled at the wake they attended. The victim was, in fact, enjoying a winning
streak when her son, Ramil Talens, came to fetch her but which he failed to do because his mother was
winning, and she refused to leave. The purse of Talens containing cash was gone when her corpse was
found in the canal with a stab wound and bruises. What was left was a safety pin which victim used to
fasten the missing purse to her clothes.
Denial is an inherently weak defense which must be buttressed by strong evidence of non-culpability to
merit credibility. Denial is negative and self-serving and cannot be given greater evidentiary weight over
the testimonies of credible witnesses who positively testified that appellant was at the locus criminis and
was the last person seen with the victim alive.
The absence of evidence showing any improper motive on the part of the principal witness for the
prosecution to falsely testify against the appellant strongly tends to buttress the conclusion that no such
improper motive exists and that the testimony of said witnesses deserve full faith and credit.
The essence of voluntary surrender is spontaneity and the intent of the accused to give himself up and
submit himself unconditionally to the authorities either because he acknowledges his guilt or he wants to
save the State the trouble of having to effect his arrest. Spontaneity and intent to give one's self up are
absent where the accused went into hiding for six months after the incident and had to resort to an alias
when he was involved in an accident being investigated by the police authorities.
Robbery with Homicide is a special complex crime against property. Homicide is incidental to the robbery
which is the main purpose of the criminal. In charging Robbery with Homicide, the onus probandi is to
establish: "(a) the taking of personal property with the use of violence or intimidation against a person; (b)
the property belongs to another; (c) the taking is characterized with animus lucrandi; and (d) on the

Page
Section 1-C, SY 06-07

153

occasion of the robbery or by reason thereof, the crime of homicide, which is used in the generic sense,
was committed." Although there was no witness as to the actual robbing of the victim, there is testimony
that the victim had more or less P2,000.00; and wore gold earrings valued at P750.00. These were never
recovered.
While there is indeed no direct proof that Virginia Talens was robbed at the time she was killed, we may
conclude from four circumstances that the robbery occasioned her killing: (1) Both appellant and victim
gambled at the wake. (2) The appellant knew that victim was winning. (3) The victim was last seen alive
with appellant. (4) The victim's purse containing her money and earrings were missing from her body
when found.
The decision of the regional trial court is affirmed. Costs against appellant. So ordered.
PEOPLE VS. MAGPALAO
[197 SCRA 79; G.R. NO. 92415; 14 MAY 1991]
Facts:
Eleven (11) people rode in a Ford Fiera going to Baguio. Namely they are: Felizardo Galvez, Jimmy
Jetwani, Simeon Calama, Rene Salonga, Eduardo Lopez, Adolfo Quiambao, Aliman Bara-akal, Anwar
Hadji Edris, Gumanak Ompa and defendant-appelants in this case, Omar Magpalao and Rex
Magumnang.
After an hour of driving, the car stopped so that one of the passengers could urinate. While the car was
stopped the Bara-akal, Edris, Ompa, Magpalao and Magumnang pointed guns and knives at the other
passengers and divested them of their properties.
On of the robbers then ordered Galvez to drive the car towards the precipice (bangin). When the car was
near the precipice, Galvez then stepped to the brakes. The other passengers jumped out of the car and
went to different directions to escape. Galvez however, was left in side the car and was stabbed by one of
the robbers. The robbers then escaped. Quiambao, who owned the car helped Galvez to get to a hospital.
Galvez died in the hospital. The robbers were then apprehended with the exception of Edris who remain
at large. Mangumnang however escaped while being in detention and Bara-akal died inside the jail. Since
Mangumnang was not arrested, the trial in absentia continued as to him. Ompa, Magpalao, and
Magumnang were all held guilty as principal by direct participation of the crime of Robbery with Homicide.
Issue:
Whether or Not the lower court erred in failing to apply the Constitutional mandate on the presumption of
innocence and proof beyond reasonable doubt when it allowed the trial in absentia to push through on the
part of defendant-appellant Magumnang.
Held:
The Court affirmed the decision of the lower court. The reason is that the lower court has jurisdiction over
Magumnang the moment the latter was in custody. Jurisdiction once acquired is not lost upon the
instance of parties but until the case is terminated. Since all the requisites of trial in absentia are
complete, the court has jurisdiction over Magumnang.
In addition, Magumnang was presumed innocent during his trial in absentia. The prosecution had strong
evidence against him as proof beyond reasonable doubt that he is a principal by direct participation in the
crime of Robbery with Homicide. Thus, the Constitutional mandate was not violated.
PEOPLE VS. ACABAL
[226 SCRA 694 ; G.R. NO. 103604, 23 SEP 1993]
Facts:
The accusatory portion in the information for murder. Facts are as follows:
"That sometime in the evening of the 28th of January, 1980, at Nagbinlod, Municipality of Sta. Catalina,
Province of Negros Oriental, Philippines, and within the jurisdiction of this Honorable Court, the accused,
including several 'John Does', conspiring and confederating with one another, with intent to kill, and with
treachery and evident premeditation and being then armed with bolos and 'pinuti', did then and there
willfully, unlawfully and feloniously attack, assault and use personal violence on the person of one

Page
Section 1-C, SY 06-07

154

San Beda College of Law Alabang


Constitutional Law 2 Case Digests
Rizalina Apatan Silvano while the latter was about to leave her house and inflicting upon her injuries, to
wit: 'right leg amputated below the knee; left leg hacked behind the knee; abdomen hacked with viscerae
evacerated,' and did then and there set the house on fire while the aforementioned Rizalina Apatan
Silvano was inside said house trying to escape therefrom, and allowing her to be burned inside said
house which was burned to the ground, thereby causing upon said Rizalina Apatan Silvano her death and
burning her beyond recognition.
But on 16 May 1987, a fire gutted the building where Branch 37 was located and the records of these two
cases were burned. The records were subsequently reconstituted upon petition of the prosecuting fiscal.
The testimonies of the witnesses were retaken, however, before it could commence, accused Engracio
Valeriano jumped bail and the warrant for his arrest issued on 16 November 1987 was returned unserved
because he could not be found. An alias warrant for his arrest was issued on 26 June 1989, but he
remains at large up to the present.
After the completion of the re-taking of the testimonies of the witnesses in Branch 37, Criminal Cases
Nos. 4584 and 4585 were re-raffled to Branch 33 of the trial court, then presided over by Judge Pacifico
S. Bulado.
The decision of the trial court, per Judge Pacifico S. Bulado, dated 31 October 1991 but promulgated on
20 December 1991, contained no specific dispositive portion. Its rulings are found in the last two
paragraphs which read as follows:
"The elements of murder in this case, Criminal Case No. 4585 for the killing of Rizalina Apatan-Silvano
having been proved by the prosecution beyond doubt, the accused JUANITO RISMUNDO, MACARIO
ACABAL and ABUNDIO NAHID, considering the attendant qualifying aggravating circumstances of
nighttime, use of fire by burning the house of victim Rizalina Apatan-Silvano in order to forcibly drive her
out of her house and hack her to death, the abuse of superior strength, the penalty impossable [sic] here
will be in its maximum degree, that is reclusion perpetua taking into account Article 248 of the Revised
Penal Code, the penalty now for murder is Reclusion Temporal to Reclusion Perpetua, and for all the
accused to indemnify the heirs of the victim the sum of Thirty Thousand (P30,000.00) Pesos since this
case occurred [sic] in 1980. For the wounding of the victim Wilson A. Silvano, this Court believes that
simple frustrated homicide only is committed by the accused Engracio Valeriano only.
But since the person who actually inflicted the injuries of victim Wilson Silvano, accused Engracio
Valeriano only is nowhere to be found, hence, not brought to the bar of justice, he being a fugitive or at
large, no penalty could be imposed on him since he is beyond the jurisdiction of this court to reach. All the
other two (2) accused, JUANITO RISMUNDO and ABUNDIO NAHID are hereby ordered and declared
absolved from any criminal responsibility from frustrated homicide.
The bail bond put up by the three accused, namely: Juanito Rismundo, Macario Acabal and Abundio
Nahid are hereby ordered cancelled and let a warrant of arrest be issued for their immediate
confinement."
Issue:
Whether or not the judgment complied with the Rules of Court.
Whether or not the cancellation of the bail bonds of the accused is valid.
Whether or not the accused may be tried in absentia.
Whether or not the accused is guilty of the crime of frustrated murder.
Held:
We find that the decision substantially complies with the Rules of Court on judgments as it did sentence
the accused-appellants to reclusion perpetua. A judgment of conviction shall state (a) the legal
qualification of the offense constituted by the acts committed by the accused, and the aggravating or
mitigating circumstances attending the commission, if there are any; (b) the participation of the accused in
the commission of the offense, whether as principal, accomplice or accessory after the fact; (c) the
penalty imposed upon the accused; and (d) the civil liability or damages caused by the wrongful act to be
recovered from the accused by the offended party, if there is any, unless the enforcement of the civil
liability by a separate action has been reserved or waived.
It is obvious that they clearly understood that they were found guilty beyond reasonable doubt of the
crime of murder and were sentenced to suffer the penalty of reclusion perpetua in Criminal Case No.

Page
Section 1-C, SY 06-07

155

4585. Were it otherwise, they would not have declared in open court their intention to appeal immediately
after the promulgation of the decision and would not have subsequently filed their written notice of appeal.
Accused-appellants contend that the trial court did not impose any sentence and so cannot cancel
anymore their bail bonds and direct their arrest and immediate commitment because it already lost
jurisdiction over their persons when they perfected their appeal.
The decision did impose the penalty of reclusion perpetua. Since the order cancelling their bail bonds and
directing their arrest is contained in the decision itself, it is apparent that their abovementioned contention
is highly illogical. At the time the order in question was made, the trial court still had jurisdiction over the
persons of the accused-appellants.
The trial court further erred in holding that no penalty could be imposed on accused Engracio Valeriano in
Criminal Case No. 4584 because he "is nowhere to be found, hence, not brought to the bar of justice, he
being a fugitive or at large." The court ignored the fact that Engracio jumped bail after he had been
arraigned, just before the retaking of evidence commenced. Paragraph (2), Section 14, Article III of the
Constitution permits trial in absentia after the accused has been arraigned provided he has been duly
notified of the trial and his failure to appear thereat is unjustified. One who jumps bail can never offer a
justifiable reason for his non-appearance during the trial.
Accordingly, after the trial in absentia, the court can render judgment in the case and promulgation may
be made by simply recording the judgment in the criminal docket with a copy thereof served upon his
counsel, provided that the notice requiring him to be present at the promulgation is served through his
bondsmen or warden and counsel.
In conclusion, because of reasonable doubt as to their guilt, the accused-appellants must be acquitted.
Every accused is presumed innocent until the contrary is proved; that presumption is solemnly
guaranteed by the Bill of Rights. The contrary requires proof beyond reasonable doubt, or that degree of
proof which produces conviction in an unprejudiced mind. Short of this, it is not only the right of the
accused to be freed; it is even the constitutional duty of the court to acquit him.
US VS. TAN TENG
[23 PHIL 145; G.R. NO. 7081; 7 SEP 1912]
Facts:
The defendant herein raped Oliva Pacomio, a seven-year-old girl. Tan Teng was gambling near the house
of the victim and it was alleged that he entered her home and threw the victim on the floor and place his
private parts over hers. Several days later, Pacomio was suffering from a disease called gonorrhea.
Pacomio told her sister about what had happened and reported it to the police.
Tan Teng was called to appear in a police line-up and the victim identified him. He was then stripped of his
clothing and was examined by a policeman. He was found to have the same symptoms of gonorrhea. The
policeman took a portion of the substance emitting from the body of the defendant and turned it over to
the Bureau of Science. The results showed that the defendant was suffering from gonorrhea.
The lower court held that the results show that the disease that the victim had acquired came from the
defendant herein. Such disease was transferred by the unlawful act of carnal knowledge by the latter. The
defendant alleged that the said evidence should be inadmissible because it was taken in violation of his
right against self-incrimination.
Issue:
Whether or Not the physical examination conducted was a violation of the defendants rights against selfincrimination.
Held:
The court held that the taking of a substance from his body was not a violation of the said right. He was
neither compelled to make any admissions or to answer any questions. The substance was taken from
his body without his objection and was examined by competent medical authority.
The prohibition of self-incrimination in the Bill of Rights is a prohibition of the use of physical or moral
compulsion to extort communications from him, and not an exclusion of his body as evidence, when it
may be material. It would be the same as if the offender apprehended was a thief and the object stolen by
him may be used as evidence against him.

Page
Section 1-C, SY 06-07

156

San Beda College of Law Alabang


Constitutional Law 2 Case Digests

VILLAFLOR VS. SUMMERS


[41 PHIL 62; G.R. NO. 16444; 8 SEP 1920]
Facts:
Petitioner Villaflor was charged with the crime of adultery. The trial judge ordered the petitioner to subject
herself into physical examination to test whether or not she was pregnant to prove the determine the
crime of adultery being charged to her. Herein petitioner refused to such physical examination interposing
the defense that such examination was a violation of her constitutional rights against self-incrimination.
Issue:
Whether or Not the physical examination was a violation of the petitioners constitutional rights against
self-incrimination.
Held:
No. It is not a violation of her constitutional rights. The rule that the constitutional guaranty, that no person
shall be compelled in any criminal case to be a witness against himself, is limited to a prohibition against
compulsory testimonial self-incrimination. The corollary to the proposition is that, an ocular inspection of
the body of the accused is permissible.
BELTRAN VS. SAMSON
[53 PHIL 570; G.R. NO. 32025; 23 SEPT 1929]
Facts:
Beltran, as a defendant for the crime of Falsification, refused to write a sample of his handwriting as
ordered by the respondent Judge. The petitioner in this case contended that such order would be a
violation of his constitutional right against self-incrimination because such examination would give the
prosecution evidence against him, which the latter should have gotten in the first place. He also argued
that such an act will make him furnish evidence against himself.
Issue:
Whether or not the writing from the fiscal's dictation by the petitioner for the purpose of comparing the
latter's handwriting and determining whether he wrote certain documents supposed to be falsified,
constitutes evidence against himself within the scope and meaning of the constitutional provision under
examination.
Held:
The court ordered the respondents and those under their orders desist and abstain absolutely and forever
from compelling the petitioner to take down dictation in his handwriting for the purpose of submitting the
latter for comparison. Writing is something more than moving the body, or the hands, or the fingers;
writing is not a purely mechanical act, because it requires the application of intelligence and attention; and
in the case at bar writing means that the petitioner herein is to furnish a means to determine whether or
not he is the falsifier, as the petition of the respondent fiscal clearly states. Except that it is more serious,
we believe the present case is similar to that of producing documents or chattels in one's possession. We
say that, for the purposes of the constitutional privilege, there is a similarity between one who is
compelled to produce a document, and one who is compelled to furnish a specimen of his handwriting, for
in both cases, the witness is required to furnish evidence against himself. It cannot be contended in the
present case that if permission to obtain a specimen of the petitioner's handwriting is not granted, the
crime would go unpunished. Considering the circumstance that the petitioner is a municipal treasurer, it
should not be a difficult matter for the fiscal to obtained genuine specimens of his handwriting. But even
supposing it is impossible to obtain specimen or specimens without resorting to the means complained
herein, that is no reason for trampling upon a personal right guaranteed by the constitution. It might be
true that in some cases criminals may succeed in evading the hand of justice, but such cases are
accidental and do not constitute the raison d' etre of the privilege. This constitutional privilege exists for
the protection of innocent persons.
PASCUAL VS. BME
[28 SCRA 345; G.R. NO. 25018; 26 MAY 1969]

Page
Section 1-C, SY 06-07

157

Facts:
Petitioner Arsenio Pascual, Jr. filed an action for prohibition against the Board of Medical Examiners. It
was alleged therein that at the initial hearing of an administrative case for alleged immorality, counsel for
complainants announced that he would present as his first witness the petitioner. Thereupon, petitioner,
through counsel, made of record his objection, relying on the constitutional right to be exempt from being
a witness against himself. Petitioner then alleged that to compel him to take the witness stand, the Board
of Examiners was guilty, at the very least, of grave abuse of discretion for failure to respect the
constitutional right against self-incrimination.
The answer of respondent Board, while admitting the facts stressed that it could call petitioner to the
witness stand and interrogate him, the right against self-incrimination being available only when a
question calling for an incriminating answer is asked of a witness. They likewise alleged that the right
against self-incrimination cannot be availed of in an administrative hearing.
Petitioner was sustained by the lower court in his plea that he could not be compelled to be the first
witness of the complainants, he being the party proceeded against in an administrative charge for
malpractice. Hence, this appeal by respondent Board.
Issue:
Whether or Not compelling petitioner to be the first witness of the complainants violates the SelfIncrimination Clause.
Held:
The Supreme Court held that in an administrative hearing against a medical practitioner for alleged
malpractice, respondent Board of Medical Examiners cannot, consistently with the self-incrimination
clause, compel the person proceeded against to take the witness stand without his consent. The Court
found for the petitioner in accordance with the well-settled principle that "the accused in a criminal case
may refuse, not only to answer incriminatory questions, but, also, to take the witness stand." If petitioner
would be compelled to testify against himself, he could suffer not the forfeiture of property but the
revocation of his license as a medical practitioner. The constitutional guarantee protects as well the right
to silence: "The accused has a perfect right to remain silent and his silence cannot be used as a
presumption of his guilt." It is the right of a defendant "to forego testimony, to remain silent, unless he
chooses to take the witness stand with undiluted, unfettered exercise of his own free genuine will."
The reason for this constitutional guarantee, along with other rights granted an accused, stands for a
belief that while crime should not go unpunished and that the truth must be revealed, such desirable
objectives should not be accomplished according to means or methods offensive to the high sense of
respect accorded the human personality. More and more in line with the democratic creed, the deference
accorded an individual even those suspected of the most heinous crimes is given due weight. The
constitutional foundation underlying the privilege is the respect a government ... must accord to the dignity
and integrity of its citizens.
PEOPLE VS. BALISACAN
[17 SCRA 1119; G.R. NO. L-26376; 31 AUG 1966]
Facts:
Aurelio Balisacan was charged with homicide in the CFI of Ilocos Norte. Upon being arraigned, he
entered into a plea of guilty. In doing so, he was assisted y counsel. At his counsel de officio, he was
allowed to present evidence and consequently testified that he stabbed the deceased in self-defense. In
addition, he stated that he surrendered himself voluntarily to the police authorities. On the basis of the
testimony of the accused, he was acquitted. Thus, the prosecution appealed.
Issue:
Whether or Not the appeal placed the accused in double jeopardy.
Held:
The Supreme Court held that it is settled that the existence of plea is an essential requisite to double
jeopardy. The accused had first entered a plea of guilty but however testified that he acted in complete
self-defense. Said testimony had the effect of vacating his plea of guilty and the court a quo should have

Page
Section 1-C, SY 06-07

158

San Beda College of Law Alabang


Constitutional Law 2 Case Digests
required him to plead a new charge, or at least direct that a new plea of not guilty be entered for him. This
was not done. Therefore, there has been no standing of plea during the judgment of acquittal, so there
can be no double jeopardy with respect to the appeal herein.
PEOPLE VS. OBSANIA
[23 SCRA 1249; G.R. L-24447; 29 JUN 1968]
Facts:
The accused was charged with Robbery with Rape before the Municipal Court of Balungao, Pangasinan.
He pleaded not guilty. His counsel moved for the dismissal of the charge for failure to allege vivid designs
in the info. Said motion was granted. From this order of dismissal the
prosecution appealed.
Issue:
Whether or Not the present appeal places the accused in Double Jeopardy.
Held:
In order that the accused may invoke double jeopardy, the following requisites must have obtained in the
original prosecution, a) valid complaint, b) competent court, c) the defendant had pleaded to the charge,
d) defendant was acquitted or convicted or the case against him was dismissed or otherwise terminated
without his express consent.
In the case at bar, the converted dismissal was ordered by the Trial Judge upon the defendant's motion to
dismiss. The doctrine of double jeopardy as enunciated in P.vs. Salico applies to wit when the case is
dismissed with the express consent of the defendant, the dismissal will not be a bar to another
prosecution for the same offense because his action in having the case is dismissed constitutes a
waiver of his constitutional right/privilege for the reason that he thereby prevents the Court from
proceeding to the trial on the merits and rendering a judgment of conviction against him.
In essence, where a criminal case is dismissed provisionally not only with the express consent of the
accused but even upon the urging of his counsel there can be no double jeopardy under Sect. 9 Rule 113,
if the indictment against him is revived by the fiscal.
PAULIN VS. GIMENEZ
[217 SCRA 386; G.R. NO. 103323; 21 JAN 1993]
Facts:
Respondent and Brgy Capt. Mabuyo, while in a jeep, were smothered with dust when they were
overtaken by the vehicle owned by Petitioner Spouses. Irked by such, Mabuyo followed the vehicle until
the latter entered the gate of an establishment. He inquired the nearby security guard for the identity of
the owner of the vehicle. Later that day, while engaged in his duties, petitioners allegedly pointed their
guns at him. Thus, he immediately ordered his subordinate to call the police and block road to prevent the
petitioners escape. Upon the arrival of the police, petitioners put their guns down and were immediately
apprehended.
A complaint grave threats was filed against the petitioners (Criminal Case No. 5204). It was dismissed
by the court acting on the motion of the petitioners. Mabuyo filed a MOR thus the dismissal was reversed.
Thereafter, petitioners filed for certiorari, prohibition, damages, with relief of preliminary injunction and the
issuance of a TRO (CEB-9207). Petition is dismissed for lack of merit and for being a prohibited pleading
and ordered to proceed with the trial of the case. Hence, this instant petition.
Issue:
Whether or Not the dismissal of 5204 was a judgment of acquittal.
Whether or Not the judge ignored petitioners right against double jeopardy by dismissing CEB-9207.
Held:
For double jeopardy to attach, the dismissal of the case must be without the express consent of the
accused. Where the dismissal was ordered upon motion or with the express assent of the accused, he

Page
Section 1-C, SY 06-07

159

has deemed to have waived his protection against double jeopardy. In the case at bar, the dismissal was
granted upon motion of the petitioners. Double jeopardy thus did not attach.
Furthermore, such dismissal is not considered as an acquittal. The latter is always based on merit that
shows that the defendant is beyond reasonable doubt not guilty. While the former, in the case at bar,
terminated the proceedings because no finding was made as to the guilt or innocence of the petitioners.
The lower court did not violate the rule when it set aside the order of dismissal for the reception of further
evidence by the prosecution because it merely corrected its error when it prematurely terminated and
dismissed the case without giving the prosecution the right to complete the presentation of its evidence.
The rule on summary procedure was correctly applied.
PEOPLE VS. COURT OF SILAY
[74 SCRA 248; G.R. NO. L-43790; 9 DEC 1976]
Facts:
That sometime on January 4,1974, accused Pacifico Sensio, Romeo Millan and Wilfredo Jochico who
were then scalers at the Hawaiian-Philippine Company, weighed cane cars No.1743,1686 and 1022
loaded with sugar canes which were placed in tarjetas (weight report cards), Apparently, it was proven
and shown that there was padding of the weight of the sugar canes and that the information on the
tarjetas were to be false making it appear to be heavier than its actual weight. The three accused then
were charged with Falsification by private individuals and use of falsified document. After the
prosecution had presented, the respondent moved to dismiss the charge against them on the ground that
the evidences presented were not sufficient to establish their guilt beyond reasonable doubt. Acting on
the motion, respondent court issued its order dismissing the case on the ground that the acts committed
by the accused do not constituted the crime of falsification as strictly enumerated in the revised penal
code defining the crime of falsification which was charged earlier and that their case be dismissed.
People asserts that the plea of double jeopardy is not tenable even if the case at bar was dismissed
because according to them, it was done with the consent of the accused therefore waiving there defense
of double jeopardy. The accused on the other hand, reiterated the fact that the dismissal was due to lack
of merits of the prosecution which would have the same effect as an acquittal which will bar the
prosecution from prosecuting the accused for it will be unjust and unconstitutional for the accused due to
double jeopardy rule thus the appeal of the plaintiff.
Issue:
Whether or Not the grant of petition by the court would place the accused Sensio, Millan and Jochico in
double jeopardy
Held:
Yes the revival of the case will put the accused in double jeopardy for the very reason that the case has
been dismissed earlier due to lack of merits. It is true that the criminal case of falsification was dismissed
on a motion of the accused however this was a motion filed after the prosecution had rested its case,
calling for the evidence beyond reasonable ground which the prosecution had not been able to do which
would be tantamount to acquittal therefore will bar the prosecution of another case. As it was stated on
the requirements of a valid defense of double jeopardy it says: That there should be a valid complaint,
second would be that such complaint be filed before a competent court and to which the accused has
pleaded and that defendant was previously acquitted, convicted or dismissed or otherwise terminated
without express consent of the accused in which were all present in the case at bar. There was indeed a
valid, legitimate complaint and concern against the accused Sensio, Millan and Jochico which was filed at
a competent court with jurisdiction on the said case. It was also mentioned that the accused pleaded not
guilty and during the time of trial, it was proven that the case used against the accused were not sufficient
to prove them guilty beyond reasonable doubt therefore dismissing the case which translates to acquittal.
It explained further that there are two instances when we can conclude that there is jeopardy when first is
that the ground for the dismissal of the case was due to insufficiency of evidence and second, when the
proceedings have been reasonably prolonged as to violate the right of the accused to a speedy trial. In
the 2 requisites given, it was the first on that is very much applicable to our case at bar where there was
dismissal of the case due to insufficiency of evidence which will bar the approval of the petition in the
case at bar for it will constitute double jeopardy on the part of the accused which the law despises.
PEOPLE VS. RELOVA
[149 SCRA 292; G.R. NO.L-45129; 6 MAR 1987]

Page
Section 1-C, SY 06-07

160

San Beda College of Law Alabang


Constitutional Law 2 Case Digests
FACTS: In this petition for certiorari and mandamus, People of the Philippines seeks to set aside the
orders of Respondent Judge Hon. Relova quashing an information for theft filed against Mr. Opulencia on
the ground of double jeopardy and denying the petitioners motion for reconsideration.. On Feb.1 1975,
Batangas police together with personnel of Batangas Electric Light System, equipped with a search
warrant issued by a city judge of Batangas to search and examine the premises of the Opulencia
Carpena Ice Plant owned by one Manuel Opulencia. They discovered electric wiring devices have been
installed without authority from the city government and architecturally concealed inside the walls of the
building. Said devices are designed purposely to lower or decrease the readings of electric current
consumption in the plants electric meter. The case was dismissed on the ground of prescription for the
complaint was filed nine months prior to discovery when it should be 2months prior to discovery that the
act being a light felony and prescribed the right to file in court. On Nov 24, 1975, another case was filed
against Mr. Opulencia by the Assistant City Fiscal of Batangas for a violation of a Batangas Ordinance
regarding unauthorized electrical installations with resulting damage and prejudice to City of Batangas in
the amount of P41,062.16. Before arraignment, Opulencia filed a motion to quash on the ground of
double jeopardy. The Assistant fiscals claim is that it is not double jeopardy because the first offense
charged against the accused was unauthorized installation of electrical devices without the approval and
necessary authority from the City Government which was punishable by an ordinance, where in the case
was dismissed, as opposed to the second offense which is theft of electricity which is punishable by the
Revised Penal Code making it a different crime charged against the 1 st complaint against Mr.Opulencia.
Issue:
Whether or Not the accused Mr. Opulencia can invoke double jeopardy as defense to the second offense
charged against him by the assistant fiscal of Batangas on the ground of theft of electricity punishable by
a statute against the Revised Penal Code.
Held:
Yes, Mr. Opulencia can invoke double jeopardy as defense for the second offense because as tediously
explained in the case of Yap vs Lutero, the bill of rights give two instances or kinds of double jeopardy.
The first would be that No person shall be twice put in jeopardy of punishment for the same offense and
the second sentence states that If an act is punishable by a law or an ordinance, the conviction or
acquittal shall bar to another prosecution for the same act. In the case at bar, it was very evident that the
charges filed against Mr. Opulencia will fall on the 2 nd kind or definition of double jeopardy wherein it
contemplates double jeopardy of punishment for the same act. It further explains that even if the offenses
charged are not the same, owing that the first charge constitutes a violation of an ordinance and the
second charge was a violation against the revised penal code, the fact that the two charges sprung from
one and the same act of conviction or acquittal under either the law or the ordinance shall bar a
prosecution under the other thus making it against the logic of double jeopardy. The fact that Mr.
Opulencia was acquitted on the first offense should bar the 2 nd complaint against him coming from the
same identity as that of the 1st offense charged against Mr.Opulencia.
ESMENA VS. POGOY
[102 SCRA 861; G.R. NO. L-54110; 20 FEB 1981]
Facts:
Petitioners Esmea and Alba were charged with grave coercion in the Court of Cebu City for allegedly
forcing Fr. Thomas Tibudan to withdraw a sum of money worth P5000 from the bank to be given to them
because the priest lost in a game of chance. During arraignment, petitioners pleaded Not Guilty. No
trial came in after the arraignment due to the priests request to move it on another date. Sometime later
Judge Pogoy issued an order setting the trial Aug.16,1979 but the fiscal informed the court that it received
a telegram stating that the complainant was sick. The accused invoked their right to speedy trial.
Respondent judge dismissed the case because the trial was already dragging the accused and that the
priests telegram did not have a medical certificate attached to it in order for the court to recognize the
complainants reason to be valid in order to reschedule again another hearing. After 27 days the fiscal
filed a motion to revive the case and attached the medical certificate of the priest proving the fact that the
priest was indeed sick of influenza. On Oct.24,1979, accused Esmea and Alba filed a motion to dismiss
the case on the ground of double jeopardy.
Issue:
Whether or Not the revival of grave coercion case, which was dismissed earlier due to complainants
failure to appear at the trial, would place the accused in double jeopardy
Held:

Page
Section 1-C, SY 06-07

161

Yes, revival of the case will put the accused in double jeopardy for the very reason that the case has been
dismissed already without the consent of the accused which would have an effect of an acquittal on the
case filed. The dismissal was due to complainants incapability to present its evidence due to non
appearance of the witnesses and complainant himself which would bar further prosecution of the
defendant for the same offense. For double jeopardy to exist these three requisites should be present,
that one, there is a valid complaint or information filed second, that it is done before a court of competent
jurisdiction and third, that the accused has been arraigned and has pleaded to the complaint or
information. In the case at bar, all three conditions were present, as the case filed was grave coercion,
filed in a court of competent jurisdiction as to where the coercion took place and last the accused were
arraigned and has pleaded to the complaint or the information. When these three conditions are present
then the acquittal, conviction of the accused, and the dismissal or termination of the case without his
express consent constitutes res judicata and is a bar to another prosecution for the offense charged. In
the case, it was evidently shown that the accused invoked their right to a speedy trial and asked for the
trial of the case and not its termination which would mean that respondents had no expressed consent to
the dismissal of the case which would make the case filed res judicata and has been dismissed by the
competent court in order to protect the respondents as well for their right to speedy trial which will be
equivalent to acquittal of the respondents which would be a bar to further prosecution.
PEOPLE VS. DE LA TORRE
[380 SCRA 586; G.R. NOS. 137953-58; 11 MAR 2002]
Facts:
Wilfredo dela Torre, appellee, has three children from a common-law relationship, the eldest of which is
Mary Rose. When Mary Rose was 7 yearsold, her mother left them together with her youngest brother so
she and her other brother were left to the care of her father.
Mary Rose was the brightest in her class despite their poverty. However, in January 1997, a sudden
change in Mary Roses behavior behavior was noticed. She was twelve years old at that time. She
appeared sleepy, snobbish and she also urinated on her panty. When confronted by her head teacher,
Mary Rose admitted that she was abused repeatedly by her father. Her father, however, denied
vehemently the charges being imputed to him by her daughter.
The RTC convicted appellee of two counts of acts of lasciviousness and four counts of murder. However,
the RTC refused to impose the supreme penalty of death on appellee. It maintained that there were
circumstances that mitigated the gravity of the offenses such as the absence of any actual physical
violence or intimidation on the commission of the acts, that after the mother of Mary Rose left the conjugal
home, for more than five years, Wilfredo, Mary Rose and her brother were living together as a family and
Mary Rose was never molested by her father.
The prosecution seeks to modify the RTC Decision by imposing the supreme penalty of death of the
accused. It argues that it has proven that the victim is the daughter of the accused, and that she was
below eighteen years old when the rapes took place. As a consequence, the trial court should have been
imposed the penalty of death pursuant to Section 11 of R.A. 7659. .
Issue:
Whether or Not the Court erred in penalizing the appellee with reclusion perpetua in each of the four
indictments of rape, instead of imposing the supreme penalty of death as mandated by R.A. 7659.
Held:
Under Section 1, Rule 122 of the 2000 Rules of Criminal Procedure, any party may appeal from a
judgment or final order unless the accused will be put in double jeopardy. In People vs. Leones, it
declared that:
while it is true that this Court is the Court of last resort, there are allegations of error
committed by a lower court which we ought not to look into to uphold the right of the
accused. Such is the case in an appeal by the prosecution seeking to increase the
penalty imposed upon the accused for this runs afoul of the right of the accused against
double jeopardyWhen the accused after conviction by the trial court did not appeal his
decision, an appeal by the government seeking to increase the penalty imposed by the
trial court places the accused in double jeopardy and should therefore be dismissed.

Page
Section 1-C, SY 06-07

162

San Beda College of Law Alabang


Constitutional Law 2 Case Digests
The ban on double jeopardy primarily prevents the State from using its criminal processes as an
instrument of harassment to wear out the accused by a multitude of cases with accumulated trials. It also
serves as a deterrent from successively retrying the defendant in the hope of securing a conviction. And
finally, it prevents the State, following conviction, from retrying the defendant again in the hope of securing
a greater penalty.
Being violative of the right against double jeopardy, the appeal of the prosecution cannot prosper.

Page
Section 1-C, SY 06-07

163

CITIZENSHIP

Page
Section 1-C, SY 06-07

164

San Beda College of Law Alabang


Constitutional Law 2 Case Digests
CITIZENSHIP
Art. 4
Sec. 1. The following are citizens of the Philippines:
(1)
Those who are citizens of the Philippines at the time of the adoption of this Constitution;
(2)
Those whose fathers or mothers are citizens of the Philippines;
(3)
Those born before January 17, 1973, of Filipino mothers, who elect Philippine citizenship
upon reaching the age of majority; and
(4)
Those who are naturalized in accordance with law.
Sec. 2. Natural-born citizens are those who are citizens of the Philippines from birth without having to
perform any act to acquire or perfect their Philippine citizenship. Those who elect Philippine citizenship in
accordance with paragraph (3), Section 1 hereof shall be deemed natural-born citizens.
Sec. 3. Philippine citizenship may be lost or reacquired in the manner provided by law.
Sec. 4. Citizens of the Philippines who marry aliens shall retain their citizenship, unless by their act or
omission they are deemed, under the law, to have renounced it.
Sec. 5. Dual allegiance of citizens is inimical to the national interest and shall be dealt with by law.
FRIVALDO VS. COMELEC
[174 SCRA 245; G.R. NO. 87193; 23 JUN 1989]
Facts:
Petitioner Juan G. Frivaldo was proclaimed governor-elect of the province of Sorsogon on January 22,
1988, and assumed office in due time. On October 27, 1988, the League of Municipalities, Sorsogon
Chapter, represented by its President, Estuye, who was also suing in his personal capacity, filed with the
COMELEC a petition for the annulment of Frivaldo; election and proclamation on the ground that he was
not a Filipino citizen, having been naturalized in the United States on January 20, 1983. In his answer
dated May 22, 1988, Frivaldo admitted that he was naturalized in the United States as alleged but
pleaded the special and affirmative defenses that he had sought American citizenship only to protect
himself against President Marcos. His naturalization, he said, was "merely forced upon himself as a
means of survival against the unrelenting persecution by the Martial Law Dictator's agents abroad." He
added that he had returned to the Philippines after the EDSA revolution to help in the restoration of
democracy. In their Comment, the private respondents reiterated their assertion that Frivaldo was a
naturalized American citizen and had not reacquired Philippine citizenship on the day of the election on
January 18, 1988. He was therefore not qualified to run for and be elected governor. They also argued
that their petition in the Commission on Elections was not really for quo warranto under Section 253 of the
Omnibus Election Code. The ultimate purpose was to prevent Frivaldo from continuing as governor, his
candidacy and election being null and void ab initio because of his alienage. Speaking for the public
respondent, the Solicitor General supported the contention that Frivaldo was not a citizen of the
Philippines and had not repatriated himself after his naturalization as an American citizen. As an alien, he
was disqualified from public office in the Philippines. His election did not cure this defect because the
electorate of Sorsogon could not amend the Constitution, the Local Government Code, and the Omnibus
Election Code. He also joined in the private respondent's argument that Section 253 of the Omnibus
Election Code was not applicable because what the League and Estuye were seeking was not only the
annulment of the proclamation and election of Frivaldo. He agreed that they were also asking for the
termination of Frivaldo's incumbency as governor of Sorsogon on the ground that he was not a Filipino.
Issue:
Whether or Not petitioner Juan G. Frivaldo was a citizen of the Philippines at the time of his election on
January 18, 1988, as provincial governor of Sorsogon.
Held:
The reason for this inquiry is the provision in Article XI, Section 9, of the Constitution that all public
officials and employees owe the State and the Constitution "allegiance at all times" and the specific
requirement in Section 42 of the Local Government Code that a candidate for local elective office must be
inter alia a citizen of the Philippines and a qualified voter of the constituency where he is running. Section
117 of the Omnibus Election Code provides that a qualified voter must be, among other qualifications, a

Page
Section 1-C, SY 06-07

165

citizen of the Philippines, this being an indispensable requirement for suffrage under Article V, Section 1,
of the Constitution.
In the certificate of candidacy he filed on November 19, 1987, Frivaldo described himself as a "naturalborn" citizen of the Philippines, omitting mention of any subsequent loss of such status. The evidence
shows, however, that he was naturalized as a citizen of the United States in 1983 per the following
certification from the United States District Court, Northern District of California, as duly authenticated by
Vice Consul Amado P. Cortez of the Philippine Consulate General in San Francisco, California, U.S.A.
The Court sees no reason not to believe that the petitioner was one of the enemies of the Marcos
dictatorship. Even so, it cannot agree that as a consequence thereof he was coerced into embracing
American citizenship. His feeble suggestion that his naturalization was not the result of his own free and
voluntary choice is totally unacceptable and must be rejected outright.
This Court will not permit the anomaly of a person sitting as provincial governor in this country while
owing exclusive allegiance to another country. The fact that he was elected by the people of Sorsogon
does not excuse this patent violation of the salutary rule limiting public office and employment only to the
citizens of this country. The qualifications prescribed for elective office cannot be erased by the electorate
alone. The will of the people as expressed through the ballot cannot cure the vice of ineligibility, especially
if they mistakenly believed, as in this case, that the candidate was qualified. Obviously, this rule requires
strict application when the deficiency is lack of citizenship. If a person seeks to serve in the Republic of
the Philippines, he must owe his total loyalty to this country only, abjuring and renouncing all fealty and
fidelity to any other state.
It is true as the petitioner points out that the status of the natural-born citizen is favored by the
Constitution and our laws, which is all the more reason why it should be treasured like a pearl of great
price. But once it is surrendered and renounced, the gift is gone and cannot be lightly restored. This
country of ours, for all its difficulties and limitations, is like a jealous and possessive mother. Once
rejected, it is not quick to welcome back with eager arms its prodigal if repentant children. The returning
renegade must show, by an express and unequivocal act, the renewal of his loyalty and love.
Petition Dismissed. Petitioner JUAN G. FRIVALDO is hereby declared not a citizen of the Philippines and
therefore disqualified from serving as Governor of the Province of Sorsogon. Accordingly, he is ordered to
vacate his office and surrender the same to the duly elected Vice-Governor of the said province once this
decision becomes final and executory.
MERCADO VS. MANZANO
[307 SCRA 630; G.R. NO. 135083; 26 MAY 1999]
Facts:
Petitioner Ernesto Mercado and Private respondent Eduardo Manzano are candidates for the position of
Vice-Mayor of Makati City in the May, 1998 elections. Private respondent was the winner of the said
election but the proclamation was suspended due to the petition of Ernesto Mamaril regarding the
citizenship of private respondent. Mamaril alleged that the private respondent is not a citizen of the
Philippines but of the United States. COMELEC granted the petition and disqualified the private
respondent for being a dual citizen, pursuant to the Local Government code that provides that persons
who possess dual citizenship are disqualified from running any public position. Private respondent filed a
motion for reconsideration which remained pending until after election. Petitioner sought to intervene in
the case for disqualification. COMELEC reversed the decision and declared private respondent qualified
to run for the position. Pursuant to the ruling of the COMELEC, the board of canvassers proclaimed
private respondent as vice mayor. This petition sought the reversal of the resolution of the COMELEC
and to declare the private respondent disqualified to hold the office of the vice mayor of Makati.
Issue:
Whether or Not private respondent is qualified to hold office as Vice-Mayor.
Held:
Dual citizenship is different from dual allegiance. The former arises when, as a result of the concurrent
application of the different laws of two or more states, a person is simultaneously considered a national by
the said states. For instance, such a situation may arise when a person whose parents are citizens of a
state which adheres to the principle of jus sanguinis is born in a state which follows the doctrine of jus
soli. Private respondent is considered as a dual citizen because he is born of Filipino parents but was
born in San Francisco, USA. Such a person, ipso facto and without any voluntary act on his part, is

Page
Section 1-C, SY 06-07

166

San Beda College of Law Alabang


Constitutional Law 2 Case Digests
concurrently considered a citizen of both states. Considering the citizenship clause (Art. IV) of our
Constitution, it is possible for the following classes of citizens of the Philippines to posses dual citizenship:
(1) Those born of Filipino fathers and/or mothers in foreign countries which follow the principle of jus soli;
(2) Those born in the Philippines of Filipino mothers and alien fathers if by the laws of their fathers
country such children are citizens of that country; (3) Those who marry aliens if by the laws of the latters
country the former are considered citizens, unless by their act or omission they are deemed to have
renounced Philippine citizenship. Dual allegiance, on the other hand, refers to the situation in which a
person simultaneously owes, by some positive act, loyalty to two or more states. While dual citizenship is
involuntary, dual allegiance is the result of an individuals volition.
By filing a certificate of candidacy when he ran for his present post, private respondent elected Philippine
citizenship and in effect renounced his American citizenship. The filing of such certificate of candidacy
sufficed to renounce his American citizenship, effectively removing any disqualification he might have as a
dual citizen.
By declaring in his certificate of candidacy that he is a Filipino citizen; that he is not a permanent resident
or immigrant of another country; that he will defend and support the Constitution of the Philippines and
bear true faith and allegiance thereto and that he does so without mental reservation, private respondent
has, as far as the laws of this country are concerned, effectively repudiated his American citizenship and
anything which he may have said before as a dual citizen. On the other hand, private respondents oath
of allegiance to the Philippine, when considered with the fact that he has spent his youth and adulthood,
received his education, practiced his profession as an artist, and taken part in past elections in this
country, leaves no doubt of his election of Philippine citizenship.
TECSON VS. COMELEC
[424 SCRA 277; G.R. No. 161434; 3 Mar 2004]
Facts:
Victorino X. Fornier, petitioner initiated a petition before the COMELEC to disqualify FPJ and to deny due
course or to cancel his certificate of candidacy upon the thesis that FPJ made a material
misrepresentation in his certificate of candidacy by claiming to be a natural-born Filipino citizen when in
truth, according to Fornier, his parents were foreigners; his mother, Bessie Kelley Poe, was an American,
and his father, Allan Poe, was a Spanish national, being the son of Lorenzo Pou, a Spanish subject.
Granting, petitioner asseverated, that Allan F. Poe was a Filipino citizen, he could not have transmitted his
Filipino citizenship to FPJ, the latter being an illegitimate child of an alien mother. Petitioner based the
allegation of the illegitimate birth of respondent on two assertions - first, Allan F. Poe contracted a prior
marriage to a certain Paulita Gomez before his marriage to Bessie Kelley and, second, even if no such
prior marriage had existed, Allan F. Poe, married Bessie Kelly only a year after the birth of respondent.

Issue:
Whether or Not FPJ is a natural born Filipino citizen.
Held:
It is necessary to take on the matter of whether or not respondent FPJ is a natural-born citizen, which, in
turn, depended on whether or not the father of respondent, Allan F. Poe, would have himself been a
Filipino citizen and, in the affirmative, whether or not the alleged illegitimacy of respondent prevents him
from taking after the Filipino citizenship of his putative father. Any conclusion on the Filipino citizenship of
Lorenzo Pou could only be drawn from the presumption that having died in 1954 at 84 years old, Lorenzo
would have been born sometime in the year 1870, when the Philippines was under Spanish rule, and that
San Carlos, Pangasinan, his place of residence upon his death in 1954, in the absence of any other
evidence, could have well been his place of residence before death, such that Lorenzo Pou would have
benefited from the "en masse Filipinization" that the Philippine Bill had effected in 1902. That citizenship
(of Lorenzo Pou), if acquired, would thereby extend to his son, Allan F. Poe, father of respondent FPJ.
The 1935 Constitution, during which regime respondent FPJ has seen first light, confers citizenship to all
persons whose fathers are Filipino citizens regardless of whether such children are legitimate or
illegitimate.
But while the totality of the evidence may not establish conclusively that respondent FPJ is a natural-born
citizen of the Philippines, the evidence on hand still would preponderate in his favor enough to hold that
he cannot be held guilty of having made a material misrepresentation in his certificate of candidacy in
violation of Section 78, in relation to Section 74, of the Omnibus Election Code.

Page
Section 1-C, SY 06-07

167

BENGZON VS. HRET


[357 SCRA 545; G. R. No. 142840; 7 May 2001]
Facts:
Respondent Teodoro Cruz was a natural-born citizen of the Philippines. He was born in San Clemente,
Tarlac, on April 27, 1960, of Filipino parents. The fundamental law then applicable was the 1935
Constitution. On November 5, 1985, however, respondent Cruz enlisted in the United States Marine
Corps and without the consent of the Republic of the Philippines, took an oath of allegiance to the United
States. As a Consequence, he lost his Filipino citizenship for under Commonwealth Act No. 63, section
1(4), a Filipino citizen may lose his citizenship by, among other, "rendering service to or accepting
commission in the armed forces of a foreign country. He was naturalized in US in 1990. On March 17,
1994, respondent Cruz reacquired his Philippine citizenship through repatriation under Republic Act No.
2630. He ran for and was elected as the Representative of the Second District of Pangasinan in the May
11, 1998 elections. He won over petitioner Antonio Bengson III, who was then running for reelection.

Issue:
Whether or Not respondent Cruz is a natural born citizen of the Philippines in view of the constitutional
requirement that "no person shall be a Member of the House of Representative unless he is a naturalborn citizen.
Held:
Respondent is a natural born citizen of the Philippines. As distinguished from the lengthy process of
naturalization, repatriation simply consists of the taking of an oath of allegiance to the Republic of the
Philippine and registering said oath in the Local Civil Registry of the place where the person concerned
resides or last resided. This means that a naturalized Filipino who lost his citizenship will be restored to
his prior status as a naturalized Filipino citizen. On the other hand, if he was originally a natural-born
citizen before he lost his Philippine citizenship, he will be restored to his former status as a natural-born
Filipino.

Page
Section 1-C, SY 06-07

168

You might also like